TP Questions for More Review Flashcards

1
Q

To be consistent with ethical requirements, David DeFoe, a psychology intern, must do which of the following when a supervisor has legal responsibility for his work?
A. He must tell his clients that he is a psychology intern.
B. He must tell his clients that his work is being supervised
C. He must provide clients with information about his education and experience
D. He must indicate his status on his business card

A

Standard 10.10 of the APA’s Ethics Code & Principle III.22 of the Canadian Code of Ethics apply to the situation described in the question.
A. He must tell his clients that he is a psychology intern - CORRECT: This answer is most consistent w/ethical guidelines which require interns (Trainees) to inform their clients of tier professional statuses. Note that Standard 10.10(c) of the Ethics Code also requires that Mr. DeFoe provide clients with the name of his supervisor.
B. He must tell his clients that his work is being supervised - Incorrect: Telling clients that he is being supervised would not be adequate since this would not clearly indicate that he is an intern.
(Ethics Question)

How well did you know this?
1
Not at all
2
3
4
5
Perfectly
2
Q

You have been seeing Leticia Lopez in therapy for several months. Leticia is 24 years old and lives with her widowed mother, who is paying for Leticia’s therapy. One day, Leticia’s mother calls and says she is very concerned
about Leticia and wants to know what she can do to help Leticia feel better about herself. Mrs. Lopez asks that you not tell Leticia that she has called. Your best course of action in this situation would be to:
A. give Mrs. Lopez the specific advice she has requested.
B. tell Mrs. Lopez to ask Leticia what she (Mrs. Lopez) can do to help her.
C. tell Mrs. Lopez that it would be best if you discussed this matter with Leticia
D. suggest that Mrs. Lopez accompany Leticia to her next therapy session.

A

Leticia is an adult and, even though her mother is paying for her therapy, this does not limit Leticia’s confidentiality.
C. tell Mrs. Lopez that it would be best if you discussed this matter with Leticia - CORRECT This course of action is most consistent with the ethical guidelines and is also likely to be in the best interests of Leticia from a clinical perspective.
a. Incorrect Discussing Leticia’s status with her mother without Leticia’s consent would represent a breach of confidentiality.
b. Incorrect This alternative does not explicitly violate ethical guidelines, but response c is a better answer.
d. Incorrect This would not be the most appropriate course of action from either an ethical or clinical perspective.
(Ethics Question)

How well did you know this?
1
Not at all
2
3
4
5
Perfectly
3
Q

Dr. Calvin Claymore, a licensed psychologist, regularly waives the co-payment for low-income clients whose therapy fees are covered by insurance. This practice is:
A. ethical only if the insurance company has agreed to this arrangement
B. ethical since it is in the best interests of low-income clients.
C. ethical as long as Dr. Claymore doesn’t bill the insurance company for an increased hourly fee in order to collect her full fee.
D. ethical as long as the insurance company does not explicitly prohibit this arrangement.

A

The practice described in this question represents insurance fraud, which is both unethical and illegal.
A. ethical only if the insurance company has agreed to this arrangement - CORRECT Most insurance companies agree to pay a specific percentage of a psychologist’s Fee but, when the co-payment is waived, this means the insurance company is paying the entire Fee. Consequently, regularly waiving the co-payment without the permission of the insurance company represents insurance fraud.
(Ethics Question)

How well did you know this?
1
Not at all
2
3
4
5
Perfectly
4
Q
Which of the following is responsible for the largest proportion of complaints filed with the APA’s Ethics Committee?
A. sexual misconduct
B. test misuse
C. breach of confidentiality 
D. discrimination
A

Although the types of actions that underlie complaints filed with the APA’s Ethics Committee vary in number from year to year, one of the actions listed in the answers to this question has consistently been the most frequent cause of complaints.
a. Sexual Misconduct - CORRECT The 2005 Report of the Ethics Committee, For example, identifies sexual misconduct of an adult as being the behavior most commonly cited in cases opened by the Ethics Committee, which is consistent with previous
reports. Test misuse, breach of confidentiality, and discrimination (answers b, c, and d) are causes of complaints but not as often as is sexual misconduct.
(Ethics Question)

How well did you know this?
1
Not at all
2
3
4
5
Perfectly
5
Q

Within the context of sexual harassment, “quid pro quo” refers to:
A. a situation in which an employee’s response to sexual demands impacts his/her job.
B. the creation of a “hostile environment” as a result of sexually offensive conduct.
C. the judgment of a “reasonable woman” that an act is offensive.
C. overt (explicit) sexual conduct.

A

“Quid pro quo” is one of two legally recognized types of sexual harassment.
A. a situation in which an employee’s response to sexual demands impacts his/her job - CORRECT Quid pro quo sexual harassment occurs when a person’s submission to or rejection of sexual advances affects his/her employment status (Hostile environment sexual harassment is the other legally recognized type of
sexual harassment.)
(Ethics Question)

How well did you know this?
1
Not at all
2
3
4
5
Perfectly
6
Q
Administering a drug that reduces dopamine to below normal levels would have which effect on the symptoms of schizophrenia?
A. exacerbate symptoms
B. decrease or eliminate symptoms 
C. have no effect on symptoms
D. affect negative symptoms only
A

According to the dopamine hypothesis, schizophrenia is due to excessive dopamine or oversensitivity to dopamine. Consequently, drugs that reduce dopamine levels will decrease the symptoms of this disorder.
B. decrease or eliminate symptoms - CORRECT The traditional antipsychotic drugs exert their beneficial effects primarily by reducing dopamine to below-normal levels.
d. Incorrect The traditional antipsychotics actually seem to have more effect on the positive symptoms than on the negative symptoms of schizophrenia.
(Abnormal Psychology Question - DSM -IV)

How well did you know this?
1
Not at all
2
3
4
5
Perfectly
7
Q

In terms of neurotransmitters, bulimia nervosa has been linked to:
A. lower-than-normal levels of serotonin and norepinephrine.
B. a lower-than normal level of serotonin and a higher-than-normal level of norepinephrine.
C. a lower-than-normal level of norepinephrine and a higher-than normal level of serotonin.
D. higher-than-normal levels of serotonin and norepinephrine.

A

Bulimia has been attributed to abnormalities in serotonin,
norepinephrine, and dopamine.
A. lower-than-normal levels of serotonin and norepinephrine - CORRECT Bulimia has been linked to low levels of both serotonin and norepinephrine; and several recent studies have confirmed that serotonin and
norepinephrine reuptake inhibitors (e.g., duloxetine) are useful For eliminating the binging and purging behaviors associated with this disorder.
(Abnormal Psychology Question - DSM -IV)

How well did you know this?
1
Not at all
2
3
4
5
Perfectly
8
Q

The assessment of patients with Alzheimer’s dementia is an ongoing process due to the degenerative nature of the disease and the consequent need to alter the nature of the treatment plan. During the 4th or 5th year of the
disorder, an assessment is most likely to reveal:
A. deficits in new learning, mild to moderate impairment in remote memory, anomia, and sadness.
B. severe impairments in recent and remote memory, fluent aphasia, indifference or irritability, and restlessness.
C. severe impairments in memory and executive functioning, motor rigidity, confusion, and delusions.
D. severely impaired intellectual functioning, limb rigidity, apathy, and seizures.

A

Alzheimer’s dementia is a degenerative disease with symptoms becoming progressively worse over time.
a. Incorrect These symptoms are more common during the first 2 to 3 years of the disorder.
B. severe impairments in recent and remote memory, fluent aphasia, indifference or irritability, and restlessness - CORRECT These are characteristic symptoms during the 3rd through 10th years.
c. Incorrect These are late-stage symptoms (8 to 12 years).
d. Incorrect Severely impaired intellectual Functioning, limb rigidity, and apathy are also later symptoms; and seizures are not necessarily associated with this disorder.
(Abnormal Psychology Question - DSM -IV)

How well did you know this?
1
Not at all
2
3
4
5
Perfectly
9
Q

According to the catecholarnine hypothesis:
A. mania is due to a deficiency in norepinephrine.
B. depression is due to a deficiency in norepinephrine
C. mania is due to excessive acetylcholine.
D. depression is due to excessive acetylcholine.

A

Knowing that the catecholamines include epinephrine, norepinephrine, and dopamine would have helped you identify the correct response to this question.
a. Incorrect The catecholamine hypothesis (Schildkraut, 1965, 1978) predicts that mania is due to excessive norepinephrine.
B. depression is due to a deficiency in norepinephrine - CORRECT According to the catecholamine hypothesis, at least some types of depression are due to a lower-than-normal level of norepinephrine.
(Abnormal Psychology Question - DSM -IV)

How well did you know this?
1
Not at all
2
3
4
5
Perfectly
10
Q
If one of the offspring in a family develops schizophrenia, what is the likelihood that his/her biological sibling will also develop this disorder?
A. 2 %
B. 10 %
C. 25%
D. 45 %
A

Although the concordance rates for schizophrenia vary somewhat from study to study, only one of the correlations given in the answers comes close to the rates reported in the literature for biological siblings.
B. 10 % - CORRECT This is the typical correlation reported in the literature for biological siblings.
(Abnormal Psychology Question - DSM -IV)

How well did you know this?
1
Not at all
2
3
4
5
Perfectly
11
Q
Research investigating comorbidity in children suggests that, when depression occurs in conjunction with \_\_\_\_\_\_\_, the depression is often associated with a different course and a different family background than
when it occurs alone.
A. an anxiety disorder
B. conduct disorder
C. somatic complaints
D. learning problems
A

This is a difficult question that requires you to be an expert on childhood depression. For these kinds of questions, if you don’t know the answer, don’t spend too much time trying to figure them out: Make an “educated guess” and move on to the next question.
a. Incorrect Anxiety commonly occurs in conjunction with depressive symptoms in children. The research suggests that when anxiety and depression occur together, the depression is similar in terms of course and family background to depression that occurs without anxiety.
B. conduct disorder - CORRECT Some experts suggest that depression occurring in conjunction with conduct disorder is actually a different type of depression since it is associated with a lower rate of depression in adulthood as well as with a lower rate of depression among relatives. See R. Harrington, Affective disorders, in M. Rutter, et al., (eds.), Child and Adolescent Psychiatry, Oxford, Blackwell Scientific Publications, 1994.
c. Incorrect There is currently no research suggesting that somatic complaints are indicative of a different form of depression.
d. Incorrect There is currently no evidence that suggests that learning problems are indicative of a different form of depression.
(Abnormal Psychology Question - DSM -IV)

How well did you know this?
1
Not at all
2
3
4
5
Perfectly
12
Q

The research suggests that the most effective treatment for nicotine dependence in terms of both short- and long-term effects combines:
A. a 12-step program with individual therapy.
B. a 12-step program with relapse prevention.
C. nicotine replacement therapy with aversion therapy.
D. nicotine replacement therapy with behavioral interventions.

A

Based on a review of the empirical research, the American Psychiatric Association (1996) concluded that a multimodal intervention is the most successful approach for treating nicotine dependence.
D. nicotine replacement therapy with behavioral interventions - CORRECT The APA Found that the most successful programs include nicotine replacement therapy, multicomponent behavior therapy, and support and assistance from a clinician.
(Abnormal Psychology Question - DSM -IV)

How well did you know this?
1
Not at all
2
3
4
5
Perfectly
13
Q
Dissociative amnesia most commonly involves:
A. short-term memory loss.
B. retrospective gaps in memory.
C. widespread retrograde amnesia.
D. a period of unconsciousness.
A

Dissociative amnesia is characterized by an inability to recall important personal information that is often related to a traumatic event.
a. Incorrect Short-term memory is usually not affected in dissociative amnesia.
B. retrospective gaps in memory - CORRECT Dissociative amnesia involves retrospective gaps in the recall of aspects of the individual’s past, often aspects related to a trauma or stressor - i.e., the individual cannot recall events related to or following exposure to a traumatic or stressful event.
c. Incorrect Memory loss is most often related to a traumatic event and does not usually entail widespread retrograde amnesia — i.e., it does not include memory
loss for events that occurred prior to the event.
d. Incorrect A period of unconsciousness is not characteristic of this disorder.
(Abnormal Psychology Question - DSM -IV)

How well did you know this?
1
Not at all
2
3
4
5
Perfectly
14
Q

Apraxia, a symptom of dementia, involves which of the following?
A. deficits in written and/or spoken language
B. inability to execute voluntary motor movements
C. incoordination, clumsiness, and a lack of balance
D. inability to recognize familiar objects or people

A

The diagnosis of dementia requires some degree of memory impairment and aphasia, apraxia, agnosia, and/or impaired executive functioning.
a. Incorrect Aphasia refers to deficits in written and/or spoken language.
B. inability to execute voluntary motor movements - CORRECT A person with apraxia has difficulty executing voluntary motor movements that is NOT due to problems in muscle strength or a lack of cooperation.
c. Incorrect Ataxia is characterized by incoordination, clumsiness, and a lack of balance.
d. Incorrect Agnosia is the inability to recognize or identify familiar objects, people, or sounds.
(Abnormal Psychology Question - DSM -IV)

How well did you know this?
1
Not at all
2
3
4
5
Perfectly
15
Q

Unilateral electroconvulsive shock therapy (ECT) to the right hemisphere would most likely produce:
A. predominantly anterograde amnesia involving nonverbal material.
B. predominantly anterograde amnesia involving verbal material.
C. predominantly retrograde amnesia involving nonverbal material.
D. predominantly retrograde amnesia involving verbal material.

A

Knowing that, in most people, the left hemisphere mediates verbal memories while the right hemisphere mediates nonverbal memories would have helped narrow the responses to a and c.
A. predominantly anterograde amnesia involving nonverbal material - CORRECT ECT can produce both anterograde and retrograde amnesia but anterograde amnesia predominates. Consequently, following unilateral ECT to the right hemisphere, a person will have the most trouble forming new nonverbal memories. See, e.g., E. Kolb and I. Q. Wishaw, Fundamentals of Human
Neuropsycnology, New York, W. H. Freeman and Co., 1995.
(Abnormal Psychology Question - DSM -IV)

How well did you know this?
1
Not at all
2
3
4
5
Perfectly
16
Q

Compared to children and adolescents in the general population, children and adolescents with a learning disorder have:
A. a lower rate of psychopathology.
B. a higher rate of psychopathology.
C. the same rate of psychopathology.
D. the same rate of psychopathology but more severe symptoms.

A

Although the specific diagnoses that commonly co—occur with the learning disorders are relatively limited in number, the rates of these diagnoses are higher than the rates in the general population.
B. a higher rate of psychopathology - CORRECT As an example, the prevalence rate for ADHD is 3 to 5% in the general population but between 20 and 25% for children with a learning disorder.
(Abnormal Psychology Question - DSM -IV)

How well did you know this?
1
Not at all
2
3
4
5
Perfectly
17
Q
Orgasmic reconditioning is considered to be an effective treatment for which of the following disorders?
A. male erectile disorder
B. male and female orgasmic disorder 
C. paraphilia
D. gender identity disorder
A

Orgasmic reconditioning is based on the assumption that orgasm reinforces sexual fantasies, and it is used to replace the stimuli that produce an organism.
C. paraphilia - CORRECT Orgasmic reconditioning is one of the techniques used to treat paraphilias and involves having the individual replace an unacceptable sexual
fantasy with a more acceptable one while masturbating.
(Abnormal Psychology Question - DSM -IV)

How well did you know this?
1
Not at all
2
3
4
5
Perfectly
18
Q

In therapy, an American Indian family is likely to respond most positively to interventions that:
A. reflect a “value—free” perspective.
B. reflect a spiritual, holistic’ orientation to life.
C. make a clear distinction between mental and physical well-being.
D. take into account the hierarchical structure of the family.

A

It is always important to take a therapy client’s culture into account when deriving a diagnosis and developing a treatment plan.
a. Incorrect When working with American Indian clients, adopting a “value-free” perspective can lead to mistrust.
B. reflect a spiritual, holistic’ orientation to life - CORRECT American Indians are likely to have a spiritual, holistic orientation to life that emphasizes harmony with nature.
c. Incorrect As a general rule, American Indians do not make clear distinctions between mental and physical health.
d. Incorrect The structure of American Indian families varies; however, families are usually not hierarchical.
(Clinical Psychology Question)

How well did you know this?
1
Not at all
2
3
4
5
Perfectly
19
Q

Teddy Roosevelt was weak and ill as a child but grew up to be a robust adult and coined the term “rugged individualism.” He also became associated with
the slogan “speak softly but carry a big stick.” This outcome is predicted by ______theory of personality.
A. Adler’s
B. Rogers’s
C. Perls’s
D. Jung’s

A

This is a fairly simple question as long as you recall that Adler is associated with the concepts of “feelings of inferiority” and “striving for superiority.”
A. Adler’s - CORRECT According to Adler, real or imagined organ inferiority leads to feelings of inferiority. However, people can respond to these feelings in a
constructive way and pursue goals that help them achieve superiority.
(Clinical Psychology Question)

How well did you know this?
1
Not at all
2
3
4
5
Perfectly
20
Q
The belief that a child‘s misbehavior has one of four goals - i.e. attention, revenge, power, or to display inadequacy -is most consistent with: 
A. Beck’s cognitive-behavioral therapy.
B. Adler’s individual psychology.
C. Perls’s Gestalt therapy.
D. Mahler’s object relations theory.
A

Adler believed that all behaviors are goal—directed and purposeful.
B. Adler’s individual psychology - CORRECT Dreikurs, an associate of Adler’s, described the misbehavior of
children as attempts to belong, which reflect faulty beliefs about what it takes to belong (e.g., “I belong only when I’m the center of attention”).
(Clinical Psychology Question)

How well did you know this?
1
Not at all
2
3
4
5
Perfectly
21
Q

Carl Jung believed that a client’s transference:
A. is a fantasy that distracts the client from reality.
B. represents “mixed feelings” toward the therapist.
C. is a form of “acting out.”
D. reflects the client’s personal and collective unconscious

A

The correct answer to this question should have been easy to identify if you’re at all familiar with Jung’s work, since only one of the answers includes language that is associated with his analytical psychology.
D. reflects the client’s personal and collective unconscious - CORRECT Jung viewed transference as the projection of both the personal and collective unconscious.
(Clinical Psychology Question)

How well did you know this?
1
Not at all
2
3
4
5
Perfectly
22
Q

A formative evaluation is conducted:
A. prior to developing an intervention to determine what the intervention should include.
B. during the delivery of the intervention to identify ways to improve it.
C. at the end of the intervention to evaluate its effectiveness.
D. at the end of the intervention to evaluate participants’ reactions to it.

A

In the evaluation literature, a distinction is made between formative and summative evaluation.
a. Incorrect This sounds more like a needs analysis.
B. during the delivery of the intervention to identify ways to improve it - CORRECT As its name suggests, a formative evaluation is conducted while an
intervention is being “formed” [developed or delivered] and is used to determine if the intervention needs to be modified in order to meet its goals.
c. Incorrect This describes a summative evaluation.
d. Incorrect This also sounds more like a summative evaluation.
(Clinical Psychology Question)

How well did you know this?
1
Not at all
2
3
4
5
Perfectly
23
Q
A practitioner of which of the following is most likely to agree that the therapist's role is to bring unconscious issues that exist within a family to a conscious level?
A. systemic family therapy
B. existential family therapy
C. structural family therapy
D. object relations family therapy
A

Practitioners of psychodynamic psychotherapy are interested in the impact of unconscious issues on current behavior; and only one of the therapies listed in the answers is a psychodynamic approach.
D. object relations family therapy - CORRECT Object relations family therapy is psychoclynamic and links
intrapsychic phenomena to current family relationships.
(Clinical Psychology Question)

How well did you know this?
1
Not at all
2
3
4
5
Perfectly
24
Q
From the perspective of Gestalt therapy, introjection is:
A. the result of an unresolved conflict.
B. an image disturbance.
C. a boundary disturbance.
D. a defense mechanism.
A

Gestaltians use the term introjection to describe an overly permeable boundary between the person and the environment.
C. a boundary disturbance - CORRECT For Gestaltians, neurotic behavior is often the result of a disturbance
in the boundary between the person and his/her external environment. Introjection is one type of boundary disturbance and occurs when a person accepts values, beliefs, etc. from the environment without actually understanding or fully assimilating them.
(Clinical Psychology Question)

How well did you know this?
1
Not at all
2
3
4
5
Perfectly
25
Q

A family therapist working from the perspective of social learning theory is likely to describe spouse abuse as:
A. a homeostatic mechanism that serves to restore the abuser’s control over the relationship.
B. the result of repetitious and dysfunctional transactional patterns.
C. an acquired response that has been maintained by its ability to reduce stress.
D. isolation of the marital dyad from outside social (corrective) influences.

A

Social learning theory emphasizes the impact of parents and others on behavior as well as the internal, vicarious, and/or external consequences of a behavior.
C. an acquired response that has been maintained by its ability to reduce stress - CORRECT This answer is the only one that addresses the impact of learning and consequences on behavior and, therefore, is the best response.
(Clinical Psychology Question)

How well did you know this?
1
Not at all
2
3
4
5
Perfectly
26
Q
From the perspective of Jay i-ia|ey’s strategic family therapy, “symptoms” are primarily \_\_\_\_\_\_\_phenomena.
A. affective
B. intrapsychic 
C. interpersonal
D. cognitive
A

Haley’s strategic family therapy was influenced by the
communication/interaction school of family therapy and emphasizes the role of interaction in family functioning.
C. interpersonal - CORRECT For strategic family therapists, a symptom is an interpersonal phenomenon that represents an attempt to control a relationship.
(Clinical Psychology Question)

How well did you know this?
1
Not at all
2
3
4
5
Perfectly
27
Q
A wife approaches her husband in an affectionate way. He pulls away from her; but when she begins to leave the room, he says in a hurt manner, "What's wrong? Don't you want to spend time with me?“ When she protests, he ignores her. This is an example of:
A. mystification.
B. a negative feedback loop. ‘
C. dysfunctional communication.
D. double-bind communication.
A

In this situation, the husband has given inconsistent (“double”) messages to his wife.
a. Incorrect Mystification is similar to a double—bind communication, but its purpose is to mask an underlying conflict.
b. Incorrect A negative feedback loop is a corrective mechanism that allows a family system to reduce deviation and maintain stability.
c. Incorrect Although this communication is dysfunctional, this answer is too general, and answer d is a better response.
D. double-bind communication - CORRECT When a person is given inconsistent messages and is unable to comment on them, this is referred to as a “double-bind.”
(Clinical Psychology Question)

How well did you know this?
1
Not at all
2
3
4
5
Perfectly
28
Q

The Racial/Cultural Identity Development Model proposed by Atkinson, Morten and Sue (1993) is based on the assumption that the stages of identity development reflect changes in:
A. attitudes toward members of minority and dominant cultures.
B. interactions with members of minority and dominant cultures.
C. level of acculturation.
D. stage of ego identity development.

A

Each stage of the Racial/Cultural Identity Development Model is characterized by a different set of attitudes toward members of one’s own minority group, members of other minority groups, and members of the dominant group.
A. attitudes toward members of minority and dominant cultures - CORRECT Each stage in this model reflects different attitudes toward members of different racial/cultural groups.
(Clinical Psychology Question)

How well did you know this?
1
Not at all
2
3
4
5
Perfectly
29
Q

Troiden’s (1988) Gay/Lesbian (Homosexual) Identity Development Model predicts that individuals in the sensitization stage:
A. realize that they are homosexual.
B. begin to “come out” to friends in the homosexual community.
C. feel different or alienated from their same—sex peers.
D. believe they are homosexual but try to “pass” as heterosexual.

A

Troiden’s (1988) Gay/Lesbian (Homosexual) Identity Development Model distinguishes between four age-related stages: sensitization, self-recognition, identity assumption, and commitment.
a. Incorrect This is characteristic of the self—recognition stage which occurs at the onset of puberty.
b. Incorrect This is characteristic of the identity assumption stage.
C. feel different or alienated from their same—sex peers - CORRECT The sensitization stage usually occurs in middle childhood. During this stage, the individual feels different from and experiences less opposite—sex
interests than his/her peers and begins to have homosexual feelings without understanding the implications of those feelings for self—identity.
c. Incorrect This is not characteristic of the sensitization stage.
(Clinical Psychology Question)

How well did you know this?
1
Not at all
2
3
4
5
Perfectly
30
Q

The Health Belief Model predicts that, to modify an individual’s health—related behaviors, an intervention should focus on:
A. the rewards and punishments associated with health—related behaviors.
B. the severity, duration, and stage of the illness.
C. the individual’s locus of control.
D. the individual’s knowledge, motivation, and self—efficacy beliefs.

A

As its name implies, the Health Belief Model focuses on the impact of an individual’s beliefs on his/her health—related behaviors.
a. Incorrect The Health Belief Model emphasizes the individual’s perceptions and beliefs rather than on external conditions [e.g., rewards and punishments] that
influence health—related behaviors.
b. Incorrect This is not a focus of interventions based on the Health Belief Model, which usually emphasize prevention of a disorder rather than the treatment of an
existing disorder.
c. Incorrect The Health Locus of Control Model emphasizes the role of locus of control beliefs.
D. the individual’s knowledge, motivation, and self—efficacy beliefs - CORRECT According to the Health Belief Model, a person’s willingness to take appropriate health—related actions is related to the person’s beliefs about (1)
his/her susceptibility to the illness; (2) the consequences of the illness; and (3) the benefits of and barriers to taking appropriate action. Interventions based on the Health Belief Model focus on the individual’s knowledge about the illness and methods for avoiding it; motivation to take action; and self—efficacy beliefs.
(Clinical Psychology Question)

How well did you know this?
1
Not at all
2
3
4
5
Perfectly
31
Q

Prochaska and DiClemente’s (1982) stages of change (transtheoretical) model predicts that a person in the _______stage plans to take action within the
next six months that will alter his/her problematic behavior.
A. action
B. contemplation
C. preparation
D. precontemplation

A

Prochaska and DiClemente’s stages of change model distinguishes between six stages of change: precontemplation, contemplation, preparation, action, maintenance, and termination.
a. Incorrect A person in the action stage is currently taking actions to alter his/her behavior.
B. contemplation - CORRECT A person in the contemplation stage intends to take action in the
next six months.
c. Incorrect A person in the preparation stage is planning to take action in the near future (usually defined as in the next month).
d. Incorrect A person in the precontemplation stage is not planning to change in the foreseeable future.
(Clinical Psychology Question)

How well did you know this?
1
Not at all
2
3
4
5
Perfectly
32
Q
From the perspective of general systems theory, if an open system is faced with an unexpected threat, it will:
A. close down.
B. attempt to maintain a steady state.
C. defend against or adapt to it.
D. break down.
A

Systems theory underlies family therapy, in which a family is viewed as an open system that interacts with the environment.
C. defend against or adapt to it - CORRECT Open systems respond to input by modifying or elaborating structural elements. Under some conditions, the system will react in a way that maintains homeostasis; in other conditions, the system changes or adapts to the new input.
(Clinical Psychology Question)

How well did you know this?
1
Not at all
2
3
4
5
Perfectly
33
Q
Atkinson, Morten, and Sue’s (1993) Racial;'Cultural Identity Development Model proposes that people in which of the following stages begin to question their rejection of all aspects of the dominant culture and absolute loyalty to their own culture?
A. encounter
B. dissonance 
C. integrative awareness
D. introspection
A

The Racial/Cultural Identity Development Model distinguishes between five stages: conformity, dissonance, resistance and immersion, introspection, and integrative awareness.
a. Incorrect The encounter stage is the second stage in Cross’s (1991) Black Racial (Nigrescence) Identity Development Model.
b. Incorrect During the dissonance stage, the individual begins to question conformity to the dominant culture.
c. Incorrect In this stage, the person bases acceptance or rejection of aspects of the majority and minority cultures on an objective evaluation.
D. introspection - CORRECT During the introspection stage, the individual begins to question the unequivocal position that he/she adopted during the previous resistance-
immersion stage with regard to his/he own culture and the dominant culture.
(Clinical Psychology Question)

How well did you know this?
1
Not at all
2
3
4
5
Perfectly
34
Q
Grief, role disputes, role transitions, and interpersonal deficits are the primary targets of:
A. transactional analysis.
B. interpersonal therapy. 
C. reality therapy.
D. solution—focused therapy.
A

The four problem areas identified in this question (grief, role disputes, role transitions, and interpersonal deficits) are all related to interpersonal issues.
B. interpersonal therapy - CORRECT Practitioners of interpersonal therapy (1’l’P} focus on one or more of
the problems areas identified in this question.
(Clinical Psychology Question)

How well did you know this?
1
Not at all
2
3
4
5
Perfectly
35
Q
The information that family members continuously exchange and that helps minimize deviation and maintain the family's stability is referred to as \_\_\_\_ feedback.
A. external
B. internal
C. negative
D. positive
A

In family systems theory, the information exchange between family members can act as either positive or negative feedback.
C. negative - CORRECT Negative feedback is corrective and helps the system return to maintain its steady state. Thus, a “negative feedback loop” helps minimize
deviation and thereby maintains the family’s stability.
d. Incorrect Positive feedback increases deviation from a steady state and, therefore, produces a change in the family’s functioning.
(Clinical Psychology Question)

How well did you know this?
1
Not at all
2
3
4
5
Perfectly
36
Q

For Carl Rogers, incongruence between self and experience:
A. motivates the change required for positive growth.
B. can lead to denial or distortion of the self or experience.
C. is a frequent cause of premature termination from therapy.
D. can lead to a “failure identity.”

A

According to Rogers, for an individual to achieve self—actualization, his/her sense of self must remain unified, organized, and whole.
B. can lead to denial or distortion of the self or experience - CORRECT Rogers believed that the self can become disorganized when there is incongruence between the self and experience. This occurs when the individual experiences conditions of worth (e.g., is given attention or affection only when he/she acts in certain ways). Incongruence can lead to anxiety, which the
individual may attempt to alleviate by denying or distorting the self and/or the experience.
(Clinical Psychology Question)

How well did you know this?
1
Not at all
2
3
4
5
Perfectly
37
Q

Which of the following is incorrectly matched?
A. Client-centered therapy: External events must be a primary focus of therapy because they are responsible for deterring people from normal healthy growth.
B. Reality therapy: Environment and person must both be considered because over—concentration on one can limit the effectiveness of therapy
C. Behavioral therapy: Environmental forces are the primary determinant of human development.
D. Psychodynamic therapy: Interactions between the person and the environment are played out largely in the person’s unconscious.

A

This question is a little tricky, and you may have had to use the process of elimination to identify the correct response.
A. Client-centered therapy: External events must be a primary focus of therapy because they are responsible for deterring people from normal healthy growth - CORRECT Although incongruence between self and experience (the
environment) is an important concept in client—centered therapy, little attention is paid to actual external events in therapy. Instead, the focus is on the individual’s perception of those events.
b. Incorrect Reality therapy emphasizes person-environment transactions, and views the difficulties that people experience as reflecting a tendency to deny the
reality of external events.
c. Incorrect This is an accurate description of the behavioral view.
d. Incorrect This accurately describes the psychodynamic approach.
(Clinical Psychology Question)

How well did you know this?
1
Not at all
2
3
4
5
Perfectly
38
Q
A "teleological" approach is most associated with:
A. Skinner.
B. Bowen.
C. Adler.
D. Freud.
A

A teleological approach views behavior and personality as being “pulled” by a subjective future rather than being “pushed” by an objective past (e.g., by heredity or environmental events).
a. Incorrect Skinner viewed behavior as the result of the past consequences of the behavior.
b. Incorrect Bowen is a family therapist who regards severe mental disorders (e.g., Schizophrenia) as the result of a multigenerational transmission process. Therefore, his approach would not be described as teleological.
C. Adler - CORRECT A distinguishing characteristic of Adler’s approach is its teleological explanation for behavior. Adler believed that behavior is determined by future goals.
d. Incorrect Freud viewed current behavior as the result of innate factors and past experiences.
(Clinical Psychology Question)

How well did you know this?
1
Not at all
2
3
4
5
Perfectly
39
Q

A family therapist using the structural approach of Salvador Minuchin would most likely:
A. clarify boundaries between family members in order to reduce enmeshment.
B. work initially with the most differentiated family member.
C. use a multiple-therapist team to prevent any one therapist from becoming “triangulated” into the family system.
D. issue specific “directives” designed to counteract dysfunctional processes.

A
As its name implies, structural family therapy focuses on altering the family's structure in order to change the behavior patterns of family members.
A. clarify boundaries between family members in order to reduce enmeshment - CORRECT Even if you are unfamiliar with Minuchin, you may have been able
to guess that structural therapy would be concerned with boundaries. Structural family therapists view family dysfunction as being related to boundaries that are
too diffuse (enmeshed) or too rigid (disengaged).
b. Incorrect This is more characteristic of Bowen's approach to family therapy.
c. Incorrect Minuchin's approach does not involve the use of multiple-therapist teams.
d. Incorrect The issuance of directives is more characteristic of strategic family therapy than of structural family therapy.
(Clinical Psychology Question)
How well did you know this?
1
Not at all
2
3
4
5
Perfectly
40
Q

Which of the following individuals is incorrectly matched with his view of personality or behavior?
A. Skinner: Personality represents a complete (or incomplete) mastery of the environment over innate drives and tendencies.
B. Rogers: Human behavior and personality depend more on subjective reality than on external reality.
C. Freud: “The child is the father of the man.“
D. Adler: Humans are motivated largely by innate social urges.

A

Identifying the correct answer to this question requires a careful reading of the responses.
A. Skinner: Personality represents a complete (or incomplete) mastery of the environment over innate drives and tendencies - CORRECT This description of Skinner’s theory is not quite accurate: Although Skinner acknowledged the existence of innate drives, he did not place as much emphasis on their role in personality development as it implies. His focus was, instead, on the external (environmental) determinants of behavior.
b. Incorrect Rogers believed it was possible to understand someone only by seeing reality as that person sees it.
c. Incorrect Freud believed that personality is pretty well established by age 5 or 6 - i.e., that adult personality is established in childhood.
d. Incorrect Adler stressed the role of social tendencies in personality development.
(Clinical Psychology Question)

How well did you know this?
1
Not at all
2
3
4
5
Perfectly
41
Q

Practitioners of Gestalt psychotherapy consider a client’s desire to talk about his/her past as:
A. a useful strategy for helping the client understand the present.
B. a way for the client to avoid coming to terms with the present.
C. the “second layer” of therapy.
D. a way for achieving “closure” near the end of therapy.

A

Knowing that Gestaltians focus on the here-and-now in therapy would have helped you identify the correct response to this question.
B. a way for the client to avoid coming to terms with the present - CORRECT Gestalt therapists do not deny the significance of the past. However, in therapy, they encourage clients to stay in the present even when
discussing past events or feelings since focusing on the past is viewed as a way to avoid the present.
(Clinical Psychology Question)

How well did you know this?
1
Not at all
2
3
4
5
Perfectly
42
Q

For Gestalt therapists, maladaptive behavior:
A. is a “developmental anomaly.”
B. reflects the adoption of “mistaken beliefs.”
C. results from the adoption of an unhealthy life script
D. reflects a “growth disorder.“

A

Like other humanistic therapists, Gestalt therapists view the individual as having an innate tendency for positive growth.
a. Incorrect This does not accurately describe the Gestaltian view of maladaptive behavior.
b. Incorrect This sounds more like cognitive views of maladaptive behavior.
c. Incorrect This is consistent with the assumptions of transactional analysis.
D. reflects a “growth disorder.“ - CORRECT For Gestaltians, neurotic behavior reflects interference with natural growth and a resulting lack of integration.
(Clinical Psychology Question)

How well did you know this?
1
Not at all
2
3
4
5
Perfectly
43
Q
Research on Helms’s White Racial Identity Development Model suggests that a White therapist will usually be most successful when working with a client from an ethnic/racial minority group when the therapist is in which stage?
A. reintegration
B. immersion—emersion
C. autonomy
D. integrative awareness
A

Helms’s White Racial Identity Development Model distinguishes between six identity statuses (stages): contact, disintegration, reintegration, pseudo—independence, immersion—emersion, and autonomy.
C. autonomy - CORRECT Perhaps not surprisingly, a White therapist is likely to be most effective when working with a client from an ethnic/racial minority group when the therapist is in the final stage of White identity development - i.e., the autonomy stage. A person in this stage has internalized a positive (non racist) White identity
that includes appreciation of and respect for racial/cultural similarities and differences.
(Clinical Psychology Question)

How well did you know this?
1
Not at all
2
3
4
5
Perfectly
44
Q

From the perspective of feminist therapy, therapist self—disclosure is:
A. contraindicated because it puts the client in a passive role.
B. necessary during the early stages of therapy to encourage the client’s participation.
C. a means of fostering a special bond between the client and the therapist
D. useful for promoting an egalitarian relationship between the therapist and client.

A

A key characteristic of feminist therapy is its emphasis on an egalitarian relationship.
a. Incorrect Promoting a passive role in a client is discouraged in Feminist therapy, but therapist self-disclosure (if done appropriately) is not considered a contributor to passivity.
b. Incorrect This describes one use of self-disclosure, but this response doesn’t best describe the role of self-disclosure in feminist therapy.
c. Incorrect Feminist therapists generally discourage developing a “special bond” because doing so can foster the client’s dependence on the therapist.
D. useful for promoting an egalitarian relationship between the therapist and client - CORRECT Feminist therapists view sharing their own life experiences with clients as a way of promoting an egalitarian relationship.
(Clinical Psychology Question)

How well did you know this?
1
Not at all
2
3
4
5
Perfectly
45
Q
A Milan systemic family therapist will emphasize the use of which of the following in therapy?
A. directives
B. "holons"
C. maneuvers, coalitions, and games ‘
D. circular questions
A

A primary goal of Milan systemic family therapy is to “help family members see their choices and to assist them in exercising their prerogative of choosing” {Gelcer, McCabe, & Smith—Resnick, 1990, p. 22).
a. Incorrect Systemic family therapists view therapy as facilitative and, therefore, avoid the use of directives.
b. Incorrect As used by Minuchin, the term “holon” refers to the concept that a phenomenon is simultaneously a whole and its parts.
c. Incorrect Maneuvers, coalitions, and games are avoided by systemic therapists because they are more directive than facilitative.
D. circular questions - CORRECT For systemic therapists, questions are not only means for obtaining information but also act as a therapeutic intervention. Circular questions help define and clarify confused ideas and behaviors and introduce new information to family members.
(Clinical Psychology Question)

How well did you know this?
1
Not at all
2
3
4
5
Perfectly
46
Q
The notion that there are three major life tasks - friendship, occupation, and love — is MOST consistent with the philosophy of:
A. Per|s
B. Berne
C. Rogers
D. Adler
A

The three tasks listed in the question (friendship, occupation, and love) all involve social interactions. Of the individuals listed, one is most associated with an emphasis on social factors (i.e., social interest).
D. Adler - CORRECT Adler believed that people are motivated primarily by an innate social interest and that the goal in life is to act in ways that fulfill social responsibilities.
(Clinical Psychology Question)

How well did you know this?
1
Not at all
2
3
4
5
Perfectly
47
Q

A feminist therapist is least likely to view which of the following as an important aspect or goal of therapy:
A. identifying the sociopolitical and interpersonal forces that underlie a woman’s symptoms.
B. reconstructing the therapeutic process so that the traditional pattern of dependence is not recreated in the therapeutic relationship.
C. using the therapeutic relationship as an opportunity to model appropriate behavior.
D. identifying and integrating the masculine and feminine aspects of the woman’s personality.

A

Feminist therapists believe that a primary goal of feminist therapy is helping the client understand and appropriately respond to forces in the environment that impact the client’s life.
a. Incorrect Feminist therapists emphasize the role of sociopolitical Factors in maladaptive behavior and, therefore, a goal of therapy is to help the client
understand that the “personal is political.”
b. Incorrect Feminist therapists regard the relationship between therapist and client as a relationship between equals and use it as a model of equality to help Female clients overcome barriers created by traditional roles and relationships.
c. Incorrect Feminist therapists view the therapeutic relationship as an opportunity to model egalitarianism and other adaptive behaviors.
D. identifying and integrating the masculine and feminine aspects of the woman’s personality - CORRECT Feminists therapist are more likely to avoid labeling aspects of the personality as “masculine” or “feminine.”
(Clinical Psychology Question)

How well did you know this?
1
Not at all
2
3
4
5
Perfectly
48
Q
Helms’s (1995) White Racial Identity Development Model distinguishes between six statuses. The first status is:
A. internalization.
B. integrative awareness.
C. reintegration.
D. contact.
A

He|ms’s six identity statuses are contact, disintegration, reintegration, pseudo-independence, immersion/emersion, and autonomy.
D. contact - CORRECT Contact is the first status in Helms’s model. It is characterized by a lack of awareness of racial identity and often by racist attitudes and beliefs.
(Clinical Psychology Question)

How well did you know this?
1
Not at all
2
3
4
5
Perfectly
49
Q

For a Gestalt therapist, a primary goal of treatment is to help the client:
A. integrate the present with his/her past and future.
B. integrate the various aspects of the self.
C. develop a “success identity.”
D. develop a “healthy style of life.”

A

For the exam, you want to know that, in Gestalt therapy, the primary goals are increased awareness and integration of all aspects of the self
a Incorrect In Gestalt therapy, the focus is on the present. In fact, its founder, Fritz Perls, argued that “nothing exists but the now.”
B. integrate the various aspects of the self - CORRECT A primary goal of gestalt therapy is to integrate all aspects of the self; i.e., one’s feelings, thoughts, and actions.
c Incorrect This is the goal of reality therapy.
d Incorrect This sounds more like Adlerian therapy.
(Clinical Psychology Question)

How well did you know this?
1
Not at all
2
3
4
5
Perfectly
50
Q

Family therapy is probably contraindicated in which of the following situations?
A. The presenting problem involves long-term marital conflict.
B. Family members attribute their problems to one of the members.
C. One of the family members has severe depression.
D. One of the family members has anorexia or bulimia.

A

Family therapy is generally contraindicated when one family member’s disturbance, while affecting the family, is basically unrelated to family processes or structure or is so disruptive that it would interfere with the success of therapy.
a. Incorrect Family therapy would be appropriate in this situation.
b. Incorrect Family therapy would be appropriate in this situation.
C. One of the family members has severe depression - CORRECT Depression has been linked to a variety of factors including heredity, biochemical abnormalities, learned helplessness, hostility or aggression turned
inward, and illogical schemata. It has not, however, been consistently linked with any particular family factor, and family therapy, therefore, would not be particularly useful as a treatment for this disorder.
d. Incorrect Family therapy would be appropriate in this situation since eating disorders (especially anorexia and bulimia) have been linked to family factors.
(Clinical Psychology Question)

How well did you know this?
1
Not at all
2
3
4
5
Perfectly
51
Q
For practitioners of humanistic psychotherapy, psychopathology is the result of:
A. blocked potential.
B. “dis-integration."
C. unresolved conflicts.
D. severe trauma.
A

Therapists classified as humanists share a belief in the inherent capacity for humans to grow toward the achievement of their potential (i.e., to achieve self-actualization).
A. blocked potential. - CORRECT Neurosis and psychosis are generally viewed as the result of interference with the natural potential for growth and self-actualization.
(Clinical Psychology Question)

How well did you know this?
1
Not at all
2
3
4
5
Perfectly
52
Q

For practitioners of Minuchin’s structural family therapy, a symptom is:
A. both the result and cause of dysfunctional communication patterns.
B. the result of a family projection process.
C. a reflection of an “impasse.”
D. a maladaptive reaction to stress.

A

A primary goal of structural family therapy is to restructure the family so that it is better able to adapt to maturational and situational stressors.
a. Incorrect This sounds more like Haley’s strategic family therapy.
b. Incorrect This sounds like Bowen’s extended family systems therapy.
c. Incorrect This isn’t consistent with structural family therapy.
D. a maladaptive reaction to stress - CORRECT For structural family therapists, developmental and environmental changes produce stress which a family can respond to in either a healthy or unhealthy way.
(Clinical Psychology Question)

How well did you know this?
1
Not at all
2
3
4
5
Perfectly
53
Q

From a psychoanalytic perspective, anxiety is due to:
A. the inability to “construe” an event.
B. a threat to one’s unified self-concept.
C. a feeling of being isolated and helpless in a hostile world
D. the inability to deal with external threats.

A

Freud’s personality theory emphasizes conflicts between the internal instincts and the demands of reality.
a. Incorrect This best describes Kelly’s definition of anxiety.
b. Incorrect This is Roger’s view of anxiety.
c. Incorrect Horney referred to this as “basic anxiety.”
D. the inability to deal with external threats - CORRECT Of the answers given, this is the only one that implies a conflict between internal and external events and, therefore, is the best description of Freud’s conceptualization of anxiety.
(Clinical Psychology Question)

How well did you know this?
1
Not at all
2
3
4
5
Perfectly
54
Q

The primary difference between feminist and nonsexist therapy is that, in contrast to nonsexist therapy, feminist therapy:
A. promotes an egalitarian therapist—client relationship.
B. emphasizes political values and social change.
C. rejects sex-role stereotypes.
D. recognizes the impact of sexism on mental health.

A

Feminist therapy and non-sexist therapy share a number of characteristics but differ in terms of their emphasis on sociopolitical contributors to mental health problems.
a. Incorrect This is characteristic of both types of therapy.
B. emphasizes political values and social change - CORRECT Feminist therapy is the more political of the two types of therapy and is based on the assumption that the “personal is political.”
c. Incorrect Both types of therapy reject traditional se)

How well did you know this?
1
Not at all
2
3
4
5
Perfectly
55
Q

In therapy, an object relations family therapist would be most interested in which of the following?
A. distinguishing between positive and negative connotations
B. integrating attitudes, feelings, and behaviors.
C. interpreting focused and contextual transferences.
D. expanding awareness and se|f-responsibility.

A

Object relations family therapy is a psychodynamic approach and relies on many of the techniques associated with psychodynamic psychotherapy
C. interpreting focused and contextual transferences - CORRECT D. E. Scharff and J. S. Scharff note that addressing transferences and countertransferences is a primary focus of object relations family therapy (Object Relations Family Therapy, Northvale, NJ, Jason Aronson, 1991). These authors also distinguish between two types of transference - focused and contextual - and state that the latter is particularly important in family therapy.
(Clinical Psychology Question)

How well did you know this?
1
Not at all
2
3
4
5
Perfectly
56
Q
Replacing a “failure identity” with a “success identity" is a goal of treatment for practitioners of \_\_\_\_\_\_\_ therapy.
A. Gestalt
B. Reality
C. So|ution—focused
D. Ad|erian
A

Of the therapies listed, only one distinguishes between success and failure identities.
B. Reality - CORRECT Glasser, the founder of reality therapy, viewed identity as a basic psychological need and distinguished between success and failure identities. A person develops a success identity when the person fulfills his/her other needs in a responsible way.
(Clinical Psychology Question)

How well did you know this?
1
Not at all
2
3
4
5
Perfectly
57
Q

When calculating an “effect size,” you would:
A. divide the difference between the means of the experimental and control groups by the “grand mean.”
B. divide the mean of the experimental group by its standard deviation and the mean of the control group by its standard deviation and sum the results.
C. add the mean difference between the experimental and control groups for each study and divide the result by the total number of studies.
D. divide the difference between the means of the experimental and control groups by the standard deviation of the control group.

A

The term “effect size” is associated with meta—analysis. Although there are several ways to calculate an effect size, only one of the responses accurately describes one of these methods.
D. divide the difference between the means of the experimental and control groups by the standard deviation of the control group - CORRECT The most commonly used effect size is a type of standard score, which means that it reports the effect of an intervention in terms of standard
deviation units.
(Clinical Psychology Question)

How well did you know this?
1
Not at all
2
3
4
5
Perfectly
58
Q
The first stage in Cross’s (1991) Black Racial Identity Development Model is:
A. conformity.
B. incorporation.
C. contact.
D. pre-encounter.
A

Cross’s Black Racial Identity Development Model distinguishes between 5 stages: pre-encounter, encounter, immersion-emersion, internalization, and internalization-commitment.
D. pre-encounter - CORRECT Cross’s model is based on the premise that African American identity development is directly linked to racial oppression. During the initial
pre-encounter stage, race has low salience.
(Clinical Psychology Question)

How well did you know this?
1
Not at all
2
3
4
5
Perfectly
59
Q

According to Atkinson, Morten, and Sue’s (1993) Racial/Cultural Identity Development Model, an African-American therapy client in the conformity stage will most likely:
A. prefer a white therapist.
B. be unwilling to self-disclose to a white therapist.
C. say his/her problems are directly related to racial oppression.
D. prefer a therapist who has expertise in working with African Americans.

A

The Racial/Cultural Identity Development Model consists of five stages: conformity, dissonance, resistance and immersion, introspection, and integrative awareness.
A. prefer a white therapist - CORRECT During the initial conformity stage, African Americans have positive
attitudes toward Whites and negative attitudes toward their own group; and, in therapy, they are likely to prefer a White therapist.
b. Incorrect This is characteristic of African American therapy clients in the dissonance, resistance and immersion, or introspection stage.
c. Incorrect This is characteristic of clients in the dissonance or resistance and immersion stage.
cl. Incorrect This is true about African American clients in the introspection or integrative awareness stage.
(Clinical Psychology Question)

How well did you know this?
1
Not at all
2
3
4
5
Perfectly
60
Q

Client-centered therapists are most likely to interpret a client’s heart palpitations, hyperventilation, tension headaches, and nausea as:
A. the result of unresolved intrapsychic conflicts.
B. manifestations of denied threats to the self-concept.
C. a response to overwhelming environmental stress.
D. the consequence of being unable to fulfill one”s own needs.

A

For practitioners of client-centered therapy, personality and behavioral problems arise when an individual’s natural tendency toward growth and actualization is disrupted by incongruence between the self and
experience.
B. manifestations of denied threats to the self-concept - CORRECT Incongruence results when the evaluations made by others are inconsistent with one’s self-concept. One way in which this incongruence is dealt with is to deny the external experience by saying it doesn’t exist or by distorting it. Rogers believed that such denial doesn’t work because it leads to anxiety which, in turn, may produce visceral symptoms.
(Clinical Psychology Question)

How well did you know this?
1
Not at all
2
3
4
5
Perfectly
61
Q

A family therapist would use the technique known as “prescribing the symptom” in order to:
A. undermine the family’s resistance to change.
B. reduce tension between family members.
C. change the meaning of a situation by changing the way it is perceived
D. educate family members about the benefits of change.

A

Prescribing the symptom is a paradoxical technique that involves instructing family members to engage in the problematic behavior, often in an exaggerated form.
A. undermine the family’s resistance to change - CORRECT The purpose of prescribing the symptom is to undermine the Family’s resistance to changing a behavior by requesting that Family members engage in the behavior. Ideally, this will cause them to rebel and engage, instead, in a more desirable behavior.
b. Incorrect This is not an accurate description of the purpose of prescribing the symptom.
c. Incorrect This is a better description of the technique known as relabeling.
d. Incorrect This is not an accurate description of the purpose of prescribing the symptom.
(Clinical Psychology Question)

How well did you know this?
1
Not at all
2
3
4
5
Perfectly
62
Q
A therapist instructs a client who suffers from insomnia to polish his hardwood floors for at least two hours whenever he wakes up during the night. Apparently this therapist is familiar with the work of:
A. Luigi Boscolo.
B. Milton Erickson.
C. Salvador Minuchin.
D. Marquis de Sade.
A

The task described in the question is an example of an “ordeal.”
a. Incorrect Luigi Boscolo is affiliated with the Milan systemic school of family therapy, which is not associated with the use of ordeals.
B. Milton Erickson - CORRECT For the exam, you should have paradoxical techniques and ordeals associated with Milton Erickson and Jay Haley (who was strongly influenced by Erickson).
c. Incorrect The use of ordeals is more associated with Erickson and Haley than with Minuchin.
(Clinical Psychology Question)

How well did you know this?
1
Not at all
2
3
4
5
Perfectly
63
Q

Criterion refers to the degree to which a supervisor’s
evaluation of an employee’s job performance is biased by the supervisor’s knowledge of the employee’s score on the selection test used to hire the employee.
A. deficiency
B. contamination
C. relevance
D. partiality

A

In this situation, the supervisor’s knowledge of the employee’s selection test score is “contaminating” his/her rating of the employee on a criterion (job performance) measure.
a. Incorrect Criterion deficiency refers to the degree to which a criterion measure does not measure all aspects of the “ultimate” criterion. The information provided in the question does not indicate that the measure of job
performance is deficient in terms of being a thorough measure of performance. It only indicates that the measure is susceptible to criterion contamination.
B. contamination - CORRECT The situation described in this question illustrates criterion contamination.
c. Incorrect Criterion relevance refers to the extent to which a criterion measure evaluates the “ultimate” criterion.
d. Incorrect Criterion partiality is a “made up” term.
(I-O Psychology)

How well did you know this?
1
Not at all
2
3
4
5
Perfectly
64
Q

Use of the Taylor-Russell tables would indicate that the decision-making accuracy of a selection test that has a low to moderate validity coefficient is greatest when:
A. the selection ratio is .90 and the base rate is .20.
B. the selection ratio is .90 and the base rate is .50.
C. the selection ratio is .10 and the base rate is .20.
D. the selection ratio is .10 and the base rate is .50.

A

The Taylor-Russell Tables provide information on a test’s decision- making accuracy for various combinations of base rates, selection ratios, and validity coefficients.
D. the selection ratio is .10 and the base rate is .50 - CORRECT A test with a low or moderate validity coefficient can improve decision-making accuracy when the selection ratio is low (e.g., .10) and the base
rate is moderate (near .50).
(I-O Psychology)

How well did you know this?
1
Not at all
2
3
4
5
Perfectly
65
Q

The function of a formative evaluation is best described as obtaining the information needed to:
A. guide program development so as to produce the best version of the program as possible.
B. determine whether the program has produced outcomes commensurate with the program’s goals.
C. initially formulate the goals and objectives of a new program.
D. identify and define the causal links between observed relationships among a set of variables.

A

A formative evaluation is what it sounds like: It is conducted while a program is being “formed.”
A. guide program development so as to produce the best version of the program as possible - CORRECT The purpose of a formative evaluation is to obtain the information needed to modify a program as it is being developed in order to determine if modifications are needed to achieve the program’s goals.
b. Incorrect This better describes a summative evaluation.
c. Incorrect This is not the function of a formative evaluation.
d. Incorrect This does not describe the purpose of a formative evaluation.
(I-O Psychology)

How well did you know this?
1
Not at all
2
3
4
5
Perfectly
66
Q

Research on job satisfaction suggests that it:
A. is a relatively stable trait and is minimally affected by job changes.
B. is relatively stable within the same job but unstable when measured across different jobs.
C. is unstable and varies over time both within the same job and across different jobs.
D. may be stable or unstable within and across jobs depending on other characteristics of the worker.

A

The studies have found that job satisfaction is a stable characteristic
A. is a relatively stable trait and is minimally affected by job changes - CORRECT The research has found that job satisfaction is relatively stable over time and across jobs. One explanation for this is that job satisfaction is
strongly related to a tendency toward positive or negative affect, which is a stable characteristic: In other words, people with negative affect tend to be dissatisfied with work, while those with positive affect tend to be satisfied.
(I-O Psychology)

How well did you know this?
1
Not at all
2
3
4
5
Perfectly
67
Q

In a study designed to evaluate the effects of rewards on productivity, employees in the experimental department receive their regular salary plus small bonuses that are contingent on their productivity, while employees in
the control department receive their regular salary but no special incentives. The productivity levels of each department and the bonuses received by the
experimental department are posted on a bulletin board in the company’s cafeteria. As expected, employees in the experimental department display an increase in productivity. However, an unexpected result is that employees in the control department do not maintain their performance at usual levels but, instead, exhibit a decrease in productivity. This latter result confirms the
predictions of which of the following?
A. ERG theory
B. equity theory
C. the John Henry effect
D. social comparison theory

A

Of the theories listed, equity theory best explains the outcomes of the study described in this question.
a. Incorrect Alderfer’s ERG theory does not apply to the situation described in this question as well as equity theory does.
B. equity theory - CORRECT Equity theory is used to predict how workers will respond in situations they perceive as equitable or inequitable. According to this theory, employees who perceive their situation as inequitable will attempt to create equity by decreasing their inputs (e.g., effort and performance) or by increasing
their outcomes (which employees in this situation cannot do).
c. Incorrect The John Henry effect predicts that employees in the control department will try to outperform employees in the experimental department.
d. Incorrect Social comparison theory doesn’t apply to this situation. It predicts that we learn about our own abilities (or other characteristics} by comparing them to those of other people.
(I-O Psychology)

How well did you know this?
1
Not at all
2
3
4
5
Perfectly
68
Q

Research comparing day, swing, and night shifts has found that:
A. the night shift is associated with the highest accident rates and has the most detrimental impact on social relations.
B. the night shift is associated with the highest accident rates, but the swing shift has the most detrimental impact on social relations.
C. the swing shift is associated with the highest accident rates, but the night shift has the most detrimental impact on social relations.
D. the swing shift is associated with the highest accident rates and has the most detrimental impact on social relations.

A

The studies indicate that the swing and night shifts are both associated with negative outcomes.
B. the night shift is associated with the highest accident rates, but the swing shift has the most detrimental impact on social relations - CORRECT Studies comparing the impact of the various work shifts have not produced entirely consistent results. However, it does appear that the night shift is associated with the highest accident rates and lowest performance, apparently as the result of sleep deprivation. In contrast, the swing shift has the most negative impact on family and other social relationships.
(I-O Psychology)

How well did you know this?
1
Not at all
2
3
4
5
Perfectly
69
Q

In the context of expectancy (VIE) theory, instrumentality refers to an employee’s:
A. belief that successful performance will lead to certain rewards.
B. belief that high effort will lead to successful performance.
C. commitment to work-related goals.
D. willingness to accept responsibility.

A

Expectancy theory predicts that employee motivation is a function of three factors: expectancy, instrumentality, and valence.
a. CORRECT In the context of expectancy theory, instrumentality refers to an employee’s beliefs about the link between performance and outcomes.
b. Incorrect This describes an employee’s expectancy beliefs.
c. Incorrect This does not describe instrumentality.
d. Incorrect This does not describe instrumentality.
(I-O Psychology)

How well did you know this?
1
Not at all
2
3
4
5
Perfectly
70
Q

From the perspective of Herzberg’s two-factor theory, the relationship between pay and motivation is best described by which of the following?
A. Pay acts as a source of motivation only after safety needs have been met.
B. Pay acts as a source of motivation only when intrinsic motivation is low
C. Pay acts as a source of motivation only for jobs that cannot be “enriched.”
D. Pay does not act as a source of motivation.

A

Herzberg‘s two-factor theory proposes that all people have two basic needs — hygiene needs and motivator needs.
a. Incorrect This isn’t consistent with Herzberg’s theory.
b. Incorrect This isn’t consistent with Herzberg’s theory.
c. Incorrect Herzberg proposed that providing employees with “enriched” jobs (jobs that provide opportunities to satisfy motivator needs) is the best way to increase satisfaction and motivation. He believed that enrichment could be applied to all types of jobs, and some of its earliest applications were to unskilled and clerical jobs.
D. Pay does not act as a source of motivation - CORRECT Herzberg’s two-factor theory identifies pay as a hygiene factor. Like other hygiene factors, pay leads to dissatisfaction when it is perceived to be inadequate but produces a state of neutrality when it is adequate. To increase motivation (and satisfaction), motivator factors must be provided by the job.
(I-O Psychology)

How well did you know this?
1
Not at all
2
3
4
5
Perfectly
71
Q
According to Tuckman and Jensen (1977), group members begin to develop close relationships with one another, trust each other, and feel as though they are part of a cohesive, effective group during the stage.
A. norming
B. forming
C. performing
D. storming
A

Tuckman and Jensen (1977) distinguish between 5 stages of group development — forming, storming, norming, performing, and adjourning.
A. norming - CORRECT The norming stage is characterized by agreement, commitment, and unity. Members listen to and support each other during this stage and develop close relationships. The norming stage may become a “period of play” with the task becoming temporarily abandoned by workers.
b. Incorrect During the forming stage, group members become acquainted and attempt to establish “ground rules” for the group.
c. Incorrect During the performing stage, group members are focused on “getting the job done.”
d. Incorrect The storming stage is characterized by power struggles and conflict
(I-O Psychology)

How well did you know this?
1
Not at all
2
3
4
5
Perfectly
72
Q
A company president is concerned about the low motivation and satisfaction of her employees and, as a result, institutes a wage and bonus (financial) incentive program for all employees. Six months later, the president finds that her efforts have not increased the employees’ job motivation or satisfaction. This result is best predicted by which of the following theories?
A. ERG theory
B. expectancy theory
C. equity theory
D. two-factor theory
A

For the exam, you want to be familiar with the basic assumptions and predictions of the major theories of motivation.
a. Incorrect ERG theory predicts that people have three basic needs — existence, relatedness, and growth. From the perspective of ERG theory, opportunities for
increased compensation may help satisfy an individual’s needs and thereby lead to increased satisfaction and motivation.
b. Incorrect Expectancy theory predicts that valued outcomes lead to increased satisfaction and motivation. There is no information provided in the question
suggesting that the wage and incentive program is not valued by employees, so this is not the best response.
c. Incorrect Equity theory predicts that motivation is related to the comparisons we make between our input/outcome ratio and the input/outcome ratios of
workers performing similar jobs. There is no information given in this questions suggesting that this type of comparison is responsible for the employees’ lack of
response to the wage and incentive program.
D. two-factor theory - CORRECT According to two-factor theory, lower-level needs such as physiological and safety needs have little effect on job satisfaction or motivation
when they are fulfilled but produce dissatisfaction when they are unfulfilled. In contrast, higher-level needs have little effect on satisfaction and motivation when they are unfulfilled but increase satisfaction and motivation when they are fulfilled. Two-factor theory would, therefore, predict that financial incentives, which address lower-level needs, would not increase motivation and satisfaction
(I-O Psychology)

How well did you know this?
1
Not at all
2
3
4
5
Perfectly
73
Q

Research investigating goal-setting as a method for increasing employee motivation and performance has demonstrated that:
A. participation in goal-setting and the provision of monetary rewards increases the probability that goals will be accepted.
B. the provision of a monetary reward (but not participation in goal-setting) increases a worker’s willingness to exert a high degree of effort to achieve goals.
C. participation in goal-setting (but not monetary rewards) increases a worker’s willingness to exert a high degree of effort to achieve goals.
D. participation in goal-setting and the provision of monetary rewards are important for motivation only when goals are difficult.

A

Although goal acceptance is considered the critical factor in the effectiveness of goal-setting for improving motivation and performance, several techniques can help promote acceptance of and commitment to goals.
A. participation in goal-setting and the provision of monetary rewards increases the probability that goals will be accepted - CORRECT This answer is most consistent with the results of the research on goal-setting theory: The studies have confirmed that participation in goal-setting is not critical for goal acceptance but can help increase it and that providing monetary rewards also increases the likelihood that goals will be accepted.
(I-O Psychology)

How well did you know this?
1
Not at all
2
3
4
5
Perfectly
74
Q
In organizations, the level of affective commitment is likely to be least predictive of which of the following?
A. job satisfaction
B. productivity
C. job motivation
D. turnover
A

Two types of organizational commitment are distinguished - continuance commitment and affective commitment. Of these, affective commitment is most predictive of work-related outcomes.
B. productivity - CORRECT Of the outcomes listed in the answers, affective commitment is least predictive of productivity.
(I-O Psychology)

How well did you know this?
1
Not at all
2
3
4
5
Perfectly
75
Q
When a predictor has a validity coefficient of .40 and the base rate is 60%, the predictor will be maximally useful for decision-making when the selection ratio is:
A. 1:2.
B. 1:20
C. 2:10.
D. 10:15.
A

The usefulness of a selection test for making hiring decisions is affected by three factors: its validity coefficient, the selection ratio, and the base
rate.
B. 1:20 - CORRECT A predictor’s decision-making accuracy (incremental validity) is maximized when the base rate is close to 50% and the selection ratio is as low
as possible. A selection ratio of 1:20 (5%) means that there are twenty applicants for every one job opening, and, of the selection ratios given, it is the lowest.
(I-O Psychology)

How well did you know this?
1
Not at all
2
3
4
5
Perfectly
76
Q

If a “process consultant” is hired by a school district to help it resolve on-going problems between school board members, the administrative staff, and the teachers, the consultant is most likely to:
A. conduct formal interviews to pinpoint problem areas and then conduct training to rectify identified problems.
B. help board members, staff, and teachers identify and modify the ways in which their behaviors are inconsistent with their goals.
C. act as a mediator or arbitrator between the board members, staff, and teachers.
D. conduct a survey to identify discrepancies between how board members, staff, and teachers view their roles and the school district’s goals.

A

Not surprisingly, process consultants focus on “processes.”
a. Incorrect Process consultation involves a less structured approach to identifying and resolving consultee problems than the approach described in this
answer and focuses on helping employees identify and resolve their own work-related problems.
B. help board members, staff, and teachers identify and modify the ways in which their behaviors are inconsistent with their goals - CORRECT Process consultants focus on processes (interactions) between people and how these processes interfere with the achievement of their goals.
c. Incorrect This is not the role that process consultants adopt.
d. Incorrect Although process consultants are concerned about consultees’ goals, the use of a survey is associated more with the technique known as “survey feedback.”
(I-O Psychology)

How well did you know this?
1
Not at all
2
3
4
5
Perfectly
77
Q
Which of the following would be most useful for establishing comparable worth?
A. job evaluation
B. job specification
C. needs assessment
D. person (employee) analysis
A

Comparable worth refers to the principle that men and women should receive comparable compensation for work that requires similar skills, knowledge, responsibilities, etc.
A. job evaluation - CORRECT A job evaluation is conducted in organizations to determine the relative worth of jobs in order to set wages and salaries. Several methods of job evaluation are useful for establishing comparable worth.
b. Incorrect A job specification is a detailed description of a job that indicates what skills and knowledge are required to perform the job satisfactorily. Although a job specification might be useful for comparing job requirements for the purpose of assessing comparable worth, answer a is a better response.
c. Incorrect A needs assessment is conducted to determine training needs.
d. Incorrect A person analysis is part of a needs assessment and involves determining which employees require training and what knowledge, skills, and
abilities they need to acquire to perform their jobs effectively.
(I-O Psychology)

How well did you know this?
1
Not at all
2
3
4
5
Perfectly
78
Q
To promote recycling in a community, you attempt to establish a collaboration between educators, community leaders, and interested community members. This approach is best described as:
A. normative-reeducative.
B. rational-empirical.
C. power-coercive.
D. reciprocal-deterministic.
A

This question is referring to Chin and Benne’s (1976) distinction between three strategies for planned change: normative-reeducative, rational-empirical, and power-coercive.
A. normative-reeducative - CORRECT A key characteristic of the normative-reeducative strategy is its focus on collaboration between individuals representing different disciplines and interests in order to use norms and peer pressure to foster change.
b. Incorrect The rational-empirical strategy utilizes information as the primary change agent.
c. Incorrect The power-coercive strategy utilizes the power and authority of leaders to facilitate change.
cl. Incorrect This is not one of the three change strategies identified by Chin and Benne.
(I-O Psychology)

How well did you know this?
1
Not at all
2
3
4
5
Perfectly
79
Q

The rational-economic model of decision-making is based on the assumption that:
A. decision-makers place more emphasis on the costs of certain decisions than on other consequences of their decisions.
B. decision-makers have complete information about all alternatives and their consequences before making decisions.
C. individuals are better than groups at making decisions, especially under stressful conditions.
D. decision-makers’ knowledge about possible alternatives is always incomplete.

A

As its name implies, the rational-economic model assumes that decisions are based on a rational process.
a. Incorrect This is not an assumption of the rational-economic model.
B. decision-makers have complete information about all alternatives and their consequences before making decisions - CORRECT From the perspective of the rational-economic model, “rational” means considering all alternatives and their consequences before making a
decision.
c. Incorrect This is not an assumption of this model.
d. Incorrect A criticism of the rational-economic model is that it does not take into account that a decision-maker’s knowledge about alternatives is often incomplete.
(I-O Psychology)

How well did you know this?
1
Not at all
2
3
4
5
Perfectly
80
Q
If you were hired by a large company to develop a new training program, your first step would probably be to conduct a:
A. needs analysis.
B. job evaluation.
C. sumrnative evaluation.
D. formative evaluation.
A

For the exam, you want to be able to distinguish between needs analysis, job analysis, and job evaluation.
a. CORRECT Training program development begins with a needs analysis (also known as a needs assessment), which usually consists of three components: an
organizational analysis, a job analysis, and a person analysis.
b. Incorrect A job evaluation is conducted to determine the appropriate compensation for a job.
c. Incorrect A summative evaluation is conducted to determine the etfects of a training program or intervention after it has been developed.
d. Incorrect A Formative evaluation is conducted while a training program or intervention is being developed to determine if modifications are required in order For the program or intervention to achieve its goals.
(I-O Psychology)

How well did you know this?
1
Not at all
2
3
4
5
Perfectly
81
Q
In organizations, person-organization (P-O) fit is achieved primarily through:
A. career/vocational counseling.
B. organizational development.
C. training and evaluation.
D. selection and socialization.
A

The person-organization fit refers to the match between the employee’s values, needs, preferences, etc. and the culture of the organization. A good P-O fit has been linked to several benefits including enhanced satisfaction, motivation, and organizational commitment and reduced
stress and turnover.
D. selection and socialization - CORRECT The experts identify selection and socialization as the primary
opportunities for ensuring a good person-organization fit (e.g., Bowen, et al., 1991; Chatman, 1991): Selection helps identify individuals whose characteristics match the characteristics of the organization, while socialization (e.g., behavioral modeling by and feedback from the supervisor, participation in work-related social events) helps employees acquire the skills, knowledge, and
attitudes that are compatible with the organization’s culture.
(I-O Psychology)

How well did you know this?
1
Not at all
2
3
4
5
Perfectly
82
Q

According to Super’s career development theory, job satisfaction is directly related to:
A. the degree of similarity between the individual’s ego identity and career identity.
B. the degree of satisfaction that the person has with his/her life in general.
C. the degree to which the person is able to implement his/her self-concept at work.
D. the degree to which the job fulfills the person’s most prepotent needs.

A

Knowing that the “self-concept” is a key concept in Super’s theory would have enabled you to identify the correct answer to this question.
a. Incorrect Ego identity is a focus of Tiedeman and O’Hara’s theory of career development.
C. the degree to which the person is able to implement his/her self-concept at work - CORRECT According to Super, the self-concept consists of the values, abilities, personality traits, needs, and interests that we believe we possess. Self-concept influences career choice; and the degree of match between the self-concept and the job affects the individual’s job satisfaction, stability, and
success.
(I-O Psychology)

How well did you know this?
1
Not at all
2
3
4
5
Perfectly
83
Q

Trainability tests are:
A. paper-and-pencil tests that assess the aptitudes required for a particular job.
B. paper-and-pencil tests that assess motivation and other job-related attitudes.
C. work samples that incorporate a structured period of learning and evaluation.
D. multimodal assessment techniques used to determine what training current workers require.

A

As their name implies, trainability tests are used to determine if individuals will benefit from training.
C. work samples that incorporate a structured period of learning and evaluation - CORRECT Trainability tests are similar to work samples except they are given to people who currently do not have sufficient skills or knowledge to perform the job. They are used to determine if a job applicant is likely to benefit from training.
(I-O Psychology)

How well did you know this?
1
Not at all
2
3
4
5
Perfectly
84
Q
Five participants in a research study work individually at computer terminals to generate solutions to novel problems. Each participant in the study simultaneously reviews the suggested solutions of other participants while entering his or her own solutions. The purpose of this study is to assess the effects of technology on:
A. social inhibition.
B. groupthink. 
C. quality circles.
D. brainstorming.
A

The phrase “generate solutions to novel problems” should have helped you identify the correct answer to this question.
a. Incorrect Social inhibition occurs when an individual’s performance on a task is negatively affected by the mere presence of other individuals.
b. Incorrect Groupthink is a suspension of critical thinking that may occur in highly cohesive groups.
c. Incorrect Quality circles are small voluntary groups of employees who work together and meet regularly to discuss and resolve work-related problems.
D. brainstorming - CORRECT Brainstorming was originally developed as a way to improve group performance on difficult or novel tasks. Although the research on brainstorming has generally found that people come up with more and better solutions when working alone than when working as a group, there is some evidence that
brainstorming by computer improves its outcomes.
(I-O Psychology)

How well did you know this?
1
Not at all
2
3
4
5
Perfectly
85
Q

Expectancy theory predicts that worker motivation depends on several factors including “valence,” which refers to:
A. the employee’s beliefs about the value of rewards provided for successful performance.
B. the employee’s beliefs about the inherent (intrinsic) value of the work itself.
C. the strength of the worker’s motivation and organizational commitment.
D. the strength of the effort (versus ability) component of the worker’s motivation.

A

According to expectancy theory, motivation is a function of three components: expectancy, instrumentality, and valence.
A. the employee’s beliefs about the value of rewards provided for successful performance - CORRECT Valence refers to the value a worker places on the outcomes that
will be provided for successful performance. When these outcomes have positive valence, this increases the worker’s motivation to perform.
(I-O Psychology)

How well did you know this?
1
Not at all
2
3
4
5
Perfectly
86
Q
Of the “Big Five” personality traits, is most predictive of job performance across different types of jobs and job settings.
A. openness to experience
B. conscientiousness
C. agreeableness
D. extraversion
A

Empirical research has identified five basic personality traits (the “Big Five”): neuroticism, extraversion, openness to experience, agreeableness, and conscientiousness.
B. conscientiousness - CORRECT Conscientiousness refers to the individual’s level of responsibility dependability, persistence, self-control, and achievement motivation. Perhaps not surprisingly, of the Big Five traits, conscientiousness has been found to the best predictor of job performance across different jobs, job settings, and
criterion measures.
(I-O Psychology)

How well did you know this?
1
Not at all
2
3
4
5
Perfectly
87
Q

Research indicates that the nature of communication networks can affect worker satisfaction, group performance, and leadership effectiveness. For
instance, when tasks are complex and unstructured:
A. a centralized communication network is associated with better group performance.
B. a centralized communication network is associated with better group performance only when the group leader is authoritarian.
C. a decentralized communication network is associated with better group performance.
D. a decentralized communication network is associated with better group performance only when the group leader is authoritarian.

A

Researchers interested in work-related communication distinguish between two types of communication networks — centralized and decentralized.
C. a decentralized communication network is associated with better group performance - CORRECT The research indicates that decentralized networks, in which no one
individual has greater access to information, are best for complex, unstructured tasks that have a number of different solutions. Centralized networks, on the other hand, are better for simple, structured tasks.
(I-O Psychology)

How well did you know this?
1
Not at all
2
3
4
5
Perfectly
88
Q

According to Fiedler’s contingency model of leadership, high LPC leaders:
A. are always more effective than low LPC leaders.
B. are more effective than low LPC leaders in moderately favorable situations.
C. elicit less trust from supervisees than do low LPC leaders.
D. elicit less intrinsic motivation from supervisees than do low LPC leaders

A

Fiedler’s LPC (least preferred coworker) scale measures the extent to which a leader describes ineffective subordinates in positive terms. A high LPC leader is one who has nice things to say even about inefficient
workers.
a. Incorrect Fiedler’s theory is a “contingency” theory and predicts that the most effective leadership style depends on the nature of the situation.
B. are more effective than low LPC leaders in moderately favorable situations - CORRECT According to Fiedler, low LPC leaders are most effective in low and high favorable situations, while high LPC leaders are most effective in
moderately favorable situations.
c. Incorrect This is not predicted by Fiedler’s theory.
d. Incorrect Fielder did not directly address the issue of job motivation but, because his theory is a contingency theory, it implies that the ability of high and low LPC leaders to foster the motivation of employees would depend on the nature of the situation.
(I-O Psychology)

How well did you know this?
1
Not at all
2
3
4
5
Perfectly
89
Q

An implication of Bandura’s social cognitive theory is that, to maximize Worker motivation, a job should be designed so that:
A. rewards are matched to the employee’s prepotent needs.
B. the job maximizes team (versus individual) responsibility.
C. job roles and responsibilities match the employee’s self-concept.
D. the job maximizes opportunities for self-regulation.

A

The most recent version of Bandura’s (1997) social learning theory - social cognitive theory - emphasizes the impact of self-regulation on behavior.
a. Incorrect This sounds like Maslow’s need hierarchy theory.
b. Incorrect Bandura’s theory Focuses more on individual behavior than on team behavior.
c. Incorrect This sounds more like Super’s theory of career development.
D. the job maximizes opportunities for self-regulation - CORRECT Bandura’s social cognitive theory distinguishes between Four processes that contribute to self-regulation: goal-setting, self-observation, self-evaluation, and self-reaction. According to this theory, self-regulation (the
exercise of influence over one’s own behavior) is a primary determinant of motivation.
(I-O Psychology)

How well did you know this?
1
Not at all
2
3
4
5
Perfectly
90
Q
When performing a(n) task, group members select a solution offered by one of the group members as the group's solution.
A. compensatory
B. disjunctive
C. conjunctive
D. additive
A

A distinction is made between four types of group tasks - compensatory, disjunctive, conjunctive, and additive.
a. Incorrect In a compensatory task, the group’s performance is the average of the effort or performance of the individual members.
B. disjunctive - CORRECT For disjunctive tasks, the group selects a solution (ideally the optimal solution) from those proposed by individual group members.
c. Incorrect On conjunctive tasks, group members act in unison, which means that the group product is limited by the performance of the weakest member.
d. Incorrect When working on an additive task, the group product is the sum of the contributions of each member.
(I-O Psychology)

How well did you know this?
1
Not at all
2
3
4
5
Perfectly
91
Q
A leacler has a clear vision for the future and communicates that a vision to followers by acting as a role model, empowering followers, and inspiring followers to replace self-interest with the interests of the group.
A. charismatic
B. transactional
C. transformational 
D. bureaucratic
A

Researchers interested in leadership styles distinguish between charismatic, transactional, and transformational leaders.
a. Incorrect Charismatic leaders share several characteristics with transformational leaders but exert influence primarily through their own personal qualities (e.g., self-confidence and charisma). As defined by some
experts, charismatic leaders have a “dark side” — e.g., they may expect unquestioning loyalty, devotion, and obedience from followers.
b. Incorrect Transactional leaders foster worker motivation and productivity primarily by simplifying and structuring tasks and providing rewards for successful performance.
C. transformational - CORRECT Transformational leaders motivate employees by recognizing the need for change, creating a vision that guides change, and transmitting that
vision to employees. In contrast to charismatic leaders, they empower followers and activate their higher-order needs.
d. Incorrect Bureaucratic leaders follow orders and directives and adhere strictly to established procedures and policies.
(I-O Psychology)

How well did you know this?
1
Not at all
2
3
4
5
Perfectly
92
Q
To alleviate the "leniency bias," you would use of which of the following?
A. double-blind technique
B. weighted BIB
C. forced distribution
D. Likert scale
A

The leniency bias is a type of rater bias that involves giving ratees high ratings on each dimension of performance regardless of their actual performance.
a. Incorrect The double-blind technique is used in research and involves keeping participants and the experimenter uninformed of which group (e.g.,
experimental versus control) participants are in.
b. Incorrect A weighted BIB is used to collect biographical information about a job applicant and would not be useful for alleviating a leniency bias.
C. forced distribution - CORRECT Rater biases can be alleviated by using relative rating scales such as the forced distribution scale.
d. Incorrect Likert scales are very susceptible to the leniency bias and other rater biases.
(I-O Psychology)

How well did you know this?
1
Not at all
2
3
4
5
Perfectly
93
Q

According to House’s (1971) path-goal theory, the optimal leadership style depends primarily on:
A. certain characteristics of the worker and the work situation.
B. the person-organization fit.
C. the degree to which the leader is task- versus person-oriented.
D. the “favorableness” of the situation for the leader.

A

House’s path-goal theory distinguishes between four leadership styles: directive, supportive, participative, and achievement-oriented.
A. certain characteristics of the worker and the work situation - CORRECT According to path-goal theory, the optimal leadership style depends on certain characteristics of the worker (e.g., self-confidence and locus of control) and the work situation (e.g., degree of task challenge and ambiguity).
b Incorrect Path-goal theory identifies worker and situational characteristics as determinants of the optimal leadership style but does not Focus on the person-
organization fit.
c. Incorrect Path-goal theory does not identify the leader’s orientation as a determinant of the optimal leadership style.
d. Incorrect This sounds more like Fiedler’s contingency model of leadership.
(I-O Psychology)

How well did you know this?
1
Not at all
2
3
4
5
Perfectly
94
Q

According to Hersey and Blanchard’s situational leadership model, “employee maturity” is a function of the employee’s:
A. stage of career development.
B. personality.
C. skills and willingness to assume responsibility.
D. beliefs about the meaning and value of work.

A

Hersey and Blanchard propose that a leader is most effective when his/her behavior matches the employee’s level of maturity.
C. skills and willingness to assume responsibility - CORRECT Hersey and Blanchard distinguish between two aspects of employee maturity: Job maturity refers to the employee’s knowledge and skills, while psychological maturity refers to the employee’s self-confidence and willingness to assume responsibility.
(I-O Psychology)

How well did you know this?
1
Not at all
2
3
4
5
Perfectly
95
Q
Job satisfaction is an accurate predictor of:
A. both job performance and health.
B. job performance but not health.
C. health but not job performance.
D. neither job performance nor health.
A

For the exam, you want to be familiar with factors that do and do not correlate with job satisfaction.
C. health but not job performance - CORRECT The research has found that job satisfaction is a good predictor of longevity and mental and physical health. However, the studies have consistently found low correlations between job satisfaction and performance.
(I-O Psychology)

How well did you know this?
1
Not at all
2
3
4
5
Perfectly
96
Q

As defined by Brousseau and Driver (1994), “career concept” refers to a person’s:
A. work-related personality characteristics.
B. work-related roles and responsibilities.
C. career identity.
D. career decisions and motives.

A

Career concept is a key concept in K. R. Brousseau and M. J. Driver’s model of career development (Enhancing informed choice: A career-concepts approach to career advisement, Selections, Spring, 24-31, 1994)
D. career decisions and motives - CORRECT As defined by Brousseau and Driver, career concept refers to an
individual’s career decisions and motives, which vary in terms of three dimensions — frequency of job change; direction of change; and type of change in job content. Status on these dimensions produces four career concepts: steady state, linear, spiral, and transitory.
(I-O Psychology)

How well did you know this?
1
Not at all
2
3
4
5
Perfectly
97
Q
According to Hersey and Blanchard's situational leadership model, a leader should use a "participative" style for subordinates who have:
A. low ability and low motivation.
B. low ability and high motivation.
C. high ability and low motivation.
D. high ability and high motivation.
A

Hersey and Blanchard’s situational leadership model proposes that a leader is most effective when his/her leadership style matches subordinates’ ability and motivation (willingness to accept responsibility).
a. Incorrect A telling style is most effective for employees low in both ability and motivation.
b. Incorrect A selling style is most effective for employees who are low in ability and high in motivation.
C. high ability and low motivation - CORRECT According to Hersey and Blanchard, a participative leadership style
is most effective when subordinates have high levels of ability and low levels of motivation.
d. Incorrect A delegating style is most effective for employees who are high in both ability and motivation.
(I-O Psychology)

How well did you know this?
1
Not at all
2
3
4
5
Perfectly
98
Q

Lewin’s force field theory describes planned change in organizations as involving which of the following stages?
A. unfreezing, changing, and refreezing
B. forming, performing, and re-forming
C. identifying, planning, and acting
D. setting the stage, problem-solving, and reaching an agreernent

A

According to Lewin (1951), organizations continuously respond to forces that either promote or resist change.
A. unfreezing, changing, and refreezing - CORRECT Unfreezing, changing, and refreezing are the three stages of planned change identified by Lewin.
d. Incorrect These are the three stages of mediation.
(I-O Psychology)

How well did you know this?
1
Not at all
2
3
4
5
Perfectly
99
Q
Female applicants consistently obtain lower scores on a selection test than do male applicants. However, when they are hired, females perform as well on the job as do males. This situation illustrates which of the following?
A. differential validity 
B. differential selection
C. adverse impact
D. unfairness
A

This is a difficult question, but you may have been able to identify the correct answer through the process of elimination.
a. Incorrect Differential validity occurs when a predictor has different validity coefficients for members of different groups. No information is given about the validity coefficients for males and females, so you cannot conclude that the selection test has differential validity.
b. Incorrect Differential selection is a threat to the internal validity of a research study and is not relevant to this situation.
c. Incorrect The situation described in the question could lead to adverse impact if females are not hired because of their low scores on the predictor. However, the question does not state that the hiring rate for females is lower than the rate for males, so this is not the best response.
d. CORRECT The situation described in the question illustrates the EEOC’s definition of unfairness: “When members of one race, sex, or ethnic group
characteristically obtain lower scores on a selection procedure than members of another group, and the differences in scores are not reflected in differences in a
measure of job performance, use of the selection procedure may unfairly deny opportunities to members of the group that obtains the lower scores” (Uniform
Guidelines on Employee Selection Procedures, Section 14.B.8.a).
(I-O Psychology)

How well did you know this?
1
Not at all
2
3
4
5
Perfectly
100
Q
A|derfer's (1972) ERG theory is best viewed as a modification of and alternative to:
A. Mas|ow’s need hierarchy theory.
B. Herzberg's two-factor theory.
C. Vroom’s expectancy theory.
D. Bandura's social cognitive theory.
A

ERG theory proposes that we have three basic needs: existence, relatedness, and growth.
A. Mas|ow’s need hierarchy theory - CORRECT Alderfer modified Mas|ow’s need hierarchy theory so that it better
corresponds to research showing that humans have three (rather than five) distinct needs and that more than one need can act as a motivator at any point in time.
(I-O Psychology)

How well did you know this?
1
Not at all
2
3
4
5
Perfectly
101
Q

According to Krumboltz (1996), a person’s career path is determined primarily by:
A. basic needs and drives.
B. ego identity development.
C. perceptions of the match between his/her abilities and the requirements of the job.
D. learning that occurs as the result of interactions with other people.

A

Krumboltz proposes that career decisions are based on what the individual has learned.
a. Incorrect Roe (1972) Focuses on the impact of basic needs on career development.
b. Incorrect Ego identity development is a focus of Tiedeman and O’Hara’s (1963) theory of career development.
c. Incorrect This does not accurately describe Krumboltz’s theory of career decision making.
D. learning that occurs as the result of interactions with other people - CORRECT Krumboltz’s social learning theory of career decision-making proposes that career decisions are based primarily on what we have learned,
especially from our interactions with others.
(I-O Psychology)

How well did you know this?
1
Not at all
2
3
4
5
Perfectly
102
Q

An important implication of Fiedler’s contingency theory of leadership is that:
A. the most effective leaders allow workers to participate in goal-setting.
B. managers must alter their behaviors to fit the demands of the situation.
C. an effective manager may become ineffective if the situation changes in certain ways.
D. to be effective, managers must be aware that different factors act as “satisfiers” for different employees.

A

Fiedler’s contingency theory of leadership proposes that a worker’s performance depends on the interaction between the leader’s leadership style (high LPC versus low LPC) and the favorableness of the situation, which refers to the extent to which the leader has influence, the tasks are structured, and the leader is in a position to reward employees for good performance.
a. Incorrect This is not a prediction of Fiedler’s contingency theory.
b. Incorrect Fiedler believed that a leader’s style is stable and cannot, therefore, be changed even when the situation changes.
C. an effective manager may become ineffective if the situation changes in certain ways - CORRECT Fiedler believed that different leadership styles are more effective in different situations. Specifically, he believed that low LPC leaders are most effective in very favorable and unfavorable situations, while high LPC leaders are
most effective in moderately favorable situations. Therefore, if the situation changes in terms of favorableness, an effective leader can actually become
ineffective (and vice versa).
d. Incorrect This is not a prediction of Fiedler’s contingency model of leadership. Fiedler was interested in the interaction between leadership style and the
favorableness of the situation.
(I-O Psychology)

How well did you know this?
1
Not at all
2
3
4
5
Perfectly
103
Q

Assessment centers are most commonly used to:
A. hire and promote clerical workers.
B. hire and promote managers.
C. train semi-skilled and skilled workers.
D. train salespeople.

A

Assessment centers were originally used during World War II as a means of selecting OSS agents. They are now used primarily for evaluating and predicting the performance of managerial-level employees.
B. hire and promote managers - CORRECT Assessment centers are used to evaluate managerial-level
employees for the purpose of selection, promotion, or training and involve having participants engage in a variety of exercises, many of which simulate the actual tasks a manager performs on-the-job.
(I-O Psychology)

How well did you know this?
1
Not at all
2
3
4
5
Perfectly
104
Q

A personnel director decides to raise a selection test’s cutoff scores when using the test to assist in hiring decisions. Most likely the personnel director has made this decision in order to:
A. increase the number of true positives.
B. decrease the number of false positives.
C. increase the number of false negatives.
D. decrease the number of true negatives.

A

Raising the predictor cutoff score decreases the number of true and false positives and increases the number of true and false negatives.
a. Incorrect Raising the predictor cutoff score would decrease the number of true positives.
B. decrease the number of false positives - CORRECT Most likely, a personnel director would raise the predictor cutoff in order to decrease the number of false positives since these individuals “cost the company money” (i.e., they would be hired on the basis of their test scores but
would be unsuccessful on the job).
c. Incorrect Although raising the predictor cutoff score does increase the number of false negatives, this would not be considered desirable in most situations - i.e., a personnel director would not want to increase the number of people who are rejected (not hired) on the basis of their selection test scores but who would
have been successful on the job if they had been hired.
d. Incorrect Raising the predictor cutoff increases the number of true negatives.
(I-O Psychology)

How well did you know this?
1
Not at all
2
3
4
5
Perfectly
105
Q
An organizational psychologist finds that his newly developed selection test has different validity coefficients for male and female applicants and that the difference is statistically significant. This suggests that the test has:
A. incremental validity.
B. discriminant validity. 
C. differential validity.
D. convergent validity.
A

The selection test described in this question has different levels of validity for males and females.
a. Incorrect A test’s incremental validity refers to the benefits that use of the test provides with regard to decision-ma king accuracy.
b. Incorrect A test has discriminant (divergent) validity when it has low correlations with tests that measure different traits, characteristics, or abilities. Discriminant validity provides evidence of a test’s construct validity.
C. differential validity - CORRECT A test has differential validity when its validity coefficient for one group is different from its validity coefficient for another group.
d. Incorrect A test has convergent validity when it correlates highly with tests that measure the same trait. Convergent validity provides evidence of construct
validity.
(I-O Psychology)

How well did you know this?
1
Not at all
2
3
4
5
Perfectly
106
Q

According to the Yerkes-Dodson law:
A. higher levels of arousal are associated with higher levels of learning and performance.
B. moderate levels of arousal are associated with the highest levels of learning and performance.
C. higher levels of satisfaction are associated with higher levels of job performance.
D. moderate levels of satisfaction are associated with the highest levels of job performance.

A

The Yerkes-Dodson law pertains to the relationship between arousal and learning or performance.
B. moderate levels of arousal are associated with the highest levels of learning and performance - CORRECT The Yerkes-Dodson law predicts an inverted U-shaped relationship between arousal and learning or performance, with moderate levels of arousal being associated with the highest levels of learning or performance.
(I-O Psychology)

How well did you know this?
1
Not at all
2
3
4
5
Perfectly
107
Q
The primary purpose of a(n) \_\_\_\_\_\_\_\_\_ is to obtain detailed information about job requirements in order to facilitate decisions related to compensation.
A. needs assessment
B. organizational analysis
C. job analysis 
D. job evaluation
A

Of the procedures listed in the answers, only one is conducted specifically to facilitate decisions related to compensation.
a. Incorrect A needs assessment is conducted to identify training needs.
b. Incorrect An organizational analysis is often part of a needs assessment and is conducted to identify the organization’s goals.
c. Incorrect A job analysis is conducted to identify the essential characteristics of a job. Job analysis serves several functions in an organization and may be the
first step in a job evaluation.
D. job evaluation - CORRECT A job evaluation is conducted specifically for the purpose of determining the relative worth of jobs in order to set wages and salaries.
(I-O Psychology)

How well did you know this?
1
Not at all
2
3
4
5
Perfectly
108
Q

A manager who adheres to the principles of “scientific management” would most likely agree with which of the following?
A. Employees view work “as natural as play” and actively seek autonomy and responsibility.
B. An employee”s relationships with his/her coworkers are important determinants of job satisfaction.
C. Because employees value money more than other incentives, pay is the most effective motivator.
D. An employee”s unfulfilled needs take precedence over other needs, and rewards should be determined accordingly.

A

Scientific management was developed by Frederick Taylor (1911) who applied the scientific method to the study of job productivity
a. Incorrect This describes McGregor’s Theory Y managers.
b. Incorrect This is an assumption of the human relations movement, which was an alternative to scientific management.
C. Because employees value money more than other incentives, pay is the most effective motivator - CORRECT Taylor believed that employees are motivated primarily by economic self-interest and, therefore, that money is the most effective motivator.
d. Incorrect This sounds like Maslow’s need hierarchy theory.
(I-O Psychology)

How well did you know this?
1
Not at all
2
3
4
5
Perfectly
109
Q

Research comparing heterogeneous and homogeneous work groups has found that, in general, heterogeneous groups:
A. are more creative and better at decision-making.
B. are more creative but less productive overall.
C. make better decisions but are less productive overall.
D. are more creative but worse at decision-making.

A

Most studies have found that groups are more effective when members are heterogeneous with regard to gender, personality, experience, skills, etc.
A. are more creative and better at decision-making - CORRECT Much of the research on group heterogeneity has focused on its effects on creativity and decision-making and has found a positive effect of heterogeneity on both measures of performance.
(I-O Psychology)

How well did you know this?
1
Not at all
2
3
4
5
Perfectly
110
Q

The bounded rationality (administrative) model predicts that decision-makers “satisfice” rather than “optimize” due to:
A. their tendency to make conservative decisions.
B. peer pressure.
C. limited time and resources.
D. insufficient response-contingent reinforcement.

A

Herbert Simon (1979) identified two models of individual decision-making: the rational-economic model and the bounded rational (administrative) model.
C. limited time and resources - CORRECT According to Simon, limited time and resources require decision-
makers to be less than totally rational. Specifically, rather than considering all alternatives before making a decision, they consider alternatives only until they
encounter one that meets minimum requirements of acceptability.
(I-O Psychology)

How well did you know this?
1
Not at all
2
3
4
5
Perfectly
111
Q

Which of the following will have the greatest impact on employees’ perceptions of distributive justice at work?
A. Employees receive the pay and benefits they believe they deserve
B. Employees participate in making decisions that affect their jobs.
C. Employees believe hiring and promotion decisions are fair.
D. Employees feel they are treated with respect and consideration by managers.

A

Researchers interested in organizational justice distinguish between three types — distributive, procedural, and interactional.
A. Employees receive the pay and benefits they believe they deserve - CORRECT Distributive justice refers to the fairness of the outcomes of organizational policies and procedures.
b. Incorrect This sounds more like procedural justice, which refers to the fairness of the ways in which procedures and policies are implemented.
c. Incorrect This also sounds like procedural justice.
d. Incorrect Interactional justice refers to how employees perceive the quality and content of their interactions with managers and other employees.
(I-O Psychology)

How well did you know this?
1
Not at all
2
3
4
5
Perfectly
112
Q

A transformational leader uses “framing” in order to:
A. make the organization’s goals more meaningful to employees.
B. clarify the consequences of undesirable performance.
C. reduce personal biases in decision-making.
D. adapt his/her leadership style to the characteristics of subordinates

A

Even if you’re unfamiliar with “framing,” you may have been able to identify the correct answer with your knowledge of transformational leadership.
A. make the organization’s goals more meaningful to employees - CORRECT Transformational leaders recognize the need for change and are able to communicate that need to employees. Framing is one technique these leaders use to inspire employees and involves describing the organization’s goals in a way that makes them more meaningful to employees.
c. Incorrect The term “framing” is also used to refer to a type of decision-making bias. In that context, framing occurs when the description of a problem in a positive or negative way influences the type of decision that is made. However, this question is asking about the use of framing by transformational leaders.
(I-O Psychology)

How well did you know this?
1
Not at all
2
3
4
5
Perfectly
113
Q

Frame-of-reference training is used to:
A. increase the use of effective rehearsal strategies in older adults.
B. improve the diagnostic accuracy of clinical psychologists.
C. improve a rater’s accuracy when evaluating a ratee’s performance
D. train and evaluate managerial-level employees.

A

Frame-of-reference (FOR) training is a type of rater training.
C. improve a rater’s accuracy when evaluating a ratee’s performance - CORRECT The goal of FOR training is to improve raters’ accuracy when completing performance appraisals by helping them identify and focus on the
most important job performance dimensions and distinguish behaviors that are indicative of good, average, and poor performance within each dimension.
(I-O Psychology)

How well did you know this?
1
Not at all
2
3
4
5
Perfectly
114
Q

“Critical incidents” are best described as:
A. instructional strategies that are critical for mastery.
B. behaviors that increase the risk for accidents and/or errors.
C. behaviors that clearly define successful and unsuccessful job performance.
D. compensatory factors that are used to establish wages and salaries

A

Critical incidents are descriptions of specific behaviors that are associated with good and poor job performance. The use of critical incidents in performance appraisals helps decrease rater biases.
C. behaviors that clearly define successful and unsuccessful job performance - CORRECT As noted above, critical incidents are descriptions of specific job
behaviors that define successful and unsuccessful job performance. When used in performance appraisal instruments, critical incidents are the “anchors” in a
graphic rating scale or the statements included in a checklist.
d. Incorrect The compensatory factors identified during the course of a job evaluation may include critical incidents but are not limited to them.
(I-O Psychology)

How well did you know this?
1
Not at all
2
3
4
5
Perfectly
115
Q

The “paired comparison” technique is used to:
A. evaluate an employee’s job performance.
B. make hiring and placement decisions.
C. identify an employee’s training needs.
D. determine the appropriate compensation for a job

A

The paired comparison technique is a method of performance appraisal
A. evaluate an employee’s job performance - CORRECT When using the paired comparison technique, an employee is compared to every other employee on each dimension of job performance.
(I-O Psychology)

How well did you know this?
1
Not at all
2
3
4
5
Perfectly
116
Q

The research indicates that, when Total Quality Management (TQM) fails to live up to its potential, this is most often because:
A. the team leader lacks adequate leadership skills.
B. there was too much reliance on team (versus individual) effort.
C. employees were not sufficiently involved in decision-making.
D. pay and other benefits were not adequately linked to team performance.

A

The primary goal of TQM is to continuously improve the quality of the organization’s products and services. This is accomplished by evaluating customer satisfaction, maximizing employee involvement and empowerment, and making continuous improvements in organizational
processes.
a. Incorrect Although leader ability is always important, it has not been identified as a particular problem in TQM, which emphasizes the involvement of employees in decision-making processes.
b. Incorrect Team work is considered a key feature of TQM; and the research has shown that, under the right circumstances, team work is associated with positive outcomes for the company.
C. employees were not sufficiently involved in decision-making - CORRECT Although TQM emphasizes team involvement in decision-making, too often decisions are made unilaterally by the supervisor.
d. Incorrect This hasn’t been identified as a problem by the research on TQM.
(I-O Psychology)

How well did you know this?
1
Not at all
2
3
4
5
Perfectly
117
Q
The is the least developed area of the brain at birth
A. brain stem
B. cerebral cortex
C. limbic system
D. cerebellum
A

Not surprisingly, it is the higher centers of the brain that are least developed at birth.
B. cerebral cortex - CORRECT The cerebral cortex, which is responsible for higher-level cognitive skills, complex motor behaviors, language, and spatial skills, is almost
completely undeveloped at birth.
(Lifespan Development)

How well did you know this?
1
Not at all
2
3
4
5
Perfectly
118
Q

Recent surveys suggest that, for women, which of the following characteristics are associated with a high risk for divorce?
A. marrying at an older age and coming from a single-parent home
B. marrying at a younger age and having a child within 7 months of marriage
C. having a lower level of education and marrying at an older age
D. having a higher level of education and cohabitating with the partner prior to marriage

A

Surveys have identified a number of demographic characteristics that are associated with risk for divorce. See, e.g., National Center for Health Statistics, Cohabitation, marriage, divorce, and remarriage in the
United States, Hyattsville, MD, Department of Health and Human Services, 2002.
a. Incorrect Although women who come from a single-parent home are at higher risk For divorce, marrying at an older age is associated with a lower risk.
B. marrying at a younger age and having a child within 7 months of marriage - CORRECT Marrying at a younger age and having a child before marriage or within 7 months of marriage is associated with a higher risk For divorce.
c. Incorrect Although having a lower level of education is associated with a higher risk for divorce, marrying at an older age is not.
d. Incorrect Although cohabitating with the partner prior to marriage is associated with a higher risk For divorce, a higher level of education is not.
(Lifespan Development)

How well did you know this?
1
Not at all
2
3
4
5
Perfectly
119
Q
According to Piaget, children begin to intentionally lie (make false statements) at about years of age.
A. 2
B. 5
C.  7
D. 10
A

Piaget believed that young children lie spontaneously (unintentionally) and that children do not begin to intentionally lie until age 7 or 8.
C. 7 years old - CORRECT This answer is consistent with Piaget’s conclusions. Note, however, that subsequent research suggests that children as young as age 3 or 4
intentionally lie.
(Lifespan Development)

How well did you know this?
1
Not at all
2
3
4
5
Perfectly
120
Q
Which of the following individuals coined the term “identity crisis”?
A. Freud
B. Mahler
C. Marcia
D. Erikson
A

The term “identity crisis” is now used by a number of psychologists (especially those interested in adolescent development), but it was coined by Erikson.
D. Erikson - CORRECT Erikson coined the term “identity crisis” and described it as the most important psychosocial crisis Faced by human beings. He also identified adolescence as the time when the search For an identity is most intense.
(Lifespan Development)

How well did you know this?
1
Not at all
2
3
4
5
Perfectly
121
Q
In terms of the parenting styles identified by Baurnrind and her colleagues (1991), independent self-confident adolescents are most likely to have parents who are:
A. authoritative.
B. authoritarian.
C. permissive.
D. traditional.
A

Baumrind and her colleagues distinguish between authoritarian, authoritative, permissive, and rejecting-neglecting parents.
A. authoritative - CORRECT OF the four parenting styles, the authoritative style is most likely to produce independent self-confident children and adolescents. Authoritative parents combine rational control with warmth, receptivity, and the encouragement of independence.
b. Incorrect Authoritarian parents impose absolute standards of control, stress obedience, and are willing to use Force to obtain compliance. Their children tend
to be dependent, passive, and submissive and display a limited sense of responsibility.
c. Incorrect Permissive parents provide their children with Few controls or demands and display moderate levels of warmth. Their children exhibit little self-reliance, exploratory behavior, or self-control.
d. Incorrect “Traditional” is not a parenting style identified by Baumrind.
(Lifespan Development)

How well did you know this?
1
Not at all
2
3
4
5
Perfectly
122
Q
A 16-month-old child who calls her pet cat “kitty” sees a dog for the first time and calls it “kitty.” In terms of Piaget's theory of cognitive development, the child is displaying:
A. assimilation.
B. accommodation. 
C. categorization.
D. centration.
A

Piaget distinguished between two complementary processes that underlie cognitive development: assimilation and accommodation.
A. assimilation - CORRECT Assimilation involves incorporating new knowledge into existing cognitive structures or schemes. The child in the question doesn’t know what a dog is and is incorporating it into her existing “kitty” scheme.
b. Incorrect Accommodation involves modifying an existing scheme or creating a new one.
c. Incorrect Piaget did not use the term “categorization” to describe the incorporation of new knowledge into an existing cognitive scheme.
d. Incorrect Centration is a limitation of the preoperational stage and refers to the tendency to Focus on the most salient aspect of an object.
(Lifespan Development)

How well did you know this?
1
Not at all
2
3
4
5
Perfectly
123
Q
Which of the following aspects of memory is likely to show the greatest age- related decline?
A. sensory memory
B. memory span
C. recent long-term memory 
D. remote long-term memory
A

Research has confirmed a predictable pattern in age-related declines in memory and other cognitive abilities.
C. recent long-term memory - CORRECT Recent long-term (secondary) memory is most affected by
increasing age, followed by working memory. The other aspects of memory listed in the answers are relatively unaffected by age.
(Lifespan Development)

How well did you know this?
1
Not at all
2
3
4
5
Perfectly
124
Q
Presbyopia refers to loss of:
A. far vision.
B. near vision.
C. color vision.
D. depth perception.
A

Presbyopia occurs when the lens of the eye loses its ability to focus. Many people begin to experience presbyopia in their mid-40’s.
B. near vision - CORRECT People with presbyopia have difficulty focusing on objects that are close.
(Lifespan Development)

How well did you know this?
1
Not at all
2
3
4
5
Perfectly
125
Q
Adults aged 65 and older usually report the largest number of memories for events that occurred in the 10 year period prior to being evaluated. For these individuals, the next largest number of memories is for events that occurred when they were between the ages of:
A. 15 and 25. 
B. 25 and 35.
C. 35 and 45.
D. 45 and 55.
A

The research has shown that older adults typically recall recent and remote events better than intermediate events.
a. CORRECT The term “reminiscence bump” is used to refer to the greater memory For events that occurred during adolescence and early adulthood.
(Lifespan Development)

How well did you know this?
1
Not at all
2
3
4
5
Perfectly
126
Q

Babies of mothers who frequently used cocaine while pregnant:
A. are lethargic and nonresponsive and have significant cognitive impairments and motor delays.
B. are irritable and restless, have an exaggerated startle response and high-pitched cry, and are difficult to soothe.
C. have hearing and other sensory impairments, slowed reactions to external stimuli, and poor muscle tone.
D. cry infrequently and have lower-than-normal respiratory and heart rates and moderate to severe mental retardation.

A

Cocaine use by pregnant women increases the risk for spontaneous abortion and stillbirth; and, for babies who survive, is associated with several physical and behavioral abnormalities.
B. are irritable and restless, have an exaggerated startle response and high-pitched cry, and are difficult to soothe - CORRECT Infants of mothers who used cocaine during pregnancy often have a low birth weight, a piercing cry, and an exaggerated startle response; are irritable and hypersensitive; and do not respond in normal ways to the sound and sight of caregivers.
(Lifespan Development)

How well did you know this?
1
Not at all
2
3
4
5
Perfectly
127
Q

Sarah and Eve are served liver for dinner. Sarah screams, “Now there’s more!” when hers is cut up into bite-size pieces. Eve says, “No, there’s not! It’s just the same.” In terms of Piaget’s stages of cognitive development:
A. Sarah is in the preoperational stage.
B. Eve is in the sensorimotor stage.
C. Sarah is in the concrete operational stage.
D. Eve is in the preoperational stage. l

A

Piaget distinguished between four stages of cognitive development: sensorimotor, preoperational, concrete operational, and formal operational.
A. Sarah is in the preoperational stage - CORRECT Sarah appears to be in the preoperational stage since she is unable to conserve and, thus, believes there is more liver on her plate when it is cut into small pieces. In contrast, Eve can conserve and is, therefore, in the concrete (or perhaps Formal) operational stage.
(Lifespan Development)

How well did you know this?
1
Not at all
2
3
4
5
Perfectly
128
Q
Research on gender differences in conversation style indicates that, when compared to men, women:
A. talk for longer periods of time.
B. ask more questions.
C. interrupt more often.
D. make more eye contact.
A

Research has identified several consistent gender differences in conversation style.
B. ask more questions - CORRECT Women are more likely to ask questions during a conversation, and men tend to talk for longer periods of time, interrupt more frequently, and make more eye contact.
(Lifespan Development)

How well did you know this?
1
Not at all
2
3
4
5
Perfectly
129
Q
According to Erikson, the successful outcome of the final stage of psychosocial development is:
A. accomplishment.
B. personal control.
C. wisdom.
D. identity.
A

Ego integrity versus despair is the final stage in Erikson’s theory of psychosocial development.
a. Incorrect A sense of accomplishment is the successful outcome of the generativity vs. stagnation stage which occurs in mid-adulthood.
b. Incorrect A sense of personal control is the successful outcome of the autonomy vs. shame and doubt stage which occurs from about ages 1 to 3.
C. wisdom - CORRECT Wisdom and integrity are the outcomes of successful resolution of the psychosocial crises of the final ego integrity vs. despair stage.
d. Incorrect A coherent sense of identity is the successful outcome of the identity vs. role confusion stage which occurs in adolescence.
(Lifespan Development)

How well did you know this?
1
Not at all
2
3
4
5
Perfectly
130
Q

The original research on the self-fulfilling prophecy effect (Rosenthal and Jacobson, 1968) examined how:
A. adolescents’ expectations about their academic performance affected their actual performance.
B. teachers’ expectations about the academic performance of children affected the children’s actual performance.
C. physicians’ diagnoses affected the health-related behaviors of patients with chronic illnesses.
D. the social context affects the way that people interpret the behaviors of others.

A

The self-fulfilling prophecy (Rosenthal) effect predicts that other people’s expectations about our performance affects our actual performance.
a. Incorrect This does not describe the self-fulfilling prophecy effect.
B. teachers’ expectations about the academic performance of children affected the children’s actual performance - CORRECT Rosenthal and Jacobson (1968) found that grade school teachers’ expectations about students had a self-fulfilling prophecy effect - i.e., when
teachers were told that the test scores of some students indicated they would be “academic spurters” (even though the students had been randomly selected), those students did well, apparently because they were treated differently by the teachers.
c. Incorrect This does not describe the self-fulfilling prophecy effect.
d. Incorrect This sounds like Rosenhan’s pseudopatient study.
(Lifespan Development)

How well did you know this?
1
Not at all
2
3
4
5
Perfectly
131
Q

Infantile amnesia refers to the inability of:
A. children to develop declarative memories until 10 to 12 months of age.
B. children to develop memories for autobiographical events until 3 or 4 years of age.
C. adolescents and adults to recall events they experienced prior to 3 or 4 years of age.
D. adults to accurately recall when their children achieved major developmental milestones.

A

Infantile amnesia refers to the inability to recall events that occurred prior to about age 3 or 4.
b. Incorrect Children under the age of 3 form autobiographical memories. However, many of these memories are “lost” in subsequent yea rs.
C. adolescents and adults to recall events they experienced prior to 3 or 4 years of age - CORRECT Although early research suggested that infantile amnesia is absolute, subsequent studies indicate that older children, adolescents, and adults are often able to recall some events that took place prior to age 3 or 4.
(Lifespan Development)

How well did you know this?
1
Not at all
2
3
4
5
Perfectly
132
Q

Adults who are classified as ________ on the Adult Attachment Interview describe their childhood relationships with parents using positive terms, but
their specific childhood memories contradict or do not support these positive descriptions.
A. autonomous
B. preoccupied
C. dismissing
D. distant

A

The Adult Attachment Interview (AAI) assesses the examinee’s childhood relationships with his/her mother and father and categorizes the examinee’s attachment style as autonomous, preoccupied, or dismissing.
a. Incorrect Examinees classified as autonomous provide coherent descriptions of their childhood relationships.
b. Incorrect Examinees classified as preoccupied become very angry or confused when describing their childhood relationships with parents and may be preoccupied with a parent.
C. dismissing - CORRECT Examinees classified as dismissing provide inconsistent and incoherent descriptions of their childhood relationships with parents
d. Incorrect Distant is not one of the AAI categories.
(Lifespan Development)

How well did you know this?
1
Not at all
2
3
4
5
Perfectly
133
Q
Klinefelter syndrome is due to
A. a dominant gene.
B. a recessive gene.
C. a chromosomal deletion.
D. extra chromosome.
A

Klinefelter syndrome is due to a chromosomal abnormality that affects males only.
D. extra chromosome - CORRECT Men with Klinefelter syndrome have an extra X chromosome. Symptoms of this disorder include low testosterone levels and infertility, breast development, and reduced body and facial hair.
(Lifespan Development)

How well did you know this?
1
Not at all
2
3
4
5
Perfectly
134
Q

According to Whorf’s linguistic relativity hypothesis:
A. culture determines language.
B. language is dependent on thought.
C. language determines the nature of thought.
D. language, thought, and culture are reciprocally determined

A

An ongoing debate in the literature focuses on the relationship between language and thought.
a. Incorrect This is essentially the opposite of Whorf’s hypothesis.
b. Incorrect This is also the opposite of Whorf’s view and is more consistent with the view of Piaget.
C. language determines the nature of thought - CORRECT According to Whorf’s hypothesis (also known as the Sapir-Whorf hypothesis), the structure and lexicon of language influences how the individual perceives, interprets, and reacts to the world.
d. Incorrect This does not accurately describe Whorf’s hypothesis.
(Lifespan Development)

How well did you know this?
1
Not at all
2
3
4
5
Perfectly
135
Q
Two-year-old Mayen uses the word “juice” to refer to all of the beverages that she drinks. This is an example of:
A. overextension.
B. overgeneralization.
C. expansion.
D. extension.
A

Children make a number of predictable errors during the process of language acquisition.
A. overextension - CORRECT Overextension occurs when a child uses a word to describe a wider range of objects or events than is appropriate.
b. Incorrect Overgeneralization occurs when the child extends grammatical rules to words that are an exception to the rule (e.g., adds “ed” to “go” to Form the past tense).
c. Incorrect Expansion refers to an adult’s response to a child’s utterance that elaborates on what the child has said.
d. Incorrect It is overextension (not extension} that describes Mayen’s inappropriate use of the word “juice.”
(Lifespan Development)

How well did you know this?
1
Not at all
2
3
4
5
Perfectly
136
Q
Ron attends a rally organized to oppose gun control. When asked why he is against gun control, Ron states, "Well, the law says people have the right to own a gun, and everybody in my hometown has a gun." Kohlberg would say that Ron is at which level of moral development?
A. preconventional 
B. conventional
C. postconventional
D. autonomous
A

Kohlberg distinguishes between three levels of moral development: preconventional, conventional, and postconventional.
a. Incorrect Preconventional moral judgments are typical of children aged 4 through 10 and reflect a desire to avoid punishment and satisfy one’s own needs
b. CORRECT The conventional level of moral development is characterized by support for and adherence to existing social rules and laws.
c. Incorrect Moral judgments at the postconventional level are based on social contracts, democratically accepted laws, and the individual’s conscience.
d. Incorrect Autonomous is one of Piaget’s stages of moral development.
(Lifespan Development)

How well did you know this?
1
Not at all
2
3
4
5
Perfectly
137
Q

As defined by Vygotsky, “scaffolding” refers to:
A. biological maturation.
B. the use of effective memory strategies.
C. attachment to caregivers.
D. assistance from more skilled individuals.

A

Scaffolding is a key concept in Vygotsky’s theory of cognitive development.
a. Incorrect Although Vygotsky recognized the role of biological maturation in cognitive development, scaffolding refers to social factors.
b. Incorrect Assistance within the zone of proximal development might include modeling effective memory strategies, but this is not the best answer of those given.
c. Incorrect Attachment is not a focus of Vygotsky’s theory.
D. assistance from more skilled individuals - CORRECT Vygotsky proposed that scaffolding helps the child progress within the zone of proximal development. It refers to the temporary support and assistance that adults and more knowledgeable and skilled peers provide to a
learner and includes modeling appropriate responses, simplifying a task so that it is more achievable, and providing instructions and cues.
(Lifespan Development)

How well did you know this?
1
Not at all
2
3
4
5
Perfectly
138
Q

The rooting reflex occurs when:
A. the newborn’s cheek is touched.
B. the newborn’s foot is tickled.
C. the newborn hears his/her caregiver’s voice.
D. the newborn hears an unexpected loud noise

A

Reflexes are unlearnecl responses to particular stimuli in the environment.
A. the newborn’s cheek is touched - CORRECT The rooting reflex occurs when the baby turns his/her head in the direction of a touch or stroke on his/her cheek.
b. Incorrect This describes the Babinski reflex.
c. Incorrect This does not describe the rooting reflex.
d. Incorrect This describes the Moro (startle) reflex.
(Lifespan Development)

How well did you know this?
1
Not at all
2
3
4
5
Perfectly
139
Q
Children begin to deliberately and regularly use rehearsal, elaboration, and organization as memory strategies by \_\_\_\_\_ years of age.
A. 2 to 3
B. 5 to 7
C. 9 to 10 
D. 13 to 15
A

The consistent and deliberate use of memory strategies does not occur until age 9 or 10.
b. Incorrect Preschoolers sometimes use memory strategies but do so in non-deliberate and often ineffective ways. In addition, children of this age can be
taught to use memory strategies for a specific task but do not apply them to new tasks.
C. 9 to 10 - CORRECT The research has shown that the deliberate and consistent use of the memory strategies listed in the question begins at about 9 to 10 years of
age.
d. Incorrect The use of these strategies continues to be “fine-tuned” during adolescence.
(Lifespan Development)

How well did you know this?
1
Not at all
2
3
4
5
Perfectly
140
Q
Which of the following is LEAST likely to be one of a child's first words?
A. fall
B. bye-bye
C. cold
D. cookie
A

A child’s first words are most likely to be social or functional expressions such as hi, bye-bye, or thanks; names for objects that are permanent, familiar, and usually movable such as dada or blanket; and action words such as push or fall.
a. Incorrect “Fall” is an action word and is likely to be a first word.
b. Incorrect “Bye-bye” is a social expression and is likely to be one of a child’s first words.
C. cold - CORRECT Words representing sensations or feelings such as “cold” are likely to appear later in the child’s linguistic development.
d. Incorrect “Cookie” is the name for a permanent, familiar, and movable object and is the type of word that is likely to be one of a child’s first words.
(Lifespan Development)

How well did you know this?
1
Not at all
2
3
4
5
Perfectly
141
Q

According to Erikson, at age two, the primary psychosocial task is to develop ________, and failure to successfully accomplish this task leads to __________.
A. an ego identity; identity diffusion
B. a capacity for industry; a sense of inferiority
C. a sense of basic trust; suspicion and mistrust
D. sense of autonomy; doubt and shame

A

Erikson identified eight stages of psychosocial development that are each characterized by a different psychosocial conflict.
a. Incorrect This is characteristic of the identity vs. identity confusion stage which occurs in adolescence.
b. Incorrect This is characteristic of the industry vs. inferiority stage which occurs from ages 6 to 11 years.
c. Incorrect This is characteristic of the basic trust vs. mistrust stage which occurs from birth to 1 year of age.
D. sense of autonomy; doubt and shame - CORRECT Erikson’s autonomy vs. shame and doubt stage occurs from 1 to 3 years of age. Successful resolution of the conflicts of this stage results in a sense of self-control (autonomy), while failure leads to shame and doubt.
(Lifespan Development)

How well did you know this?
1
Not at all
2
3
4
5
Perfectly
142
Q
According to Daniel Levinson's (1986) Seasons of a Man's Life model, the mid-life transition is characterized by:
A. following the dream
B. formulation of the dream
C. re-evaluation of the dream
D. realization of the dream
A

Levinson’s model identifies several stages and transitional periods. The mid-life transition occurs between the ages of 40 and 45.
a. Incorrect Following the dream (goal or vision for the future) is characteristic of the “entry life structure for early adulthood” stage, which occurs from ages 22 to 28.
b. Incorrect Formulation of the dream is characteristic of the “early adult transition,” which occurs from age 17 to 22.
C. re-evaluation of the dream - CORRECT The “mid-life transition” is characterized by a realization that the
dream is not satisfying and/or will not be accomplished.
d. Incorrect Realization of the dream is characteristic of the “culmination of early adult life structure” stage, which occurs from ages 33 to 45.
(Lifespan Development)

How well did you know this?
1
Not at all
2
3
4
5
Perfectly
143
Q

Thomas and Chess’s (1977) “goodness-of-fit” model predicts that maladjustment in children is due to:
A. a mismatch between the child s basic temperament and his/her parents childrearing practices.
B. a mismatch between the child’s attachment behaviors and his/her parents’ response to those behaviors.
C. inconsistencies in the parenting styles of the child’s caregivers.
D. inconsistencies in the child’s sense of self and the feedback the child receives from caregivers about his/her behavior.

A

Knowing that Thomas and Chess studied temperament would have enabled you to identify the correct response to this question.
A. a mismatch between the childs basic temperament and his/her parents childrearing practices - CORRECT Based on their research, Thomas and Chess concluded that it is not the chi|d’s early temperament that predicts Future adjustment but the match between the child’s temperament and his/her caregivers’ childrearing practices.
(Lifespan Development)

How well did you know this?
1
Not at all
2
3
4
5
Perfectly
144
Q

The most common symptoms of congenital cytomegalovirus (CMV) are:
A. immunologic abnormalities and delayed physical and cognitive development.
B. hyperactivity, irritability, and irregular sleep and feeding habits.
C. learning disabilities, facial deformities, and hyperactivity.
D. mental retardation and hearing and visual impairments.

A

Congenital cytomegalovirus (CMV) occurs when CMV, a type of herpes virus, is passed from an infected pregnant woman to her fetus through the placenta.
a. Incorrect These are signs of HIV infection in childhood.
b. Incorrect These symptoms are associated with prenatal malnutrition but not with congenital CMV.
c. Incorrect These are symptoms of fetal alcohol syndrome.
D. mental retardation and hearing and visual impairments - CORRECT These are the most common symptoms of congenital CMV.
(Lifespan Development)

How well did you know this?
1
Not at all
2
3
4
5
Perfectly
145
Q

An individual’s phenotype:
A. is attributable solely to his or her genetic inheritance.
B. refers to characteristics that are determined by environmental factors
C. is determined by a combination of genetic and environmental factors.
D. refers to the unobservable factors that contribute to behavior, personality, etc.

A

Researchers interested in the relative contributions of genetics and environment to human characteristics distinguish between genotype and phenotype.
a. Incorrect Genotype refers to the individual’s genetic endowment.
C. is determined by a combination of genetic and environmental factors - CORRECT Phenotype refers to observable characteristics that are attributable to the individual’s genotype (genetic make-up) and environment.
(Lifespan Development)

How well did you know this?
1
Not at all
2
3
4
5
Perfectly
146
Q
According to Kohlberg, a person in the sixth (final) stage of moral development is guided in his or her moral judgments primarily by:
A. laws and legally binding contracts.
B. principles of conscience.
C. cultural standards and values.
D. religious doctrine.
A

Kohlberg’s original theory of moral development included six stages. The sixth stage is the “morality of individual principles of conscience” stage.
B. principles of conscience - CORRECT In the sixth stage of development, moral judgments are based on self-chosen principles that are considered universal in application.
(Lifespan Development)

How well did you know this?
1
Not at all
2
3
4
5
Perfectly
147
Q

By 18 months of age, the average child is:
A. speaking in single words and walking while holding onto furniture.
B. speaking in two-word phrases and walking independently.
C. speaking in multi-word phrases and walking up and down the stairs without help.
D. speaking in short sentences and walking up and down the stairs with help.

A

For the exam, you’ll want to be familiar with the major developmental milestones of early childhood.
a. Incorrect This is more characteristic of children at 12 months.
B. speaking in two-word phrases and walking independently - CORRECT Two-word phrases and the ability to walk independently are exhibited by most children by the age of 18 months.
c. Incorrect This is more characteristic of 2-year-olds.
d. Incorrect Speaking in short sentences is more characteristic of 3-year-olds.
(Lifespan Development)

How well did you know this?
1
Not at all
2
3
4
5
Perfectly
148
Q

A child would be most likely to do which of the following first?
A. pull him/herself to standing by holding onto the furniture.
B. show separation anxiety
C. pretend that a wood cylinder is a cup
D. say her first word

A

To answer this question, you must be familiar with the developmental milestones that occur during the first 6 to 12 months of life.
a. Incorrect Children typically pull themselves to a standing position by holding onto the Furniture by 9 to 10 months of age.
B. show separation anxiety - CORRECT Separation anxiety usually first appears between the ages of 6 to 8
months of age.
c. Incorrect Symbolic play usually begins around 11 to 12 months of age.
d. Incorrect Children usually say their first recognizable words at about 12 months of age.
(Lifespan Development)

How well did you know this?
1
Not at all
2
3
4
5
Perfectly
149
Q
Habituation-dishabituation research has demonstrated that, at about \_\_\_ months of age, infants begin to exhibit recognition memory for a visual stimulus for up to 24 hours following presentation of the stimulus.
A. 3
B. 6
C. 9
D. 11
A

Researchers use several techniques to study memory in infants including habituation-dishabituation.
A. 3 - CORRECT By three months of age, infants habituate to visual stimuli — i.e., show less response to the second presentation of a stimulus for periods up to 24 hours. This is interpreted as indicating that the infant recognizes the stimulus.
(Lifespan Development)

How well did you know this?
1
Not at all
2
3
4
5
Perfectly
150
Q

Auditory localization:
A. develops gradually in the first six months of life.
B. is present to some degree at birth but then seems to disappear between the ages of 2 and 4 months and then reappears.
C. is present to some degree at birth but then seems to disappear between the ages of 9 and 12 months and then reappears.
D. is not evident in the first few months of life but develops gradually and is not fully developed until 24 months of age.

A

Auditory localization refers to ability to orient toward the direction of a sound.
B. is present to some degree at birth but then seems to disappear between the ages of 2 and 4 months and then reappears - CORRECT Although auditory localization is apparent soon after birth, it seems to disappear between 2 and 4 months of age, and then reappears and improves
during the rest of the first year of life.
(Lifespan Development)

How well did you know this?
1
Not at all
2
3
4
5
Perfectly
151
Q
According to Noam Chomsky's nativist theory, language is largely the result of:
A. innate factors.
B. operant conditioning.
C. parental modeling. 
D. cognitive development.
A

As a nativist, Noam Chomsky views language as an innate skill.
A. innate factors - CORRECT Chomsky proposed an innate language acquisition device that consists of language rules and that allows children to acquire language just by being exposed to it.
(Lifespan Development)

How well did you know this?
1
Not at all
2
3
4
5
Perfectly
152
Q

Children ages 7 to 9 years of age are most likely to say that:
A. people on TV can see into their homes.
B. people who die on TV are really dead.
C. people in TV shows are actors.
D. people on TV aren’t alive.

A

Researchers interested in the impact of television on children have attempted to determine the degree to which children of different ages distinguish between fact and fantasy.
a. Incorrect This is characteristic children aged 3 and 4.
b. Incorrect This is also characteristic of younger children.
C. people in TV shows are actors - CORRECT Children begin to understand that characters on television are
actors who are playing roles by 7 to 9 years of age.
d. Incorrect Children ages 7 to 9 understand that people on TV aren’t the characters they are playing, but they don’t believe that the people on TV “aren’t alive.”
(Lifespan Development)

How well did you know this?
1
Not at all
2
3
4
5
Perfectly
153
Q
Code-switching is associated with:
A. learning disabilities.
B. gender identity disorder.
C. encoding in long-term memory.
D. bilingualism.
A

Code-switching refers to the alternation between languages during a conversation.
D. bilingualism - CORRECT Code switching is used by bilinguals for several purposes — e.g., to establish rapport with a listener or to better communicate one’s attitude toward the listener.
(Lifespan Development)

How well did you know this?
1
Not at all
2
3
4
5
Perfectly
154
Q

The outer limit of Vygotsky’s zone of proximal development is defined as:
A. what a child can do with the assistance of an adult or more capable peer.
B. what the child can do with maximal effort and motivation.
C. the limitations determined by the child’s current biological maturation.
D. the limitations determined by the child’s current level of psychosocial development.

A

The “zone of proximal development” is a key concept in Vygotsky’s theory of cognitive development.
A. what a child can do with the assistance of an adult or more capable peer - CORRECT In Vygotsky’s theory of cognitive development, the zone of proximal development refers to the gap between what the child can currently do alone and what he/she can do with the assistance of a more skilled person.
(Lifespan Development)

How well did you know this?
1
Not at all
2
3
4
5
Perfectly
155
Q

Carol Gilligan (1982) criticized Kohlberg’s theory of moral development on the ground that it is:
A. less applicable to females than to males.
B. less applicable to members of minority groups than to Whites.
C. more applicable to highly educated individuals than to those with less education.
D. more applicable to religious than to nonreligious individuals.

A

Gilligan points out that Kohlberg’s original research participants were males and argues that, as a result, his theory of moral development is more applicable to males than to females.
A. less applicable to females than to males - CORRECT Based on her research, Gilligan concludes that the moral
judgments of Females are often influenced by concerns related to caring, compassion, and responsibility to others.
(Lifespan Development)

How well did you know this?
1
Not at all
2
3
4
5
Perfectly
156
Q
Child abuse and neglect are most associated with which of the following attachment patterns?
A. disorganized/disoriented
B. distant/disengaged
C. insecure/avoidant
D. insecure/ambivalent
A

Children who have been maltreated often exhibit disorganized, contradictory behaviors in the presence of their caregivers — e.g., they might reach out for the caregiver while backing away.
A. disorganized/disoriented - CORRECT Mary Main, a student of Ainsworth, Found that about 80% of maltreated children exhibit a disorganized/disoriented attachment pattern.
(Lifespan Development)

How well did you know this?
1
Not at all
2
3
4
5
Perfectly
157
Q

According to Piaget, when making moral judgments, children in the autonomous stage:
A. focus on the consequences of the behavior only.
B. consider relevant laws and rules.
C. focus on their own emotional reactions to the behavior.
D. consider the intentions of the actor.

A

Piaget distinguished between two stages of moral development: heteronomous and autonomous.
a. Incorrect This is characteristic of the heteronomous stage.
b. Incorrect This is more characteristic of the heteronomous than the autonomous stage.
c. Incorrect This is not characteristic of the autonomous stage.
D. consider the intentions of the actor - CORRECT The autonomous stage begins by age 10 or 11. Children in this
stage base their judgments of behavior primarily on the intentions of the actor.
(Lifespan Development)

How well did you know this?
1
Not at all
2
3
4
5
Perfectly
158
Q

Research has shown that teachers criticize boys more than girls and that:
A. girls receive more praise and positive feedback than boys do.
B. boys receive more praise for ability but girls receive more praise for effort.
C. boys receive more praise for effort but girls receive more praise for ability.
D. boys and girls both receive more praise for effort than for ability.

A

While boys receive more criticism from teachers overall, patterns of criticism (and praise) differ for boys and girls.
a. Incorrect Many teachers seem to take the “good behavior” of girls for granted; and, consequently, it is not generally true that girls receive more praise than boys do.
B. boys receive more praise for ability but girls receive more praise for effort - CORRECT Patterns of criticism and praise by teachers are usually gender- related: Boys are praised more by teachers for their ability, while girls are more likely to receive praise for their effort.
(Lifespan Development)

How well did you know this?
1
Not at all
2
3
4
5
Perfectly
159
Q

Babbling ordinarily begins at about 4 to 5 months of age and initially includes:
A. phonemes from all languages.
B. phonemes from the child’s native language only.
C. morphemes from all languages.
D. morphemes that the child has heard most frequently.

A

Babbling involves the repetition of vowel and consonant sounds — e.g., bi-bi-bi and da-da-da.
A. phonemes from all languages - CORRECT Babbling initially consists of phonemes from all languages; but, by
about 9 months, it narrows to include the sounds of the child’s native language only.
(Lifespan Development)

How well did you know this?
1
Not at all
2
3
4
5
Perfectly
160
Q
When asked if he has thought about college, Hakaru H., age 16, says, “My parents want me to be a lawyer, so I'm planning to go to law school." According to Marcia, Hakaru is exhibiting which of the following?
A. identity moratorium
B. identity diffusion
C. identity achievement
D. identity foreclosure
A

Marcia distinguishes between 4 identity statuses that reflect the degree to which the individual has experienced an identity crisis and is committed to an identity.
a. Incorrect Identity moratorium occurs when the individual is experiencing an identity crisis and is actively exploring alternative identities.
b. Incorrect Identity diffusion occurs when the individual has not yet experienced an identity crisis and is not committed to a particular identity.
c. Incorrect Identity achievement occurs when the individual has resolved the identity crisis and is committed to an identity.
D. identity foreclosure - CORRECT Identity foreclosure occurs when the individual has not experienced an identity crisis but has chosen an identity (e.g., career) that has been imposed by others.
(Lifespan Development)

How well did you know this?
1
Not at all
2
3
4
5
Perfectly
161
Q

A child with an insecure/anxious-avoidant pattern of attachment:
A. is very distressed when separated from his/her mother and continues to be highly anxious when she returns.
B. is more anxious when his/her mother is present than absent and resists contact with her when she returns following separation.
C. is very distressed when separated from his/her mother but pulls away from her when she returns.
D. shows little distress when separated from his/her mother and turns away from her when she returns.

A

Attachment patterns are categorized in terms of 4 types: secure, insecure/anxious-avoidant, insecure/anxious-ambivalent, and disorganized-disoriented.
D. shows little distress when separated from his/her mother and turns away from her when she returns - CORRECT In Ainsworth’s “strange situation,” insecure/anxious-avoidant
children are uninterested in exploration, show little distress when separated from their mothers, and avoid her when she returns.
(Lifespan Development)

How well did you know this?
1
Not at all
2
3
4
5
Perfectly
162
Q
Kohlberg's cognitive-developmental theory identifies which of the following as the initial stage of gender-role acquisition?
A. gender affirmation
B. gender identity 
C. gender constancy
D. gender stability
A

Kohlberg’s cognitive-developmental model identifies three stages of gender-role acquisition — gender identity, gender stability, and gender constancy.
B. gender identity - CORRECT According to Kohlberg (1969), gender identity is the first stage in the acquisition of gender-role behaviors and occurs at about age 2 to 3. It is followed by gender stability (age 4) and then gender constancy (ages 6 to 7).
(Lifespan Development)

How well did you know this?
1
Not at all
2
3
4
5
Perfectly
163
Q
A baby with \_\_\_\_\_\_\_\_\_ attachment is mildly upset when separated from his/her mother and then actively seeks contact with her when she returns.
A.insecure/ambivalent
B. insecure/avoidant
C. secure
D. stable
A

Research on early attachment has identified four basic attachment patterns — secure, insecure/ambivalent, insecure/avoidant, and disorganized/disoriented.
a. Incorrect A baby with insecure/ambivalent attachment is very distressed when separated from his/her mother but is angry and resistant to contact when she returns.
b. Incorrect A baby with insecure/avoidant attachment shows little distress when separated from his/her mother and ignores or avoids her when she returns.
C. secure - CORRECT The behavior described in this question is characteristic of babies with secure attachment.
d. Incorrect Stable is not one of the four types of attachment patterns identified by researchers.
(Lifespan Development)

How well did you know this?
1
Not at all
2
3
4
5
Perfectly
164
Q
By about 8 to 12 months of age, babies actively seek an object that has been hidden from view. According to Piaget, this ability is clue to the development of:
A. working memory.
B. metamemory.
C. object attachment.
D. the object concept.
A

This question describes the ability to recognize that objects continue to exist even when they are no longer in view.
D. the object concept - CORRECT Piaget referred to this ability as the object concept (or object permanence). It begins to develop when the child is between 8 and 12 months of age.
(Lifespan Development)

How well did you know this?
1
Not at all
2
3
4
5
Perfectly
165
Q
To study social referencing, a developmental psychologist would most likely use:
A. an habituation task.
B. pattern recognition.
C. a mobile.
D. the visual cliff.
A

Social referencing refers to relying on the emotional response of another person (e.g., caregiver) to decide how to respond in an ambiguous situation.
D. the visual cliff - CORRECT Of the techniques listed in the answers, the visual cliff is the only one that has been used to study social referencing. Research using this technique has found that a baby’s willingness to cross the visual cliff depends on his/her mother’s facial expression.
(Lifespan Development)

How well did you know this?
1
Not at all
2
3
4
5
Perfectly
166
Q
Of the senses, is least well-developed at birth
A. vision
B. audition
C. taste
D. smell
A

The infant’s senses develop rapidly in the first few months of life.
A. vision - CORRECT OF the senses, vision is least well-developed at birth. In terms of visual acuity, newborns see at 20 Feet what an adult with normal vision sees at
200 feet.
(Lifespan Development)

How well did you know this?
1
Not at all
2
3
4
5
Perfectly
167
Q

Garin G., age 3.5 years, sometimes repeats words and phrases or leaves out words and sounds when he’s talking about something that he’s very excited about. At this age, this type of speech dysfluency is usually:
A. suggestive of an emotional problem.
B. indicative of a speech disability.
C. the result of inadequate language training.
D. a normal developmental phenomenon.

A

The speech dysfluency exhibited by Garin is normal for children between the ages 18 months and 5 years.
D. a normal developmental phenomenon - CORRECT Speech dysfluency is usually considered indicative of a problem only if it continues beyond the age of 5.
(Lifespan Development)

How well did you know this?
1
Not at all
2
3
4
5
Perfectly
168
Q
As described by Piaget, the concrete operational stage extends from ages:
A. birth to 2 years.
B. 2 to 7 years.
C. 7 to 11 years.
D. 11 to 15 years.
A

Piaget distinguished between 4 stages of cognitive development: sensorimotor, preoperational, concrete operational, and formal operational. Note that the exact age at which each stage begins varies, depending on the individual’s experiences. However, of the age ranges
listed, only one is most associated with the concrete operational stage.
a. Incorrect The sensorimotor stage extends from birth to 2 years of age.
b. Incorrect The preoperational stage extends from 2 to 7 years of age.
C. 7 to 11 years - CORRECT Piaget’s concrete operational stage extends from 7 to 11 years of age.
d. Incorrect The Formal operational stage extends from 11 years on.
(Lifespan Development)

How well did you know this?
1
Not at all
2
3
4
5
Perfectly
169
Q

Maternal malnutrition has the most severe consequences for the developing brain of the fetus when it occurs:
A. from conception to first month of prenatal development.
B. from the first month to third month of prenatal development.
C. from the third month to sixth month of prenatal development.
D. from the sixth month to ninth month of prenatal development.

A

Prenatal malnutrition has its most severe impact, especially on the fetus’s brain, during the later stages of prenatal development.
D. from the sixth month to ninth month of prenatal development - CORRECT The brain grows most rapidly during the Final trimester, and prenatal malnutrition during this period is associated with a low brain weight and abnormalities in the organization of the brain’s neurons.
(Lifespan Development)

How well did you know this?
1
Not at all
2
3
4
5
Perfectly
170
Q

During the second stage of Kohlberg’s preconventional level of moral development, children obey rules because:
A. they feel they have a personal duty to uphold rules and laws.
B. “everyone else is doing it.”
C. doing so helps them avoid punishment.
D. doing so helps them satisfy their personal needs.

A

The preconventional level of moral development consists of two stages: punishment and obedience and instrumental hedonism.
a. Incorrect This is characteristic of the second stage of the conventional level.
b. Incorrect This doesn’t describe the second stage of the preconventional level.
c. Incorrect This is characteristic of the initial punishment and obedience stage of the preconventional level.
D. doing so helps them satisfy their personal needs - CORRECT During the instrumental hedonism stage, children consider the correct action to be the one that best satisfies their own personal needs. To illustrate this stage, Kohlberg described a 10-year old boy who, when asked what it means to be a good son, replied, “Be good to your Father and he will be good to you.”
(Lifespan Development)

How well did you know this?
1
Not at all
2
3
4
5
Perfectly
171
Q
Research investigating father-child attachment suggests that it depends most on:
A. nurturance and caregiving.
B. authority.
C. play activities.
D. maternal absence.
A

Fathers and their children often develop a strong bond, but the behaviors that contribute to father-child attachment differ from those that are most important for mother-child attachment.
C. play activities - CORRECT The research indicates that the father-child bond is related more to the father’s role as “playmate” than to nurturance and caregiving, which underlie the mother-child bond.
(Lifespan Development)

How well did you know this?
1
Not at all
2
3
4
5
Perfectly
172
Q

According to Michael Rutter (1979), an increased risk for psychopathology in children and adolescents is associated with which of the following factors?
A. low socioeconomic status, large family size, and maternal psychopathology
B. large family size, paternal psychopathology, and difficult temperament
C. maternal psychopathology, low socioeconomic status, and racial/ethnic minority status
D. low socioeconomic status, single-parent family, and racial/ethnic minority status

A

Rutter’s indicators for psychopathology are low SES, marital discord, large family size and overcrowding, paternal criminality, maternal psychiatric disorder, and being placed in foster care [M. Rutter, Protective factors in children’s responses to stress and disadvantage, in
M. W. Garmezy and M. Rutter (eds.), Stress, Coping, and Development in Children (Vol. 3), McGraw-Hill, NY, 1979].
A. low socioeconomic status, large family size, and maternal psychopathology - CORRECT These are three of the six risk factors identified by Rutter.
b. Incorrect Paternal psychopathology and difficult temperament are not risk factors identified by Rutter.
c. Incorrect Racial/ethnic minority status was not identified by Rutter as a risk factor.
d. Incorrect Single-parent family and racial/ethnic minority status are not risk factors identified by Rutter.
(Lifespan Development)

How well did you know this?
1
Not at all
2
3
4
5
Perfectly
173
Q
Information processing theories of cognitive development focus on:
A. developmental stages.
B. mental rules and strategies.
C. social interactions.
D. critical periods.
A

Information processing theories conceptualize the human mind as a computer.
a. Incorrect In contrast to Piagetians, information processing theorists are not interested in stages of cognitive development.
B. mental rules and strategies - CORRECT Information processing theorists focus on mental operations and
view cognitive development as due to changes in mental capacity and increasing sophistication in the use of relevant rules and strategies.
c. Incorrect Social interactions are central to Vygotsky’s sociocultural theory but are not a focus for information processing theorists.
d. Incorrect Information processing theorists view development as continuous and do not propose that it occurs only during critical periods.
(Lifespan Development)

How well did you know this?
1
Not at all
2
3
4
5
Perfectly
174
Q
Which aspect of long-term memory is usually the first to be adversely affected by increasing age?
A. sensory
B. semantic
C. procedural
D. episodic
A

Long-term memory is divided into procedural and declarative memory, and declarative memory is further divided into semantic and episodic (autobiographical) memory.
a. Incorrect Sensory memory is not a type of long-term memory.
b. Incorrect Semantic memory is less atfected by normal aging than is episodic memory.
c. Incorrect Procedural memory is relatively unaffected by normal aging.
D. episodic - CORRECT Episodic memory is memory For personally experienced events and is the first aspect of long-term memory that shows age-related declines.
(Lifespan Development)

How well did you know this?
1
Not at all
2
3
4
5
Perfectly
175
Q
For most children, stranger anxiety begins at about \_\_\_\_\_ months of age
A. 2 to 3
B. 4 to 6
C. 8 to 10
D. 12 to 14
A

Stranger anxiety involves anxious, fearful responses in the presence of a stranger, especially when a caregiver is not present.
C. 8 to 10 - CORRECT Although the experts differ somewhat with regard to the exact age of onset of stranger anxiety, most agree that it begins during the second half of the first year.
(Lifespan Development)

How well did you know this?
1
Not at all
2
3
4
5
Perfectly
176
Q
In terms of age, Erikson's stage of psychosocial development corresponds to Freud's phallic stage.
A. industry vs. inferiority
B. autonomy vs. shame and doubt
C. basic trust vs. mistrust
D. initiative vs. guilt.
A

Erikson’s psychosocial theory proposes that personality development occurs throughout the lifespan and involves eight stages.
a. Incorrect The industry vs. inferiority stage is characteristic of children aged 6 to 11 years and corresponds to Freud’s latency stage.
b. Incorrect The autonomy vs. shame and doubt stage occurs between the ages of 1 and 3 years and corresponds to Freud’s anal stage.
c. Incorrect The basic trust vs. mistrust stage occurs from birth to 1 year of age and corresponds to Freud’s oral stage.
D. initiative vs. guilt - CORRECT The initiative vs. guilt stage is characteristic of children aged 3 to 6 years and corresponds to Freud’s phallic stage.
(Lifespan Development)

How well did you know this?
1
Not at all
2
3
4
5
Perfectly
177
Q
A key accomplishment of Piaget's preoperational stage is the emergence of:
A. conservation.
B. the object concept. 
C. formal operations.
D. the symbolic function.
A

According to Piaget, the preoperational stage is characterized by the development of the symbolic function, which permits language use, symbolic play, and deferred imitation.
a. Incorrect Conservation refers to the knowledge that physical properties of objects remain invariant with transformations that do not involve addition or subtraction. Such knowledge is acquired between the ages of 7 and 11 years during the concrete operational period.
b. Incorrect The object concept develops during the sensorimotor stage, which is characteristic of children from birth to two years.
c. Incorrect Formal operations are characteristic of the Final (formal operational) stage of cognitive development.
D. the symbolic function - CORRECT The ability For symbolic representation, which is necessary for language use, symbolic play, and deferred imitation, develops during the preoperational stage, which is characteristic of children aged 2 through 7.
(Lifespan Development)

How well did you know this?
1
Not at all
2
3
4
5
Perfectly
178
Q
According to Bronfenbrenner’s (1979) ecological model, the \_\_\_\_\_ consists of interactions between elements of the microsystem (e.g., between the family and the school).
A. mesosystem 
B. exosystem
C. macrosystem
D. endosystem
A

Bronfenbrenner’s ecological model distinguishes between 4 layers (levels) that influence a child’s development: microsystem, mesosystem, exosystem, and macrosystem.
A. mesosystem - CORRECT The mesosystem is the second layer and consists of interactions between components of the microsystem — e.g., the parents’ involvement in the child’s school experiences and interactions between the child’s church and community.
b. Incorrect The exosystem consists of elements in the social setting that indirectly affect the child’s development (e.g., the parents’jobs).
c. Incorrect The macrosystem is the outermost layer and includes the values, laws, customs, etc. of the child’s culture.
d. Incorrect The endosystem is not one of the levels identified by Bronfenbrenner.
(Lifespan Development)

How well did you know this?
1
Not at all
2
3
4
5
Perfectly
179
Q
Two-word sentences are characteristic of the speech of children at \_\_\_\_\_ months of age.
A. 8 to 12
B. 12 to 18
C. 18 to 24
D. 24 to 30
A

Two-word sentences are referred to as telegraphic speech.
C. 18 to 24 - CORRECT Telegraphic speech is characteristic of children between the ages of 18 and 24 months, and consists of the most important words (e.g., “my doll” and “give cake”).
(Lifespan Development)

How well did you know this?
1
Not at all
2
3
4
5
Perfectly
180
Q
Research evaluating the psychological and behavioral effects of child sexual abuse have found that all of the following except \_\_\_\_\_ are associated with more severe outcomes.
A. male gender
B. earlier age of onset of abuse
C. abuse by a family member
D. abuse by a much older perpetrator
A

Like much research in psychology, studies on the outcomes of child sexual abuse have not produced consistent results.
A. male gender - CORRECT Most studies have not found a consistent difference in outcomes for male and Female victims of sexual abuse. However, the studies that have found a gender difference and have found the outcomes to be worse for Females than for males.
b. Incorrect An earlier age of onset of the abuse has been linked to worse outcomes.
c. Incorrect Abuse by a parent, parent-figure, or other Family member is associated with worse outcomes than is abuse by a stranger.
d. Incorrect Abuse by a much older perpetrator is associated with worse outcomes. See, e.g., National Research Council, Understanding Child Abuse and
Neglect, Washington, DC, National Academy Press, 1993.
(Lifespan Development)

How well did you know this?
1
Not at all
2
3
4
5
Perfectly
181
Q
An increased risk for juvenile delinquency is most associated with which of the following parenting styles?
A. authoritarian
B. authoritative
C. permissive/nondirective 
D. rejecting/neglecting
A

Several parental behaviors have been linked to an increased risk for juvenile delinquency.
D. rejecting/neglecting - CORRECT A consistent finding about antisocial behavior in adolescents is that it is predicted by low parental warmth, high parental conflict, and inconsistent discipline. Of the parenting styles listed in the answers, these parental behaviors are most characteristic of the rejecting/neglecting style.
(Lifespan Development)

How well did you know this?
1
Not at all
2
3
4
5
Perfectly
182
Q

Recent research suggests that play in childhood:
A. is best described as an imitation of adult behavior
B. contributes to many aspects of development.
C. is essentially a form of entertainment.
D. helps children develop rule-governed behaviors.

A

Although play, by definition, does not have a purpose, the research has found that it serves important functions for both humans and animals.
a. Incorrect This describes sociodramatic play and does not apply to all types of play.
B. contributes to many aspects of development - CORRECT Current theories about play during childhood regard it as important for a child’s physical, intellectual, social, and emotional development.
d. Incorrect This is true about some types of play only and, therefore, is not the best answer of those given.
(Lifespan Development)

How well did you know this?
1
Not at all
2
3
4
5
Perfectly
183
Q
In her longitudinal study of high-risk infants, Werner (1993) found that positive outcomes for these infants were most associated with:
A. small family size.
B. good physical health.
C. precocious physical development.
D. an easy temperament.
A

Werner (1993) identified several factors that contribute to resilience in high-risk infants.
D. an easy temperament - CORRECT Werner concludes that resilience is most associated with (1) fewer stressors following birth, (2) stable support from a parent or other caregiver, and (3) an easy temperament.
(Lifespan Development)

How well did you know this?
1
Not at all
2
3
4
5
Perfectly
184
Q

Raymond is a high-energy four-year old who is always asking questions and likes to help with everything. A problem, however, is that when Raymond starts helping one family member, he is often distracted by other activities and may run off to help another member of the family with whatever he or she is doing. Raymond also likes to pretend that he’s one of his favorite characters on TV and can often be seen trying to “arrest” his younger
brother or save his friends from an imaginary “bad guy.” In terms of Erikson’s stages of psychosocial development, Raymond appears to be in which stage of development?
A. autonomy vs. shame and doubt
B. initiative vs. guilt
C. trust vs. mistrust
D. industry vs. inferiority

A

Erikson identified 8 stages of psychosocial development that encompass the lifespan. According to Erikson, each stage is characterized by a psychosocial crisis that must be appropriately resolved in order for healthy growth and maturation to occur.
a. Incorrect Autonomy vs. shame is the second stage of psychosexual development and occurs between the ages of 1 and 3 years.
B. initiative vs. guilt - CORRECT The stage of initiative vs. guilt begins around age three. It is characterized by increasing locomotion, imagination, curiosity, and exploration. Raymond’s age and behavior suggest that he is in this stage.
c. Incorrect The stage of trust vs. mistrust is the first stage of psychosocial development.
d. Incorrect Industry vs. inferiority is the fourth stage of development and usually begins at around age six.
(Lifespan Development)

How well did you know this?
1
Not at all
2
3
4
5
Perfectly
185
Q
You conduct a study to compare gifted and average children. Most likely the results of your study will indicate that gifted children are superior in terms of which of the following?
A. divergent thinking 
B. response speed
C. metacognition
D. parallel processing
A

Recent research on giftedness has focused on the role of metacognition.
a. Incorrect Divergent thinking is associated with creativity and can be found in both average and gifted individuals.
b. Incorrect In fact, superior response accuracy is more characteristic of gifted children than is response speed.
C. metacognition - CORRECT The research has consistently shown that gifted children are more aware of their cognitive processes and better at choosing and applying cognitive strategies and evaluating the effectiveness of their choice.
d. Incorrect The term parallel processing has several meanings. In the context of physiological psychology, it refers to the flow of information from the source to the recipient via more than one route. Parallel processing has not been linked to giftedness.
(Lifespan Development)

How well did you know this?
1
Not at all
2
3
4
5
Perfectly
186
Q
Down Syndrome is caused by:
A. a dominant gene.
B. two recessive genes.
C. a chromosomal abnormality.
D. an X-linked gene.
A

Down Syndrome is a chromosomal condition that is associated with mental retardation, specific facial features, and certain physical abnormalities (e.g., hypotonia, heart defects, and hearing impairment)
C. a chromosomal abnormality - CORRECT Down syndrome is most often due to a failure of the 21st pair of
chromosomes to split during meiosis, resulting in an extra chromosome 21.
(Lifespan Development)

How well did you know this?
1
Not at all
2
3
4
5
Perfectly
187
Q

According to Gerald Patterson and his colleagues (1992), which of the following is a key contributor to aggressive behavior in children?
A. the child’s intelligence
B. the child’s “basic personality”
C. the strength of the parent-child attachment
D. the parents’ childrearing behaviors

A

The social learning approach of Patterson and his colleagues focuses on the family factors that contribute to aggressiveness in children.
D. the parents’ childrearing behaviors - CORRECT Patterson et al.’s “coercive family interaction model” attributes aggression in children primarily to interactions between parents and their children, especially interactions involving the parents’ childrearing behaviors.
(Lifespan Development)

How well did you know this?
1
Not at all
2
3
4
5
Perfectly
188
Q
The studies have generally shown that children have the most difficulty adjusting to the remarriage of their parents when they are in \_\_\_\_\_ the time of remarriage.
A. preschool
B. elementary school
C. early adolescence
D. late adolescence
A

Hetherington (1993) and others have found that children’s adjustment to parental remarriage is affected by several factors including the child’s age.
C. early adolescence - CORRECT Hetherington concludes that individuals in early adolescence have the most difficulty adapting to the remarriage of a parent because the introduction of a stepparent at this time may exacerbate the normal adjustment problems associated with this age.
(Lifespan Development)

How well did you know this?
1
Not at all
2
3
4
5
Perfectly
189
Q

Kohlberg argued that there is a monotonic relationship between moral judgment and moral action and proposed that, as one moves from a lower to a higher stage of moral development:
A. the range of possible moral actions becomes narrower and the individual assumes greater responsibility for relating his/her judgments to actions.
B. the range of possible moral actions becomes broader and the individual assumes greater responsibility for relating his/her judgments to actions.
C. the range of possible moral actions remains the same but the individual assumes greater responsibility for relating his/her judgments to actions.
D. the range of possible moral actions becomes narrower but the individual’s sense of responsibility for relating his/her judgments to actions remains about the same..

A

Although Kohlberg believed that there is a positive relationship between moral judgment and moral action, he proposed that this relationship is much stronger at higher levels of moral development.
A. the range of possible moral actions becomes narrower and the individual assumes greater responsibility for relating his/her judgments to actions - CORRECT The stronger relationship between judgment and action at higher stages is attributable to two factors: first, the range of actions suggested by the individual’s judgment narrows; and, second, the individual experiences an increasing sense of responsibility to act.
(Lifespan Development)

How well did you know this?
1
Not at all
2
3
4
5
Perfectly
190
Q

Longitudinal studies of children who attended Head Start programs have found that:
A. early IQ gains and improvements in academic achievement persist into adolescence and early adulthood.
B. early IQ gains decline but improvements in academic achievement persist into adolescence.
C. early improvements in academic achievement decline by adolescence but initial IQ gains persist into adulthood.
D. attendees are indistinguishable from nonattendees on measures of IQ and academic achievement by adolescence.

A

Longitudinal research has found that the short- and long-term effects of Head Start and similar compensatory education programs differ.
B. early IQ gains decline but improvements in academic achievement persist into adolescence - CORRECT While early research on Head Start found that it has positive
effects on IQ, subsequent longitudinal studies found that IQ gains usually decline over time but that beneficial effects on academic achievement and success persist into adolescence and early adulthood. See, e.g., S. W. Barnett, Long-term cognitive and academic effects of early childhood education on children in poverty, Preventive Medicine, 27(2), 204-207, 1998.
(Lifespan Development)

How well did you know this?
1
Not at all
2
3
4
5
Perfectly
191
Q
A \_\_\_\_\_ is a restricted interval of time during which a particular kind of learning or experience must occur in order for development to proceed normally.
A. sensitive period
B. critical period 
C. refractory period
D. incubation period
A

Researchers interested in early development distinguish between critical and sensitive periods.
a. Incorrect A sensitive period is an interval of time when certain behaviors are more easily learned or acquired or when the organism is especially receptive to certain types of experience. Sensitive periods are more flexible than critical periods.
B. critical period - CORRECT A critical period is a limited period of time in which certain learning or experiences must occur in order for normal development to take place. There is evidence that, in humans, there is a critical period for language acquisition.
c. Incorrect The term refractory period is used to describe the interval of time after orgasm during which another orgasm is not possible.
d. Incorrect Incubation period has several meanings. One definition is the time interval between exposure to a disease and the development of symptoms of that disease.
(Lifespan Development)

How well did you know this?
1
Not at all
2
3
4
5
Perfectly
192
Q

Studies comparing children who are actively rejected or neglected by their peers have found that:
A. children who are actively rejected have worse outcomes.
B. children who are neglected have worse outcomes.
C. children who are actively rejected or neglected have similar outcomes.
D. children who are actively rejected or neglected have different short-term outcomes but similar long-term outcomes.

A

The research has shown that being actively rejected by peers is associated with worse outcomes than is being neglected by peers.
A. children who are actively rejected have worse outcomes - CORRECT Children who are actively rejected by peers exhibit a number of negative social behaviors, while those who are neglected are relatively well-adjusted. In addition, neglected children are more likely to experience an improvement in their peer status when they change social groups (e.g., change schools).
(Lifespan Development)

How well did you know this?
1
Not at all
2
3
4
5
Perfectly
193
Q
As defined by Piaget, a \_\_\_\_\_ circular reaction occurs when a baby explores the characteristics of familiar objects by deliberately using them in new ways.
A. reflexive
B. primary 
C. secondary
D. tertiary
A

According to Piaget, much of the cognitive development that occurs during the sensorimotor stage is the result of circular reactions, in which the child repeats or reproduces a pleasurable or interesting experience.
a. Incorrect Piaget considered reflexes to be the “building blocks” of cognitive development from birth to 1 month.
b. Incorrect Primary circular reactions predominate from ages 1 to 4 months. They involve simple motor habits that center around the baby’s own body (e.g., thumb sucking].
c. Incorrect Secondary circular reactions predominate from ages 4 to 8 months and are actions involving other people or objects.
D. tertiary - CORRECT Tertiary circular reactions predominate from ages 12 to 18 months and involve varying an original or usual action on an object to see what happens
(Lifespan Development)

How well did you know this?
1
Not at all
2
3
4
5
Perfectly
194
Q
The relationship of preadolescent siblings is best described as:
A. distant/aloof.
B. conflictual/hostile.
C. competitive and cooperative.
D. close and conflictual.
A

Even if you’re unfamiliar with the research, it makes sense that preadolescent relationships (or any relationship for that matter!) would involve a mixture of positive and negative aspects.
D. close and conflictual - CORRECT The research indicates that sibling relationships often involve
contradictions, but the combination of closeness and conflict seems to be particularly characteristic of siblings during preadolescence.
(Lifespan Development)

How well did you know this?
1
Not at all
2
3
4
5
Perfectly
195
Q
Sandra Bem’s (1984) theory of gender role identity development emphasizes which of the following?
A. psychosexual crises
B. rewards ancl punishments
C. early object relations
D. gender schemas
A

The exam occasionally includes a question that requires you to be recognize the name and work of a specific theorist or researcher; and this is an example of this type of question.
a. Incorrect Psychosexual crises are a focus of Freud’s theory of gender-role identity development.
b. Incorrect Rewards and punishments are not central to Bem’s theory.
c. Incorrect The role of early object relations in the development of a gender identity is of concern to feminist object relations theorists.
D. gender schemas - CORRECT Bem’s gender schema theory combines social learning theory and cognitive-developmental theory and focuses on the role of gender schemas, which are cognitive structures of masculinity and femininity that influence how the child interprets the world.
(Lifespan Development)

How well did you know this?
1
Not at all
2
3
4
5
Perfectly
196
Q

Research confirming which of the following provides support for the encoding specificity principle?
A. Elaborative rehearsal is more effective than maintenance rehearsal.
B. Retroactive interference has a greater impact on recent (versus remote) memories.
C. Recognition memory is ordinarily better than recall memory.
D. Most memory loss is consistent with the typical forgetting curve.

A

The encoding specificity principle (Tulving & Thomson, 1973) predicts that recall is improved when the same information is available at encoding and retrieval.
a. Incorrect Although this is true (elaborative rehearsal is more effective), it does not support the encoding specificity principle.
b. Incorrect Research indicating that retroactive interference has a greater impact on recent memories supports the consolidation theory of memory.
C. Recognition memory is ordinarily better than recall memory - CORRECT The research has consistently found that recognition memory is better than recall memory, and this has been attributed to the fact that recognition items provide more cues than do recall items (i.e., there is greater overlap between the information in the test items and the information contained in the memory trace).
d. Incorrect Research showing that most memory loss is consistent with the typical forgetting curve supports the consolidation theory of memory.
(Learning Theories & CBT)

How well did you know this?
1
Not at all
2
3
4
5
Perfectly
197
Q

According to Albert Ellis, maladaptive behavior is caused primarily by which of the following?
A. learned helplessness
B. boundary disturbances
C. a lack of response-contingent reinforcement
D. irrational thoughts and beliefs

A

Knowing that Ellis’s approach is classified as a type of cognitive-behavioral therapy would have helped you identify the correct response to this question.
D. irrational thoughts and beliefs - CORRECT The basic premise of Ellis’s rational-emotive behavioral therapy
(REBT) is that maladaptive behaviors and emotions are the result of irrational thoughts and beliefs. Thus, the primary goal of REBT is to replace the client’s
irrational thoughts and beliefs with more rational, adaptive ones.
(Learning Theories & CBT)

How well did you know this?
1
Not at all
2
3
4
5
Perfectly
198
Q
Studies comparing the recall ability of participants who were or were not allowed to sleep following memorization of a list of words have provided support for which of the following theories of forgetting?
A. trace decay theory
B. interference theory
C. motivated forgetting theory
D. retrieval failure theory
A

The research referred to in this question found that participants who slept remembered more than those who stayed awake.
a. Incorrect Trace decay theory predicts that forgetting is due to the passage of time. If this theory were correct, participants who did or did not sleep would have exhibited similar levels of recall.
B. interference theory - CORRECT The results of this research suggest that forgetting is not due simply to the passage of time but, instead, to the events that follow learning. In other words, memory loss is due to the interference of other events.
c. Incorrect Freud proposed that motivated forgetting is due to the repression of unconscious desires.
d. Incorrect Retrieval failure theory predicts that a loss of memory is not due to a failure to store information in long-term memory but, instead, to an inability to retrieve that information.
(Learning Theories & CBT)

How well did you know this?
1
Not at all
2
3
4
5
Perfectly
199
Q
Sensate focus was developed by Masters and Johnson (1970) as a technique for reducing performance anxiety related to sexual intercourse. If viewed as a behavioral technique, sensate focus is best described as a form of:
A. classical extinction.
B. operant extinction.
C. counterconditioning.
D. covert sensitization.
A

Sensate focus involves pairing pleasurable sensations with performance anxiety in order to reduce the latter.
C. counterconditioning - CORRECT Counterconditioning is based on the principles of classical conditioning and involves pairing two incompatible responses in order to
eliminate one of the responses.
(Learning Theories & CBT)

How well did you know this?
1
Not at all
2
3
4
5
Perfectly
200
Q
Which of the following types of biofeedback has been found to be most effective for the treatment of Raynaud’s disease?
A. EEG
B. GSR
C. thermal
D. respiration
A

Knowing that Raynaud’s disease involves a constriction of blood vessels in the extremities would have helped you identify the correct answer to this question.
a. Incorrect EEG (electroencephalogram) biofeedback provides information about brain activity and has not been found useful for treating Raynaud’s disease.
b. Incorrect GSR (galvanic skin response) biofeedback has been found useful for treating stress, anxiety, ADHD, and stuttering.
C. thermal - CORRECT Thermal (temperature) biofeedback has been found to be an effective treatment for a number of circulatory disorders, including Raynaud’s disease.
d. Incorrect Respiration biofeedback provides information on breathing rate and is used to treat anxiety, asthma, and hyperventilation.
(Learning Theories & CBT)

How well did you know this?
1
Not at all
2
3
4
5
Perfectly
201
Q
Edward Tolrnan is best known for his research on learning
A. discovery
B. observational
C. latent
D. insight
A

Tolman believed that learning could occur without being manifested in performance — i.e., that learning could be latent.
a. Incorrect Bruner was a major contributor to knowledge about discovery (inquiry) learning.
b. Incorrect Bandura conducted research on observational learning.
C. latent - CORRECT Tolman’s research with rats in mazes indicated that the rats had learned the maze (i.e., had formed “cognitive maps”) without exhibiting that
learning until they were reinforced for doing so. He referred to this as latent learning.
d. Incorrect Kohler conducted research on insight learning.
(Learning Theories & CBT)

How well did you know this?
1
Not at all
2
3
4
5
Perfectly
202
Q

Rehm’s (1977) self-control model of depression focuses on which the following?
A. beliefs about the self, the world, and the future
B. integration of thoughts, feelings, and actions
C. levels of knowledge, skills, and attitudes

A

Rehrn’s model of depression distinguishes between six processes that involve three areas of self-control.
D. processes related to self-monitoring, self-evaluation, and self- reinforcement - CORRECT According to Rehm, deficits in these three aspects of self-control increase a person’s vulnerability to depression. See, L. P. Rehm, A self-control model of depression, Behavior Therapy, 1977, 8, 787-804.
(Learning Theories & CBT)

How well did you know this?
1
Not at all
2
3
4
5
Perfectly
203
Q

When using protocol analysis, an individual is asked to:
A. “brainstorm” with other problem-solvers.
B. “think aloucl” while solving a complex problem.
C. identify alternative strategies for solving the problem.
D. construct a cognitive map of a problem.

A

Protocol analysis was developed by cognitive psychologists as a method for obtaining information about a person’s thoughts while solving problems.
B. “think aloucl” while solving a complex problem - CORRECT Protocol analysis involves having the individual “think aloud” while solving a problem while an observer records what the individual says. The observer then analyzes the individual’s verbalizations in order to identify the cognitive processes involved in solving the problem.
(Learning Theories & CBT)

How well did you know this?
1
Not at all
2
3
4
5
Perfectly
204
Q
A behavior that has been reinforced on which of the following schedules is most resistant to extinction?
A. fixed ratio
B. variable ratio
C. fixed interval
D. variable interval
A

The intermittent schedules of reinforcement listed in the answers are associated with different rates of acquisition and extinction.
a. Incorrect The acquisition of a response is rapid when using a fixed ratio schedule of reinforcement. However, because the relationship between responding and reinforcement is predictable, extinction is faster for this schedule than for the variable ratio schedule.
B. variable ratio - CORRECT A variable ratio schedule produces behaviors that are most resistant to extinction because, when using this schedule, the relationship
between responding and reinforcement is unpredictable.
c. Incorrect The rate of extinction is faster with this schedule than with the variable ratio schedule.
d. Incorrect The rate of extinction is faster with this schedule than with the variable ratio schedule.
(Learning Theories & CBT)

How well did you know this?
1
Not at all
2
3
4
5
Perfectly
205
Q
Whenever Bahir B., age 5, has tantrums in his kindergarten class, his teacher removes him from all opportunities for reinforcement by having him sit in a small empty room adjacent to the classroom for ten minutes. The teacher is using which of the following techniques to reduce Bahir's tantrums?
A. positive punishment
B. negative reinforcement
C. time-out 
D. overcorrection
A

Interventions based on operant conditioning increase behaviors through reinforcement or decrease behaviors through punishment or extinction.
a. Incorrect Positive punishment involves applying a stimulus following a behavior in order to decrease the occurrence of that behavior. In the situation described in this question, opportunities for reinforcement are being removed following the behavior.
b. Incorrect Negative reinforcement involves removing a stimulus following a behavior in order to increase the occurrence of that behavior. The goal in this case is to decrease a behavior.
C. time-out - CORRECT Time-out involves removing an individual from all possible sources of positive reinforcement for a brief period of time following the occurrence of a behavior in order to decrease that behavior. Time-out is usually classified as a type of negative punishment.
d. Incorrect Overcorrection involves having the individual engage in alternative behaviors after performing the undesirable behavior (i.e., returning the environment to its previous condition andfor practicing correct behaviors).
(Learning Theories & CBT)

How well did you know this?
1
Not at all
2
3
4
5
Perfectly
206
Q
Which of the following interventions does not use classical extinction to eliminate an undesirable anxiety response?
A. covert sensitization
B. graduated exposure
C. flooding
D. implosive therapy
A

When using classical extinction to eliminate an anxiety response, the individual is exposed to the anxiety-arousing stimulus (which is assumed to be a conditioned stimulus) in the absence of the unconditioned stimulus.
A. covert sensitization - CORRECT Covert sensitization uses counterconditioning to eliminate undesirable self-reinforcing behaviors (e.g., drug use, paraphilias).
b. Incorrect Graduated exposure involves exposing the individual to anxiety-arousing stimuli in a gradual way — i.e., by beginning with stimuli that produce the least amount of anxiety and then gradually progressing to stimuli that evoke higher levels of anxiety.
c. Incorrect Flooding, which is also known as in vivo exposure with response prevention, involves exposing the individual to fear-arousing stimuli while preventing him/her from making his/her usual avoidance response.
d. Incorrect Implosive therapy involves exposing the individual to fear-arousing stimuli in imagination.
(Learning Theories & CBT)

How well did you know this?
1
Not at all
2
3
4
5
Perfectly
207
Q
The use of shaping to establish a complex behavior depends on which of the following?
A. desensitization
B. positive reinforcement
C. alternate response training
D. higher-order conditioning
A

Shaping is based on operant conditioning and involves reinforcing successive approximations to the target (desired) behavior.
a. Incorrect The term desensitization has several meanings depending on the context, but it is not relevant to the process of shaping.
b. CORRECT When using shaping to establish a new behavior, responses that come closer and closer to the desired behavior are successively reinforced.
c. Incorrect Alternate response training involves training the individual to engage in a response that interferes with or is incompatible with the target (undesirable) response.
d. Incorrect Higher-order conditioning is a type of classical (not operant) conditioning and is not a component of shaping.
(Learning Theories & CBT)

How well did you know this?
1
Not at all
2
3
4
5
Perfectly
208
Q
Research on EMDR (eye movement desensitization and reprocessing) suggests that its effectiveness for alleviating the symptoms of PTSD is attributable to which of the following?
A. imaginal exposure
B. higher-orcler conditioning
C. relaxation training
D. fixed visual attention
A

EMDR was originally developed as a treatment for PTSD and combines rapid lateral eye movements (which purportedly trigger adaptive neurophysiological information-processing mechanisms) with exposure
and other techniques.
a. CORRECT Studies that used a dismantling strategy or compared EMDR to exposure-based treatments have generally confirmed that the essential component of EMDR is exposure to the feared stimulus in imagination rather than to lateral eye movements. See, e.g., S. Taylor et al., Comparative efficacy, speed, and adverse effects of three PTSD treatments: Exposure therapy, EM DR, and relaxation, Journal of Clinical and Consulting Psychology,
2003, 72, 330-338.
(Learning Theories & CBT)

How well did you know this?
1
Not at all
2
3
4
5
Perfectly
209
Q

When using punishment to modify behavior, habituation is most likely to be a problem when:
A. the intensity of the punishment is gradually increased over time.
B. the individual is given brief “vacations” from punishment.
C. different methods of punishment are alternately applied to the target behavior.
D. the use of punishment is restricted to one or two (versus numerous) behaviors.

A

In the context of punishment, habituation refers to a reduction in the effectiveness of punishment.
A. the intensity of the punishment is gradually increased over time. - CORRECT Gradually increasing the intensity of punishment over time (i.e., beginning with a weak Form of punishment and then gradually increasing its intensity) increases the likelihood that habituation will occur. In contrast, the methods listed in answers b, c, and d are useful For reducing habituation. See, e.g., D. C. Lerman and C. M. Vorndran, On the status of knowledge for using punishment: Implications for treating behavior disorders, Journal of Applied Behavioral Anaiysis, 2002, 35, 431-464.
(Learning Theories & CBT)

How well did you know this?
1
Not at all
2
3
4
5
Perfectly
210
Q

Bandura’s social learning theory posits that:
A. people can acquire behaviors without actually performing them
B. reinforcement is unnecessary for learning to occur.
C. learning reflects a series of internal S-R connections.
D. people can passively acquire behavior through observation.

A

Bandura’s social learning theory is distinguished from behavioral theories by its recognition of the role of cognitive (covert) factors in learning.
A. people can acquire behaviors without actually performing them - CORRECT Bandura distinguished between learning and performance and proposed that people can learn (acquire) behaviors without actually performing them by observing others perform those behaviors.
b. Incorrect Although Bandura downplays the importance of external reinforcement, he does not entirely abandon the idea that reinforcement enhances the acquisition and performance of behaviors.
c. Incorrect This doesn’t accurately describe Bandura’s theory.
d. Incorrect Bandura describes learning as an active, not passive, process.
(Learning Theories & CBT)

How well did you know this?
1
Not at all
2
3
4
5
Perfectly
211
Q
A functional analysis is conducted in order to identify \_\_\_\_\_ variables
A. controlling
B. moderator 
C. dependent
D. organisrnic
A

The term “functional analysis” is associated with behavioral assessment and involves identifying the antecedents and consequences of the target behavior.
a. CORRECT Functional analysis is used to identify the Factors (antecedents and consequences) that are controlling the target behavior so that those Factors can
be altered.
(Learning Theories & CBT)

How well did you know this?
1
Not at all
2
3
4
5
Perfectly
212
Q
A behavioral psychologist warns that "a single exception can reinstate the target response and may even maintain it for an extended period of time." Apparently, the psychologist is referring to the use of which of the following operant techniques to eliminate an undesirable response?
A. extinction 
B. shaping
C. negative reinforcement
D. chaining
A

The wording of this question implies that the correct answer must be a technique that is used to eliminate a response.
A. extinction - CORRECT Intermittent reinforcement produces behaviors that are very resistant to extinction. Therefore, when using extinction to eliminate a behavior, all reinforcement for that behavior must be consistently withheld. If the behavior is reinforced even once, the behavior may be reinstated and the extinction process will have to begin again.
b. Incorrect Shaping is used to establish a behavior and involves reinforcing successive approximations to the desired behavior.
c. Incorrect Negative reinforcement is also used to increase a behavior and involves removing an undesirable consequence when the behavior is performed.
d. Incorrect Chaining is used to establish complex behaviors.
(Learning Theories & CBT)

How well did you know this?
1
Not at all
2
3
4
5
Perfectly
213
Q

Operant extinction would NOT be the treatment-of-choice when:
A. an alternative behavior cannot be identified.
B. the behavior has been reinforced on a continuous schedule.
C. a temporary increase in the behavior cannot be tolerated.
D. the baseline levels of the behavior cannot be established.

A

Extinction is accomplished by withholding all reinforcement from a previously reinforced behavior.
a. Incorrect It would be helpful to be simultaneously reinforcing an alternative behavior, but the absence of an alternative behavior would not rule out the use of extinction.
b. Incorrect Behaviors that have been reinforced on a continuous schedule are easier to extinguish than those that have been reinforced on an intermittent schedule.
C. a temporary increase in the behavior cannot be tolerated - CORRECT Extinction often produces an initial “extinction burst” (temporary increase in the target behavior). Consequently, it would be contraindicated when a temporary increase in the behavior cannot be tolerated.
d. Incorrect It would be useful to have a baseline reading to determine the effects of the extinction procedure, but the absence of a baseline measure would not rule out the use of extinction.
(Learning Theories & CBT)

How well did you know this?
1
Not at all
2
3
4
5
Perfectly
214
Q

When using reinforcement to establish a new behavior, the most effective approach is to begin with a continuous schedule of reinforcement and then, when the behavior has reached the desired level, to switch to an intermittent
schedule. The process of switching from a continuous to an intermittent schedule is referred to as:
A. fading.
B. desensitizing.
C. thinning.
D. blocking.

A

This question is asking for the term that is used to describe reducing the proportion of reinforcement.
a. Incorrect Fading refers to the gradual removal of a prompt.
b. Incorrect Desensitizing is not the term used to describe reducing the proportion of reinforcement.
c. CORRECT This question provides an example of the technique known as thinning (or schedule thinning).
d. Incorrect Blocking is a classical conditioning term and occurs when two different conditioned (neutral) stimuli are paired with the unconditioned stimulus.
(Learning Theories & CBT)

How well did you know this?
1
Not at all
2
3
4
5
Perfectly
215
Q

The assumption underlying the use of implosive therapy for treating a phobia is that:
A. extinction of a conditioned response occurs when positive reinforcement for that response is repeatedly withheld.
B. extinction of a conditioned response occurs when the CS is repeatedly presented without the US.
C. consistent application of an aversive stimulus following a response results in elimination of that response.
D. pairing a maladaptive response with an alternative adaptive response eliminates the maladaptive response

A

Implosive therapy uses classical extinction to eliminate an undesirable anxiety response.
B. extinction of a conditioned response occurs when the CS is repeatedly presented without the US - CORRECT Implosive therapy involves having the client repeatedly imagine a feared stimulus (CS) in the absence of the stimulus (US) that initially evoked the fear in order to eliminate the conditioned Fear response.
(Learning Theories & CBT)

How well did you know this?
1
Not at all
2
3
4
5
Perfectly
216
Q

Which of the following is an example of anterograde amnesia?
A. As the result of a head injury he received in a car accident, a man cannot remember where he was going before the accident occurred.
B. A woman cannot remember how she got to the hospital or other events that occurred during the 24 hours after she was sexually assaulted.
C. A high school junior who learned Spanish her freshman year is having trouble learning French because she keeps substituting Spanish words for French ones.
D. A college student can remember information related to an important event but cannot remember how he acquired that information.

A

Anterograde amnesia involves a loss of memory for events that occur after the event that caused the memory loss.
a. Incorrect A loss of memory for events that occurred prior to the trauma that caused the memory impairment is referred to as retrograde amnesia.
B. A woman cannot remember how she got to the hospital or other events that occurred during the 24 hours after she was sexually assaulted - CORRECT This answer provides an example of anterograde amnesia — i.e., the woman has no memory for events that occurred after the assault.
c. Incorrect This is an example of proactive interference.
d. Incorrect This is an example of source amnesia.
(Learning Theories & CBT)

How well did you know this?
1
Not at all
2
3
4
5
Perfectly
217
Q

The technique known as stress inoculation consists of three overlapping phases. These are:
A. preparation, action, and reflection.
B. engagement/motivation, behavior change, and generalization.
C. initiation of treatment, primary treatment/rehabilitation, and aftercare
D. conceptualization/education, skill acquisition and rehearsal, and application.

A

Stress inoculation (Meichenbaum & Jaremko, 1982) was designed to help people deal with stressful events by increasing their coping skills.
D. conceptualization/education, skill acquisition and rehearsal, and application - CORRECT Stress inoculation is a cognitive-behavioral technique that consists of the three phases listed in this response.
(Learning Theories & CBT)

How well did you know this?
1
Not at all
2
3
4
5
Perfectly
218
Q
Which of the following is responsible for your ability to remember to call your friend next week in order to wish her a “happy birthday”?
A. prospective memory
B. meta-memory
C. secondary memory
D. working memory
A

If you’re not familiar with the type of memory being asked about in this question, you may have been able to identify the correct answer through the process of elimination.
a. CORRECT Prospective memory refers to the ability to “remember to remember” — i.e., to remember to do something in the future.
b. Incorrect Meta-memory refers to awareness and understanding of one’s own memory processes. It includes the ability to know what memory strategies to
use for specific tasks.
c. Incorrect Secondary memory is used by some authors as a synonym for long-term memory in general, while others define it less broadly as a synonym for recent long-term memory.
d. Incorrect Working memory is the aspect of short-term memory that is used to perform certain mental tasks (e.g., to solve a simple arithmetic problem “in your head”).
(Learning Theories & CBT)

How well did you know this?
1
Not at all
2
3
4
5
Perfectly
219
Q

The various intermittent schedules of reinforcement are associated with different response patterns. Which of the following schedules and patterns is incorrectly matched?
A. variable ratio: stable rates of responding with high resistance to extinction
B. variable interval: smooth, stable rates of responding with moderate to low resistance to extinction
C. fixed ratio: post-reinforcement response pauses with moderately high resistance to extinction
D. fixed interval: smooth but slow rates of responding with moderate resistance to extinction

A

The specific intermittent schedule of reinforcement used influences how quickly an operant behavior is acquired and how well it is maintained. Note that this question is asking for the schedule that is incorrectly matched with its description.
a. Incorrect When using a variable ratio schedule, reinforcement is provided after a variable number of responses. This schedule produces stable rates of
responding and responses that are highly resistant to extinction.
b. Incorrect Use of a variable interval schedule involves providing reinforcement after random or variable time intervals. It produces stable rates of responding and low to moderate resistance to extinction.
c. Incorrect Use of a fixed ratio schedule entails consistently providing reinforcement after a fixed number of correct responses. It produces a “scalloped” response pattern characterized by brief pauses in responding
following reinforcement and a moderately high resistance to extinction.
D. fixed interval: smooth but slow rates of responding with moderate resistance to extinction - CORRECT When using a fixed interval schedule, reinforcement is provided after a fixed interval of time as long as the response occurred at least once during that interval. It produces a “scalloped” (not smooth] response pattern characterized by brief pauses in responding following reinforcement and low resistance to extinction.
(Learning Theories & CBT)

How well did you know this?
1
Not at all
2
3
4
5
Perfectly
220
Q

Betsy B., age 9, is given a quarter by her parents each time she practices her violin and another quarter each time she completes her homework. If Betsy’s parents stop paying her for completing her homework, they are most
likely to find that Betsy:
A. spends less time doing homework and less time practicing the violin.
B. spends less time doing homework but more time practicing the violin.
C. spends less time doing homework but about the same amount of time practicing the violin.
D. spends about the same amount of time doing homework but more time practicing the violin.

A

Betsy has received reinforcement for two different behaviors, but the reinforcement for one behavior will be terminated.
B. spends less time doing homework but more time practicing the violin - CORRECT This question is asking about “positive behavioral contrast,” which occurs when reinforcement for one behavior is withdrawn while reinforcement for another behavior is continued. In this situation, the behavior that is no longer reinforced will decrease, while the behavior that continues to be reinforced will increase.
(Learning Theories & CBT)

How well did you know this?
1
Not at all
2
3
4
5
Perfectly
221
Q
According to the levels of processing model of memory, words and other information may be encoded at different levels, with \_\_\_\_\_ processing being the deepest level.
A. orthographic
B. phonemic
C. semantic
D. perceptual
A

Craik and Lockhart’s (1972) levels of processing model proposes that the level (depth) of processing of information affects how well that information is retained.
C. semantic - CORRECT Craik and Lockhart distinguished between shallow and deep processing. According to their theory, shallow processing (processing words on
the basis of their phonemic or orthographic characteristics) is less effective than deep processing (processing words at the semantic or meaning-based level).
d. Incorrect Perceptual processing includes orthographic and phonemic processing, which are shallow levels.
(Learning Theories & CBT)

How well did you know this?
1
Not at all
2
3
4
5
Perfectly
222
Q
Within the Pavlovian paradigm, experimental neurosis is the result of:
A. unavoidable aversive stimuli.
B. difficult discriminations.
C. accidental conditioning.
D. higher-order conditioning.
A

Experimental neurosis was exhibited by dogs in Pavlov’s research when they were presented with stimuli very similar to the original CS.
B. difficult discriminations - CORRECT When required to make difficult discriminations between the CS and stimuli very similar to it, the normally docile animals in Pavlov’s studies became very agitated. Pavlov referred to this as experimental neurosis.
(Learning Theories & CBT)

How well did you know this?
1
Not at all
2
3
4
5
Perfectly
223
Q
Research using the dismantling strategy has found that the effectiveness of systematic desensitization as a treatment for anxiety is attributable to which of the following?
A. escape conditioning
B. extinction 
C. satiation
D. higher-order conditioning
A

Systematic desensitization was developed as an application of reciprocal inhibition (counterconditioning). However, research using the dismantling strategy suggests that reciprocal inhibition is not the essential component.
B. extinction - CORRECT The results of research using the dismantling strategy has led researchers to conclude that repeated exposure to feared stimuli leads to extinction of the undesirable anxiety response and is responsible For the benefits of systematic desensitization.
(Learning Theories & CBT)

How well did you know this?
1
Not at all
2
3
4
5
Perfectly
224
Q
The law of effect was proposed by:
A. Tolman.
B. Thorndike.
C. Watson.
D. Bandura.
A

According to the law of effect, a response that is followed by a “satisfying state of affairs” is more likely to be repeated.
a. Incorrect Tolman is associated with Latent learning/cognitive maps.
B. Thorndike - CORRECT Thorndike’s research established a connection between the consequences of a behavior and the likelihood that the behavior will be
repeated, and he referred to this connection as the law of effect.
c. Incorrect Watson used Pavlovian (classical) conditioning to explain the development of fear responses.
d. Incorrect Bandura is most associated with social (observational) learning theory.
(Learning Theories & CBT)

How well did you know this?
1
Not at all
2
3
4
5
Perfectly
225
Q
Prior to taking an important test, Delwood D. feels very anxious and, on the basis of this anxiety, concludes that he's not prepared for the test and will fail it. Delwood is exhibiting which of the following?
A. polarized thinking
B. demanding expectations
C. catastrophizing
D. emotional reasoning
A

Delwood is exhibiting a cognitive distortion that involves making a judgment on the basis of his emotions.
a. Incorrect Polarized (dichotomous) thinking involves seeing things as either “black or white.”
b. Incorrect Demanding expectations are beliefs that there are rules or laws that must always be obeyed.
c. Incorrect Catastrophizing involves believing that an event will have catastrophic consequences when, in fact, it’s more likely to be only irritating or inconvenient. Although Delwood is, in a sense, catastrophizing, answer d is a better response because his conclusions about the test are derived from his emotional state.
D. emotional reasoning - CORRECT Emotional reasoning is what it sounds like — i.e., concluding that what you feel must be the truth.
(Learning Theories & CBT)

How well did you know this?
1
Not at all
2
3
4
5
Perfectly
226
Q

Research on the serial position effect provides support for the distinction between:
A. iconic and echoic memory.
B. distributed and parallel processing.
C. short- and long-term memory.
D. proactive and retroactive interference.

A

The serial position effect refers to the tendency to recall items in the beginning and end of a list better than items in the middle of the list, especially when recall occurs immediately after rehearsing the list.
C. short-and long-term memory - CORRECT One explanation for the serial position effect is that items in the beginning of the list have been stored in long-term memory, while those in the end of the list are still present in short-term memory.
(Learning Theories & CBT)

How well did you know this?
1
Not at all
2
3
4
5
Perfectly
227
Q
The highest levels of learning and performance are usually associated with:
A. low levels of arousal.
B. moderate levels of arousal. 
C. high levels of arousal.
D. variability in levels of arousal.
A

Even if you’re not familiar with the Yerkes-Dodson law (which proposes that learning and performance are curvilinearly related to arousal), you may have been able to identify the correct answer on the basis of common sense - i.e., most people perform best when their level of
arousal is somewhere between “none” and “a lot.”
a. Incorrect A low level of arousal can lead to boredom which can negatively affect learning and performance.
B. moderate levels of arousal - CORRECT Moderate levels of arousal are associated with the highest levels of
learning and performance. This is particularly true when moderate levels of arousal are combined with moderate task difficulty.
c. Incorrect A high level of arousal can lead to stress and fatigue, which can negatively affect learning and performance.
d. Incorrect Varying levels of arousal would not be optimal for learning and performance.
(Learning Theories & CBT)

How well did you know this?
1
Not at all
2
3
4
5
Perfectly
228
Q

In Pavlov’s research, experimental animals began to salivate in response to a tone after the tone had been presented repeatedly with meat powder. In this
situation, the tone was a(n) _____ stimulus.
A. conditioned
B. second-order
C. unconditioned
D. discriminative

A

In classical conditioning, a conditioned stimulus (CS) elicits a conditioned response (CR) as a result of its pairing with an unconditioned stimulus (US) that naturally elicits the unconditioned response (UR).
A. conditioned - CORRECT In Pavlov’s studies, the tone was a conditioned stimulus (CS) - i.e., it elicited salivation only as the result of being paired with an unconditioned
stimulus (meat powder) that naturally elicited salivation.
b. Incorrect In second-order conditioning (also known as higher-order conditioning), a previously conditioned stimulus is paired with a neutral stimulus so that presentation of the neutral stimulus alone elicits a conditioned response. The study described in this question is not an example of second-order conditioning.
c. Incorrect The unconditioned stimulus (US) elicits a response without conditioning. In Pavlov’s studies, the unconditioned stimulus was the meat powder.
d. Incorrect A discriminative stimulus is a cue that signals that a reinforcer will be provided when the individual performs a specific behavior.
(Learning Theories & CBT)

How well did you know this?
1
Not at all
2
3
4
5
Perfectly
229
Q

Masaaki M. has not yet learned to distinguish between a cow and a horse. When he sees a cow, he sometimes says “horse” and he sometimes calls a horse a “cow.” To help him learn the difference between these two animals
Masaaki’s parents make the sounds “moo” or “neigh,” as appropriate, whenever the boy encounters a cow or horse. Masaaki’s parents are using which of the following techniques?
A. shaping
B. prompting
C. chaining
D. thinning

A

Masaaki’s parents are providing the child with prompts.
a. Incorrect Shaping is used to help an individual acquire a complex behavior and involves reinforcing successive approximations to the target behavior.
B. prompting - CORRECT Prompting involves providing an individual with appropriate cues to help elicit the desired behavior.
c. Incorrect When using chaining to establish a complex behavior consisting of a sequence of responses, each response acts as reinforcement for the previous response and as a discriminative stimulus for the next response.
d. Incorrect Thinning is the process of gradually reducing the frequency of reinforcement (e.g., of switching from a continuous to an intermittent schedule of reinforcement).
(Learning Theories & CBT)

How well did you know this?
1
Not at all
2
3
4
5
Perfectly
230
Q

From the perspective of Beck’s cognitive-behavioral therapy, the modification of maladaptive cognitions requires which of the following?
A. a balance of intrinsic and extrinsic reinforcement
B. the experience of affective arousal
C. an adequate level of self-efficacy
D. a sense of personal responsibility

A

This is a difficult question unless you are very familiar with Beck’s approach.
B. the experience of affective arousal - CORRECT According to Beck, the modification of dysfunctional cognitions “can only occur if the patient is engaged in the problematic situation and experiences affective arousal” (A. T. Beck and M. Weishaar, Cognitive therapy, in A.
Freeman, et al., eds., Comprehensive Handbook of Cognitfve Therapy, New York, Plenum Press, 1989, p. 29).
(Learning Theories & CBT)

How well did you know this?
1
Not at all
2
3
4
5
Perfectly
231
Q
Samuel S., age 43, says, "Even though I've just been given a bonus at work, I feel like I'm about to be fired.” As defined by Aaron Beck, Samuel is exhibiting which of the following cognitive distortions?
A. mustabatory thinking
B. polarized thinking 
C. personalization
D. arbitrary inference
A

The phenomena listed in the answers to this question are all cognitive distortions that have been identified as contributors to maladaptive behavior.
a. Incorrect “Mustabatory thinking” is a term used by Ellis and refers to the belief that certain conditions must be met - e.g., “I must be liked by everyone; if not, I’m a terrible person.”
b. Incorrect Polarized (dichotomous) thinking involves seeing things as either “black or white.”
c. Incorrect Pesonalization involves mistakenly viewing oneself as the source of an event that actually had another cause.
D. arbitrary inference - CORRECT As defined by Beck, arbitrary inference involves drawing a conclusion that is made in the absence of supporting evidence or in the presence of contradictory evidence.
(Learning Theories & CBT)

How well did you know this?
1
Not at all
2
3
4
5
Perfectly
232
Q
From a behavioral perspective, phobic reactions to benign objects or events are acquired when those objects or events become associated with a stimulus that naturally produces a fear response. In other words, phobic reactions are due to:
A. negative reinforcement. 
B. negative punishment.
C. stimulus generalization.
D. classical conditioning.
A

There are several explanations for the phobic reactions. The explanation provided in this question is consistent with the Pavlovian view.
D. classical conditioning - CORRECT According to the Pavlovian (classical conditioning) explanation for phobias, a Fear response to a previously neutral object or event is a classically conditioned response in which Fear is attached to the neutral stimulus (the CS) through its pairing with a stimulus that naturally elicits Fear (the US).
(Learning Theories & CBT)

How well did you know this?
1
Not at all
2
3
4
5
Perfectly
233
Q

Images and sounds are maintained in sensory memory for:
A. an indefinite period of time.
B. minutes to days, depending on the depth of encoding.
C. about 60 to 90 seconds.
D. less than 5 seconds.

A

Sensory memory provides very brief storage of incoming sensory input.
D. less than 5 seconds - CORRECT Sensory memory includes a separate store for each sense, and the
duration of sensory memory varies somewhat, depending on the sense. For iconic (visual) memories, the duration is about .5 to 1.0 seconds; for echoic (auditory) memories, the duration is up to 4 or 5 seconds.
(Learning Theories & CBT)

How well did you know this?
1
Not at all
2
3
4
5
Perfectly
234
Q
In a research study, Variable B is paired with Variable A so that, eventually, the response automatically produced by Variable A is also produced by Variable B. Then, Variable C is paired with Variable B so that it produces the same response. Based on this information, you can conclude that this study was designed to investigate which of the following?
A. pseudoconditioning
B. spontaneous recovery
C. two-factor learning
D. higher-order conditioning
A

The term “paired with” suggests that this study is investigating some form of classical conditioning - i.e., in classical conditioning, presentation of a conditioned stimulus is paired with presentation of an unconditioned stimulus so that the conditioned stimulus produces a
response that is similar to the one produced by the unconditioned stimulus.
a. Incorrect In classical conditioning, pseudoconditioning occurs when (1) the neutral stimulus is accidentally paired with the US and, as a result, produces a response similar to the UR or (2) repeated presentation of the US increases the likelihood that the individual will respond to a novel stimulus (not just the US or CS) with a response similar to the UR, especially in the same context in which
the US was presented
b. Incorrect Spontaneous recovery refers to the return of a classically conditioned response or an operant response following extinction.
c. Incorrect Two-factor learning (e.g., avoidance conditioning) combines classical and operant condition, which is not occurring in the study described in this
question.
D. higher-order conditioning - CORRECT In higher-order conditioning, the initial conditioned stimulus (CS) is
treated as an unconditioned stimulus (US) and paired with a neutral stimulus so that the neutral stimulus also produces a conditioned response. This question describes the procedure used in higher-order conditioning.
(Learning Theories & CBT)

How well did you know this?
1
Not at all
2
3
4
5
Perfectly
235
Q
When using aversive counterconditioning as a treatment for cocaine abuse, cocaine would be the:
A. conditioned stimulus.
B. unconditioned stimulus. 
C. positive reinforcer.
D. negative reinforcer.
A

Aversive counterconditioning involves pairing a stimulus (US) that produces an aversive response (UR) with the undesirable stimulus (CS) so that the undesirable stimulus alone elicits an aversive response (CR).
A. conditioned stimulus - CORRECT In the example given in this question, cocaine would be the CS while the US would be electric shock or other aversive stimulus.
(Learning Theories & CBT)

How well did you know this?
1
Not at all
2
3
4
5
Perfectly
236
Q
Rehrn’s self-control therapy begins with self-monitoring, which involves maintaining a daily record of:
A. positive activities.
B. autornatic thoughts.
C. distressful thoughts.
D. antecedent events.
A

One of the assumptions underlying Rehm’s approach is that depressed individuals pay more attention to negative events than to positive ones
A. positive activities - CORRECT To alter a depressed patient’s focus, therapy begins with a task that helps the patient shift his/her attention to positive events.
b. Incorrect Keeping a daily record of automatic thoughts is associated with Beck’s cognitive-behavioral therapy.
(Learning Theories & CBT)

How well did you know this?
1
Not at all
2
3
4
5
Perfectly
237
Q

Which of the following is an example of semantic memory?
A. remembering that you have a doctor’s appointment next Monday.
B. remembering what you did on your last birthday.
C. remembering the definition of “semantic memory.”
D. remembering how to ride a bicycle after not doing so for several years

A

Semantic memory refers to memory for facts, concepts, and other types of information that are independent of time and context.
a. Incorrect This is an example of prospective memory, which is usually classified as a separate aspect of long-term memory or as part of episodic memory.
b. Incorrect This is an example of episodic memory.
C. remembering the definition of “semantic memory.” - CORRECT Definitions are stored in semantic memory.
d. Incorrect This is an example of procedural memory.
(Learning Theories & CBT)

How well did you know this?
1
Not at all
2
3
4
5
Perfectly
238
Q
"If a child's whining and crying secures his parents’ attention, the child is likely to whine and cry again." Which of the following individuals is most likely to agree with this statement?
A. Bandura
B. Ebbinghaus
C. Watson
D. Thorndike
A

The statement in this question asserts that a behavior followed by a positive (satisfying) consequence will increase. This is a premise of operant conditioning and is referred to as positive reinforcement.
a. Incorrect Although Bandura recognizes the role of reinforcement in the acquisition and performance of behavior, his social learning theory proposes that people can acquire behaviors simply by observing others perform those behaviors. In addition, Bandura places more emphasis on the role of internal (versus external) reinforcement.
b. Incorrect Ebbingha us is associated with research on memory and forgetting.
c. Incorrect Watson is associated with classical conditioning, not operant conditioning.
D. Thorndike - CORRECT The first name that probably came to mind when you read this question was Skinner, but, unfortunately, his name is not one of the answers.
However, Thorndike was a predecessor of Skinner and proposed the law of effect, which predicts that behaviors that are followed by satisfying consequences will tend to occur again.
(Learning Theories & CBT)

How well did you know this?
1
Not at all
2
3
4
5
Perfectly
239
Q

From the perspective of learning theory, phobic anxiety is:
A. elicited by unconditioned (aversive) stimuli or stimuli that have been associated with them.
B. elicited by secondary punishing, frustrating, or otherwise aversive stimuli.
C. the result of a low level of response-contingent reinforcement.
D. the result of “accidental” pairings between dysfunctional levels of internal arousal and reinforcing stimuli.

A

Knowing that learning theorists consider anxiety to be the result of classical conditioning would have helped you identify the correct response to this question.
A. elicited by unconditioned (aversive) stimuli or stimuli that have been associated with them - CORRECT The classical conditioning explanation for anxiety is that it is
produced by exposure to an unconditioned stimulus that naturally elicits anxiety or, perhaps more often, by exposure to a conditioned stimulus that has been
paired with an unconditioned stimulus that naturally elicits anxiety.
b. Incorrect The term “secondary” is ambiguous in this response. Also, as noted above, anxiety can be due to aversive (unconditioned) stimuli or to conditioned
stimuli that have been associated with aversive stimuli.
c. Incorrect A low level of response-contingent reinforcement has been linked to depression, not to phobic anxiety.
d. Incorrect This answer does not accurately describe the learning theory view of phobic anxiety.
(Learning Theories & CBT)

How well did you know this?
1
Not at all
2
3
4
5
Perfectly
240
Q

When using the Premack Principle to modify a behavior, the reinforcer is:
A. applied intermittently.
B. a generalized conditioned reinforcer.
C. a behavior that occurs frequently.
D. a stimulus that naturally elicits the desired behavior.

A

The Premack Principle is a type of positive reinforcement.
C. a behavior that occurs frequently - CORRECT When using the Premack Principle, a high-frequency behavior is
used as the reinforcer for a low-frequency behavior. The Premack Principle is also known as “Grandma’s Rule” - e.g., “you can go out and play once you’ve cleaned your room.”
(Learning Theories & CBT)

How well did you know this?
1
Not at all
2
3
4
5
Perfectly
241
Q
Elmo E., age 5, has learned that, if he has a tantrum whenever his parents want him to eat food he doesn't like at mealtime, his parents will let him eat something else instead in order to get Elmo to stop crying. In this situation, the parents’ behavior (letting Elmo eat something other than the disliked food) is being maintained by which of the following?
A. escape conditioning 
B. avoidance conditioning
C. stimulus discrimination
D. stimulus generalization
A

In this situation, the parents’ behavior (letting Elmo eat something other than the disliked food) results in termination of Elmo’s tantrum.
A. escape conditioning - CORRECT Escape conditioning occurs when a person’s behavior is maintained
because it allows the person to escape an undesirable stimulus. Escape conditioning is a type of negative reinforcement.
b. Incorrect In avoidance conditioning, a cue signals that the undesirable stimulus is about to begin. In this situation, the individual can avoid the stimulus
completely by performing the appropriate behavior as soon as he/she perceives the cue.
c. Incorrect In stimulus discrimination, a cue signals whether a US (classical conditioning) or a reinforcer (operant conditioning) will be delivered or provided
d. Incorrect In stimulus generalization, the individual responds with a CR to stimuli that are similar to the original CS.
(Learning Theories & CBT)

How well did you know this?
1
Not at all
2
3
4
5
Perfectly
242
Q
In a study on learning and memory, participants in the experimental group learned two lists of nonsense syllables (first List A, then List B) and were then asked to recall List A. Participants in the control group also learned List A but, instead of learning List B, were asked to count backward from 10 numerous times before being asked to recall List A. Results of the study indicated that participants in the control group recalled more words from List A than did participants in the experimental group. These results are best explained by which of the following?
A. proactive interference
B. retroactive interference 
C. trace decay
D. serial position effect
A

The purpose of the study described in this question was apparently to investigate how subsequent experience affects prior learning.
a. Incorrect Proactive interference occurs when prior learning interferes with subsequent (new) learning.
B. retroactive interference - CORRECT Retroactive interference occurs when subsequent learning interferes with prior learning and is most likely to occur when the subsequent and prior learning are similar in terms of content. In the study described in this question, participants in the experimental group learned a second list of nonsense syllables after learning the first list and were then asked to recall the first list. In contrast, participants in the control group engaged in an unrelated task after learning the original list to ensure that they did not continue to
rehearse the list prior to testing. Because control group participants recalled more words from List A than did participants in the experimental group, the results confirmed that retroactive interference contributes to forgetting.
c. Incorrect Trace decay theory predicts that forgetting is due to the passage of time and, therefore, that participants in the two groups would have had similar
levels of recall.
d. Incorrect The serial position effect does not explain the results of this study. It refers to the tendency to recall the first and last items in a list better than items in the middle of the list.
(Learning Theories & CBT)

How well did you know this?
1
Not at all
2
3
4
5
Perfectly
243
Q
Increasing knowledge and understanding of our own thinking can, according to some authorities, improve our ability to reason effectively. In other words improving our \_\_\_\_\_ can have beneficial effects on our ability to think clearly, solve problems, and make decisions.
A. semantic memory
B. syntactic knowledge
C. meta-cognition 
D. introspection
A

This question is asking about the term that is used to describe “thinking about thinking.”
a. Incorrect Semantic memory is memory for Facts, concepts, and other knowledge.
b. Incorrect Syntactic knowledge is knowledge about how sounds combine to Form language.
C. meta-cognition - CORRECT Meta-cognition refers to our knowledge or awareness of our own cognitive processes.
d. Incorrect Although meta-cognition involves introspection (self-examination and self-reflection), introspection is a broader term than meta-cognition and,
therefore, not the best answer to this question since it is asking specifically about self-understanding related to thinking.
(Learning Theories & CBT)

How well did you know this?
1
Not at all
2
3
4
5
Perfectly
244
Q

According to Wolfgang Kohler, learning:
A. is largely the result of trial-and-error.
B. is the result of repeated exposure to the same or similar conditions.
C. involves using effective encoding and retrieval cues.
D. involves discovering relationships between elements of the problem

A

Kohler is associated with insight learning, which is a sudden understanding of relationships among a set of elements relevant to a problem that leads to recognition of the solution to the problem.
a. Incorrect Kohler did not assume that learning is the result of trial-and-error.
b. Incorrect This isn’t compatible with Kohler’s theory.
c. Incorrect This doesn’t describe insight learning.
D. involves discovering relationships between elements of the problem - CORRECT Insight learning occurs when there are relationships in the problem to be discovered and the discovery of those relationships is within the organism’s cognitive capacity.
(Learning Theories & CBT)

How well did you know this?
1
Not at all
2
3
4
5
Perfectly
245
Q

Socratic dialogue and collaborative empiricism are most associated with which of the following therapeutic approaches?
A. Kelly’s personal construct therapy
B. Glasser’s reality therapy
C. Ellis’s rational-emotive behavioral therapy
D. Becks cognitive-behavioral therapy

A

Only one of the approaches listed in the answers conceptualizes therapy as involving collaborative empiricism and uses Socratic dialogue as a method for encouraging collaboration between the therapist and client.
D. Becks cognitive-behavioral therapy - CORRECT Beck’s cognitive-behavioral therapy is referred to as collaborative
empiricism because it is founded on a collaborative therapist-client relationship that involves testing hypotheses about the client’s beliefs and assumptions.
Socratic dialogue is an important aspect of cognitive-behavioral therapy and is used to help the client question and challenge his/her underlying beliefs, obtain
disconfirming evidence for those beliefs, and identify alternative beliefs.
(Learning Theories & CBT)

How well did you know this?
1
Not at all
2
3
4
5
Perfectly
246
Q

When conducting a study to investigate the phenomenon known as “blocking,” you will:
A. present the CS prior to the US; then, once the CR is established, present the US prior to the CS.
B. present the CS prior to the US; then, once the CR is established, present the CS prior to a second US.
C. present the CS prior to the US; then, once the CR is established, present the original CS simultaneously with a second CS prior to the US.
D. present the CS prior to the US then, once the CR is established, l present a second CS prior to the original CS.

A

Blocking occurs when two different conditioned stimuli are paired with the unconditioned stimulus.
C. present the CS prior to the US; then, once the CR is established, present the original CS simultaneously with a second CS prior to the US - CORRECT This is the procedure that is used to investigate blocking. Apparently, when this procedure is used, the second CS never becomes associated with the US and does not elicit a CR. One explanation for blocking is that it occurs because the second CS does not provide information beyond what is already provided by the original CS.
(Learning Theories & CBT)

How well did you know this?
1
Not at all
2
3
4
5
Perfectly
247
Q
Charging telephone customers for using directory assistance to obtain phone numbers in order to decrease their use of directory assistance is an application of which of the following?
A. Premack Principle
B. overcorrection
C. negative reinforcement
D. negative punishment
A

In this situation, money is being taken away following a behavior (use of directory assistance) in order to decrease that behavior.
a. Incorrect The Premack Principle is a type of positive reinforcement and is used to increase a behavior.
b. Incorrect Overcorrection involves having the individual engage in alternative behaviors (e.g., “fixing” the consequences of the undesirable behavior and for practicing appropriate behaviors) whenever he/she performs the undesirable behavior.
c. Incorrect Negative reinforcement is used to increase a behavior and involves removing an undesirable stimulus when the target behavior is performed.
D. negative punishment - CORRECT Negative punishment occurs when the removal of a stimulus
following a behavior decreases the behavior.
(Learning Theories & CBT)

How well did you know this?
1
Not at all
2
3
4
5
Perfectly
248
Q

Stress inoculation is best viewed as a combination of which of the following techniques?
A. cognitive restructuring and graded rehearsal
B. role-playing and flooding
C. classical extinction and psychoanalysis
D. counterconditioning and flooding

A

Stress inoculation is a cognitive-behavioral strategy that involves teaching clients the cognitive and behavioral skills they need to cope with future stressful situations.
a. CORRECT During the course of stress inoculation, the client learns to distinguish between ineffective and effective thoughts, actions, etc. and is taught alternative cognitive and behavioral skills that lead to effective coping in stressful situations. Once these skills have been learned, the client begins to apply them to real-life situations, usually beginning with the least stress-producing situations and then gradually working up to the most stress-producing situations.
(Learning Theories & CBT)

How well did you know this?
1
Not at all
2
3
4
5
Perfectly
249
Q

In his studies on memory and forgetting, Ebbinghaus:
A. used hungry cats as research subjects.
B. used himself as the research subject.
C. had volunteers memorize word pairs.
D. had volunteers “think aloud” while solving complex problems

A

Ebbinghaus was among the first to conduct empirical studies on human memory.
B. used himself as the research subject - CORRECT Ebbinghaus was the participant in his own research, which involved memorizing lists of nonsense syllables and subsequently recalling the words to evaluate retention. His research led to the identification of learning and forgetting curves.
(Learning Theories & CBT)

How well did you know this?
1
Not at all
2
3
4
5
Perfectly
250
Q
The process of gradually removing a prompt until it is no longer provided is referred to as:
A. extinction.
B. shaping.
C. fading. 
D. thinning.
A

The process of gradually eliminating prompts is referred to as “fading.” a. Incorrect Extinction refers to the removal of an unconditioned stimulus (classical conditioning) or a reinforcer (operant conditioning).
b. Incorrect Shaping involves reinforcing successive approximations to the desired behavior.
C. fading - CORRECT The gradual reduction or withdrawal of prompts is called fading.
d. Incorrect Thinning is the process of gradually reducing the frequency of reinforcement (e.g., of switching from a continuous to an intermittent schedule of reinforcement).
(Learning Theories & CBT)

How well did you know this?
1
Not at all
2
3
4
5
Perfectly
251
Q

Secondary reinforcers:
A. are applied after a primary reinforcer.
B. are applied on an intermittent schedule.
C. have inherent (innate) reinforcing value.
D. acquire their reinforcing value through conditioning

A

Behaviorists distinguish between primary and secondary reinforcers.
a. Incorrect This answer does not accurately describe the use of secondary reinforcers.
b. Incorrect Secondary reinforcers may be applied on a continuous or intermittent schedule.
c. Incorrect This answer describes primary reinforcers.
D. acquire their reinforcing value through conditioning - CORRECT This answer describes secondary reinforcers, which are also known as conditioned reinforcers because they acquire their reinforcing value through their association with primary reinforcers (i.e., as the result of being “paired with” primary reinforcers). Money is a secondary reinforcer that has value only because it enables us to purchase primary reinforcers, which have inherent reinforcing properties.
(Learning Theories & CBT)

How well did you know this?
1
Not at all
2
3
4
5
Perfectly
252
Q
Research has confirmed that behavioral techniques that reduce or eliminate a behavior are more effective when they are combined with techniques that increase alternative behaviors. This probably explains why \_\_\_\_\_ is
often an effective intervention.
A. differential reinforcement 
B. response cost
C. covert sensitization
D. negative practice
A

Of the interventions listed in the responses, only one involves eliminating an undesirable behavior while establishing more desirable behaviors.
A. differential reinforcement - CORRECT Differential reinforcement not only uses extinction to eliminate an
undesirable behavior but also reinforces the individual for engaging in alternative behaviors.
b. Incorrect Response cost is a type of negative punishment and is used to eliminate a behavior.
c. Incorrect Covert sensitization is a classical conditioning technique used to eliminate an undesirable behavior.
d. Incorrect When using negative practice, the individual is asked to repeatedly perform the undesirable behavior. Ideally, this will give the individual greater control over the behavior or, alternatively, lead to habituation or satiation.
(Learning Theories & CBT)

How well did you know this?
1
Not at all
2
3
4
5
Perfectly
253
Q
When using covert sensitization to treat a sexual fetish, the fetish object acts as a(n):
A. primary reinforcer.
B. secondary reinforcer.
C. unconditioned stimulus.
D. conditioned stimulus.
A

The goal of covert sensitization is to eliminate a response by replacing it with an incompatible response.
a. Incorrect A primary reinforcer is rewarding because of its inherent value. Food is a primary reinforcer for a hungry organism.
b. Incorrect A secondary reinforcer acquires its reinforcing value through its pairing with a primary reinforcer. Money is an example of a secondary
reinforcer: It is valuable only because it can be exchanged for primary reinforcers.
c. Incorrect When using covert sensitization to reduce an undesirable behavior, the aversive stimulus is the unconditioned stimulus.
D. conditioned stimulus - CORRECT When using covert sensitization, the target stimulus (in this case the fetish object) is paired in imagination with an aversive stimulus. Eventually the target stimulus elicits the same unpleasant response as the aversive stimulus and, as a consequence, will be avoided. In this situation, the CS is the target stimulus, while the US is the aversive stimulus.
(Learning Theories & CBT)

How well did you know this?
1
Not at all
2
3
4
5
Perfectly
254
Q

You are using in vivo aversive counterconditioning to reduce a client’s consumption of alcohol. To maximize the effectiveness of the treatment, the aversive stimulus (electric shock) should be applied:
A. when the drink is first presented to the client.
B. right before the client takes a drink.
C. just as the client begins drinking.
D. immediately after the client has finished a drink.

A

Aversive counterconditioning is a type of classical conditioning, and classical conditioning is most effective when a “delay” procedure is used.
b. Incorrect This describes backward conditioning, which is usually ineffective.
C. just as the client begins drinking - CORRECT Delay conditioning involves applying the CS before the start of the US so that presentation of the two stimuli overlaps and both are terminated together. Of the various types of forward conditioning, delay conditioning is usually most effective.
d. Incorrect This sounds like trace conditioning, which is ordinarily less effective than delay conditioning.
(Learning Theories & CBT)

How well did you know this?
1
Not at all
2
3
4
5
Perfectly
255
Q

A number of relapse prevention models have been proposed for alcoholism, cigarette smoking, and other addictive disorders. According Marlatt and Gordon’s (1985) model, the most effective way to increase the likelihood of recovery after relapse is to:
A. increase access to social support.
B. emphasize the negative consequences of relapse.
C. shift attention from internal to external antecedents.
D. address motivational issues related to abstinence.

A

Marlatt and Gordon’s (1985) model emphasizes the impact of cognitive and situational factors on relapse.
C. shift attention from internal to external antecedents - CORRECT Marlatt and Gordon’s (1985) “abstinence violation effect “ (AVE) model considers recovery after relapse to be related to attributions about the cause of the relapse — i.e., successful recovery is more likely when the person attributes relapse to external, unstable, and specific (high-risk) Factors than when he/she attributes it to internal, stable, and global factors.
(Learning Theories & CBT)

How well did you know this?
1
Not at all
2
3
4
5
Perfectly
256
Q
John Watson found that, after Little Albert had been classically conditioned to respond with fear to a white rat, the child also responded with fear to other white furry objects. Little Albert’s reaction to other white furry objects was the result of:
A. higher-order conditioning.
B. negative transfer.
C. pseudoconditioning.
D. stimulus generalization.
A

Little Albert’s response to a particular stimulus (the white rat) generalized to similar stimuli (other white furry objects).
a. Incorrect Higher-order conditioning is a classical conditioning technique in which a neutral stimulus elicits a conditioned response as a result of its pairing with a conditioned stimulus. In this situation, other white Furry objects were not “paired” with the white rat.
b. Incorrect Negative transfer occurs when previous learning interferes with present learning.
c. Incorrect Pseudoconditioning occurs when a neutral stimulus elicits a response due to the accidental pairing of the stimulus with another stimulus that evokes that response or when repeated exposure to a US increases the likelihood that the individual will respond to a neutral stimulus with a response similar to the UR.
D. stimulus generalization - CORRECT Stimulus generalization occurs when an individual responds with a conditioned response to stimuli that are similar to the original conditioned stimulus.
(Learning Theories & CBT)

How well did you know this?
1
Not at all
2
3
4
5
Perfectly
257
Q
Research investigating the usefulness of modeling for treating specific phobias has found that which of the following methods is generally most effective?
A. covert modeling
B. symbolic modeling
C. participant modeling
D. vicarious modeling
A
Bandura and others have compared the effectiveness of different types of modeling for reducing phobias and other anxiety responses.
a. Incorrect When using covert modeling, the individual is asked to imagine someone else engaging in the target behavior.
b. Incorrect Symbolic modeling involves observing models indirectly — e.g., in films, videotapes, or books.
C. participant modeling - CORRECT When using participant modeling, the model first engages in the
target behavior (or an aspect of it) and then prompts the individual to engage in the same behavior. The research has found that participant modeling is the most effective form of modeling for treating phobias.
d. Incorrect Vicarious (overt) modeling involves observing others successfully perform the target behavior. Vicarious modeling plus guided participation (i.e., participant modeling) is more effective for treating phobias than is vicarious modeling alone.
(Learning Theories & CBT)
How well did you know this?
1
Not at all
2
3
4
5
Perfectly
258
Q
When using a continuous schedule of reinforcement to increase the occurrence of a desirable behavior, you would be most concerned about which of the following?
A. satiation
B. sensitization
C. experimental neurosis
D. spontaneous recovery
A

A continuous schedule of reinforcement involves reinforcing the individual each time he/she emits the target response.
A. satiation - CORRECT Satiation occurs when a reinforcer loses its reinforcing value because the organism has become satisfied (satiated). It is a potential problem when using positive reinforcement, especially when the reinforcer is a primary reinforcer and is delivered on a continuous schedule.
b. Incorrect In the context of learning theory, the term sensitization has at least two meanings: It refers to either the initial increase in responsiveness to a stimulus when it is presented repeatedly or to the increase in responsiveness to a stimulus that occurs after exposure to a very intense or irritating stimulus.
c. Incorrect Experimental neurosis occurs in classical conditioning when the organism is required to discriminate between very similar stimuli.
d. Incorrect Spontaneous recovery refers to the return of a conditioned response following the apparent extinction of that response.
(Learning Theories & CBT)

How well did you know this?
1
Not at all
2
3
4
5
Perfectly
259
Q
When using higher-order conditioning to establish a conditioned response:
A. a US serves as a CS.
B. a US serves as a UR.
C. a UR serves as a CR.
D. a CS serves as a US.
A

Higher-order conditioning is a type of classical conditioning.
D. a CS serves as a US - CORRECT In higher-order conditioning, a conditioned stimulus (CS) is paired
with a neutral stimulus so that the neutral stimulus also elicits the conditioned response. In this situation, the CS is acting as a US (unconditioned stimulus) when it is paired with the neutral stimulus.
(Learning Theories & CBT)

How well did you know this?
1
Not at all
2
3
4
5
Perfectly
260
Q
Although he was unable to carry out his plans, John Watson outlined several methods for removing Little Albert’s classically-conditioned fear reaction to white rats. One plan involved repeatedly confronting Albert with a white rat without the loud noise in order to produce extinction of his fear response. This technique now underlies the behavioral treatment known as:
A. covert sensitization.
B. flooding.
C. response cost.
D. stress inoculation.
A

The intervention described in this question involves exposing Little Albert to the feared object.
a. Incorrect When using covert sensitization, the target stimulus is paired in imagination with an aversive stimulus. Eventually, the target stimulus elicits the
same unpleasant response as the aversive stimulus and, as a consequence, will be avoided.
B. flooding - CORRECT Flooding (which is also known as in vivo exposure with response prevention) involves the extinction of a conditioned response through prolonged
exposure to the conditioned stimulus without the unconditioned stimulus.
c. Incorrect Response cost involves removing a specific reinforcer following a behavior in order to decrease the behavior.
d. Incorrect Stress inoculation training is used to help people deal with stressful events by increasing their coping skills.
(Learning Theories & CBT)

How well did you know this?
1
Not at all
2
3
4
5
Perfectly
261
Q
You witness a four-car collision on your way to work and find yourself recalling the incident frequently during the next several weeks. Which aspect of your long-term memory is responsible for your memory of the accident?
A. implicit memory
B. semantic memory
C. episodic memory
D.  Sensory memory
A

In this situation, you are recalling a personally experienced event.
a. Incorrect Implicit memory is memory that occurs automatically (without conscious effort).
b. Incorrect Semantic memory is memory for the meaning of words and concepts.
C. episodic memory - CORRECT Episodic memory is memory for episodes (events) that have been
personally experienced.
d. Incorrect Sensory memory refers to brief impressions formed by input from the senses.
(Learning Theories & CBT)

How well did you know this?
1
Not at all
2
3
4
5
Perfectly
262
Q
To maximize the effectiveness of overcorrection, its restitution and positive practice phases are often combined with:
A. differential reinforcement.
B. activity scheduling.
C. physical guidance.
D. stimulus control.
A

To ensure that the person actually performs the behaviors required during the restitution and positive practice phases of overcorrection, “extra help” is sometimes needed.
C. physical guidance - CORRECT It may be necessary to physically guide the person through the behaviors required during restitution and for positive practice when the person is resistant to perform them on his/her own. Because physical guidance may be necessary, overcorrection is contraindicated for children who have experienced abuse or who are more physically powerful than the person administering the treatment.
(Learning Theories & CBT)

How well did you know this?
1
Not at all
2
3
4
5
Perfectly
263
Q

Whenever four-year-old William W. whines, his parents try to ignore him Occasionally, however, William’s mother or father picks him up when he whines, and, as a result, William’s whining has increased. The increase in
William’s whining is due to which of the following?
A. differential reinforcement
B. response generalization
C. intermittent reinforcement
D. response discrimination

A

William’s whining is being reinforced whenever his parents pick him up after he begins to whine.
a. Incorrect Differential reinforcement involves reinforcing behaviors other than the target one in order to decrease that behavior.
b. Incorrect Response generalization occurs when a CS produces responses similar to the CR.
C. intermittent reinforcement - CORRECT William’s parents are reinforcing the boy’s whining on an
intermittent schedule, which is producing an increase in his behavior.
d. Incorrect Response discrimination is the opposite of response generalization and occurs when the individual responds to the CS with a specific response (the original CR).
(Learning Theories & CBT)

How well did you know this?
1
Not at all
2
3
4
5
Perfectly
264
Q

EMG frontalis biofeedback is considered by a number of authorities to be the treatment-of-choice for tension (muscle contraction) headaches. However, some research suggests that which of the following interventions is as
effective, if not even more effective, than EMG feedback for this type of headache?
A. thermal feedback
B. relaxation training
C. alternate response training
D. covert sensitization

A

Several techniques have been found to be as effective as or more effective than EMG biofeedback for alleviating tension headaches.
a. Incorrect Thermal (temperature) feedback has been found to be an effective treatment for migraine headaches but not for tension headaches.
B. relaxation training - CORRECT A number of studies have found relaxation training to be as effective as (or even more effective than) EMG biofeedback for tension headaches.
c. Incorrect Alternate response training is a self-control technique that is used primarily to alleviate anxiety.
d. Incorrect When using covert sensitization, the target stimulus is paired in imagination with an aversive stimulus so that the target stimulus elicits the same unpleasant response as the aversive stimulus and, as a consequence, will be avoided.
(Learning Theories & CBT)

How well did you know this?
1
Not at all
2
3
4
5
Perfectly
265
Q
A person who argues that "all psychopathology has a similar set of underlying dysfunctional beliefs" is rnost likely sympathetic with the views of:
A. Beck.
B. Ellis.
C. Glasser.
D. Meichenbaum.
A

The term “dysfunctional beliefs” should have pointed to Ellis as the correct response.
a. Incorrect An important distinction between Beck and Ellis is that the former views each mental disorder as being characterized by a unique set of faulty cognitions, while the latter assumes that all disorders derive from the same set of basic irrational beliefs.
B. Ellis - CORRECT Ellis’s assumption that all disorders can be attributed to the same set of irrational beliefs is considered by some to be a shortcoming of his approach.
(Learning Theories & CBT)

How well did you know this?
1
Not at all
2
3
4
5
Perfectly
266
Q

In Albert Ellis’s A-B-C model, B refers to:
A. beliefs about an event.
B. behaviors that occur in response to an event
C. biological predispositions.
D. baseline rates of responding.

A

Ellis’s rational-emotive behavior therapy is based on the assumption that behavioral and emotional reactions to events are clue to beliefs about those events rather than to the events themselves.
A. beliefs about an event - CORRECT Ellis depicts the relationship between antecedent events, beliefs
about those events, and reactions to those events in terms of an A-B-C model, where A represents the antecedent event, B represents beliefs about the event,
and C represents the behavioral or emotional consequence of the beliefs.
(Learning Theories & CBT)

How well did you know this?
1
Not at all
2
3
4
5
Perfectly
267
Q
You are hired by an organization to develop an incentive program for salespeople. The personnel manager suggests that employees be given a bonus every time their monthly sales exceed their previous month's sales by at least 10%. However, you decide to give each salesperson a bonus after every sixth sale which means that most salespeople will receive a bonus every six to eight weeks. Your incentive program represents which schedule of reinforcement?
A. variable interval
B. variable ratio
C. fixed interval
D. fixed ratio
A

An important thing to notice when reading this question is that it is asking about the schedule you are recommending.
D. fixed ratio - CORRECT In this situation, the delivery of reinforcement depends on the number of responses, with reinforcement being given for every sixth response.
This represents a fixed ratio schedule. (Don’t be misled by the statement “every six to eight weeks.” It is the number of responses, not the time interval that determines when salespeople will receive a bonus.)
(Learning Theories & CBT)

How well did you know this?
1
Not at all
2
3
4
5
Perfectly
268
Q

Which of the following best describes elaborative rehearsal?
A. making new information meaningful
B. consciously repeating new information
C. practicing a new skill past the point of mastery
D. “thinking aloud” while studying

A

Researchers interested in memory distinguish between two types of rehearsal — elaborative and maintenance.
A. making new information meaningful - CORRECT The term elaborative rehearsal is used to describe the process of making new information meaningful in order to enhance its retention and
retrieval. Relating new information to previously acquired information is one type of elaborative rehearsal.
b. Incorrect This sounds more like maintenance rehearsal which involves the rote repetition of information.
c. Incorrect This describes “overlearning.”
d. Incorrect This does not describe elaborative rehearsal.
(Learning Theories & CBT)

How well did you know this?
1
Not at all
2
3
4
5
Perfectly
269
Q
In a residential facility, children are rewarded with a specific number of tokens each time they perform desirable behaviors but must return a certain number of tokens when they engage in undesirable behaviors. The latter technique (returning tokens) exemplifies which of the following?
A. negative reinforcement
B. response cost
C. Premack Principle
D. differential reinforcement
A

This question is asking which technique involves removing a stimulus following a behavior in order to reduce or eliminate that behavior.
a. Incorrect Negative reinforcement involves removing a stimulus following a behavior, but its goal is to increase the behavior, not to decrease it.
B. response cost - CORRECT Response cost is used to eliminate an undesirable behavior and involves imposing a penalty or fine (i.e., removing a specific stimulus) whenever the undesirable behavior occurs. Response cost is an application of negative punishment.
c. Incorrect The Premack Principle involves using a high frequency behavior as a reinforcer for a low frequency behavior in order to increase the low frequency
behavior.
d. Incorrect Differential reinforcement is used to decrease an undesirable behavior by reinforcing alternative behaviors. It does not involve removing a specific stimulus (e.g., tokens) following the undesirable behavior, which is what is occurring in the situation described in this question.
(Learning Theories & CBT)

270
Q

Which of the following best describes “backward conditioning”?
A. It involves presentation of the unconditioned stimulus prior to presentation of the conditioned stimulus and is rarely effective.
B. It involves presentation of the unconditioned stimulus prior to presentation of the conditioned stimulus and is very effective when the stimuli are similar in kind.
C. It involves presentation of the conditioned stimulus prior to presentation of the unconditioned stimulus and is effective only when the two stimuli occur naturally in that order.
D. It involves presentation of the reinforcer prior to the target behavior and is effective only when the reinforcer is highly desirable.

A

The term backward conditioning is associated with classical conditioning and involves presenting the US (unconditioned stimulus) prior to the CS (conditioned stimulus).
A. It involves presentation of the unconditioned stimulus prior to presentation of the conditioned stimulus and is rarely effective - CORRECT To establish a CR [conditioned response), the CS must be presented before or at the same time as the US. Backward conditioning — in which the US is presented prior to the CS — is not an effective strategy.
(Learning Theories & CBT)

271
Q
\_\_\_\_\_ memory is the aspect of memory that is involved in the recall of information acquired within the past few hours to days.
A. Working
B. Sensory
C. Long-term
D. Prospective
A

The three-store model of memory divides it into three components: sensory, short-term, and long-term memory.
a. Incorrect Working memory is part of short-term memory and is responsible for the temporary storage and manipulation of information (e.g., remembering a phone number until you have dialed it).
b. Incorrect Sensory memory stores sensory information for very brief periods (up to several seconds).
C. Long-term - CORRECT Information acquired in the past few hours to days would be part of recent long-term memory.
d. Incorrect Prospective memory refers to the ability to “remember to remember” (e.g., to remember a future appointment).
(Learning Theories & CBT)

272
Q

Tommy Tyrant, a bright four-year-old, has learned that if he starts crying as soon as his mother begins to yell at him for misbehaving, his mother will stop yelling and will play with him instead, which is what Tommy wanted in
the first place. In this situation, the mother’s yelling is acting as:
A. a discriminative stimulus.
B. an unconditioned stimulus.
C. a conditioned response.
D. a primary reinforcer.

A

In the situation described in this question, the mother’s yelling is indicating to Tommy that crying will result in reinforcement.
A. a discriminative stimulus - CORRECT A discriminative stimulus is a cue (antecedent) that signals that a particular response will be Followed by a reinforcer.
b. Incorrect An unconditioned stimulus naturally elicits the target response without learning or conditioning trials.
c. Incorrect A conditioned response is a response that occurs in the presence of a stimulus (the conditioned stimulus) as the result of pairing of that stimulus with
an unconditioned stimulus.
d. Incorrect The mother’s yelling is not acting as a reinforcer; it is her attention that has reinforcing value For Tommy.
(Learning Theories & CBT)

273
Q
To make sure that the housekeeper you hired does a good job, you decide to pay him for each chore he does well rather than giving him an hourly or weekly salary. You have apparently been influenced by which of the following individuals?
A. Pavlov 
B. Skinner
C. Wolpe
D. Tolrnan
A

This question is asking, “Which of the following individuals would recommend clearly linking reinforcement to the behavior that is being reinforced?”
a. Incorrect Pavlov is associated with classical conditioning, which involves pairing a conditioned stimulus with an unconditioned stimulus so that the
conditioned stimulus alone produces a response similar to the response produced by the unconditioned stimulus. This type of conditioning is not relevant to this situation.
B. Skinner - CORRECT Skinner believed that behavior is determined by the consequences that follow it and that, when a behavior is reinforced, it increases. According to
the principles of Skinner’s operant conditioning, clearly linking reinforcement to performance of the behavior (e.g., by providing reinforcement immediately after the behavior) is one way to maximize the effectiveness of reinforcement.
c. Incorrect Wolpe is probably most associated with systematic desensitization, which is based on the principles of classical conditioning.
d. Incorrect Tolman is best known for his research on rats in mazes, which led to his idea of cognitive maps and cognitive theory of learning.
(Learning Theories & CBT)

274
Q
A mother yells at her 4-year old son every time he runs while he is in the house. She quickly notices that her son’s running is increasing in frequency despite her reprimands. Apparently, the boy’s increased running is the result of:
A. negative reinforcement. 
B. positive reinforcement.
C. punishment.
D. second-order conditioning.
A

In the situation described in this question, the increase in the boy’s behavior is apparently due to the stimulus (the mother’s yelling) that is applied after he engages in the behavior.
a. Incorrect Negative reinforcement occurs when the removal of a stimulus following a behavior increases that behavior.
B. positive reinforcement - CORRECT Positive reinforcement is occurring when a behavior increases
because it is followed by the application of a stimulus. Note that this example illustrates the fact that a positive reinforcer doesn’t have to be considered positive or pleasant by everyone.
c. Incorrect Punishment decreases a behavior. Although the mother intends for her yelling to act as punishment, it is acting as reinforcement instead.
d. Incorrect Second-order conditioning occurs when presentation of the original conditioned stimulus is paired with a neutral stimulus so that the neutral stimulus
produces a conditioned response. Second-order conditioning is not relevant to the situation described in this question.
(Learning Theories & CBT)

275
Q
Which of the following types of biofeedback is likely to be most effective for reducing migraine headaches?
A. skin conductance
B. blood volume pulse
C. ENG (electrornyography) 
D. EEG (electroencephalograph)
A

Several types of biofeedback have been evaluated as treatments for migraine headaches.
a. Incorrect Skin conductance biofeedback provides information on autonomic arousal and is used primarily to reduce stress and anxiety reactions.
B. blood volume pulse - CORRECT Several recent studies indicate that blood volume pulse biofeedback is as effective as or more effective than skin temperature biofeedback for treating migraine headaches. Blood volume pulse is measured
using a photoplethysmograph (PPG) that monitors change in blood flow in the blood vessels beneath the skin. See, e.g., Y. Nestoriuc &A. Martin, Efficacy of biofeedback for migraine: A meta-analysis, Pain (in press).
c. Incorrect EMG biofeedback is useful for muscle tension headaches but has not been found to be more effective than blood volume pulse biofeedback for treating migraine headaches.
d. Incorrect EEG biofeedback provides information on brain activity and is not used as a treatment for migraine headaches
(Learning Theories & CBT)

276
Q

The keyword method is most useful for:
A. remembering word pairs.
B. remembering a long list of unrelated words
C. recalling the order of mathematical operations
D. mastering spatial relationships.

A

The keyword method is mnemonic device that involves forming an image that links two words. For example, to remember that “ranidae” is the scientific term for common frogs, you might create an image of a frog sitting in the rain under an umbrella.
A. remembering word pairs - CORRECT The keyword method is useful for learning pairs of words such as
the English and Spanish equivalents for words or the capitals of states.
b. Incorrect An acronym is more useful for learning lists of unrelated words.
c. Incorrect The method of loci and the pegword method are useful for recalling ordered information.
d. Incorrect The keyword method is not useful for this purpose.
(Learning Theories & CBT)

277
Q

A behavior therapist working with a child who bites her nails sets a timer so that it rings every 5 minutes. The child is then given a reward for every 5-minute period that she doesn’t bite her nails but engages in alternative
activities instead. The therapist is using which of the following techniques?
A. differential reinforcement
B. response cost
C. overcorrection
D. Prernack Principle

A

The child is being reinforced for every 5-minute period during which she engages in behaviors other than nailbiting.
A. differential reinforcement - CORRECT Differential reinforcement is what it sounds like — i.e., it involves
reinforcing behaviors other than (different from) the target behavior.
a. Incorrect Response cost involves removing a specific reinforcer following a behavior in order to decrease the behavior.
c. Incorrect Overcorrection is used to reduce undesirable behaviors and involves having the individual engage in alternative behaviors after performing the undesirable behavior (i.e., returning the environment to its previous condition and/or practicing correct behaviors).
d. Incorrect When using the Premack Principle, a high-frequency behavior is used as a reinforcer for a low-frequency behavior.
(Learning Theories & CBT)

278
Q

Which of the following is an example of negative reinforcement?
A. A child continues to misbehave in class because of the negative attention he gets from his teacher whenever he does so.
B. A husband stops smoking because, whenever he smokes, his children and wife berate him and tell him he’s going to die from lung cancer.
C. A student stops misbehaving in class because his teacher makes him sit in the corner for 10 minutes whenever he misbehaves.
D. A graduate student smokes cigarettes in order to alleviate anxiety he feels about his ability to receive good grades in his classes.

A

Negative reinforcement occurs when the removal of a stimulus following a behavior increases the likelihood that the behavior will be repeated.
a. Incorrect This is an example of positive reinforcement: The child’s misbehavior is continuing because a stimulus is applied (the teacher pays attention to him) whenever he does so.
b. Incorrect This is an example of positive punishment.
c. Incorrect This is an example of a time-out, which is a form of negative punishment.
D. A graduate student smokes cigarettes in order to alleviate anxiety he feels about his ability to receive good grades in his classes - CORRECT In this situation the student’s smoking continues because smoking removes (alleviates) anxiety.
(Learning Theories & CBT)

279
Q

Lewinsohn’s behavioral model proposes a causal link between _____ and depression.
A. a lack of reinforcement from the environment
B. “accidental” external reinforcement for depressogenic behaviors
C. a high need for perfectionism
D. chronic physical and psychological stress

A

Lewinsohn’s (1974) behavioral model is based on operant conditioning and focuses on the consequences of behavior.
A. a lack of reinforcement from the environment - CORRECT Lewinsohn attributes depression primarily to a low rate of response-contingent reinforcement and proposes that, when a person’s behaviors are not reinforced, those behaviors are extinguished and the person is at increased risk for pessimism, low self-esteem, and other symptoms associated with depression.
(Learning Theories & CBT)

280
Q

Emelina E., age 4, has learned that, when she approaches her father while he is watching a football game on TV, her father ignores her, but when she approaches her father while he is watching any other type of TV show, he is
willing to talk to and play with her. As a result, Emelina only approaches her father when he is not watching football. Emelina’s behavior illustrates which of the following?
A. stimulus fading
B. shaping
C. stimulus control
D. pseucloconclitioning

A

Emelina has learned that she will not be reinforced for approaching her father when he is watching a football game but will be reinforced if she approaches him when he is watching other types of TV shows.
a. Incorrect Stimulus fading refers to the gradual elimination of a discriminative stimulus. In this situation, the discriminative stimulus (type of TV show) is not
being gradually eliminated.
b. Incorrect Shaping is a type of positive reinforcement in which successive approximations to the desired behavior are reinforced.
C. stimulus control - CORRECT A behavior is “under stimulus control” when it is more likely to occur in the presence of certain stimuli than in the presence of other stimuli because the stimuli signal whether or not reinforcement for the behavior will be provided.
d. Incorrect Pseudoconditioning occurs when a neutral stimulus elicits a response due to the accidental pairing of the stimulus with another stimulus that evokes
that response or when repeated exposure to a US increases the likelihood that the individual will respond to a neutral stimulus with a response similar to the UR.
(Learning Theories & CBT)

281
Q

When treating a client with panic disorder, a practitioner of Beck’s cognitive-behavioral therapy would initially help the client:
A. identify methods for alleviating his/her symptoms.
B. understand how symptoms are controlling different aspects of his/her life.
C. identify the underlying causes of his/her symptoms.
D. see how he/she misinterprets the meaning of his/her symptoms.

A

As defined by Aaron Beck, the “goals of cognitive-behavior therapy are to correct faulty information processing and to modify dysfunctional beliefs and assumptions that maintain maladaptive behaviors and
emotions” (Comprehensive Handbook of Cognitive Therapy, 1989, p. 28).
D. see how he/she misinterprets the meaning of his/her symptoms - CORRECT From the perspective of cognitive-behavior therapy, panic disorder stems from “catastrophic misinterpretations” of bodily sensations and mental
experiences. Therefore, the first few sessions of therapy focus on clarifying the nature of the client’s symptoms and how he/she misinterprets them
(Learning Theories & CBT)

282
Q
A practitioner of cognitive therapy is most likely to conceptualize a personality disorder as a consequence of:
A. impulsive action tendencies.
B. cognitive enactments.
C. maladaptive life scripts. 
D. compelling schemata.
A

Even if you are unfamiliar with the cognitive view of the personality disorders, your familiarity with cognitive therapy in general should have helped you identify the correct answer to this question.
D. compelling schemata - CORRECT Although the word “compelling” might be unfamiliar, the word “schemata” is one you should have associated cognitive therapy. According to A. Freeman and R. C. Leaf, people with personality disorders are “generally governed by very compelling schemata,” which are not easily challenged or
surrendered” [Cognitive therapy applied to personality disorders, in A. Freeman, et al. (eds.), Comprehensive Handbook of Cognitive Therapy, New York: Plenum
Press, 1989].
(Learning Theories & CBT)

283
Q
When using operant extinction to decrease or eliminate a previously reinforced behavior, the first thing you will likely observe is:
A. an initial increase in the behavior.
B. a sharp reduction in the behavior.
C. a gradual decline in the behavior.
D. no change in the behavior.
A

Operant extinction involves removing reinforcement from a previously reinforced behavior in order to reduce or eliminate that behavior.
A. an initial increase in the behavior - CORRECT When using extinction, the withdrawal of reinforcement does not usually result in an immediate cessation of the behavior. Instead, the behavior often becomes more forceful and frequent at first. This phenomenon is referred to as a response (or extinction) burst.
(Learning Theories & CBT)

284
Q
Meichenbaurn and Goodman (1971) originally developed self-instructional training as an intervention for:
A. impulsivity.
B. schizophrenia.
C. depression.
D. obesity.
A

Self-instructional training is a cognitive-behavioral technique that is used to teach people to make positive and helpful self-statements when they encounter problematic situations.
A. impulsivity - CORRECT Although self-instructional training has subsequently been used to treat a variety of disorders, it was originally described by D. H. Meichenbaum and J. Goodman as a method for helping children control their impulsive behaviors (Training impulsive children to talk to themselves: A means of
developing self-control, Journal of Abnormal Psychology, 1971, 77, 115-126).
(Learning Theories & CBT)

285
Q

A key factor in distinguishing between implicit and explicit memory is:
A. the duration of the memory.
B. the sensory system(s) involved in the initial processing of the information.
C. the type of encoding used to transfer the information from short- to long-term memory.
D. the degree to which recall of the information involves conscious awareness.

A

Researchers interested in long-term memory often distinguish between implicit and explicit memory.
D. the degree to which recall of the information involves conscious awareness - CORRECT Recalling information from explicit memory requires conscious awareness and intentional or deliberate effort, while recall from implicit memory is automatic (done without conscious awareness or intentional or deliberate effort).
(Learning Theories & CBT)

286
Q
When using classical extinction to eliminate a learned (conditioned) response:
A. the CS is presented before the US.
B. the CS is presented without the US.
C. the CS is presented after the US.
D. the US is presented without the CS.
A

Knowing that classical conditioning involves pairing a CS with a US so that the CS produces a response (CR) that is similar to the UR would have helped you identify the correct answer to this question.
B. the CS is presented without the US - CORRECT In classical conditioning, a CS (conditioned stimulus) elicits a CR (conditioned response] as a result of its pairing with a US (unconditioned stimulus). To extinguish the CR, the CS is repeatedly presented without the US
(Learning Theories & CBT)

287
Q
A \_\_\_\_\_ stimulus signals that performance of a behavior will result in reinforcement.
A. conditioned
B. unconditioned
C. discriminative
D. second-order
A

This question is asking which type of stimulus affects the occurrence of an operant response.
C. discriminative - CORRECT A discriminative stimulus (SD) is an environmental event or condition that signals that reinforcement (or punishment) is contingent on the
performance of a particular response.
(Learning Theories & CBT)

288
Q

The mother of a teen-age girl complains that all her daughter wants to do is talk on the telephone and that, as a result, the girl is not studying and her grades have been declining. The psychologist suggests that the daughter
adhere to a schedule that allows her to talk on the telephone only after she has studied for a specified amount of time. The psychologist is recommending
the mother use which of the following strategies?
A. overcorrection
B. Premack Principle
C. response cost
D. flooding

A

The psychologist is suggesting that the mother use a high probability behavior (talking on the telephone) to reinforce a low probability behavior (studying). Note that the goal of the schedule is to increase studying and not, as might be inferred, to decrease talking on the
telephone.
a. Incorrect Overcorrection is used to decrease a behavior and involves having the client correct the ill-effects of the target behavior and practice more
appropriate behaviors.
B. Premack Principle - CORRECT The Premack Principle is a type of positive reinforcement in which a high frequency behavior is used to reinforce, and thus increase, a low frequency behavior.
c. Incorrect Response cost is used to reduce a behavior and involves removing a positive reinforcer each time the target behavior is performed.
d. Incorrect Flooding involves exposing a person to a conditioned stimulus without the unconditioned stimulus so that the conditioned response is extinguished.
(Learning Theories & CBT)

289
Q

Which of the following theorists and explanations for the etiology of depression are incorrectly matched?
A. Beck: errors in thinking
B. Lewinsohn: inadequate response-contingent reinforcement
C. Seligman: learned helplessness
D. Wolpe: loss of self-control

A

A number of cognitive and behavioral explanations have been offered for the etiology of depression.
a. Incorrect For Beck, depression is due to negative (depressogenic) thoughts and cognitive distortions.
b. Incorrect Lewinsohn describes depression as being related to a lack of response-contingent reinforcement.
c. Incorrect Seligman is associated with the learned helplessness theory of depression.
D. Wolpe: loss of self-control - CORRECT According to Wolpe, depression, anxiety, and other undesirable
states are the result of classical conditioning.
(Learning Theories & CBT)

290
Q
Using a tachistoscope, a researcher flashes a rectangle containing 12 letters of the alphabet to a participant for 50 milliseconds and then asks the participant to say what he or she has just seen. Apparently, the researcher is
investigating which of the following?
A. flashbulb memory 
B. recognition memory
C. iconic memory
D. echoic memory
A

In the study described in this question, participants are asked to recall a visual image.
a. Incorrect Flasnbulb memories are vivid memories of surprising, emotional events.
b. Incorrect Recognition memory refers to the ability to recall information from long-term memory when given appropriate cues.
C. iconic memory - CORRECT Iconic memory is an aspect of sensory memory and refers to memory for visual images (icons).
d. Incorrect Echoic memory refers to auditory sensory memory.
(Learning Theories & CBT)

291
Q
A person consistently avoids an object that produces a classically conditioned anxiety response because avoidance is:
A. positively reinforced. 
B. negatively reinforced.
C. a conditioned response.
D. an unconditioned response.
A

This question is asking what maintains a person’s avoidance of an object that produces anxiety.
a. Incorrect Positive reinforcement does increase a behavior but involves the application of a stimulus following the behavior.
B. negatively reinforced - CORRECT In this situation, the person’s avoidance behaviors allow him/her to reduce or alleviate anxiety. In other words, avoidance behaviors are being negatively reinforced because a stimulus is removed whenever the behaviors are performed.
c. Incorrect Avoidance behaviors are the result of operant (not classical) conditioning. Consequently, a person’s avoidance behavior is not a conditioned (or unconditioned) response.
(Learning Theories & CBT)

292
Q
A psychologist who views learning as the result of operant conditioning would likely attribute the acquisition of complex behaviors to which of the following?
A. behavioral chaining
B. stimulus substitution
C. higher-order conditioning
D. stimulus generalization
A

From the perspective of operant conditioning, new behaviors are acquired as the result of reinforcement.
A. behavioral chaining - CORRECT When using chaining (also known as behavioral chaining), each response in the chain of responses required to learn and perform a complex behavior serves as reinforcement For the previous response in the chain.
b. Incorrect Pavlov used the term stimulus substitution to describe the connection between a CS and US — i.e., as the result of this connection, the CS substitutes for the US.
c. Incorrect Higher-order conditioning is a Form of classical conditioning involving the pairing of a neutral stimulus with a conditioned stimulus so that the neutral
stimulus also elicits the conditioned response.
d. Incorrect Stimulus generalization is occurring when stimuli similar to the conditioned stimulus also elicit the conditioned response. It explains why behaviors occur in situations outside the context in which they were originally learned.
(Learning Theories & CBT)

293
Q

A person with a temporal lobe lesion will most likely exhibit which of the following?
A. memory impairment, depression or anxiety, and hallucinations
B. poor judgment, impulsivity, and inappropriate jocularity
C. homonymous hemianopia, prosopagnosia, and irritability and suspiciousness
D. dysgraphia, depression and apathy, and denial of his/her problems

A

The symptoms listed in each answer to this question are associated with lesions in a different lobe of the cerebral cortex.
A. memory impairment, depression or anxiety, and hallucinations - CORRECT The temporal lobe mediates auditory perception and memory, and a lesion in this lobe affects memory, may produce hallucinations (especially
auditory hallucinations), and often affects mood.
b. Incorrect These symptoms are associated with lesions in certain areas of the frontal lobe.
c. Incorrect These symptoms are more likely to be caused by an occipital lobe lesion.
cl. Incorrect A lesion in the parietal lobe may produce these symptoms.
(Physiological & Psychopharm)

294
Q

Damage to Broca’s area produces _____, while damage to Wernicke’s area results in _____.
A. expressive aphasia; conduction aphasia
B. conduction aphasia; transcortical aphasia
C. expressive aphasia; receptive aphasia
D. receptive aphasia; expressive aphasia

A

Broca’s and Wernicke’s areas are the two primary language areas of the brain.
a. Incorrect Damage to Broca’s area does ca use expressive aphasia, but damage to Wernicke’s area does not produce conduction aphasia. Conduction (associative) aphasia is characterized by relatively intact comprehension and fluent speech with anomia and an inability to repeat words. It is caused by damage to the arcuate Fasciculus, which connects Wernicke’s and Broca’s areas.
b. Incorrect Damage to Broca‘s area does not cause conduction aphasia, and damage to Wernicke’s area does not ca use transcortical aphasia, which results when lesions outside Broca’s and Wernicke’s areas isolate these areas from other regions of the brain.
C. expressive aphasia; receptive aphasia - CORRECT Damage to Broca’s area causes an inability to produce language (expressive aphasia), while damage to Wernicke’s area is associated with an inability to understand written or spoken language with fluent but unintelligible speech (receptive aphasia).
d. Incorrect This is the opposite of what is true.
(Physiological & Psychopharm)

295
Q
The benzodiazepines exert their therapeutic effects by:
A. decreasing GABA levels.
B. increasing GABA activity.
C. decreasing ACh levels.
D. increasing ACh activity.
A

Knowing that GABA is an inhibitory neurotransmitter and that the benzodiazepines are anti-anxiety drugs would have helped you identify the correct answer to this question.
B. increasing GABA activity - CORRECT The benzodiazepines enhance the efficiency of GABA activity at receptor sites and thereby decrease anxiety and enhance sleep.
(Physiological & Psychopharm)

296
Q
Intense hunger, headaches, anxiety, dizziness, weakness, heart palpitations, and confusion are symptoms of which of the following endocrine disorders?
A. hyperthyroidism
B. hyperglycemia 
C. hypothyroidism
D. hypoglycemia
A

The endocrine disorders produce a combination of physical and neuropsychiatric symptoms.
a. Incorrect Hyperthyroidism is characterized by heat intolerance, tachycardia, hyperactive reflexes, distractibility, and impaired problem-solving.
b. Incorrect Symptoms of hyperglycemia (high blood sugar) include polydipsia, polyuria, polyphagia, Fatigue, weight loss, and recurrent infections.
c. Incorrect Hypothyroidism is characterized by cold intolerance, bradycardia, fatigue, mental slowing, and decreased libido.
D. hypoglycemia - CORRECT The symptoms described in this question are characteristic of hypoglycemia (low blood sugar), which has several causes including excessive production of insulin by the pancreas.
(Physiological & Psychopharm)

297
Q

If you’re in an accident that causes damage to your cerebellum, you’re most likely to:
A. have trouble playing tennis.
B. forget what occurred just prior to the accident.
C. have trouble identifying the source of familiar odors.
D. find it difficult to do simple arithmetic calculations.

A

The cerebellum is involved in the regulation of balance and coordination.
A. have trouble playing tennis - CORRECT OF the activities listed in the answers, playing tennis is most likely to be adversely affected by damage to the cerebellum.
(Physiological & Psychopharm)

298
Q
Long-term potentiation is believed to play a critical role in which of the following?
A. emotional experience and expression
B. hunger and thirst
C. sexual behavior
D. learning and memory
A

Long-term potentiation (LTP) is a physiological process that involves physical modification of nerve synapses, especially at glutamate receptors in the hippocampus.
D. learning and memory - CORRECT LTP is a widely accepted model For the synaptic changes that underlie learning and memory.
(Physiological & Psychopharm)

299
Q
There are two major types of color-blindness: Monochrornats are totally color-blind, while dichromats are insensitive to one of two color pairs (red/green or blue/yellow). The existence of dichromats supports which of the following theories of color vision?
A. trichrornat theory
B. opponent-process theory
C. place theory
D. dual-process theory
A

There are two major theories of color vision: trichromatic and opponent-process.
a. Incorrect Trichromatic theory postulates that the human eye has three types of color receptors (red, green, and blue), and that the perception of different colors is due to a combination of the responses of these three receptors.
B. opponent-process theory - CORRECT According to opponent-process theory, there are three sets of cones: one that is sensitive to white and black, another to red and green, and a third to yellow and blue. Color-blindness results from the absence of one or more of these sets of cones and, therefore, explains why dichromats cannot
distinguish between either red and green or blue and yellow.
c. Incorrect Place theory is a theory of audition, not vision.
d. Incorrect According to dual process theory, a response made by an individual to a stimulus involves two stages - a decision as to whether or not to respond and a decision as to which alternative response to make. It is not a theory of color vision.
(Physiological & Psychopharm)

300
Q
Agraphia, acalculia, finger agnosia, and right-left confusion are symptoms of which of the following?
A. Creutzfeldt-Jakob disease
B. Addison's disease
C. Gerstmann’s syndrome 
D. Geschwind syndrome
A

The four symptoms listed in this question are caused by damage to the left (dominant) angular gyrus, which is located at the point of convergence of the parietal, occipital, and temporal lobes.
a. Incorrect Creutzfeldt-Ja kob disease is a rare degenerative brain disease that usually leads to dementia and is characterized by memory loss, behavioral changes, loss of coordination, and visual disturbances.
b. Incorrect Addison’s disease is due to a lower-tha n-normal production of cortisol by the adrenal glands. Its symptoms include weight loss, orthostatic hypotension, weakness, Fatigue, and hyperpigmentation.
C. Gerstmann’s syndrome - CORRECT These are the characteristic symptoms of Gerstmann’s syndrome.
d. Incorrect Gerschwind syndrome is a personality syndrome associated with temporal lobe epilepsy and is characterized by circumstantiality, hypergraphia,
alterations in sexuality, emotional volatility, and, in some cases, hyperreligiosity.
(Physiological & Psychopharm)

301
Q
A person with Wernicke's aphasia has impaired language comprehension and:
A. nonfluent and unintelligible speech.
B. nonfluent but intelligible speech.
C. fluent but unintelligible speech.
D. fluent and intelligible speech.
A

Wernicke’s aphasia is characterized by a disruption in the ability to produce and comprehend language.
C. fluent but unintelligible speech - CORRECT People with Wernicke’s aphasia have Fluent speech, but their
speech is unintelligible because it contains multiple errors including sound and word substitutions.
(Physiological & Psychopharm)

302
Q

The initial therapeutic effect of a conventional (traditional) antipsychotic drug will most likely be:
A. an increase in goal-directed behaviors.
B. an improvement in dysphoria.
C. a reduction in memory deficits.
D. a reduction in hallucinations and delusions.

A

Knowing that the conventional antipsychotics are most effective for reducing the positive symptoms of schizophrenia would have helped you identify the correct answer to this question.
D. a reduction in hallucinations and delusions - CORRECT The positive symptoms of schizophrenia represent an excess or distortion of normal Functions and include hallucinations and delusions, while the negative symptoms represent a reduction or absence of normal Functions and include avolition, anhedonia, and alogia.
(Physiological & Psychopharm)

303
Q
The research has most consistently linked which of the following characteristics of the Type A personality pattern to an increased risk for coronary heart disease?
A. a sense of time urgency
B. hostility and anger
C. perfectionisrn
D. competitiveness and ambitiousness
A

People with a Type A personality pattern are very competitive and achievement-oriented, have a sense of time urgency, and tend to be irritable, angry, and hostile.
B. hostility and anger - CORRECT Most of the research on the Type A personality has found that, in general, it is not predictive of an increased risk For coronary heart disease. However, anger and hostility, which are characteristic of the Type A personality, have been linked to this disease and other stress-related illnesses. Note that some recent research has found that an elevated risk For coronary heart disease is associated with the Type D personality pattern, which is characterized by chronic suppression of emotional distress.
(Physiological & Psychopharm)

304
Q
The research has confirmed that the \_\_\_\_\_ is responsible for our ability to recall pleasant and unpleasant events better than neutral events.
A. amygclala 
B. medulla oblongata
C. substantia nigra
D. putamen
A

Knowing that only one of the brain structures listed in the answers is associated with emotion would have helped you identify the correct answer to this question.
A. amygclala - CORRECT The amygdala is believed to be responsible for emotional memory, which refers to memory for events that evoke strong emotions.
(Physiological & Psychopharm)

305
Q

During the night, the mother of an 8-month old baby sleeps soundly. Although street noise and the barking of the neighbor’s dog do not awaken her, she immediately awakens to the sound of her baby’s cries. The brain
structure most responsible for this phenomenon is the:
A. frontal lobe.
B. ARAS.
C. hippocampus.
D. substantia nigra.

A

Knowing that ARAS stands for “ascending reticular activating system” would have helped you identify the correct answer to this question.
a. Incorrect Certain areas of the Frontal lobe are involved in emotion, language memory, and higher-order cognitive abilities.
B. ARAS - CORRECT The ARAS acts as a “sensory screening system” that arouses the brain whenever important information must be processed.
c. Incorrect The hippocampus plays an important role in memory consolidation.
d. Incorrect The substantia nigra is part of the basal ganglia and is involved in movement and certain types of learning. Abnormalities in the substantia nigra have been linked to Parkinson’s disease, schizophrenia, and epilepsy.
(Physiological & Psychopharm)

306
Q
L-dopa is used to alleviate the symptoms of:
A. tardive dyskinesia.
B. Parkinson's disease.
C. schizophrenia.
D. Tourette’s syndrome.
A

L-dopa (levodopa) is a precursor to dopamine.
a. Incorrect In some patients with tardive dyskinesia, L-dopa actually exacerbates symptoms, apparently because the long-term use of an antipsychotic drug has increased their sensitivity to dopamine.
B. Parkinson’s disease - CORRECT Parkinson’s disease is due to deficiencies of dopamine in certain areas of the brain; and administering L-dopa increases dopamine levels and alleviates the symptoms of the disorder, especially in its initial stages.
c. Incorrect Schizophrenia is due to oversensitivity to dopamine, and L-dopa would exacerbate its symptoms.
d. Incorrect L-dopa may actually induce the symptoms of Tourette’s syndrome
(Physiological & Psychopharm)

307
Q
The sleep-wake cycle and other circadian rhythms are controlled by which of the following structures of the brain?
A. extrapyrarniclal system
B. reticular activating system 
C. lateral geniculate nucleus
D. suprachiasrnatic nucleus
A

The suprachiasmatic nucleus (SCN) is located in the hypothalamus just above the optic chiasm.
a. Incorrect The extra pyramidal system is a network of neurons that is involved in movement.
b. Incorrect The reticular activating system is a network of neurons that extends from the spinal cord into the brain and is involved in motivation and arousal.
c. Incorrect The lateral geniculate nucleus is part of the thalamus and is involved in the processing of information related to vision.
D. suprachiasrnatic nucleus - CORRECT The SCN is the “primary circadian pacemaker” in mammals and is
responsible for circadian rhythms related to sleep, temperature, blood pressure, the production of hormones, and other functions.
(Physiological & Psychopharm)

308
Q
Sperry and Gazzaniga’s (1967) research on “split-brain” patients provided evidence for which of the following?
A. hemispheric specialization 
B. neural networks
C. pleasure centers in the brain
D. the fight-or-flight response
A

Sperry and Gazzaniga’s “split-brain” patients had their corpus callosums severed in order to control severe epilepsy. A consequence of the surgery was that communication between the two hemispheres was
reduced.
A. hemispheric specialization - CORRECT Results of the studies on these patients confirmed that each hemisphere specializes with regard to certain functions - e.g., in most people, the left hemisphere is dominant for language and logical, analytical thinking, while the right hemisphere is primarily responsible For spatial perception, holistic thinking, and creativity.
(Physiological & Psychopharm)

309
Q
Which of the following psychiatric symptoms are most associated with Huntington's disease?
A. depression and irritability
B. schizophrenia and delirium
C. anorexia and anxiety
D. sleep and sexual disturbances
A

Mood symptoms, especially dysthymia and depression, are early symptoms in about 40% of cases of Huntington’s disease.
A. depression and irritability - CORRECT Early reports linked Huntington’s disease to psychotic symptoms,
but more recent, better-controlled studies have found that affective symptoms are more common and are often the first signs of the disorder.
(Physiological & Psychopharm)

310
Q
According to , physical stimulus changes are logarithmically related to their psychological sensations.
A. Weber's law
B. Fechner's law
C. Lashley's law of equipotentiality
D. Thornclike's law of effect
A

This question is asking about one of the psychophysical laws that attempt to explain the relationship between sensation and perception.
a. Incorrect According to Weber’s law, the more intense the stimulus, the greater the increase in stimulus intensity required For the increase to be perceived.
B. Fechner’s law - CORRECT Fechner’s law proposes that sensation is a logarithmic function of stimulus intensity - i.e., that a person’s experience of stimulus intensity
increases arithmetically as the stimulus intensity increases geometrically.
c. Incorrect Lashley’s law of equipotentiality is not a psychophysical law but proposes that, when an area of the cortex is damaged, other areas can take over the functions of the damaged area.
d. Incorrect Thorndike’s law of el‘l’ect is not a psychophysical law but, instead, a precursor to Skinner’s operant conditioning. It predicts that behaviors that are
followed by “satisfying consequences” are likely to occur again.
(Physiological & Psychopharm)

311
Q

Hans Selye’s (1956) general adaptation syndrome predicts that we respond to stress with a predictable pattern that involves which of the following three stages?
A. disorganization, mobilization, and adaptation
B. exhaustion, accommodation, and recovery
C. preparation, survival, and rebuilding
D. alarm, resistance, and exhaustion

A

Selye’s general adaptation syndrome (GAS) is based on the assumption that people respond to all types of prolonged stress in the same manner.
a. Incorrect These have been identified as stages in reaction to a crisis.
b. Incorrect These are not the three stages identified by Selye.
c. Incorrect These have been identified as the three stages of responding to a disaster.
D. alarm, resistance, and exhaustion - CORRECT Alarm, resistance, and exhaustion are the three stages identified by Selye. Each is defined by different physiological responses that are mediated primarily by the adrenal and pituitary glands.
(Physiological & Psychopharm)

312
Q
Damage to the reticular activating system is most likely to cause which of the following?
A. deficits in explicit memory
B. disruptions in the sleep-wake cycle 
C. impaired balance and coordination
D. Kluver-Bucy syndrome
A

The reticular activating system (RAS) is a complex, net-like system of nuclei and fibers that extends from the spinal cord into the brain.
a. Incorrect Damage to the hippocampus is associated with deficits in explicit memory.
B. disruptions in the sleep-wake cycle - CORRECT Damage to the RAS disrupts the sleep-wake cycle and can produce a permanent, coma-like state.
c. Incorrect Damage to the cerebellum would cause these symptoms.
d. Incorrect Kluver-Bucy syndrome is characterized by reduced fear, increased docility, compulsive oral behaviors, altered dietary habits, an inability to recognize visual stimuli, and hypersexuality. It is caused by lesions in amygdala, hippocampus, and certain areas of the temporal lobe.
(Physiological & Psychopharm)

313
Q
Constructional apraxia is caused by damage to the:
A. corpus callosum.
B. parietal lobe.
C. basal ganglia.
D. temporal lobe.
A

Constructional apraxia is characterized by an inability to copy or draw figures or to arrange blocks in a pattern.
B. parietal lobe - CORRECT Apraxia involves an inability to perform skilled movements that are not due to muscle weakness, sensory loss, general intellectual deterioration
or lack of cooperation. Most forms of apraxia (including constructional apraxia) are caused by damage to the parietal lobe.
(Physiological & Psychopharm)

314
Q
According to Weber's law, the more intense a stimulus, the greater the increase in stimulus intensity required for the increase to be perceived. In other words, Weber's law deals with:
A. just noticeable differences.
B. absolute thresholds.
C. stimulus intensities.
D. absolute limens.
A

Psychophysics is the study of the relationship between physical stimulus magnitudes and their corresponding psychological sensations. There are several psychophysical laws that you should be familiar with for the exam.
A. just noticeable differences - CORRECT A just noticeable difference is the minimum difference in stimulus intensity required for an individual to notice a difference from the previous intensity. For instance, if you are in a room lit by a 120 watt light bulb, and you don’t notice a difference in brightness until the wattage is increased to 130, the just noticeable difference is 10 watts. Weber’s law states that the more intense a stimulus, the greater the increase in stimulus intensity must be for the increase to be noticed.
b. Incorrect An absolute threshold is the minimum stimulus intensity required to produce a specific sensation. For example, if you don’t notice any light in a room unless the wattage of a light bulb is two or more, then two watts is the absolute threshold.
c. Incorrect Stimulus intensities are measured in terms such as watts, decibels, and Hz. While Weber’s law is concerned with stimulus intensities, it more specifically deals with just noticeable differences.
d. Incorrect Absolute limen is another term for absolute threshold.
(Physiological & Psychopharm)

315
Q

Damage to the hippocampus is most likely to interfere with the ability to:
A. recall information stored in remote long-term memory.
B. manipulate information while it is in short-term (working) memory.
C. transfer information from short- to long-term memory.
D. retrieve implicit memories, even when given adequate cues.

A

The hippocarnpus is involved in memory consolidation.
a. Incorrect Although the hippocampus contributes to memory consolidation, it does not appear to be involved in the storage of long-term memories.
b. Incorrect The hippocampus has not been found to contribute to the ability to manipulate information in working memory.
C. transfer information from short- to long-term memory - CORRECT The research has shown that the hippocampus is involved in explicit (conscious) memory and, more specifically, in transferring explicit memories from short- to long-term memory.
d. Incorrect The hippocampus appears to be more involved in explicit than in implicit memory.
(Physiological & Psychopharm)

316
Q
Spinal cord injury at the level is most likely to result in
quaclriplegia.
A. lumbar
B. sacral
C. cervical
D. thoracic
A

The 31 pairs of spinal nerves are divided into five groups or regions. These are, from the top to the bottom of the spinal cord, cervical, thoracic, lumbar, sacral, and coccygeal.
a. Incorrect Damage at the lumbar level is likely to cause some loss of functioning in the hips and legs.
b. Incorrect Damage at the sacral level also causes loss of Functioning in the hips and legs.
C. cervical - CORRECT Quadriplegia (the loss of sensory and motor Functioning in the arms and legs) results from damage at the cervical level.
d. Incorrect Paraplegia (loss of Functioning in the legs) is caused by damage at the thoracic level.
(Physiological & Psychopharm)

317
Q

Gate control theory predicts that:
A. a neural mechanism in the spinal cord either blocks pain signals or allows them to be transmitted to the brain.
B. although each sensory neuron is sensitive to all forms of energy, it always evokes the same sensation.
C. some auditory stimuli are encoded in terms of the frequency of neural firing.
D. motivation arises from constant imbalances in homeostasis.

A

Knowing that gate control theory is a theory of pain would have helped you eliminate answers c and d as correct responses.
A. a neural mechanism in the spinal cord either blocks pain signals or allows them to be transmitted to the brain - CORRECT According to gate control theory, certain areas of the spinal cord receive input not only from pain receptors but also from other skin receptors; and if these other receptors are sufficiently active, they “close” the gate to pain signals and, thereby, keep those signals from reaching the brain. Gate control theory explains why applying heat or cold to the site of an injury immediately
relieves the pain.
b. Incorrect This is predicted by Muller’s doctrine of specific nerve energies.
c. Incorrect This describes frequency (volley) theory, which is a theory of audition.
d. Incorrect This sounds like drive reduction theory.
(Physiological & Psychopharm)

318
Q
During the initial \_\_\_\_\_ stage of the development of the central nervous system, new cells are produced inside the neural tube.
A. differentiation
B. proliferation
C. synaptogenesis
D. apoptosis
A

The development of the central nervous system involves five stages: proliferation, migration, differentiation, myelination, and synaptogenesis.
a. Incorrect Differentiation is the third stage in brain development and occurs when neurons of different types develop their own unique features.
B. proliferation - CORRECT This question accurately describes the first stage of brain development — i.e., the generation of new cells is referred to as proliferation.
c. Incorrect Synaptogenesis is the formation of synapses and is the last of the five stages.
d. Incorrect Apoptosis refers to cell death. During the development of the CNS too many cells are produced, and apoptosis is a normal process.
(Physiological & Psychopharm)

319
Q
Tourette’s syndrome has been most consistently linked to abnormalities in which of the following structures of the brain?
A. dentate gyrus
B. pons
C. basal ganglia 
D. mamrnillary bodies
A

Tourette’s syndrome is a tic disorder that involves a combination of motor and vocal tics.
a. Incorrect The dentate gyrus is part of the hippocampus and is believed to be involved in memory consolidation.
b. Incorrect The pons is a hindbrain structure that relays sensory information between the cerebellum and cortex, mediates reflexes related to breathing, and may play a role in sleep and dreaming.
C. basal ganglia - CORRECT The basal ganglia are involved in the control of movement and include the corpus striatum, caudate nucleus, putamen, and globus pallidus. Basal ganglia dysfunction has been implicated in a number of disorders including Tourette’s syndrome, ADHD, OCD, and schizophrenia.
d. Incorrect The ma mamillary bodies are connected to the hypothalamus, are involved in memory, and may play a role in the anterograde amnesia associated with Korsakoff’s syndrome.
(Physiological & Psychopharm)

320
Q

_________ has been used as a pharmacologic model for
schizophrenia because drugs that alleviate the former also reduce the symptoms of the latter.
A. Serotonin syndrome
B. neuroleptic malignant syndrome
C. Alcohol delirium tremens
D. Amphetamine psychosis

A

The research has identified several similarities between the effects of amphetamines and the symptoms of schizophrenia.
D. Amphetamine psychosis - CORRECT Research on amphetamine psychosis has provided support for the
dopamine hypothesis for schizophrenia: First, amphetamines exert their effects by altering dopamine activity. Second, a high dose of amphetamine produces
symptoms similar to those associated with schizophrenia, and amphetamines exacerbate the symptoms of schizophrenia. Third, drugs that reduce dopamine
levels reduce the symptoms of both amphetamine psychosis and schizophrenia.
(Physiological & Psychopharm)

321
Q
Many psychoactive drugs produce \_\_\_\_\_ side effects such as dry mouth, blurred vision, urinary retention, constipation, decreased perspiration, and tachycardia.
A. serotoninergic
B. cholinergic
C. anticholinergic
D. dopaminergic
A

Antipsychotics, antidepressants, and a number of other drugs produce the side effects listed in this question.
C. anticholinergic - CORRECT Anticholinergic side effects result when the amount of acetylcholine (ACh) in the central and peripheral nervous systems is reduced. Anticholinergic side effects are usually more severe in older adults due to the general decrease in ACh production associated with increasing age.
(Physiological & Psychopharm)

322
Q

Lesions in the right hemisphere of the cerebral cortex are most likely to have which of the following effects?
A. increase catastrophic reactions to benign events
B. produce indifference or excessive cheerfulness
C. produce excessive fear and anger
D. decrease talkativeness

A

Although the studies suggest that simple left-right distinctions about the brain are not entirely accurate, some generalizations can be made with regard to emotions, the left hemisphere mediates positive
emotions, while the right hemisphere mediates negative emotions.
a. Incorrect This is associated with damage to the left hemisphere.
B. produce indifference or excessive cheerfulness - CORRECT The right hemisphere governs negative emotions, and lesions to this side of the brain produce an opposite effect - i.e., indifference, emotional liability, and atypical humor (e.g., either not responding to jokes or cartoons or laughing even when they are not understood).
c. Incorrect This is the opposite of what is true.
d. Incorrect This is also the opposite of what is true. Lesions in the right hemisphere often increase talkativeness.
(Physiological & Psychopharm)

323
Q

According to Schachter and Singer (1962), a person’s subjective emotional response to an event:
A. is determined entirely by environmental cues.
B. is determined entirely by the person’s physiological reaction to the event.
C. depends primarily on the simultaneous activation of certain peripheral and brain mechanisms.
D. depends on a combination of the person’s physical reaction to and cognitive interpretation of the event.

A

The theories of emotion differ with regard to the degree to which they emphasize the role of peripheral and central factors.
a. Incorrect Schachter and Singer’s theory recognizes the role of environmental factors in the experience of emotion, but does not attribute emotion entirely to
environmental cues.
b. Incorrect This sounds like the James-Lange theory of emotion, which stresses the importance of peripheral factors.
c. Incorrect This sounds like the Cannon-Bard theory of emotions.
D. depends on a combination of the person’s physical reaction to and cognitive interpretation of the event - CORRECT According to Schachter and Singer’s (1962) two-factor theory, subjective emotional experience is the consequence of a combination of physical arousal and cognitive interpretation of that arousal and the context in which it occurs.
(Physiological & Psychopharm)

324
Q
The primary symptoms of Korsakoff’s syndrome are:
A. amnesia and confabulation.
B. visual and auditory hallucinations.
C. irritability and aggression.
D. aphasia and disorientation.
A

Korsakoff’s syndrome is a brain disorder that is caused by a thiamine deficiency, often related to chronic alcoholism.
A. amnesia and confabulation - CORRECT The primary symptoms of Korsakoff’s syndrome are anterograde
and retrograde amnesia and confabulation (inventing events to fill in memory gaps)
(Physiological & Psychopharm)

325
Q

A stroke involving the left middle cerebral artery is most likely to cause which of the following?
A. right-sided hemiplegia or herniparesis, nystagmus, and vertigo
B. left-sided hemiplegia or herniparesis, double vision, and facial paralysis
C. right-sided hemiplegia or herniparesis, right-sided hyposthesia, and aphasia
D. left-sided herniplegia or herniparesis, left-sided hyposthesia, and prosopagnosia

A

Knowing that the left hemisphere controls functions on the right side of the body, while the right hemisphere controls functions on the left side of the body would have helped you eliminate answers b and d.
a. Incorrect Nystagmus and vertigo are signs of a stroke involving the vertebral artery.
b. Incorrect Double vision and facial paralysis are signs of a stroke involving the basilar artery.
C. right-sided hemiplegia or herniparesis, right-sided hyposthesia, and aphasia - CORRECT The middle cerebral artery provides blood to many regions of the
brain, and the most common symptoms of stroke involving this artery include contralateral hemiplegia or hemiparesis and hyposthesia (lack of sensation). In
addition, when the stroke involves the i cerebral artery, aphasia and apraxia may also occur.
d. Incorrect These symptoms would be caused by a stroke involving the right middle cerebral artery.
(Physiological & Psychopharm)

326
Q
Agranulocytosis, a potential side effect of the atypical antipsychotic clozapine (Clozaril) is a \_\_\_\_\_ disorder.
A. seizure
B. hormonal
C. gastrointestinal
D. blood
A

One of the disadvantages of some of the atypical antipsychotic drugs is that they may cause agranulocytosis.
D. blood - CORRECT Agranulocytosis is an abnormally low level of white blood cells. It is a rare but dangerous side effect of clozapine.
(Physiological & Psychopharm)

327
Q
Damage to the temporal lobe is most likely to result in:
A. contralateral neglect.
B. apraxia.
C. memory loss.
D. homonymous hemianopsia.
A

The temporal lobe plays an important role in several important functions including auditory sensation and perception, language comprehension, and long-term memory.
a. Incorrect Contralateral neglect is a possible consequence of parietal lobe damage.
b. Incorrect Apraxia is caused by damage to the parietal or Frontal lobe.
C. memory loss - CORRECT Damage to the dominant temporal lobe may cause verbal memory loss, while damage to the nondominant temporal lobe may produce nonverbal memory loss.
cl. Incorrect This refers to loss of one-half of the visual Field in each eye and is due to damage to the occipital lobe.
(Physiological & Psychopharm)

328
Q

In terms of total sleep time, the proportion of Stage 4 sleep in adulthood:
A. increases from early to late adulthood.
B. decreases until middle age (about age 45) and thereafter stabilizes.
C. remains stable until late adulthood (age 70) and thereafter increases.
D. decreases from early to late adulthood.

A

Stage 4 sleep is the deepest level of non-REM sleep. Like the other sleep stages, it follows a predictable pattern over the lifespan.
D. decreases from early to late adulthood - CORRECT Stages 3 and 4 (especially Stage 4) sleep show a gradual decline over the lifespan, with older adults spending most of their sleep time in Stages 1 and 2 sleep.
(Physiological & Psychopharm)

329
Q
A primary concern about the long-term use of chlorpromazine is that it may result in the development of which of the following?
A. acute hypertensive crisis 
B. profound anterograde amnesia
C. tarclive dyskinesia
D. renal toxicity
A

This question requires you to know that chlorpromazine is an antipsychotic drug and that antipsychotics are associated with a number of adverse side effects.
a. Incorrect This is a side effect of the MAOIs.
b. Incorrect This is a potential side effect of triazolam and other benzodiazepines.
C. tarclive dyskinesia - CORRECT The long-term use of an antipsychotic drug - especially chlorpromazine or other conventional antipsychotic — can result in tardive
dyslcinesia, a neurological movement disorder involving involuntary movements of the lips, tongue, face, trunk, and extremities (e.g., facial grimacing, tongue protrusion, guitar and piano-playing movements).
d. Incorrect This is associated with the use of lithium.
(Physiological & Psychopharm)

330
Q

Which of the following describes the “rebound effect” that is associated with the use of a benzodiazepine as a treatment for anxiety?
A. the persistence of symptoms when the drug is prescribed in high dose
B. the re-emergence of symptoms after long-term use of the drug
C. an initial paradoxical increase in symptoms when the drug is prescribed in a low dose
D. a temporary increase in the severity of symptoms when the drug is discontinued

A

Discontinuation of a benzodiazepine may result in a “rebound effect.”
D. a temporary increase in the severity of symptoms when the drug is discontinued - CORRECT A person taking a benzodiazepine for anxiety, for example, may experience “rebound anxiety” when he or she stops taking the drug, especially when cessation is abrupt rather than gradual. Rebound symptoms are often more severe than the original symptoms.
(Physiological & Psychopharm)

331
Q

Complex partial seizures originating in the temporal lobe are most likely to cause which of the following symptoms?
A. a “drop attack”
B. pain or tingling on one side of the body
C. a feeling of déja vu or jamais vu
D. visual hallucinations or vision loss

A

Complex partial seizures are due to abnormal electrical activity in one region of the brain.
a. Incorrect A “drop attack” is characteristic of atonic seizures and involves sudden falling down.
b. Incorrect Pain, tingling, or other bodily sensations are caused by abnormal electrical activity in the parietal lobe.
C. a feeling of déja vu or jamais vu - CORRECT Typical symptoms of seizure activity in the temporal lobe are a
change in perception, a sudden alteration in emotions and/or a sense of déja vu or jamais vu (a sense of familiarity or unfamiliarity, respectively).
d. Incorrect Visual abnormalities (e.g., hallucinations, flashing lights, or vision loss) are symptoms of abnormal electrical activity in the occipital lobe.
(Physiological & Psychopharm)

332
Q
Following abrupt cessation of a sedative after long-term use, REM sleep would be expected to:
A. increase to above normal levels.
B. increase to normal levels.
C. decrease to below normal levels.
D. decrease to normal levels.
A

Although sedative drugs increase the total sleep time, they decrease the amount or proportion of REM sleep.
A. increase to above normal levels - CORRECT Abrupt cessation of a sedative following long-term use typically
results in a “REM rebound,” which is characterized by an above-normal amount or proportion of REM sleep.
(Physiological & Psychopharm)

333
Q

Unilateral damage to the left (dominant) hemisphere is least likely to result in an inability to:
A. memorize vocabulary words.
B. construct a daily schedule.
C. read a map.
D. use the process of elimination to answer multiple-choice questions.

A

Although the two hemispheres of the cerebral cortex play a role in most behaviors, each hemisphere specializes with regard to certain functions. For most people language, math, and analytical thinking are left
hemisphere functions, while the understanding of spatial relations, creativity, intuitive thinking, and the ability to recognize faces are right hemisphere functions.
C. read a map - CORRECT Of the activities listed in the answers, the ability to read a map would most likely be impaired by damage to the right hemisphere. In contrast,
damage to the left hemisphere would have the effects listed in answers a, b, and d.
(Physiological & Psychopharm)

334
Q
The thalamus acts as a "relay station" for incoming signals from all of the senses except:
A. olfaction.
B. gustation.
C. vision.
D. audition.
A

The thalamus receives afferent impulses from all of the senses except olfaction and transmits this information to appropriate areas in the cerebral cortex.
A. olfaction - CORRECT As noted above, incoming signals related to smell (olfaction) are not relayed to higher areas of the brain through the thalamus. Instead, olfactory
signals are sent to the olfactory bulb which then forwards the signals to other areas of the brain.
(Physiological & Psychopharm)

335
Q
The \_\_\_\_\_\_ predicts that the size of an action potential is independent of the intensity of the stimulus that initiated it.
A. all-or-none principle
B. rate law
C. principle of equipotentiality
D. doctrine of specific nerve energies
A

For the exam, you want to be familiar with the basic properties of the neuron.
a. CORRECT According to the all-or-none principle, as long as a nerve cell receives the minimal degree of stimulation required to trigger an action potential, the action potential is the same size regardless of the amount of stimulation.
b. Incorrect The rate law predicts that variations in the intensity of a stimulus are coded by the frequency of the axon’s firing — i.e., the more intense the stimulus, the greater the rate of responding.
c. Incorrect The principle (law) of equipotentiality proposes that, when an area of the cortex is damaged, other intact areas of the cortex can take over the
functions of the damaged area.
d. Incorrect The doctrine of specific nerve energies predicts that each sensory neuron is sensitive to all forms of energy but always evokes the same sensation.
(Physiological & Psychopharm)

336
Q
Oversecretion of \_\_\_\_\_\_ by the hypothalarnic-pituitary-adrenal axis has been linked to depression.
A. glutamate
B. cortisol
C. adrenalin
D. melatonin
A

The structures of hypothalamic-pituitary-adrenal (HPA) axis interact and regulate a number of important functions including the body’s reaction to stress and the activities of the immune system. Recently the HPA axis has also been linked to mood and anxiety disorders.
a. Incorrect Although there is some evidence that glutamate plays a role in depression, it is not secreted by the HPA axis.
b. CORRECT Secretion of cortisol, a stress hormone, by the adrenal gland is influenced by the activity of the hypothalamus and pituitary gland. Although cortisol plays an important role in helping the body respond to stress, recent research has found that chronic excessive cortisol levels are associated with disturbances in sleep, decreased appetite, a reduced sex drive, and other
symptoms of depression.
c. Incorrect Lower-than-normal levels of noradrenaline — but not adrenaline - in noradrenergic centers in the brain (locus coeruleus and caudal raphe nucleus) have been linked to depression
d. Incorrect Abnormalities in melatonin have been linked to seasonal affective disorder but not to depression more generally. In addition, melatonin is secreted by the pineal gland.
(Physiological & Psychopharm)

337
Q
Eating aged cheese, soy products, processed meats, fermented foods, or other foods containing tyramine while taking an MAOI is contraindicated because doing so can cause which of the following?
A. rebound effect
B. agranulocytosis
C. tardive dyskinesia
D. hypertensive crisis
A

MAOI’s are a type of antidepressant drug. Combining these drugs with certain foods or other drugs can produce potentially fatal side effects.
d. CORRECT Taking an MAOI in combination with foods containing tyramine can cause a hypertensive crisis which involves elevated blood pressure, severe
headache, nausea and vomiting, palpitations, blurred vision, stiff neck, sweating, and confusion.
(Physiological & Psychopharm)

338
Q
Parkinson's disease is caused by a progressive degeneration of dopamine-producing cells in the:
A. suprachiasmatic nucleus.
B. insular cortex.
C. substantia nigra.
D. entorhinal cortex.
A

Although the cause of Parkinson’s disease is still unknown, it has been linked to the degeneration of nerves cells in specific regions of the brain.
a. Incorrect The suprachiasmatic nucleus (SCN) is located in the hypothalamus and is involved the regulation of the body’s circadian rhythms.
b. Incorrect The insular cortex is involved in speech and in the processing of autonomic and sensory information.
c. CORRECT Knowing the substantia nigra is involved in the control of movement would have helped you identify the correct answer to this question. The degeneration of dopamine-producing cells in the substantia nigra results in the motor symptoms associated with this disorder.
d. Incorrect The entorhinal cortex plays a role (in conjunction with the hippocampus) in memory consolidation and is one of the first areas of the brain
to be affected by Alzheimer’s disease.
(Physiological & Psychopharm)

339
Q
Generally, the \_\_\_\_\_ branch of the autonomic nervous system activates bodily functions while the \_\_\_\_\_ branch returns the body to a resting state.
A. peripheral; central
B. central; peripheral
C. sympathetic; parasympathetic 
D. parasympathetic; sympathetic
A

The autonomic nervous system is divided into two branches: the parasympathetic and sympathetic.
c. CORRECT The sympathetic nervous system mediates arousal and expenditure of energy and prepares the body for action, while the parasympathetic nervous
system allows the body to increase its store of energy and is active during states of relaxation.
(Physiological & Psychopharm)

340
Q
Which of the following neuroimaging techniques would not be useful for studying the functional activity of the brain (e.g., blood flow, glucose metabolism, or oxygen consumption)?
A. CT
B. SPECT
C. fMRI
D. PET
A

The various neuroimaging techniques can be categorized as either structural or functional techniques.
a. CORRECT CT (computed tomography) is a structural technique and provides information on brain structure. In contrast, the techniques listed in answers b, c, and d are functional techniques and provide information on the functional activities of the brain.
(Physiological & Psychopharm)

341
Q
While relaxing in front of the TV, Dexter D. is suddenly startled by strange noises coming from his basement and, as a result, his blood pressure increases, his heart starts racing, and he begins to sweat. Dexter's physical reactions to the strange noises are mediated by his \_\_\_\_\_ nervous system.
A. enteric
B. somatic
C. sympathetic 
D. parasympathetic
A

Dexter is exhibiting physical signs associated with the “fight-or-flight” response.
a. Incorrect The enteric nervous system is sometimes categorized as part of the autonomic nervous system. It is a network of nerves located in the gastrointestinal tract and is believed to be involved in several functions including gastrointestinal motility and secretions, activities of the endocrine system, and regulation of immune and inflammatory processes.
b. Incorrect The somatic nervous system consists of sensory nerves that carry signals from the sensory receptors to the CNS and motor nerves that carry
signals from the CNS to the skeletal muscles. It is responsible for activities that are ordinarily considered voluntary.
c. CORRECT The sympathetic nervous system is a branch of the autonomic nervous system. It mediates arousal and the expenditure of energy and prepares the body for fight or flight.
d. Incorrect The parasympathetic nervous system is another branch of the autonomic nervous system. It is involved in the conservation of energy and is active during digestion and periods of rest and relaxation.
(Physiological & Psychopharm)

342
Q

_____, a late-appearing side effect of the conventional
antipsychotic drugs, is characterized by repetitive, involuntary tic-like movements of the face, eyelids, mouth, tongue, extremities, and/or trunk.
A. Acute dystonia
B. Parkinsonism
C. Tardive dyskinesia
D. Akathisia

A

For the exam, you’ll want to be familiar with the symptoms of all of the conditions listed in the answers to this question.
a. Incorrect Acute dystonia is an early-onset side effect of the antipsychotics. Its primary symptoms are muscle spasms and slow abnormal movements, most often in the eyes, jaw, and tongue.
b. Incorrect Parkinsonism is an intermediate-onset side effect of the antipsychotic drugs. Its symptoms include muscle rigidity, impaired balance, gait changes, tremors, changes in facial expressions and speech, and muscle cramps.
c. CORRECT Tardive dyskinesia is a serious and potentially irreversible side effect of the antipsychotics (especially the conventional drugs), and the question
describes its primary symptoms.
d. Incorrect Akathisia is another intermediate-onset side effect of the antipsychotics. It involves an uncomfortable sense of restlessness that may be accompanied by foot tapping, leg shaking, or pacing.
(Physiological & Psychopharm)

343
Q
\_\_\_\_\_ is a communication disorder that is characterized by difficulties in regulating the rate, rhythm, pitch, and loudness of speech
A. Dysarthria
B. Paraphasia
C. Dysprosody
D. Adynamia
A

The pattern of stress and intonation in speech is referred to as prosody.
a. Incorrect Dysa rthria is a motor speech disorder that produces problems related to articulation.
b. Incorrect Pa raphasia is a type of aphasia that involves the use of unintended syllables, words, or phrases (e.g., unintentionally substituting one word for another).
c. CORRECT Dysprosody is a disruption in speech melody and rhythm and is characteristic of Broca’s aphasia and other nonfluent aphasias. Although
dysprosody has traditionally been linked to right hemisphere damage, its presence in Broca’s aphasia suggests that prosody is also governed, to some
degree, by the left hemisphere.
d. Incorrect Adynamia is a type of aphasia involving ditficulty in initiating speech
(Physiological & Psychopharm)

344
Q

The potential adverse side effects of the benzodiazepines include:
A. insomnia, appetite loss, and restlessness.
B. confusion, memory loss, and psychomotor impairment.
C. muscle rigidity, catalepsy, and clammy skin.
D. bradycardia, shortness of breath, and depression.

A

The benzodiazepines are anxiolytics that are used primarily as treatments for anxiety and sleep disturbances.
a. Incorrect These are common side effects of the psychostimulant drugs.
b. CORRECT Common side effects of the benzodiazepines include drowsiness, impaired psychomotor ability, disorientation and confusion, anterograde amnesia, and depression.
c. Incorrect These are signs of narcotic-analgesic overdose.
d. Incorrect These are side effects of the beta-blocker propranolol.
(Physiological & Psychopharm)

345
Q
Early memory loss associated with Alzheimer's dementia is believed to be caused by deterioration of neurons in the hippocampus that secrete:
A. GABA.
B. serotonin.
C. glycine.
D. acetylcholine.
A

For the exam, you should be familiar with the functions that have been associated with the neurotransmitters listed in the answers to this question.
a. Incorrect GABA is an inhibitory neurotransmitter. It is believed to play a role in anxiety, sleep, and eating disorders and is affected by CNS depressants.
b. Incorrect Serotonin has been linked to a variety of functions including memory. However, abnormalities in serotonin levels have not been as consistently linked to the early memory loss of Alzheimer’s dementia as have
been abnormalities in acetylcholine. Instead, serotonin is believed to contribute to other symptoms of this disorder including sensory disturbances, mood swings, and sleep disruption.
c. Incorrect Glycine plays a role in spinal reflexes and motor behavior.
d. CORRECT Acetylcholine (ACh) mediates motor functions as well as memory processes, and decreased levels of ACh in the hippocampus have been linked to
the memory loss associated Alzheimer’s dementia.
(Physiological & Psychopharm)

346
Q

A lesion in the right parietal lobe is most likely to cause which of the following?
A. unilateral neglect.
B. apperceptive agnosia
C. receptive aphasia
D. pseudoclepression or pseuclopsychopathy

A

Knowing that the parietal lobe controls the somatosensory functions of the body would have helped you identify the correct answer to this question.
a. CORRECT Unilateral (contralateral) neglect involves a lack of attention to or interest in one side of the body. It is usually results from damage to the right parietal lobe and involves the left side of the body.
b. Incorrect Apperceptive agnosia (an inability to recognize the basic shape of objects) is caused by lesions in certain areas of the occipital lobe.
c. Incorrect Receptive (Wernicke’s} aphasia is ca used by damage to Wernicke’s area, which is in the temporal lobe.
d. Incorrect Pseudodepression and pseudopsychopathy are ca used by damage to certain areas of the Frontal lobe.
(Physiological & Psychopharm)

347
Q

Propranolol and other beta-aclrenergic blocking drugs:
A. are most effective for reducing palpitations, tachycardia, tremor, and other somatic manifestations of anxiety.
B. are most effective for reducing worry, apprehension, and other psychic (subjective) manifestations of anxiety.
C. are equally effective for reducing the somatic and the psychic manifestations of anxiety.
D. are equally ineffective for reducing the somatic and the psychic manifestations of anxiety.

A

Beta-adrenergic blocking drugs (beta-blockers) are used to treat a number of disorders including anxiety.
a. CORRECT The beta-blockers reduce the somatic (peripheral) symptoms of anxiety but do not directly affect its subjective manifestations.
b. Incorrect Through their effects on the somatic symptoms of anxiety, these drugs may indirectly affect (reduce) the subjective experience of anxiety. However, their direct effects are on the somatic symptoms of anxiety.
(Physiological & Psychopharm)

348
Q

Which of the following is consistent with the predictions of the James-Lange theory?
A. I think, therefore I am.
B. I’m trembling, so I must be scared.
C. It’s noon so I must be hungry.
D. That was a great birthday surprise, so I must be happy.

A

The James-Lange theory is a peripheralist theory of emotion.
b. CORRECT According to the James-Lange theory, the psychological experience of emotion Follows bodily responses to external stimuli.
(Physiological & Psychopharm)

349
Q
There is evidence that the \_\_\_\_\_ plays a role in the etiology of seasonal affective disorder (SAD).
A. red nucleus
B. suprachiasmatic nucleus
C. tecturn
D. striatum
A

Seasonal affective disorder is a recurrent type of depression that occurs in the fall and winter months and has been linked to reduced exposure to daylight.
a. Incorrect The red nucleus is a midbrain structure and is involved in motor coordination.
b. CORRECT Knowing that the suprachiasmatic nucleus (SCN) mediates the sleep-wake cycle and other circadian rhythms may have helped you identify the correct answer to this question. The SCN’s role in SAD is believed to be related to its impact on melatonin levels, which are generally higher in individuals with this disorder and are atfected by exposure to light.
c. Incorrect The tectum is part of the midbrain and is involved in auditory and visual processing.
d. Incorrect The striatum is part of the basal ganglia and is involved in movement and certain cognitive processes.
(Physiological & Psychopharm)

350
Q
When in a restful, relaxed state, your brain emits regular high-amplitude low frequency \_\_\_\_\_ waves.
A. beta
B. delta
C. theta
D. alpha
A

Different states of wakefulness and sleep are associated with different brain wave patterns.
a. Incorrect Beta waves occur during awake, alert states.
b. Incorrect Delta waves are characteristic of Stage 3 and Stage 4 (deep) sleep.
c. Incorrect Theta waves are associated with deep relaxation and drowsiness and predominate during Stage 2 sleep.
d. CORRECT Alpha waves are regular high-amplitude, low frequency waves that are recorded during states of restfulness and relaxation.
(Physiological & Psychopharm)

351
Q

Which of the following is an example of sexual dimorphism?
A. The hypothalamus, corpus callosum, and several other regions of the human brain exhibit consistent gender-related differences in structure and activity.
B. The gonads in human males and females are undifferentiated until six to eight weeks after conception.
C. Sex hormones play a more important role in the adult sexual behavior of lower animals than in the adult sexual behavior of humans.
D. The effects of spinal cord injury on sexual functioning depend on the severity and location of the injury.

A

Sexual dimorphism refers to systematic gender-related differences in physical characteristics. Recent research has confirmed that certain structures in the brain are sexually dimorphic.
a. CORRECT The hypothalamus is one of the structures of the human brain that is sexually dimorphic. For example, the size of the hypothalamus and its secretion of growth hormone differs for males and females.
(Physiological & Psychopharm)

352
Q
Propranolol hydrochloride (Inderal) is most likely to be prescribed as a treatment for which of the following?
A. hypotension 
B. essential tremor
C. bronchial asthma
D. diabetes
A

Propranolol hydrochloride is a beta-blocker and is used to treat cardiovascular disorders, tremors, migraine headaches, and the physical symptoms of anxiety.
a. Incorrect Propra nolol HCI is used to treat hypertension and is contraindicated For hypotension.
b. CORRECT Essential tremor (tremor associated with purposeful movement) is one of the conditions that propranolol HCI is used to treat.
c. Incorrect Propranolol HCI is contraindicated for bronchial asthma.
d. Incorrect Propranolol HCI is not used as a treatment for diabetes.
(Physiological & Psychopharm)

353
Q
The research suggests that RNA (ribonucleic acid) plays an important role in which of the following?
A. the sleep-wake cycle
B. learning and memory
C. creativity
D. motivation
A

RNA is involved in protein synthesis and other cell activities.
b. CORRECT Some investigators suggest that the coding of memory may involve changes in protein molecules and, consequently, that RNA acts as a chemical mediator For learning and memory. RNA has not been linked to the
sleep-wake cycle, creativity, or motivation.
(Physiological & Psychopharm)

354
Q
Huntington's disease is an:
A. autosomal dominant disorder.
B. autosomal recessive disorder
C. X-linked dominant disorder.
D. X-linked recessive disorder.
A

Huntington’s disease is an inherited neurodegenerative disorder that is transmitted by a single dominant gene.
a. CORRECT The term “autosome” refers to a chromosome that is not an X or Y sex chromosome. Huntington’s disease is transmitted by a single dominant
autosomal chromosome.
(Physiological & Psychopharm)

355
Q
The \_\_\_\_\_\_ mediates the body's physical homeostasis
A. cingulate gyrus
B. hypothalamus
C. cerebellum
D. adrenal medulla
A

Homeostasis refers to maintaining functions within a fixed or optimal range.
a. Incorrect The cingulate gyrus is part of the limbic system and is believed to be responsible for focusing attention on emotionally significant events, linking
sensory input to emotions, mediating emotional reactions to pain, and regulating aggressive behavior.
b. CORRECT A primary function of the hypothalamus is to maintain the body’s homeostasis with regard to temperature, blood pressure, fluid and electrolyte balance, and other bodily states.
c. Incorrect The cerebellum is important for balance and posture and is vital to the performance of coordinated and refined motor movements.
d. Incorrect The adrenal medulla is the core of the adrenal gland and is involved in several functions that are similar to those of the sympathetic nervous system. It plays an important role in the body’s reaction to stress.(Physiological & Psychopharm)

356
Q
Carbamazepine and other anticonvulsant drugs were originally used to treat seizure disorders but are now also used to treat;
A. paraphilias.
B. obsessive-compulsive disorder. 
C. bipolar disorder.
D. autism.
A

Carbamazepine and other anticonvulsants (e.g., valproic acid and clonazepam) have been identified as effective treatments for only one of the disorders listed in the answers to this question.
c. CORRECT Carbamazepine and other anticonvulsant drugs are often useful for treating bipolar disorder for individuals who have not responded to lithium, and
there is some evidence that they are particularly effective as mood stabilizers for “rapid cyclers” (individuals who experience frequent mood swings).
(Physiological & Psychopharm)

357
Q

ECT (electroconvulsive therapy) is most likely to produce permanent:
A. anterograde amnesia for certain types of verbal information.
B. anterograde amnesia for certain types of nonverbal information.
C. retrograde amnesia for autobiographical memories from the recent past.
D. retrograde amnesia for autobiographical memories from the remote past.

A

Memory deficits following ECT are more common with bilateral ECT than with unilateral ECT to the nondominant hemisphere.
c. CORRECT Anterograde amnesia does occur with ECT but generally clears up within six months. However, loss of memory for past events (events prior to ECT) is sometimes permanent and most often involves autobiographical memories from the recent past.
(Physiological & Psychopharm)

358
Q
Jennifer J. always experiences certain colors in response to certain sounds (e.g., the meow of a cat always elicits a reddish-pink color while the chirp of a bird elicits light blue). Jennifer's experience is referred to as:
A. synesthesia.
B. somesthesis.
C. anosognosia.
D. alexithymia
A

For Jennifer, stimulation of one sensory modality triggers sensation in another sensory modality.
a. CORRECT Synesthesia occurs when two or more senses are automatically and involuntarily joined so that stimulation of one sense automatically elicits another sense (e.g., a certain sound elicits a specific color).
b. Incorrect Somesthesis refers to the sensory perception associated with the skin, muscles, joints, and viscera and includes touch-pressure, warmth-coolness, and proprioception.
c. Incorrect Anosognosia is a lack of awareness of one’s own disability or illness.
d. Incorrect Alexithymia is an inability to describe one’s own feelings.
(Physiological & Psychopharm)

359
Q
When an individual’s alcohol consumption affects the normal functioning of his or her cerebellum, the individual may exhibit which of the following?
A. ataxia
B. aphagia
C. akathisia 
D. apraxia
A

Knowing that the cerebellum is involved in balance and posture would have helped you identify the correct answer to this question.
a. CORRECT Ataxia involves the loss of ability to coordinate voluntary movements and can be caused by a variety of factors including damage to the cerebellum, the ingestion of certain drugs, and diseases that affect muscle
coordination (e.g., cerebral palsy).
b. Incorrect Aphagia (also known as dysphagia) refers to difficulty in swallowing or eating.
c. Incorrect Akathisia is characterized by motor restlessness, especially in the arms and legs. It is one of the side effects of the neuroleptic drugs.
d. Incorrect Apraxia involves an inability to perform skilled movements that is not due to muscle weakness, a loss of motor coordination, sensory loss, or a lack of comprehension or cooperation.
(Physiological & Psychopharm)

360
Q

An agonist drug exerts its effects by:
A. blocking an endogenous neurotransmitter’s access to a receptor site.
B. exerting the opposite effect of an endogenous neurotransmitter.
C. facilitating the reuptake of an endogenous neurotransmitter.
D. mimicking the effect of an endogenous neurotransmitter at a receptor site.

A

Agonists produce effects similar to neurotransmitters.
a. Incorrect This is one of the ways that an antagonist drug exerts its effects.
b. Incorrect This answer describes the action of an inverse agonist.
c. Incorrect This does not describe the action of an agonist drug.
d. CORRECT Agonist drugs exert their effects by mimicking a neurotransmitter and thereby activating or stimulating receptor cells or by facilitating the action of
a neurotransmitter at receptor sites.
(Physiological & Psychopharm)

361
Q
Disinhibition, apathy, and deficits in executive functioning are most associated with damage to the \_\_\_\_\_ lobes.
A. parietal
B. frontal
C. temporal
D. occipital
A

Familiarity with the primary functions of each of the lobes of the cerebral cortex would have helped you identify the correct answer to this question.
a. Incorrect Damage to the parietal lobes affects somatosensory functioning.
b. CORRECT The prefrontal area of the frontal lobes is involved in a variety of complex behaviors including emotions, memory, self-awareness, and executive
cognitive functions. Depending on its location, damage to this area can cause disinhibition (impulsivity, loss of tact], apathy (limited motivation and initiative) and/or deficits in higher-order cognitive skills.
c. Incorrect Temporal lobe damage may cause memory loss and auditory deficits.
d. Incorrect Damage to the occipital lobes produces disturbances in visual processing.
(Physiological & Psychopharm)

362
Q

One of the characteristic symptoms of Kluver-Bucy syndrome is agnosia, which involves:
A. abnormalities in motor functioning.
B. alterations in dietary habits.
C. an inability to recognize familiar objects.
D. an impaired sense of touch.

A

Note that this question is asking specifically about agnosia, not the Kluver-Bucy syndrome.
a. Incorrect This is known as parakinesia.
b. Incorrect Kluver-Bucy syndrome does involve an alteration in dietary habits, but this is not referred to as agnosia.
c. CORRECT Agnosia (“psychic blindness”) is a characteristic symptom of Kluver-Bucy syndrome and involves an inability to recognize or interpret visual stimuli. Other symptoms of Kluver-Bucy syndrome include reduced fear, increased docility, compulsive oral behaviors, markedly different dietary habits, and
hypersexuality.
d. Incorrect This is referred to as dysaphia.
(Physiological & Psychopharm)

363
Q
Selye (1956) proposed that which of the following endocrine glands mediate the general adaptation syndrome?
A. thyroid and thymus
B. adrenal and pituitary
C. thyroid and parathyroid
D. pancreas and pineal
A

According to Selye (1956), the general adaptation syndrome consists of three stages: alarm, resistance, and exhaustion.
b. CORRECT Selye proposed that the general adaptation syndrome is mediated primarily by the adrenal and pituitary glands. For example, during the initial alarm stage, the hypothalamus activates the adrenal medulla which increases the production of epinephrine.(Physiological & Psychopharm)

364
Q
The effectiveness of clomipramine as the treatment for obsessive-compulsive disorder (OCD) provides support for the hypothesis that this disorder is related to abnormalities in brain levels of:
A. acetylcholine.
B. epinephrine.
C. dopamine. 
D. serotonin.
A

Clorniprarnine and other antidepressants exert their effects by altering levels of norepinephrine, serotonin, and/or dopamine.
D. CORRECT A current theory about the cause of OCD is that it stems from low serotonin levels. This theory is supported by the fact that clomipramine (a tricyclic antidepressant that has anti-obsessional effects) blocks the reuptake of serotonin by presynaptic neurons.
(Physiological & Psychopharm)

365
Q

The neurotransmitter acetylcholine:
A. is involved in voluntary movements.
B. decreases alertness and increases the duration of sleep
C. is implicated in the development of the psychotic disorders
D. is involved in the control of pain.

A

Acetylcholine (ACh) has been linked to several functions including control of skeletal and smooth muscles, learning and memory, and the activities of the parasympathetic nervous system.
a. CORRECT The skeletal muscles, which are involved in voluntary movement, are activated by the release of ACh by motor neurons.
b. Incorrect This due to the effects of serotonin.
c. Incorrect This describes dopamine.
d. Incorrect This describes the endorphins.
(Physiological & Psychopharm)

366
Q
As the result of a brain tumor, Alma A. is no longer able to recognize her family members or friends by looking at their faces. Alma's condition is known as:
A. prosopoplegia.
B. anosmia.
C. prosopagnosia.
D. anosognosia.
A

Alma is unable to recognize familiar faces, a condition that is caused by damage to certain areas of the occipital and temporal lobes.
a. Incorrect Prosopoplegia is facial paralysis.
b. Incorrect Anosmia is the absence of the sense of smell.
c. CORRECT Prosopagnosia is a type of visual agnosia that involves an inability to recognize familiar faces, often including one’s own face.
d. Incorrect Anosognosia is the inability to recognize (or the denial of) one’s own neurological symptoms.
(Physiological & Psychopharm)

367
Q

Which of the following is incorrectly matched?
A. migraine headache: unilateral throbbing pain, nausea, irritability, and pain that worsens with routine physical activity
B. cluster headache: dull throbbing pain that begins with an aura and is exacerbated by bright light
C. tension headache: bilateral steady dull pain and a feeling of tightness in the head that may be accompanied by sleep disturbances and impaired concentration
D. sinus headache: throbbing pain that is localized around the eyes and is made worse by bending over

A

The various types of headache listed in the answers involve different symptoms. Note that this question is asking which type of headache is described incorrectly.

a. Incorrect This accurately describes a migraine headache.
b. CORRECT A cluster headache is characterized by sharp, penetrating, or burning pain behind the eye or in the temple, a stuffy or runny nose, and a red, flushed face.
c. Incorrect This accurately describes a tension headache.
d. Incorrect This accurately describes a sinus headache.(Physiological & Psychopharm)

368
Q
When used as a treatment for ADHD, the most common initial side effects of methylphenidate (Ritalin) are:
A. skin rash and itching.
B. paresthesias.
C. joint pain and muscle cramps.
D. insomnia and reduced appetite.
A

This is a relatively easy question as long as you’re familiar with the common side effects of methylphenidate and other CNS stimulants.
a. Incorrect Skin rash and itching may occur but are less common that insomnia and appetite loss.
b. Incorrect Paresthesias are not a side effect associated with methylphenidate.
c. Incorrect Joint pain and muscle cramps may also occur, but they are much less common than insomnia and appetite loss.
d. CORRECT For most individuals taking methylphenidate as a treatment for ADHD, the drug’s initial side effect is difficulty sleeping. Other common side effects include decreased appetite, nervousness, irritability, and nausea and abdominal pain.
(Physiological & Psychopharm)

369
Q

As defined in DSM-IV-TR, postconcussional disorder (syndrome) is characterized by which of the following symptoms?
A. diaphoresis, dysphagia, confusion, and tachycardia
B. slurred speech, incoordination, nystagmus, and impaired memory
C. dysphoria, nausea and vomiting, diarrhea, and insomnia
D. headache, vertigo or dizziness, sleep disturbances, and affective lability

A

Postconcussional disorder is included in the DSM-IV-TR with “Criteria Sets and Axes Provided for Further Study.”
a. Incorrect These are characteristic symptoms of neuroleptic malignant syndrome.
b. Incorrect These are symptoms of alcohol intoxication.
c. Incorrect These are characteristic symptoms of opioid withdrawal.
d. CORRECT A diagnosis of postconcussional disorder requires a history of closed head trauma that ca used significant cerebral concussion, disturbances in
attention or memory, and at least three characteristic symptoms that have persisted for at least three months. The characteristic symptoms include those listed in this answer. Other symptoms are becoming fatigued easily, apathy or reduced spontaneity, and changes in personality.
(Physiological & Psychopharm)

370
Q
\_\_\_\_\_ involves partial or complete colorblindness clue either to a loss of functioning of the cone cells or damage to the occipitotemporal region of the brain.
A. Anosognosia
B. Alexia
C. Achromatopsia
D. Akinetopsia
A

For the exam, you’ll want to be familiar with the characteristics of all of the conditions listed in the answer to this question.
a. Incorrect Anosognosia is the failure to recognize one’s own neurological symptoms.
b. Incorrect Alexia is an inability to read or understand written words.
c. CORRECT Even if you’re unfamiliar with achromatopsia, you would have been able to identify this as the correct answer if you know that “chroma” refers to
color.
d. Incorrect Akinetopsia is an inability to see objects that are in motion.
(Physiological & Psychopharm)

371
Q

For the assessment of Huntington’s disease, magnetic resonance imaging (MRI) is:
A. not useful because it is not sufficiently sensitive to detect the brain anomalies associated with this disorder.
B. not useful because it provides information only on brain morphology, which is not helpful for assessing this disorder.
C. useful for distinguishing between active and older inactive plaques in the periventricular regions.
D. useful for detecting volume reduction in the basal ganglia, even in some asymptomatic individuals.

A

Knowing that Huntington’s disease is caused by abnormalities in the basal ganglia would have helped you identify the correct answer to this question.
a. Incorrect An advantage of MRI is that it’s very sensitive to anomalies and has better resolution than a CT scan.
b. Incorrect As noted below, information about structure is useful in the diagnosis of Huntington’s disease.
c. Incorrect Plaques in the periventricular regions are characteristic of multiple sclerosis, not Huntington’s disease.
d. CORRECT Structural and functional brain imaging techniques have both been used to assess brain pathology related to Huntington’s disease. There is
evidence that MRI (a structural technique) can identify reduced volume in the basal ganglia among genetically at-risk individuals who have not yet exhibited overt symptoms of the disorder.
(Physiological & Psychopharm)

372
Q

The neurotransmitter glutamate is believed to be responsible for:
A. the effects of alcohol on cognitive abilities.
B. the increased sociability that sometimes accompanies alcohol use
C. alcohol cravings in alcohol abusers.
D. dietary changes associated with chronic alcohol use.

A

Glutamate plays a role in learning and memory and, more specifically, in long-term potentiation which is believed to mediate the formation of long-term memories.
a. CORRECT Alcohol alters glutamate levels in the brain, which can cause memory impairment and may play a role in alcohol-related blackouts.
b. Incorrect GABA is an inhibitory neurotransmitter; and alcohol enhances GABA activity which may contribute to the increased sociability associated with alcohol consumption.
c. Incorrect Serotonin is believed to be involved in alcohol craving.
d. Incorrect Dietary changes have not been linked to glutamate.
(Physiological & Psychopharm)

373
Q
Prospective memory is the ability to remember to execute an intended act in the future and is believed to be supported by activity in which of the following areas of the brain?
A. basal ganglia
B. corpus callosum
C. prefrontal cortex
D. raphe nuclei
A

A number of brain regions and structures are involved in memory including the hippocampus, amygdala, thalamus, basal ganglia, and prefrontal cortex.
a. Incorrect The basal ganglia play a role in procedural and implicit memory.
b. Incorrect The corpus callosum has not been identified as a mediator of prospective memory.
c. CORRECT Areas in the prefrontal cortex play an important role in several aspects of memory including prospective, episodic, and working memory.
d. Incorrect The raphe nuclei are located in the reticular Formation and play a role in sleep and arousal that may be involved in depression.
(Physiological & Psychopharm)

374
Q
Drowsiness (sedation) is most likely to be an initial side effect of which of the following antidepressants?
A. fluoxetine (Prozac) 
B. paroxetine (Paxil)
C. sertraline (Zoloft)
D. doxepin (Sinequan)
A

The tricyclic antidepressants are, in general, more sedating than the SSRIs.
d. CORRECT Doxepin is a tricyclic antidepressant, and drowsiness is one of the most common side effects of this drug (although it tends to decrease over time) In contrast, the drugs listed in answers a, b, and c are SSRIs and are less likely than doxepin to produce sedation.
(Physiological & Psychopharm)

375
Q

As the result of a head injury sustained in a car accident, a young woman exhibits both retrograde and anterograde amnesia. Once her memories begin to return, which of the following is likely to be recovered first?
A. memory for events that occurred several weeks before the accident
B. memory for events that occurred during the hour before the accident
C. memory for events related to the accident
D. memory for events that occurred after the accident

A

The pattern of memory impairment and recovery following head injury was originally described by Ribot in 1882.
a. CORRECT Recent memories (including memory for the traumatic event) are most susceptible to disruption as the result of trauma. Remote memories are less affected and, when they are lost, are ordinarily the first to be recovered.
(Physiological & Psychopharm)

376
Q

The most common cause of cerebral palsy is:
A. a congenital endocrine dysfunction.
B. brain damage during fetal development.
C. nutritional deficiencies immediately following birth.
D. early exposure to a teratogen that affects the body’s muscles

A

Cerebral palsy includes a number of neurological disorders that permanently impair the ability to control voluntary muscle movements. In most cases of cerebral palsy, the exact cause is unknown.
b. CORRECT Cerebral palsy is the result of brain damage, and, in the majority of cases, the damage occurs prior to birth. One cause of the damage is a lack of oxygen (asphyxia) prior to birth or during the birth process. Other possible causes include exposure to infections, head injury, intracranial hemorrhage, and genetic mutations.
(Physiological & Psychopharm)

377
Q
Sweating, speeded up mental processes, inability to sleep or relax, trembling, nervousness, palpitations, and loss of weight despite increased food intake are characteristic of which of the following endocrine disorders?
A. hyperglycemia
B. hyperthyroidism
C. adrenal insufficiency
D. hypoparathyroidism
A

For the licensing exam, you want to be familiar with the symptoms of the endocrine disorders listed in the answers to this question.
a. Incorrect Hyperglycemia is due to excessive glucose in the blood. Its symptoms include fatigue, polydipsia, polyuria, polyphagia, weight loss, poor wound healing, and recurrent infections.
b. CORRECT The symptoms listed in this question are characteristic of hyperthyroidism, which is ca used by excessive secretion of thyroxin by the thyroid gland.
c. Incorrect Adrenal insufficiency (also known as Addison’s disease) is due to a lower-than-normal production of cortisol by the adrenal glands and is
characterized by weight loss, orthostatic hypotension, weakness, fatigue, and hyperpigmentation.
d. Incorrect Hypoparathyroidism is ca used by lower-than-normal levels of parathyroid hormone. Its symptoms include muscle cramping and twitching, tingling in the lips and fingers, hair loss, and dry skin.
(Physiological & Psychopharm)

378
Q
After the image of an object is projected exclusively to Olivia O.'s left visual field (right hemisphere), Olivia is unable to verbally name the object but can pick the object out of a group of objects hidden from view with her left hand. Damage to which of the following would explain this phenomenon?
A. visual association cortex
B. prefrontal association cortex
C. arcuate fasciculus
D. corpus callosum
A

Olivia’s behavior demonstrates the lateralization of function in the brain. Olivia’s inability to verbally name the object demonstrates that its name was not transferred from her right to left hemisphere (which controls language). However, her ability to select the object with her left hand (which is controlled by the right hemisphere) demonstrates that her recognition of the object was not impaired.
a. Incorrect Damage to the visual association cortex can result in visual agnosia (an inability to recognize a familiar object by sight). Olivia recognizes the object but cannot verbally name it.
b. Incorrect The prefrontal association cortex is involved in complex functions related to perception, emotion, memory, and thinking. Damage to this area may produce subtle personality and behavioral changes (e.g., apathy,
impulsivity, inappropriate silliness).
c. Incorrect The arcuate fasciculus connects the brain’s receptive language center (Wernicke’s area) with its expressive language center (Broca’s area). Damage to the arcuate fasciculus ca uses conduction aphasia.
d. CORRECT The corpus callosum is the major bundle of fibers that connects the right and left hemispheres of the brain, and damage to this structure reduces interhemispheric communication and can produce the behavior described in this question.
(Physiological & Psychopharm)

379
Q

Damage to the right hippocampus is most likely to cause:
A. impairments in motor skills involving the left side of the body
B. interruption of the normal sleep-wake cycle.
C. deficits in spatial memory.
D. deficits in procedural memory.

A

The hippocampus is vital to learning and memory.
a. Incorrect The hippocampus is involved in declarative memory (learning of new Facts, words, names, etc.) but is not involved in the acquisition or performance of motor skills.
b. Incorrect The research has not linked the hippocampus to the sleep-wake cycle.
c. CORRECT The right hippocampus is involved in memory for nonverbal information, including spatial and temporal memory. For example, in a study of cabdrivers, PET scans revealed that the right hippocampus was responsible for the drivers’ ability to Form and recall complex routes.
d. Incorrect The hippocampus plays a role in declarative memory but does not appear to be responsible for the Formation of procedural memories.
(Physiological & Psychopharm)

380
Q
The hypothalamus is believed to be involved in all of the following functions EXCEPT:
A. hunger and thirst.
B. sexual arousal.
C. sleep-wake cycle.
D. vision and audition.
A

The hypothalamus, although small in size, is one of the most important areas of the brain.
d. CORRECT The hypothalamus mediates a wide range of autonomic, endocrine, and behavioral functions including emotional states, the sleep-wake cycle, hunger and thirst, sexual behavior and reproduction, and temperature.
However, it is not involved in vision or audition.(Physiological & Psychopharm)

381
Q
Which of the following would be most useful for alleviating an attack of hyperventilation?
A. an ice pack
B. an antacid
C. a glass of water
D. a paper bag
A

Hyperventilation is a rapid deep-breathing attack that involves a drop in carbon dioxide, leading to respiratory alkalosis and cerebral hypoxia. Symptoms include chest pain, shortness of breath, tingling and numbness in the hands and feet, dizziness, and impaired concentration
and memory.
d. CORRECT Breathing into a paper bag alleviates hyperventilation by increasing the level of carbon dioxide in the bloodstream.
(Physiological & Psychopharm)

382
Q
Soon after starting to take an antidepressant, a young woman develops several undesirable side effects including dry mouth, urinary retention, constipation, and blurred vision. Which of the following would be most useful for alleviating these symptoms?
A. caffeine
B. antihistamine
C. dopamine agonist
D. cholinergic agonist
A

The woman’s symptoms are anticholinergic side effects that are caused by the antidepressant drug’s interference with acetylcholine activity.
d. CORRECT Cholinergic agonists (e.g., bethanechol) increase acetylcholine activity by enhancing the activity of endogenous acetylcholine or by binding to and activating acetylcholine receptors. A cholinergic agonist would help alleviate the woman’s anticholinergic side effects.
(Physiological & Psychopharm)

383
Q
The onset of puberty in humans occurs when gonadotropin-releasing hormone (GnRH) is secreted by the:
A. hippocampus
B. hypothalamus
C. adrenal glands
D. thalamus
A

At puberty, the gonads (testes and ovaries) produce hormones that, in turn, are responsible for physical sexual maturation.
b. CORRECT Secretion of gonadotropin-releasing hormone by the hypothalamus stimulates production and release of the gonadotropic hormones by the pituitary
gland. These hormones then stimulate the gonads to release the sex hormones which trigger sexual development.
(Physiological & Psychopharm)

384
Q
A(n) seizure is characterized by a loss of consciousness without prominent motor symptoms.
A. atonic
B. absence
C. clonic
D. simple partial
A

Each type of seizure listed in the answers is associated with a different etiology and combination of symptoms.
a. Incorrect Atonic seizures involve a sudden loss of muscle tone that may cause the individual to fall down. The individual usually remains conscious during the
seizure.
b. CORRECT The question accurately describes an absence (petit mal) seizure.
c. Incorrect Clonic seizures are characterized by jerky movements.
d. Incorrect A simple partial seizure begins on one side of the brain and, at least initially, affects only one side of the body. It causes changes in movement and sensation but does not produce a loss of consciousness.
(Physiological & Psychopharm)

385
Q
As the result of a brain injury, Walter W. cannot recognize familiar objects by touch. Most likely, Walter has damage to his lobes.
A. parietal
B. occipital
C. temporal
D. frontal
A

Walter is exhibiting somatosensory (tactile) agnosia.
a. CORRECT The parietal lobes contain the somatosensory cortex, which processes information from the skin senses (touch, temperature, pressure, etc.)
Damage to the parietal lobes may produce somatosensory agnosia which is characterized by a lack of awareness of one side of the body andfor an inability
to recognize familiar objects by touch.
(Physiological & Psychopharm)

386
Q

When taken by an older adult as a treatment for bipolar disorder, lithium is most likely to produce which of the following side effects?
A. fine hand tremor, polyuria, and confusion
B. elevated blood pressure and tachycardia
C. nausea, appetite loss, and headache
D. confusion, memory loss, and ataxia

A

As a general rule, older adults are more susceptible than younger adults to the therapeutic and side effects of many drugs.
a. CORRECT Relatively low doses of lithium can produce these side effects in older adults.
b. Incorrect Stimulant drugs are associated with these side effects.
c. Incorrect These are side effects of the SSRIs.
d. Incorrect These are common side effects of the benzodiazepines.
(Physiological & Psychopharm)

387
Q

Endocrine disorders often mimic psychiatric disorders. For example hypothyroidism often includes symptoms suggestive of:
A. a phobic disorder, delirium, or mania.
B. mania, anorexia, or OCD.
C. OCD, depression, or a phobic disorder.
D. depression, psychosis, or dementia.

A

Hypothyroidism and other endocrine disorders may produce symptoms that are characteristic of certain psychiatric disorders.
d. CORRECT The symptoms of hypothyroidism (e.g., impaired memory and concentration, depression, apathy, confusion, thought disorders, hallucinations) overlap with those associated with depression, psychosis, and dementia.
(Physiological & Psychopharm)

388
Q
The is the large band of nerve fibers that connects the left and right cerebral hemispheres and permits interhemispheric communication of sensory, motor, and higher-order information.
A. corpus callosum
B. arcuate fasciculus
C. cingulate gyrus
D. globus pallidus
A

For the exam, you want to be familiar with the primary functions of the brain structures listed in the answers to this question.
a. CORRECT The corpus callosum is the largest band of fibers connecting the two hemispheres of the brain. Abnormalities in corpus callosum Functioning have
been linked to several disorders including dyslexia, ADH D, Tourette’s syndrome, and schizophrenia.
b. Incorrect The arcuate Fasciculus connects Wernicke’s and Broca’s areas and carries signals related to the production and comprehension of language.
c. Incorrect The cingulate gyrus is part of the limbic system and is believed to be responsible for Focusing attention on emotionally significant events, linking
sensory input to emotions, mediating emotional reactions to pain, and regulating aggressive behavior.
d. Incorrect The globus pallidus is part of the basal ganglia and is involved in motor control.
(Physiological & Psychopharm)

389
Q
\_\_\_\_\_ is caused by an abnormal accumulation of cerebrospinal fluid in the ventricles.
A. Spina bifida
B. Meningitis
C. Hydrocephalus
D.Encephalitis
A

Knowing that hydrocephalus means “water head” would have helped you identify it as the correct answer to this question.
a. Incorrect Spinal bifida is a neural tube defect caused by a Failure of the Fetus’s spine to close appropriately during early development.
b. Incorrect Meningitis is inflammation of the membranes of the brain and spinal cord and is a possible cause of acquired hydrocephalus.
c. CORRECT The accumulation of cerebrospinal Fluid in the ventricles (usually as the result of an obstruction) causes hydrocephalus which results in increased
intracranial pressure and can lead to brain damage.
d. Incorrect Encephalitis is an inflammation of the brain that is usually caused by a virus.
(Physiological & Psychopharm)

390
Q

The neurotransmitter GABA is a(n):
A. amino acid that plays an inhibitory role regardless of its location in the brain.
B. amino acid that plays an excitatory role regardless of its location in the brain.
C. amino acid that plays an inhibitory or excitatory role, depending on its location in the brain.
D. neuropeptide that plays an inhibitory or excitatory role, depending on its location in the brain.

A

The neurotransmitters can be classified in terms of their chemical make-up and their effects on post-synaptic neurons.
a. CORRECT Many neurotransmitters serve both excitatory and inhibitory functions. GABA, an amino acid, is one of the neurotransmitters that serves only one of these functions — i.e., it is an inhibitory neurotransmitter.
d. Incorrect GABA is an amino acid, not a neuropeptide. (The neuropeptide neurotransmitters include substance P and the endorphins.)
(Physiological & Psychopharm)

391
Q
Nausea, stomach cramps, excessive thirstiness, increased frequency of urination, muscle weakness, impaired memory, fine hand tremor, and weight gain are side effects most associated with which of the following?
A. Lithium
B. carbarnazepine
C. methylphenidate
D. fluoxefine
A

Unfortunately, some of the symptoms included in the question are side effects of more than one of the drugs listed in the answers, which makes this a difficult question.
a. CORRECT The symptoms given in the question are potential side effects of lithium, which is used to treat bipolar disorder.
b. Incorrect Carbamazepine (Tegretol), an anticonvulsant, is also used to treat bipolar disorder. However, its common side effects include dizziness, drowsiness, nausea and vomiting, headache, and ataxia.
c. Incorrect Methylphenidate, a CNS stimulant, is associated with insomnia, reduced appetite, nervousness, tremor, and cardiac arrhythmias.
d. Incorrect Nausea, appetite loss, anxiety, tremor, dry mouth, sweating, tiredness or weakness, and disturbances in sexuality are common side effects of
fluoxetine and other SSRIs.
(Physiological & Psychopharm)

392
Q

Which of the following is incorrectly matched?
A. alpha waves: relaxed alertness
B. beta waves: waking consciousness
C. delta waves: deep sleep
D. theta waves: sudden arousal from sleep

A

Each of the brain waves listed in the answers is associated with a different physiological state. Note that this question is asking which response represents an incorrect match.
a. Incorrect Alpha waves predominate during states of relaxed wakefulness.
b. Incorrect Beta waves predominate during states of alert wakefulness.
c. Incorrect Delta waves occur during periods of deep sleep.
d. CORRECT Theta waves occur when a person is in a deeply relaxed, drowsy state. Theta waves predominate during Stage 2 sleep.
(Physiological & Psychopharm)

393
Q

When reviewing the data she has collected from her study’s three treatment groups, a researcher discovers that the standard deviations of the three groups differ. If the researcher is planning to use the t-test for independent samples to analyze her data, this is:
A. not a problem.
B. not likely to be a problem as long as the three groups have the same number of participants.
C. not likely to be a problem as long as all three distributions have a rectangular shape.
D. not likely to be a problem as long as the data represent an interval or ratio scale of measurement.

A

The parametric tests are fairly “robust” with regard to a violation of the assumptions that should be met in order to use them, and their robustness is increased under certain conditions.
b. CORRECT When groups are equal in size, a violation of the assumption of homogeneity of variances will probably not substantially affect the results of the t-test.
c. Incorrect One of the assumptions of the t-test is that the distribution of scores is normally shaped.
cl. Incorrect Use of the t-test requires that the data represent an interval or ratio scale.
(Stats & Research Design)

394
Q
If you want to determine the degree of association between variable X and variable Y when the effects of a third variable (Z) have been statistically removed from both X and Y, you would use:
A. zero-order correlation.
B. second-order correlation.
C. multiple correlation.
D. partial correlation.
A

You may have been able to identify the correct answer to this question even if you have never heard of partial correlation since it sounds like what it is.

a. Incorrect A zero-order correlation is the correlation between two variables.
b. Incorrect In the context of correlation, “order” refers to the number of variables that are being controlled for. A first-order correlation controls for one other variable (which is what is described in this question), while a second-order correlation controls for two other variables.
c. Incorrect A multiple correlation coefficient is the correlation between three or more variables.
d. CORRECT Partial correlation, a type of first-order correlation, is used to determine the degree of association between two variables when the effects of a third variable have been removed from both X and Y.(Stats & Research Design)

395
Q
Autocorrelation is most likely to be a problem when using which of the following research designs?
A. time-series
B. factorial
C. between groups
D. Solomon four-group
A

Autocorrelation refers to the correlation between measurements of the dependent variable when it is repeatedly administered to the same participants. Autocorrelation is a problem in repeated measures designs because it can artificially inflate the value of the inferential statistic and thereby increase the probability of making a Type I error.
a. CORRECT Of the research designs listed in the answers, repeated measurement of the dependent variable is characteristic only of the time series design, which is a type of repeated measures (within subjects) design.
b. Incorrect Factorial designs include two or more independent variables and do not necessarily involve repeated measurements of the dependent variable.
(Stats & Research Design)

396
Q

If a teacher adds 10 points to each score in a distribution of scores, this will:
A. affect the mean of the distribution but not its standard deviation or variance.
B. affect the mean and range of the distribution but not its standard deviation or variance.
C. affect the standard deviation and variance of the distribution but not its mean.
D. not affect the mean, range, standard deviation, or variance of the distribution.

A

For the exam, you want to be familiar with the effects of adding or subtracting a constant to each score in a distribution or multiplying or dividing each score by a constant.
a. CORRECT Adding a constant to each score in the distribution affects the distribution’s mean (the new mean is the original mean plus the constant). However, adding a constant does not affect the variability of scores — i.e., it does not change the range, standard deviation, or variance.
(Stats & Research Design)

397
Q

“Sampling error” refers to:
A. the tendency of sample statistics to vary from one another due to unintended effects of the treatment.
B. the tendency of sample statistics to vary from population parameters due to the effects of systematic error.
C. the tendency of sample statistics to vary from population parameters due to the effects of random error.
D. the tendency of sample statistics to vary from population parameters due to the unreliability of the measuring instruments.

A

Sampling error is due to chance variations that can be expected in samples that have been randomly drawn from the population.
c. CORRECT Sampling error refers to the discrepancies between sample values and corresponding population values (parameters) that are due to chance factors in the selection process. Note that, by definition, sampling error is random, not systematic.
d. Incorrect Error due to the unreliability of the measuring instruments is referred to as measurement error, not sampling error.
(Stats & Research Design)

398
Q

You would use the Solomon four-group design in order to:
A. eliminate carryover effects.
B. reduce demand characteristics.
C. evaluate the impact of pretesting.
D. evaluate the effects of history and maturation.

A

The Solomon four-group design combines the pretest-posttest control group design with the posttest only control group design.
c. CORRECT The purpose of the Solomon Four-group design is to evaluate the impact of pretesting on a study’s internal and external validity.
(Stats & Research Design)

399
Q

Heteroscedasticity in a scattergram suggests that:
A. the relationship between the predictor and criterion cannot be described by a straight line.
B. there is a restriction of range of scores on the predictor and/or the criterion.
C. the variability (range) of scores on the criterion varies for different scores on the predictor.
D. there is a statistically significant correlation between the predictor and criterion.

A

“Heteroscedasticity” means unequal variability.
a. Incorrect This is not a conclusion that can be drawn from knowing that there is heteroscedasticity in a scattergram.
b. Incorrect This is not a conclusion that can be drawn from knowing that there is heteroscedasticity in a scattergram.
c. CORRECT Heteroscedasticity is occurring when variability of scores on the criterion differs for different scores on the predictor — e.g., when there is a narrow range of criterion scores for low and high predictor scores but a wide range of criterion scores for moderate predictor scores.
d. Incorrect This is not a conclusion that can be drawn from knowing that there is heteroscedasticity in a scattergram.
(Stats & Research Design)

400
Q

The scattergram for scores on a measure of test anxiety and a measure of math achievement reveals that students with moderate levels of anxiety obtain the highest achievement scores, while students with low and high levels of anxiety obtain lower achievement scores. If the Pearson r is used to measure the degree of the association between test anxiety and math achievement scores, the resulting correlation coefficient will:
A. underestimate the relationship between the variables.
B. overestimate the relationship between the variables.
C. under or overestimate the relationship between the variables.
D. accurately estimate the relationship between the variables.

A

An assumption underlying the use of the Pearson r is that the relationship between variables is linear.
a. CORRECT In this situation, the relationship between the variables in nonlinear (i.e., the assumption of linearity has been violated). As a result, the Pearson r will underestimate the relationship between test anxiety scores and math achievement scores.
(Stats & Research Design)

401
Q
Research participants act as their own no-treatment “controls" in which of the following types of research?
A. single-subject
B. cross-sectional
C. ex post facto
D. double-blind
A

In research, a control group is a comparison group that does not receive treatment or, alternatively, receives a standard treatment.
a. CORRECT In single-subject research, a participant’s behavior during the baseline (no treatment) and treatment phases is compared. Thus, when using single-subject designs, participants act as their own no-treatment controls.(Stats & Research Design)

402
Q
In a cross-sectional study designed to assess the effects of age on reaction time, you have adults belonging to different age groups respond to visual stimuli on a computer screen. A colleague suggests that the results of your study might be confounded by the fact that older participants may have less experience using a computer than do younger participants. This potential source of error is best described as a:
A. Rosenthal effect.
B. Hawthorne effect.
C. carryover effect.
D. cohort effect.
A

Your colleague is predicting that your research findings will be confounded by differences in computer experience that are related to differences in age. This is often a problem in cross-sectional research.
a. Incorrect The Rosenthal effect is another name for the self-fulfilling prophecy effect and is not relevant to this situation.
b. Incorrect The Hawthorne effect refers to the tendency of research participants to act in atypical ways because they are participating in a research study (e.g., because of the novelty of the situation or the special attention they are receiving).
c. Incorrect A carryover effect is a potential problem in within-subjects research. It occurs when exposure to one level of an independent variable impacts how participants respond to another level of the independent variable.
d. CORRECT “Cohort effect” refers to the confounding effects of intergenerational differences in experience or other characteristics. Cohort effects can be a source of error in cross-sectional studies (such as the study
described in this question).
(Stats & Research Design)

403
Q
For ethical reasons, a researcher would be most likely to use which of the following single-subject designs when conducting a study to evaluate the effects of a behavioral treatment for the control of severe self-destructive behaviors?
A. reversal
B. multiple sequential withdrawal
C. multiple baseline
D. Latin square
A

When evaluating a treatment to eliminate self-destructive behaviors, a researcher would not want to use a research design that requires removal or withdrawal of a successful treatment during the course of
the study.
a. Incorrect Reversal designs (e.g., ABA, ABAB) require withdrawal of the treatment during subsequent baseline (A) phases and, therefore, would not be an appropriate design in this situation.
b. Incorrect As its name implies, this design also requires withdrawal of the treatment and, therefore, would not be appropriate.
c. CORRECT The multiple baseline design would be an acceptable design in this situation since it does not require withdrawal of the treatment from a baseline once it has been applied to that baseline.
d. Incorrect The Latin square design is not a single-subject design but is a group design that is used to control order effects.
(Stats & Research Design)

404
Q

Cluster analysis would be most useful for:
A. developing a classification scheme for individuals with various combinations of depressive symptoms.
B. testing causal hypotheses about the factors that underlie the development of depression.
C. identifying the optimal combination of tests to use to predict an individual’s risk for developing major depressive disorder.
D. systematically recording behavioral data while observing individuals with depressive symptoms.

A

As its name suggests, cluster analysis is used to “cluster” or categorize individuals. Cluster analysis can be either exploratory or confirmatory. In the first case, the goal is to identify clusters of people or other
variables; in the latter, the goal is to confirm a pre-existing classification scheme.
a. CORRECT Of the situations described in the answers, this one is best suited for cluster analysis, which is useful for confirming or developing a classification scheme.
b. Incorrect Cluster analysis is not used to test causal hypotheses.
c. Incorrect Cluster analysis would not be useful for this purpose.
d. Incorrect Cluster analysis is a type of statistical analysis and is not used record behavioral observations.
(Stats & Research Design)

405
Q
A teacher converts students’ raw scores on a 200-item test to percent correct scores. The converted scores represent which scale of measurement?
A. ordinal 
B. interval
C. ratio
D. nominal
A

Be careful not to confuse percentile ranks and percent correct scores.
c. CORRECT Percent correct scores represent a ratio scale of measurement - i.e., they have the properties of order, equal intervals, and an absolute 0 point Consequently, it is possible to conclude that someone who obtains a score of 50% got twice as many items correct as someone whose score is 25%.
(Stats & Research Design)

406
Q

An analysis of covariance (ANCOVA) is used to:
A. analyze the effects of an independent variable on two or more dependent variables.
B. statistically control variability in the dependent variable due to the effects of an extraneous variable.
C. adjust for systematic differences between groups that occurred as part of the experimental treatment.
D. determine if there is a significant interaction between two or more independent variables.

A

The ANCOVA is used to statistically remove the effects of an extraneous variable on the dependent variable.
a. Incorrect This describes the MANOVA.
b. CORRECT When using the ANCOVA, the extraneous variable is the “covariate,” and its effects on the dependent variable are statistically removed so that it is easier to detect the effects of the independent variable on the dependent variable.
c. Incorrect This does not describe the ANCOVA.
d. Incorrect A factorial ANOVA is useful for determining if there is an interaction between independent variables. (There is a factorial ANCOVA that can be used when a study includes two or more independent variables, but this question is asking about the ANCOVA more generally.)
(Stats & Research Design)

407
Q
Zelda Z. obtains a score of 41 on a test that has a mean of 50 and a standard deviation of 6. If all of the scores in the distribution are transformed so that the test now has a mean of 100 and a standard deviation of 12, Zelda’s score in the new distribution would be:
A. 91.
B. 82.
C. 41.
D. 20.5.
A

To identify the correct answer to this question, you’d have to recognize that Zelda’s original score was 1-1/2 standard deviations below the mean.
b. CORRECT A score of 82 is 1-1/2 standard deviations below the mean of the new distribution and, therefore, equivalent to a score of 41 in the original distribution.
(Stats & Research Design)

408
Q

Parametric statistical tests are usually preferable to non-parametric tests because they are more “powerful.” This means that the use of a parametric test to analyze the data collected in a research study helps ensure that:
A. a false null hypothesis will be retained.
B. a true null hypothesis will be retained.
C. a false null hypothesis will be rejected.
D. the null hypothesis will be rejected.

A

Statistical power is the ability to detect a false null hypothesis.
a. Incorrect This answer describes a Type II error.
b. Incorrect This is not a description of power.
c. CORRECT This is an accurate description of power.
d. Incorrect Power refers to the ability to reject a false null hypothesis, not just to the ability to reject any null hypothesis. Therefore, answer c is the best response.
(Stats & Research Design)

409
Q

The least squares criterion is used for:
A. isolating latent traits in a factor analysis.
B. predicting “true” scores from obtained scores.
C. estimating “shrinkage” when cross-validating a multiple regression equafion.
D. identifying the “line of best fit” in a regression analysis.

A

For the licensing exam, you want to have the least squares criterion associated with regression analysis.
d. CORRECT The least squares criterion is used to locate the regression line in a scatterplot so that the amount of error in prediction is minimized when using the regression line or its equation to predict criterion scores.(Stats & Research Design)

410
Q
Which of the following behavioral observation techniques would be most useful for evaluating the extent to which a student remains on-task while his teacher is talking?
A. event recording
B. latency recording
C. interval recording
D. duration recording
A

Commonly used behavioral observation techniques include event recording, interval recording, time sampling, latency recording, and duration recording.
a. Incorrect Event recording (sa mpling) is useful for behaviors that occur infrequently, have a long duration, andfor leave a permanent record or other product.
b. Incorrect Latency recording is used to determine how long it takes for a behavior to begin after a specific event has occurred (e.g., after a verbal demand or request has been made).
c. CORRECT Interval recording is useful for behaviors that have no clear beginning or end (e.g., on-task behavior) and involves recording whether or not the behavior occurred during each interval.
d. Incorrect Duration recording indicates how long a behavior lasts and is useful for behaviors that have a clear beginning and end.
(Stats & Research Design)

411
Q
The results of a research study indicate that stress inoculation is most effective for people with mild to moderate anxiety, while pharmacotherapy plus stress inoculation |s most effective for people with severe anxiety. In this situation, level of anxiety is a:
A. moderator variable
B. mediator variable
C. control variable
D. suppressor variable
A

In the study described in this question, there is an interaction between type of therapy and level of anxiety.
a. CORRECT A moderator variable influences the nature of the relationship between an IV and DV. In this situation, level of anxiety affects (moderates) the relationship between type of therapy and therapy outcome.
b. Incorrect A mediating (intervening) variable is responsible for the relationship between an IV and DV. In the study described in the question, level of anxiety affects the relationship between therapy and anxiety, but it does not cause that relationship.
c. Incorrect A control variable is an extraneous variable that an investigator does not want to examine in his/her investigation.
d. Incorrect A suppressor variable reduces or conceals the relationship between two variables. In this situation, level of anxiety is moderating (not suppressing) the relationship between type of therapy and therapy outcome.
(Stats & Research Design)

412
Q

In the context of research design, external validity refers to the researcher’s ability to:
A. manipulate the independent variable.
B. randomly assign participants to treatment groups.
C. determine whether there is a causal relationship between variables.
D. generalize the results of the study to other individuals, settings, and conditions.

A

Two standards are used to judge the validity of a research study - internal and external validity.
a. Incorrect The ability to manipulate the independent variable has a direct impact on a study’s internal validity, which refers to the ability to determine if the independent variable has affected the dependent variable.
b. Incorrect Random assignment is important for internal validity.
c. Incorrect This response describes internal (not external) validity.
d. CORRECT External validity refers to the ability to generalize the results of a study to groups, settings, and conditions other than those used in the study. Random selection of participants from the population is one factor that helps ensure adequate external validity in terms of generalizability to other individuals.
(Stats & Research Design)

413
Q

A psychologist decides to use a single-subject design to assess the impact of an educational intervention designed to increase on-task behavior. When conducting her study, the psychologist will measure the participant’s on-task behavior:
A. once at the beginning and once at the end of the study.
B. once during the baseline phase and once during the treatment phase of the study.
C. immediately after and several weeks or months after the end of the study.
D. at regular intervals during the baseline and the treatment phases of the study.

A

The various single-subject research designs share two characteristics: First, each design includes at least one baseline (no treatment) phase and one treatment phase; and, second, the dependent variable is
measured at regular intervals during each phase.
d. CORRECT This answer is the only one of those given that accurately describes measurement of the dependent variable when using a single-subject research design.
(Stats & Research Design)

414
Q

Squaring the multiple correlation coefficient (R) produces a measure of the:
A. relative amount of dispersion (variability) in a set of scores.
B. degree to which a true score can be expected to vary from an obtained score.
C. correlation between two variables when the effects of a third variable have been statistically removed.
D. the amount of variability shared by three or more variables.

A

Squaring a correlation coefficient produces a measure of shared variability.
a. Incorrect The coefficient of variation, an infrequently used measure of variability, provides this information.
b. Incorrect This describes the standard error of measurement.
c. Incorrect This describes first-order correlation.
d. CORRECT The multiple correlation coefficient indicates the degree of association between three or more variables. Like any other correlation coefficient, it can be squared in order to obtain a measure of shared variability
(Stats & Research Design)

415
Q

To analyze the data collected in a study that included a single independent variable and three dependent variables, a researcher would use a MANOVA (multivariate analysis of variance) rather than three separate one-way ANOVAs because:
A. she wants to statistically remove the effects of a confounding variable.
B. at least one of the dependent variables is an extraneous variable.
C. she wants to reduce the probability of making a Type I error.
D. she doesn’t want to have to conduct a post-hoc analysis.

A

The MANOVA is used to simultaneously assess the effects of one or more independent variables on two or more dependent variables.
a. Incorrect The analysis of covariance (ANCOVA) is used to statistically remove the effects of an extraneous variable on the dependent variable.
b. Incorrect This is a distracter and doesn’t make sense in the context of MANOVA.
c. CORRECT If three separate ANO\/As were conducted at a level of significance of .05 each, the “experimentwise error rate” (probability of making a Type I error) would be increased to approximately .15. By conducting a single MANOVA at the .05 level, the experimentwise error rate remains at .05.
d. Incorrect If the MANOVA yields significant results, the researcher will have to conduct post hoc tests (probably three separate ANO\/As) to determine if the independent variable affected all or only some of the dependent variables.
(Stats & Research Design)

416
Q
A researcher would use an ABAB design rather than an AB design in order to control which of the following threats to his study’s validity?
A. diffusion
B. attrition
C. instrumentation
D. history
A

An ABAB design is a single-subject design that involves collecting baseline data, administering the independent variable, removing the independent variable, and then readministering the independent
variable.
a. Incorrect Diffusion is a threat to a study’s validity when participants in one group (often a no-treatment control group) benefit from the intervention administered to another group — i.e., when participants in the control group inadvertently learn about the treatment or are accidentally exposed to it.
b. Incorrect Attrition is a threat to a study’s internal validity. It occurs when participants who drop out of one group differ in a systematic way from those who drop out of another group and this difference affects the study’s results.
c. Incorrect Instrumentation is a threat to internal validity and refers to any change in tests or other measuring devices administered to participants during the course of the study that confounds the study’s results. An ABAB design would not help control instrumentation effects.
d. CORRECT The use of an ABAB design helps to determine whether or not an observed change in the dependent variable is due to the independent variable or to an external event (history). Specifically, if the participant exhibits the same change after the independent variable is administered the second time that he/she exhibited after it was administered the first time, that change can be assumed to have been ca used by the independent variable rather than by history.
(Stats & Research Design)

417
Q

Statistical regression is a potential threat to a study’s internal validity when:
A. there is a statistically different number of participants in the study’s treatment groups.
B. there is a statistically significant difference in the number of participants who drop out of the different treatment groups during the course of the study.
C. participants are selected for inclusion in the study because of their extremely high or low scores on the pretest.
D. the researcher is unable to randomly assign participants to the different treatment groups.

A

Statistical regression refers to the tendency of extreme scores to “regress to the mean” on retesting — i.e., for examinees who obtained extremely high or extremely low scores on a measure to obtain scores closer to the mean when retested.
c. CORRECT When participants are chosen for inclusion in a study because of their extreme scores on the pretest, it is likely that their scores will “regress to the mean” on the post-test, regardless of the effects of the treatment. This can make it difficult to ascertain the effects of the independent variable on the dependent variable.
(Stats & Research Design)

418
Q
A “normal” distribution is:
A. flat (rectangular).
B. bell-shaped.
C. S-shaped.
D. bimodal.
A

A normal distribution of scores is symmetrical and bell-shaped
b. CORRECT By definition, a normal distribution is bell-shaped and, consequently, is also known as the “bell curve.”
(Stats & Research Design)

419
Q

A researcher would use the randomized block ANOVA to analyze the data she has collected in order to:
A. analyze the main and interaction effects of an extraneous variable.
B. control measurement (random) error.
C. evaluate the effects of an independent variable on multiple dependent variables.
D. evaluate the effects of one or more independent variables on a nominal dependent variable.

A

As its name suggests, the randomized block ANOVA is the appropriate inferential statistical test when blocking has been used to control an extraneous variable.
a. CORRECT When using the randomized block ANOVA, the extraneous variable is treated like an independent variable so that its main and interaction effects on the dependent variable can be evaluated.
b. Incorrect A randomized block ANOVA would not be useful for this purpose.
c. Incorrect A MANOVA would be useful for this purpose.
d. Incorrect The chi-square test would be useful for this purpose.
(Stats & Research Design)

420
Q
To determine the degree of association between gender and reaction time (as rneasured in seconds), you would use which of the following bivariate correlation coefficients?
A. Spearman rho
B. point biserial
C. phi coefficient
D. Pearson r
A

The scale of measurement of the data is the primary consideration when choosing a correlation coefficient.
a. Incorrect The Spearman rho is used when both variables are measured in terms of ranks.
b. CORRECT The point-biserial correlation coefficient is appropriate when one variable is a true dichotomy (e.g., gender) and the other is measured on an interval or ratio scale (in this case, reaction time is measured on a ratio scale)
c. Incorrect The phi coefficient is appropriate when both variables are true dichotomies (e.g., undergraduate degree/no undergraduate degree and employed/unemployed).
d. Incorrect The Pearson r is the appropriate correlation coefficient when both variables are continuous (measured on an interval or ratio scale).
(Stats & Research Design)

421
Q
When the relationship between the predictor (the X variable) and the criterion (the Y variable) is curvilinear and both variables are measured on an interval or ratio scale, the appropriate correlation coefficient is:
A. phi.
B. tau.
C. rho.
D. eta.
A

The choice of a correlation coefficient is based on several factors including the scale of measurement of the variables and the shape of the relationship between them (linear vs. non-linear).
a. Incorrect The phi coefficient is appropriate when both variables are true (natural) dichotomies.
b. Incorrect Kendall’s tau is appropriate when both variables are measured on an ordinal scale.
c. Incorrect Rho (also known as the Spearman rank-order correlation coefficient) is used when both variables are measured as ranks.
d. CORRECT Eta is used to measure the relationship between two continuous (interval or ratio) variables when their relationship is nonlinear.
(Stats & Research Design)

422
Q
When a distribution of scores is normally shaped, approximately what percent of scores fall between a z-score of 0 and a z-score of +1.0?
A. 25
B. 34
C. 50
D. 68
A

A z-score of 0 is equivalent to the mean score and a z-score of +1.0 is equivalent to the score that is one standard deviation above the mean.
b. CORRECT When a distribution is normally shaped, approximately 34% of scores Fall between a z-score of 0 and a z-score of +1.0 (i.e., between the mean score and the score that is one standard deviation above the mea n).
d. Incorrect In a normal distribution, approximately 68% of scores Fall between z-scores of -1.0 and +1.0.
(Stats & Research Design)

423
Q

The Central Limit Theorem predicts that the sampling distribution of means increasingly approaches:
A. a normal shape as the sample size increases regardless of the shape of the distribution of scores in the population.
B. a normal shape as the number of samples drawn from the population increases regardless of the shape of the distribution of scores in the population.
C. the shape of the population distribution as the size of the sample increases.
D. the shape of the population distribution as the number of samples drawn from the population increases.

A

The Central Limit Theorem makes three predictions about the sampling distribution of means: (a) if repeated random samples of size N are drawn from the population, as N increases in size, the sampling distribution of means increasingly approaches normal regardless of the shape of the population distribution; (b) the mean of the sampling distribution of means equals the population mean; and (c) the standard deviation of the sampling distribution of means equals the population standard deviation divided by the square root of N.
a. CORRECT This response is consistent with the first prediction of the Central Limit Theorem.
b. Incorrect The Central Limit Theorem assumes an infinite number of samples. In other words, it is the size of the samples, not the number of samples, that impacts the shape of the sampling distribution.
c. Incorrect This is not what is predicted by the Central Limit Theorem. Instead, the Central Limit Theorem predicts that, even when the population distribution is not normal, the sampling distribution of means will increasingly approach normal as the sample size increases.
d. Incorrect This is not what is predicted by the Central Limit Theorem.
(Stats & Research Design)

424
Q

The magnitude of the standard error of the mean increases as:
A. the sample size increases and the population variance decreases
B. the sample size increases and the population variance increases.
C. the sample size decreases and the population variance decreases.
D. the sample size decreases and the population variance increases.

A

Knowing the formula for the standard error of the mean would have helped you identify the correct answer to this question.
d. CORRECT The standard error of the mean is equal to the population standard deviation divided by the square root of the sample size. This formula indicates that, as the population standard deviation increases and/or the sample size decreases, the standard error increases in magnitude.(Stats & Research Design)

425
Q

An educational psychologist conducts a study to assess the effects of an educational program designed to improve the academic performance of children who have been diagnosed as mildly mentally retarded. To do so, the psychologist matches 30 children with mild mental retardation on the basis of their IQ test scores and then randomly assigns the members of each matched pair so that one member is assigned to the experimental group and the other is assigned to the control group. She then administers the program to the children in the experimental group and, following completion of the program, administers an achievement test to all children and calculates a total correct score for each child. To assess the effects of the treatment on
achievement test scores, the psychologist should use which of the following?
A. t-test for a single sample
B. t-test for independent (unrelated) samples
C. t-test for dependent (related) samples
D. two-way ANOVA

A

This study includes one independent variable that has two levels (program and no program); the data to be analyzed represent a ratio scale of measurement (total number of correct items); and matching
was used to control an extraneous variable (IQ score).
a. Incorrect The t-test For a single sample is used to compare the mean derived from a single sample to a known population mean.
b. Incorrect The t-test for independent samples is used compare the means of two unrelated samples. The study described in this question involves comparing means obtained from dependent (related) samples.
c. CORRECT The t-test For dependent (related) samples is used when the data are collected from two correlated groups, which is the case when matching has been used to assign participants to groups.
d. Incorrect A two-way ANOVA is used when a study involves two independent variables. This study has one independent variable (educational program) with two levels.
(Stats & Research Design)

426
Q
In a normally-shaped distribution, the percentile rank equivalent of a z-score of +2.0 is approximately:
A. 2.5.
B. 50.0.
C. 84.0.
D. 97.5.
A

The licensing exam often includes a question or two on the areas under the normal curve. If you remember that about 68% of scores fall between +1.0 and -1.0 standard deviations from the mean, that 95% fall between +2.0 and -2.0 standard deviations from the mean, and that 99% fall between +3.0 and -3.0 standard deviations from the mean, you should be able to identify the correct answer these questions.
a. Incorrect A percentile rank of 2.5 is equivalent to a z-score of about -2.0.
b. Incorrect A percentile rank of 50.0 is equivalent to a z-score of 0.
c. Incorrect A percentile rank of 84 is equivalent to a z-score of +1.0.
d. CORRECT In a normal distribution, a percentile rank of 97.5 is equivalent to a z-score of about +2.0. Note that knowing that 50% of scores Fall below the mean and that 47.5% of scores Fall between the mean and the score that is 2 standard deviations above the mean would have enabled you to identify the correct
answer to this question.
(Stats & Research Design)

427
Q
According to Albert Ellis, our emotional and behavioral reactions to an event are due to our beliefs about the event rather than to the event itself. In other words, our beliefs act as a:
A. moderator variable.
B. mediator variable.
C. latent variable.
D. suppressor variable.
A

Ellis’s theory proposes that beliefs mediate (are responsible for) the impact of an event on our emotional and behavioral reactions to that event
a. Incorrect A moderator variable is a variable that affects the strength of the relationship between two other variables. For example, if the size of the correlation between a predictor and criterion differs for older and younger adults, age is a moderator variable. Note that some authors use the terms “moderator variable” and “extraneous variable” interchangeably.
b. CORRECT A mediator variable accounts for (is responsible for) the relationship between two variables.
c. Incorrect A latent variable is a theoretical variable that is believed to underlie a measured or observed variable.
d. Incorrect A suppressor variable reduces or conceals the relationship between two variables. Consequently, statistically removing the effects of a suppressor variable increases the correlation between the two variables.
(Stats & Research Design)

428
Q

LISREL (linear structural relations analysis) would be the appropriate method of analysis if your causal model:
A. includes measured (observed) attributes only.
B. includes latent traits only.
C. includes both measured (observed) attributes and latent traits.
D. includes both correlated and uncorrelated attributes.

A

LISREL, a structural equation (causal) modeling technique, is used to test causal hypotheses about relationships among measured variables and the latent traits those variables are believed to measure.
c. CORRECT In contrast to path analysis models, which predict the causal relationships among measured attributes only, LISREL models incorporate both measured attributes and latent traits.
(Stats & Research Design)

429
Q
To reduce the likelihood that experimenter expectancy will bias the results of a research study, you would use which of the following techniques?
A. single-blind
B. double-blind
C. random selection
D. random assignment
A

Experimenter expectancy refers to the bias that results when the experimenter’s expectations about the outcomes of the study influence the actual outcomes (e.g., as the result of cues the experimenter
inadvertently provides participants).
a. Incorrect In a single-blind study, the participants do not know what group they are in. This procedure would not eliminate experimenter expectancies.
b. CORRECT When a double-blind procedure is used, research participants and the experimenter are “blind” to the experimental conditions — i.e., they do not know what group participants are in. Because the researcher does not know whether individual participants are in the experimental or control group, this reduces the likelihood that he/she will provide cues to participants that could bias the results of the study.
c. Incorrect Random selection of participants to groups would not eliminate experimenter expectancy.
d. Incorrect Random assignment of participants to groups would not eliminate experimenter expectancy.
(Stats & Research Design)

430
Q

A distribution of 250 scores is positively skewed and has a range of 1 to 50. Which of the following describes the relationship of the three measures of central tendency in this distribution?
A. The median is the highest score and the mean is the lowest score.
B. The mode is the highest score and the mean is the lowest score.
C. The mean is the highest score and the mode is the lowest score.
D. The mean is the highest score and the median is the lowest score.

A

In a positively skewed distribution, the “tail” is extended on the high side of the distribution — i.e., most of the scores (outliers) are “piled up” in the low end of the distribution, while a few scores are in its high end.
c. CORRECT In a positively skewed distribution, the mean has the highest value; the mode has the lowest value; and the median is between the mean and the mode.
(Stats & Research Design)

431
Q

A researcher makes a Type II error when he/she:
A. rejects the null hypothesis when it is true.
B. rejects the null hypothesis when it is false.
C. retains the null hypothesis when it is true.
D. retains the null hypothesis when it is false.

A

In inferential statistics, there are two types of decision errors - rejecting a true null hypothesis and retaining a false null hypothesis.
a. Incorrect Rejecting a true null hypothesis is referred to as a Type I error.
b. Incorrect This would be a correct decision.
c. Incorrect This is also a correct decision.
d. CORRECT Retaining a false null hypothesis is referred to as a Type II error
(Stats & Research Design)

432
Q
To assess the effectiveness of a behavioral intervention on the caloric intake of young women with anorexia nervosa, you obtain a sample of women who have just received this diagnosis and determine the average number of calories each woman consumes each day during the week before and the week after the intervention is applied. To analyze the data you obtain, you will use which of the following statistical tests?
A. t-test for a single sample 
B. t-test for dependent samples
C. t-test for independent samples
D. t-test for multiple samples
A

Each version of the t-test is used to compare two means.
a. Incorrect The t-test For a single sample is used to compare an obtained sample mean to a known population mean.
b. CORRECT The t-test for dependent (related) samples is the appropriate test when the two means to be compared are related, which would be the case when they have been obtained from the same group of participants.
c. Incorrect The t-test for independent samples is used to compare the means obtained by two unrelated groups.
d. Incorrect There is no “t-test for multiple samples.”
(Stats & Research Design)

433
Q
When the correlation between high school students’ GPA and amount of time they spend studying each week is \_\_\_\_\_, this means that about 15% of variability in GPA is accounted for by amount of time spent studying.
A. .02
B. .15 
C. .39
D. .85
A

To determine the amount of variability in one variable that is explained by variability in another variable (i.e., the amount of shared variability), you must square the correlation coefficient. Conversely, to determine the correlation coefficient when the shared variability is known, you must calculate the square root of the shared variability.
c. CORRECT To simplify the task for this question, you could calculate the square root of .16 (rather than .15). The square root of .16 is .40, so the square root of .15 will be slightly less than .40. Therefore, of the numbers listed in the answers, the best estimate of shared variability is .39.
(Stats & Research Design)

434
Q

The denominator term in the F-ratio is reduced in magnitude by:
A. decreasing within-group variability.
B. increasing within-group variability.
C. increasing between-group variability and decreasing within-group variability.
D. decreasing between-group variability and increasing within-group variability.

A

The mean square within (MSW) is the denominator of the F-ratio and, as its name implies, is a measure of within-group variability.
a. CORRECT Within-group variability is a measure of error; and decreasing within-group variability decreases error and the magnitude of the denominator of the F-ratio.
b. Incorrect This would increase the size of the denominator of the F-ratio.
c. Incorrect MSW is a measure of within-group variability and isn’t affected by between-group variability.
d. Incorrect See explanation above.
(Stats & Research Design)

435
Q
A researcher fears that the results of her study have been biased by aspects of the experimental situation that provided participants with cues about the responses or reactions that were expected of them. This researcher is concerned about which of the following?
A. treatment diffusion
B. demand characteristics
C. confirmation bias
D. placebo effect
A

This question describes a type of reactivity in which the behavior participants is affected by cues in the experimental setting that indicate what behaviors are expected of them.
a. Incorrect Treatment diffusion (also known as treatment spillover] occurs when participants in one group inadvertently receive (or learn about) the intervention that has been applied to participants in another group.
b. CORRECT Cues in the environment that inform participants about the purpose of the study or indicate what behavior is required of them are referred to as demand characteristics.
c. Incorrect The confirmation bias is the tendency to pay attention to information that confirms one’s current beliefs or preconceptions.
d. Incorrect The placebo effect refers to an improvement in physical or mental health that occurs in response to a “fake” treatment (e.g., a sugar pill), apparently because of the individual’s expectation that the treatment will be effective.
(Stats & Research Design)

436
Q

A factorial design:
A. always includes two or more dependent variables.
B. always includes two or more independent variables
C. is used to control order effects.
D. is used to evaluate the effects of pretesting.

A

The term “factor” in factorial design refers to independent variables.
b. CORRECT A factorial research design is any design that includes two or more “factors” (independent variables).
(Stats & Research Design)

437
Q

A researcher investigates the nature of client-therapist interactions during the initial therapy session by having graduate students in psychology assume the role of either the therapist or therapy client and role-play the first
session of therapy. The researcher’s study is best described as which of the following?
A. nonparametric
B. cross-sectional
C. causal comparative
D. analogue

A

In the situation described in this question, the researcher is studying a phenomenon (therapist-client interactions) under conditions that only resemble or approximate actual clinical conditions.
d. CORRECT In clinical research, an analogue study is a study in which the conditions are in some way an analogue (approximation) of actual clinical practice.
(Stats & Research Design)

438
Q

When using multiple regression, multicollinearity occurs when:
A. the predictors are highly correlated with one another.
B. the criterion measures are highly correlated with one another.
C. there is a low correlation between the predictors and the criterion.
D. there is a high (significant) correlation between the predictors and the criterion.

A

When choosing predictors for a multiple regression equation, the optimal condition is for each predictor to explain unique variability in the criterion. When predictors are correlated, this condition is violated
a. CORRECT The term multicollinearity is used to describe high correlations between predictors (X variables).
(Stats & Research Design)

439
Q

When using stepwise multiple regression, the addition of predictors to the equation is usually based on:
A. a predefined model or theory.
B. the magnitude of R-squared.
C. the proportion of accurate versus inaccurate decisions made.
D. the magnitude of the eigenvalues.

A

The types of multiple regression include simple, hierarchical, and stepwise. As its name implies, stepwise regression involves adding (or subtracting) predictors to the multiple regression equation one at a time.
a. Incorrect This describes hierarchical multiple regression.
b. CORRECT When using multiple regression, each predictor is retained in the equation as long as it contributes significantly to the total amount of variability in the criterion that is explained by the combined predictors. The proportion of explained variability is measured by R-squared.
c. Incorrect This sounds like the basis For determining if predictors have incremental validity, which may be a consideration in multiple regression. However, this is not the best answer to this question.
d. Incorrect This refers to Factor analysis.
(Stats & Research Design)

440
Q

The correlation between two variables is equal or close to zero when, in a scatterplot:
A. the range ofY scores at each value of X is equal to the total range of Y scores.
B. the range of Y scores at each value of X is the about the same.
C. the range of Y scores is equal to the range of X scores.
D. the range of Y scores at each value of X is different.

A

The correlation for two variables is zero when scores on one variable are not related to scores on the other variable.
a. CORRECT This is the only answer of those given that describes a lack of correlation between X and Y. When the range of Y scores at each value of X equals the total range of Y scores, the scattergram for X and Y is rectangular in shape, indicating a correlation of (or near) zero.
b. Incorrect This describes homoscedasticity which is good as long as the range of Y scores at each value of X is not equal to the total range of Y scores.
c. Incorrect This condition does not imply a zero correlation between X and Y.
d. Incorrect This describes heteroscedasticity. Although heteroscedasticity can produce an inaccurate estimate of the degree of association between variables, it does not necessarily result in a correlation coefficient of zero.
(Stats & Research Design)

441
Q

When using a “multiple-baseline” design, a researcher will:
A. apply and then withdraw a treatment multiple times.
B. change the performance criterion gradually over the course of the study.
C. compare two or more treatments that are simultaneously applied.
D. sequentially apply a treatment to different behaviors.

A

The multiple-baseline design is a type of single-subject design.
a. Incorrect This describes the reversal (withdrawal) design.
b. Incorrect This describes the changing criterion design.
c. Incorrect This does not describe the multiple baseline design.
d. CORRECT When using the multiple-baseline design, a treatment is sequentially applied to different baselines — i.e., to different behaviors, to the same behavior in different settings, or to the same behavior performed by
different individuals. Unlike a “reversal” design (which is described by answer a), the multiple-baseline design does not require withdrawal of the treatment during
the course of the study.
(Stats & Research Design)

442
Q
To analyze the relationship between gender and Holland's six occupational themes, the appropriate statistical test would be which of the following?
A. single-sample chi-square
B. multiple-sample chi-square
C. one-way ANOVA
D. factorial ANOVA
A

The study described in this question has one independent variable (gender) and one dependent variable (Holland’s six occupational themes), and the dependent variable is measured on a nominal scale.
a. Incorrect The single-sample chi-square test is appropriate for descriptive studies that include a single nominal variable.
b. CORRECT The multiple-sample chi-square test is appropriate For studies that include two or more variables and the data to be analyzed represent a nominal scale.
c. Incorrect The one-way ANOVA is appropriate for studies assessing the impact of a single independent variable on a single dependent variable that is measured
on an interval or ratio scale.
d. Incorrect The factorial ANOVA is appropriate For studies assessing the impact of two or more independent variables on a single dependent variable that is measured on an interval or ratio scale.
(Stats & Research Design)

443
Q
To test the validity of a model that describes the causal relationships among a set of variables, you would use which of the following techniques?
A. path analysis
B. logistic regression
C. cluster analysis
D. multitrait-multimethod matrix
A

All of the techniques listed in the answers are multivariate technique, but only one is used to test causal hypotheses or models.
a. CORRECT Path analysis and other structural equation modeling techniques are used to determine if sample data fit a predefined model that indicates the direction of the causal links between variables.
b. Incorrect Logistic regression is a multivariate techniques used to identify the combination of predictors that most accurately categorizes individuals into criterion groups.
c. Incorrect Cluster analysis is used to “cluster” or categorize individuals.
d. Incorrect The multitrait-multimethod matrix is used to evaluate a measure’s construct validity and provides information on the test’s convergent and divergent validity.
(Stats & Research Design)

444
Q
A research participant’s score on the dependent variable is the amount of time (minutes) it took him or her to complete a task. When assigning scores to participants, the researcher discovers that three of the 60 participants did not complete the task, and he assigns them the maximum amount of time given to participants to work on the task. The best measure of central tendency for the data collected in this study would be which of the following?
A. harmonic mean
B. arithmetic mean
C. mode
D. median
A

Ordinarily, the best measure of central tendency for interval or ratio data is the arithmetic mean. However, in the situation described in this question, the mean would not be the most accurate measure because the distribution of data includes three scores that are estimates (rather than actual measures) of the value of interest
c. Incorrect Because the mode provides less information about a distribution than the median does (i.e., the mode only indicates the most frequently occurring score in the distribution), the median would be a better measure of
central tendency.
d. CORRECT The mean is affected by the magnitude of every score in a distribution, but the median is not. Consequently, in this situation, the median would not be affected by the magnitude of the three missing scores and, therefore, would be a more accurate measure of central tendency.
(Stats & Research Design)

445
Q
To increase statistical power, you would:
A. increase beta from .001 to .01.
B. increase alpha from .01 to .05.
C. use a nonparametric statistical test.
D. decrease the sample size.
A

Power refers to the ability to detect (reject) a false null hypothesis.
a. Incorrect Beta is the probability of retaining a false null hypothesis (i.e., of making a Type II error) — and the larger the size of beta, the greater the probability of retaining (not rejecting) the false null hypothesis. (Also note that researchers do not have direct control over the magnitude of beta.)
b. CORRECT The ability to reject the null hypothesis is increased by increasing alpha. When the null hypothesis is, in fact, false, increasing the size of alpha also increases power.
c. Incorrect Parametric tests are more powerful than are nonparametric tests.
d. Incorrect Decreasing the sample size would decrease power.
(Stats & Research Design)

446
Q

An investigator would use the split-plot ANOVA to analyze the data he has collected for which of the following studies?
A. a study in which each participant receives only one level of Variable A and one level of Variable B
B. a study in which participants who received level 1 of Variable A also received either level 1 or 2 of Variable B, while participants who received level 2 of Variable A received either level 3 or 4 of Variable B
C. a study in which each participant received all levels of Variable A but in a different order
D. a study in which each participant received only one level of Variable A but all levels of Variable B

A

Split-plot is another name for “mixed”; and a researcher would use a split-plot ANOVA when the study uses a mixed design - i.e., it includes both between-groups and within-subjects comparisons.
a. Incorrect This answer describes a study that includes two between-groups variables.
b. Incorrect This answer describes a nested design.
c. Incorrect This answer describes a counterbalanced design with a single independent variable.
d. CORRECT This is only answer that describes a mixed design: Variable A is a between-groups variable, while Variable B is a within-subjects variable.
(Stats & Research Design)

447
Q
If your data analysis involves calculating an "effect size," you are conducting which of the following?
A. meta-analysis
B. path analysis
C. factor analysis
D. discriminant analysis
A

Only one of the statistical techniques listed in the answers involves calculating an effect size, which is a measure of the magnitude of the relationship between two variables (e.g., a treatment and an outcome)
a. CORRECT Meta-analysis is used to combine the results of multiple studies. For example, a meta-analysis might be conducted to combine the results of several different studies that investigated the effects of cognitive therapy on depression. Use of this technique involves calculating an effect size for each study and then calculating a mean effect size for all of the studies.
(Stats & Research Design)

448
Q
A psychologist compares the effectiveness of three brief interventions for obsessive-compulsive disorder by randomly assigning adults who have received this diagnosis to one of the three interventions and measuring their symptoms prior to the beginning of treatment and one week, one month, and six months following treatment. The psychologist is using which of the following types of research design?
A. between -groups 
B. within-subjects
C. mixed
D. counterbalanced
A

The study described in this question has two independent variables - type of treatment and time.
c. CORRECT Type of treatment is a between-groups variable in this study since each participant is assigned to one of the three interventions. In contrast, time is
a within-subjects variable since all participants are being evaluated on the dependent variable at four different times. When a study includes both between-groups and within-subjects variables, it is referred to as a mixed design.
d. Incorrect A counterbalanced design is a type of within-subjects design that involves administering the different levels of the independent variable to different groups of participants in a different order.
(Stats & Research Design)

449
Q

When designing a research study, an investigator would be concerned about the impact of pretest sensitization on his study’s external validity if:
A. the investigator’s knowledge of how participants did on the pretest affects how he scores their post-tests.
B. the study’s participants know which group (experimental or control) they have been assigned to.
C. the investigator revises the post-test on the basis of how participants responded to the pretest.
D. taking the pretest changes how the participants react to the treatment.

A
Pretest sensitization (also known as an interaction between testing and treatment) is a potential threat to a study’s external validity whenever,a study includes administering a pretest.
d. CORRECT Pretest sensitization occurs when the administration of a pretest “sensitizes” participants to the purpose of the research study and, as a result, alters their reaction to the treatment. In this case, the results of the study cannot be generalized to situations in which a pretest is not administered prior to delivery of the treatment.
(Stats & Research Design)
450
Q
A "cover story" that causes research participants to think that the purpose of the study they are participating in is something other than what it really is would be most helpful for controlling which of the following?
A. Hawthorne effect
B. Rosenthal effect
C. demand characteristics
D. differential attrition
A

In the situation described in this question, participants will not know the real purpose of the study. As long as you’re familiar with the phenomena listed in the answers, you should have been able to recognize which phenomenon would be impacted by this lack of knowledge.
a. Incorrect The Hawthorne effect is the tendency for the performance of research participants to improve as the result of the attention they receive as research participants. In other words, it is knowing that they are in a study (not knowing the purpose of the study) that impacts performance.
b. Incorrect The Rosenthal (self-fulfilling prophecy) effect refers to the impact of a teacher’s expectations about the performance of his/her students on their actual performance.
c. CORRECT Demand characteristics are cues in the experimental setting that inadvertently convey information to participants about what behavior is
expected. If participants do not know what the purpose of the study is, they will be less susceptible to the effects of any cues that happen to be present.
d. Incorrect Differential attrition occurs when participants who drop out of one group differ in a systematic way from those who drop out of another group and this difference affects the study’s results.
(Stats & Research Design)

451
Q

You are conducting a multiple regression analysis and find that one of your predictors has a negative regression coefficient. This means that:
A. you should eliminate that predictor from the regression equation.
B. the predictor has an inverse relationship with the criterion.
C. the predictor is uncorrelated with the other predictors included in the analysis.
D. you have made a mistake in your data entry or calculations.

A

The size and sign of a predictor’s regression coefficient are related to the nature of the correlation between the predictor and criterion.
a. Incorrect A negative sign does not mean there is anything wrong with the predictor.
b. CORRECT A negative sign means that there is an inverse (negative) correlation between the predictor and the criterion.
c. Incorrect This is not implied by a negative regression coefficient.
d. Incorrect A negative regression coefficient is a possibility and, therefore, is not necessarily indicative of a data entry or calculation error.
(Stats & Research Design)

452
Q

On the basis of the results of the t-test a psychologist uses to analyze the data she collects, the psychologist concludes that her results are “significant at the .01 level.” This means that:
A. there is a 1% chance that she will incorrectly reject the null hypothesis.
B. there is a 1% chance that she will incorrectly retain the null hypothesis
C. there is a 99% chance that she will incorrectly reject the null hypothesis.
D. there is a 99% chance that she will incorrectly retain the null hypothesis.

A

The level of significance (alpha) determines the location of the boundary between the regions of likely and unlikely values in the sampling distribution. When results are significant at the chosen level of significance, this means that the results are in the region of unlikely
values and that the null hypothesis should be rejected.
a. CORRECT Significance at the .01 level means that there is a 1% chance that the obtained value (e.g., the mean or the difference between means) could have
occurred by chance alone given the value specified in the null hypothesis. In other words, there is a 1% probability that the null hypothesis will be incorrectly
rejected (that a Type I error will be made).
b. Incorrect The probability of incorrectly retaining the null hypothesis (i.e., of retaining a false null hypothesis) is equal to beta, not alpha.
d. Incorrect Because the results are statistically significant, the psychologist will reject (not retain) the null hypothesis.
(Stats & Research Design)

453
Q

Trend analysis is a type of analysis of variance that is used when:
A. a study’s independent variable is quantitative.
B. a study’s dependent variable is qualitative.
C. a cross-sectional research design has been used.
D. a factorial research design has been used.

A
Trend analysis is a type of analysis of variance that is used to evaluate linear and nonlinear trends.
a. CORRECT Trend analysis is used when a study involves a quantitative independent variable, and the researcher wants to determine the shape of the
relationship between the independent and dependent variables. The results of the trend analysis indicate whether or not there is a statistically significant linear
or nonlinear (quadratic, cubic, quartic) trend.
(Stats & Research Design)
454
Q

An investigator uses a factorial ANOVA to assess the effects of two independent variables on a dependent variable and obtains significant main and interaction effects. When interpreting the results of her study, the
investigator should:
A. ignore the interaction since the main effects are significant.
B. interpret the main effects with caution since the interaction is significant.
C. interpret the interaction with caution since the main effects are significant.
D. re-analyze the data since these results are not possible.

A

Main effects refer to the effects of one independent variable on the dependent variable when considered alone, while interaction effects refer to the effects of one independent variable at different levels of another independent variable.
b. CORRECT When the interaction is significant, this means that the effects of one independent variable differ for different levels of another independent variable. Thus, it is not possible to conclude that the independent variable has consistent main effects. For example, a study might find that, overall, Teaching Method #1 is superior to Teaching Method #2 (i.e., there is a main effect of
teaching method). However, there might also be an interaction between teaching method and level of self-esteem — for example, Teaching Method #1 might be more effective for students with high and moderate self-esteem, while Teaching Method #2 is more effective for students with low self-esteem. In this situation, the main effect of teaching method would have to be interpreted with caution.
d. Incorrect It is possible to have any combination of significant main and/or interaction effects.
(Stats & Research Design)

455
Q
Of 100 students surveyed on a college campus, 48 voted Republican and 52 voted Democratic in the last election. On another college campus, 63 of the 100 students surveyed voted Republican and 37 voted Democratic. Which of the following statistical tests is the appropriate technique for determining if the difference in voting preferences at the two colleges is significant?
A. two-way ANOVA
B. student's t-test
C. chi-square test
D. Mann-Whitney U test
A

In this situation, the analysis will involve comparing the frequencies (numbers) of Republican and Democrat voters at the two schools.
a. Incorrect The two-way ANOVA is used when a study involves two independent variables and data on the dependent variable represent an interval or ratio scale.
b. Incorrect The t-test is used to compare two means.
c. CORRECT The chi-square test is used when data are reported as the frequency of observations, which is the case in this study - i.e., the analysis will involve comparing the number of Democrats versus Republicans at the two
colleges.
d. Incorrect The Mann-Whitney U test is used when the analysis involves comparing rank-ordered data obtained from two independent samples.
(Stats & Research Design)

456
Q

A researcher would use “counterbalancing” in order to:
A. control multiple treatment interference.
B. ensure an equal number of participants in each group.
C. match subjects on a confounding variable. ‘
D. ensure that the sample is representative of the population from which it was drawn.

A

Counterbalancing involves administering the treatments (different levels of the IV) in different orders to different groups of participants.
a. CORRECT Counterbalancing helps control multiple treatment interference (also known as carry-over or order effects) that may result when multiple levels of the independent variable(s) are administered to the same participants.
(Stats & Research Design)

457
Q

A researcher reports that she calculated a Cohen’s d of .50 for the data she collected in a study that compared two brief treatments for generalized anxiety disorder. This means that:
A. there was a difference of one-half standard deviation between the means of the two groups.
B. there was a difference of one-fourth standard deviation between the means of the two groups.
C. the percent of variance accounted for by the difference in treatments was 50%
D. the percent of variance accounted for by the difference in treatments was 25%.

A

Cohen’s d is a measure of effect size. It indicates the difference between the means of two groups in terms of standard deviations.
a. CORRECT A Cohen’s d of .50 indicates that one group obtained a mean that is one-half standard deviation higher than the mean obtained by the other group.
(Stats & Research Design)

458
Q

A researcher uses the chi-square test to determine if the ethnic/racial make-up (i.e., White, African-American, Latino/Hispanic, Asian, Other) of a particular community on the West Coast differs significantly from a similar-
sized community on the East Coast. Ten years later, she returns to the West Coast community and finds that it has not changed dramatically in size but has changed in terms of racial and ethnic group membership. The researcher
now wants to determine if the West Coast community’s current ethnic/racial make-up differs significantly from the make-up she observed in that community ten years earlier and plans to compare the number of individuals
in each ethnic/racial category for the two years (1997 and 2007). The researcher cannot use a chi-square test to analyze the data for her current study because:
A. the study now includes two independent variables.
B. the data violate the assumption of independence.
C. the data violate the assumption of random assignment.
D. all of the above.

A

The use of any statistical test requires not only that the data are measured on a particular scale but also that certain assumptions be met.
a. Incorrect A multiple-sample chi-square test can be used when a study includes two or more independent variables.
b. CORRECT An assumption For the chi-square test is that observations are independent, which means that each individual can appear in only one category. In this study, people who lived in the community in both 1997 and 2007 would appear in two categories, which would violate this assumption.
c. Incorrect Random assignment is not an assumption for the chi-square test.
(Stats & Research Design)

459
Q

The “hallmark” of true experimental research is that, in contrast to quasi-experimental research, it permits the researcher to:
A. operationally define the independent and dependent variables.
B. use parametric tests to analyze the obtained data.
C. randomly select participants from the population.
D. randomly assign participants to treatment groups.

A

True experimental research provides the researcher with greater experimental control than does quasi-experimental research.
a. Incorrect An operational definition is a precise description of a variable in a way that Facilitates its measurement. Variables are operationally defined when
using either a true or quasi-experimental research strategy.
b. Incorrect The choice of a parametric or nonparametric statistical test is based on the characteristics of the data, not the research strategy used.
c. Incorrect Random selection is not a requirement For either true or quasi-experimental research.
d. CORRECT The key feature that distinguishes true and quasi-experimental research is the ability to randomly assign participants to groups in true experimental research, which makes it possible to draw causal inferences about the relationships between variables.
(Stats & Research Design)

460
Q

An organizational psychologist has been hired by a manufacturing company to develop a selection battery that will help the company answer the following question: “What group (assembly-line workers or clerical workers)
does the job applicant most closely resemble?” Which of the following multivariate techniques would be useful for this purpose?
A. discriminant analysis
B. multiple regression
C. factor analysis
D. path analysis

A

There are several methods for combining predictor scores to estimate performance or status on a criterion. When the criterion (Y variable) is group membership, discriminant analysis is an appropriate method.
a. CORRECT A discriminant analysis (also known as discriminant Function analysis) involves using scores on two or more predictors to predict an individual’s membership in a criterion group - i.e., it is used when the criterion is measured on a nominal scale.
b. Incorrect Multiple regression is used to combine scores on several predictors to predict or estimate performance or status on a criterion that is measured on
an interval or ratio scale.
c. Incorrect Factor analysis is used to determine the underlying Factors (attributes) that are assessed by set of measures.
d. Incorrect Path analysis is used to test a model of the causal links among a set of variables.
(Stats & Research Design)

461
Q
Which of the following techniques would be most useful for obtaining information on the cognitive processes used by students while they solve complex problems?
A. event sampling
B. situation sampling
C. functional analysis
D. protocol analysis
A

Of the techniques listed in the answers, only one is used to study cognitive processes.
a. Incorrect Event sampling is a behavioral observation technique that is useful when the behavior of interest occurs infrequently.
b. Incorrect Situation sampling is a behavioral observation technique that involves observing a behavior in a variety of situations.
c. Incorrect A functional analysis is used to identify the functions of a behavior - i.e., the antecedents and consequences that maintain the behavior.
d. CORRECT Protocol analysis (also known as the “think aloud” technique) is used to identify the mental processes that occur during the solution of problems or the performance of tasks.
(Stats & Research Design)

462
Q
A bivariate correlation coefficient of .60 indicates times as much shared variability between variables as does a bivariate correlation coefficient of .30.
A. two
B. three
C. four
D. six
A

The correlation coefficient is squared to determine the amount of shared variability.
c. CORRECT A correlation coefficient of .60 indicates that 36% of variability is shared variability, while a correlation of .30 indicates that 9% of variability is shared variability. Since 36% is four times larger than 9%, this means that a
coefficient of .60 indicates four times as much shared variability as does a coefficient of .30.
(Stats & Research Design)

463
Q

According to classical test theory, total variability in test scores is due to:
A. true score variability plus systematic error.
B. true score variability plus random error.
C. relevant variability plus irrelevant variability.
D. relevant variability plus confounding variability.

A

As defined by classical test theory, total variability in test scores is due to a combination of truth and error.
b. CORRECT This answer accurately describes how total variability is conceptualized in classical test theory. This conceptualization is represented by the formula: X = T + E, where X is the total variability in test scores; T is true
score variability; and E is variability due to measurement (random) error.
(Test Construction)

464
Q

An advantage of using Cohen’s kappa rather than percent agreement when assessing a test’s inter-rater reliability is that the former:
A. is easier to calculate.
B. corrects for chance agreement.
C. corrects for small sample size.
D. takes into account the effects of multicollinearity.

A

If you know that the problem with percent agreement as a measure of inter-rater reliability is that it is inflated by chance agreement, you would have been able to answer this question correctly even if you had never heard of Cohen’s kappa.
a. Incorrect Cohen’s kappa is actually more difficult to calculate than is percent agreement.
b. CORRECT Cohen’s kappa (also known as the kappa statistic or kappa coefficient) provides a more accurate estimate of reliability than percent agreement because its calculation includes removing the effects of chance
agreement.
c. Incorrect This does not describe Cohen’s kappa.
cl. Incorrect This does not describe Cohen’s kappa.
(Test Construction)

465
Q
When a predictor's reliability coefficient is .81, its criterion-related validity coefficient can be:
A. no less than .81.
B. no greater than .81.
C. no less than .90.
D. no greater than .90.
A

Knowing the formula for the relationship between reliability and validity would have helped you identify the correct answer to this question.
d. CORRECT A predictor’s validity coefficient can be no greater than the square root of its reliability coefficient. In this situation, the predictor’s reliability coefficient is .81, so its validity coefficient can be no greater than the square root of .81, which is .90.
(Test Construction)

466
Q
An item discrimination index (D) of indicates that all examinees in the high-scoring group and none in the low-scoring group answered the item correctly.
A. +1.0
B. -1.0
C. .50
D. 0
A

The item discrimination index ranges from -1.0 to +1.0 and is calculated by subtracting the percent of low-scoring examinees who answered the item correctly (examinees who obtained low total scores on the test) from the percent of high-scoring examinees who answered
the item correctly (examinees who obtain high total scores on the test).
a. CORRECT An item discrimination index of +1.0 indicates that all of the high-scoring and none of the low-scoring examinees answered the item correctly.
b. Incorrect An item discrimination index of -1.0 indicates that all of the low-scoring and none of the high-scoring examinees answered the item correctly.
c. Incorrect An item discrimination index of .50 indicates that the 50% fewer of the examinees in the low-scoring group (versus the high-scoring group) answered the item correctly.
d. Incorrect An item discrimination index of 0 indicates that the same percent of examinees in the low- and high-scoring groups answered the item correctly.
(Test Construction)

467
Q

A test designed to measure knowledge of clinical psychology is likely to have the highest reliability coefficient when:
A. the test consists of 30 items and the tryout sample consisted of individuals who are heterogeneous in terms of knowledge of clinical psychology.
B. the test consists of 30 items and the tryout sample consisted of individuals who are homogeneous in terms of knowledge of clinical psychology.
C. the test consists of 80 items and the tryout sample consisted of individuals who are heterogeneous in terms of knowledge of clinical psychology.
D. the test consists of 80 items and the tryout sample consisted of individuals who are homogeneous in terms of knowledge of clinical psychology.

A

The reliability of a test is affected by several factors including the length of the test and the heterogeneity of the sample in terms of the abilities or other attributes measured by the test items.
c. CORRECT All other things being equal, longer tests are more reliable than shorter tests. In addition, the reliability coefficient (like any other correlation coefficient) is larger when there is an unrestricted range of scores — i.e., when
the tryout sample contains examinees who are heterogeneous with regard to the attribute(s] measured by the test.
(Test Construction)

468
Q
A reliability coefficient is best defined as a measure of:
A. relevance.
B. consistency.
C. interpretability.
D. generalizability.
A

A test is reliable when it provides consistent results, with inconsistency in test scores being the result of random factors that are present at the time of testing.
b. CORRECT A reliability coefficient indicates the proportion of variance in test scores that is consistent (i.e., is due to true score variability rather than to measurement error).
(Test Construction)

469
Q
In the rnultitrait-rnultimethocl matrix, which of the following coefficients provides information about a test’s convergent validity?
A. heterotrait-heteromethod
B. heterotrait-monomethod
C. monotrait-heteromethod
D. monotrait-monornethod
A

The multitrait-multimethod matrix contains four types of correlation coefficients.
a. Incorrect The heterotrait-heteromethod coefficient is a measure of divergent validity.
b. Incorrect This heterotrait-monomethod coefficient is also a measure of divergent validity.
c. CORRECT The monotrait-heteromethod coefficient is a measure of convergent validity. It indicates the correlation between the test that is being validated and another measure of the same trait (monotrait) that uses a different method of measurement (heteromethod).
d. Incorrect The monotrait-monomethod coefficient indicates the correlation of the test with itself and is a measure of the test’s reliability.
(Test Construction)

470
Q
Content sampling is not a potential source of measurement error for which of the following methods for evaluating a test's reliability?
A. coefficient alpha and alternate forms
B. alternate forms and test-retest ‘
C. split-half only
D. test-retest only
A

Content sampling refers to the extent to which test scores depend on factors specific to the particular items included in the test (i.e., to its content). Note that this question is asking about the type of reliability
that is g affected by content sampling.
a. Incorrect Coefficient alpha and alternate forms reliability are both subject to content sampling error. Coefficient alpha, a measure of internal consistency, will yield a low reliability coefficient if items are not internally consistent (i.e., if items do not measure the same content domain}; alternate forms reliability will be low if the two forms do not assess the same content.
b. Incorrect Although test-retest reliability is not affected by content sampling error (see response cl], as noted above, alternate forms reliability is.
c. Incorrect Split-half reliability would yield a low reliability coefficient if the two halves of the test did not assess the same content.
d. CORRECT Because test-retest reliability involves administering the same test (i.e., the same content} twice, content sampling is n_ot a source of error.
(Test Construction)

471
Q
A college freshman obtains a score of 150 on his English final exam, a score of 100 on his math exam, a score of 55 on his chemistry exam, and a score of 30 on his history exam. The means and standard deviations for these tests are, respectively, 125 and 20 for the English exam, 90 and 10 for the math exam, 45 and 5 for the chemistry exam, and 30 and 5 for the history exam. Based on this information, you can conclude that the young man's test performance was best on which exam?
A. English
B. math
C. chemistry
D. history
A

The raw scores on different tests may be compared by converting the scores to z-scores, which is done by subtracting the mean from the examinee’s raw score to calculate a deviation score and then dividing the deviation score by the standard deviation.
c. CORRECT In this case, the student’s English score is equivalent to a z-score of +1.25, his math score is equivalent to a z-score of +1.0, his chemistry score is equivalent to a z-score of +2.0, and his history score is equivalent to a z-score of 0. Therefore, the student obtained the highest score on the chemistry test.
(Test Construction)

472
Q

Test-score banding is used to:
A. alleviate criterion contamination.
B. equate an examinee’s scores on different tests
C. facilitate criterion-referenced interpretation.
D. reduce adverse impact.

A

Test-score banding is a method of score interpretation that involves considering people who obtain scores within a specific range (band) as having equivalent scores.
d. CORRECT Banding has been identified as a method for reducing biases in testing that contribute to group differences in test scores. Its advocates argue that it is an effective way for reducing adverse impact.
(Test Construction)

473
Q

In factor analysis, when two factors are “orthogonal,” this means that:
A. the factors are correlated.
B. the factors are uncorrelated.
C. the factors explain a statistically significant amount of variability in test scores.
D. the factors do not explain a statistically significant amount of variability in test scores.

A

For the exam, you’ll want to know the difference between orthogonal and oblique factors.
b. CORRECT In factor analysis, orthogonal factors are uncorrelated (independent) and oblique factors are correlated (dependent).
(Test Construction)

474
Q
Which of the following would be useful for determining the extent to which an examinee's actual criterion score is likely to deviate from his/her predicted criterion score?
A. standard error of estimate
B. standard error of measurement
C. standard error of the difference
D. standard error of the mean
A

Because the relationship between a predictor and a criterion is never “perfect,” any predicted criterion score for an individual may or may not be his/her actual criterion score. Consequently, a good strategy is to
interpret an examinee’s obtained score in terms of a confidence interval.
a. CORRECT The standard error of estimate is used to construct the range within which an examinee’s true criterion score is likely to fall given his/her predicted criterion score. This range is referred to as a confidence interval.
b. Incorrect The standard error of measurement is used to construct a confidence interval around an examinee’s obtained (not predicted) score.
c. Incorrect The standard error of difference is used to construct a confidence interval around a difference score; i.e., a score that is calculated by subtracting scores on two tests (for example, by subtracting Performance IQ from Verbal IQ).
d. Incorrect The standard error of the mean is the standard deviation of the sampling distribution of means and is not used to construct a confidence interval around a predicted score.
(Test Construction)

475
Q

In terms of magnitude, the standard error of measurement can be:
A. no greater than 1.0.
B. no less than 1.0.
C. no greater than the standard deviation of the test scores.
D. no less than the standard deviation of the test scores.

A

Knowing the formula for the standard error of measurement would have helped you identify the correct answer to this question. The standard error of measurement equals the standard deviation of the test (predictor) scores times the square root of one minus the reliability coefficient.
a. Incorrect The standard error can be larger than 1.0.
b. Incorrect The standard error can also be less than 1.0.
c. CORRECT The maximum value for the standard error of measurement is the value of the standard deviation of the test scores. The standard error is equal to the standard deviation when the reliability coefficient is zero.
d. Incorrect The standard error of measurement is equal to 0 when the test’s reliability coefficient is +1.0. In this situation, if the test’s standard deviation is larger than 0 (which it usually is), the standard error will be less than the standard deviation.
(Test Construction)

476
Q

In principal components analysis, each eigenvalue indicates:
A. the total variance explained by an identified component.
B. the total variance explained by all of the identified components.
C. the variance shared by each test and an identified component.
D. the total variance shared by all of the tests and all of the identified components.

A

Principal components analysis is similar to factor analysis and involves identifying the components (factors) that underlie or explain the variability observed in a set of tests or other variables.
a. CORRECT An eigenvalue for each principal component is calculated by squaring the correlation between each test and the component and then summing the results. The resulting number indicates the total amount of
variability in the tests that is explained by the principal component.
b. Incorrect This answer does not describe an eigenvalue.
c. Incorrect This answer does not describe an eigenvalue.
d. Incorrect This answer does not describe an eigenvalue.(Test Construction)

477
Q

When the kappa statistic for a measure is .90, this indicates that the measure:
A. has adequate inter-rater reliability.
B. has adequate internal consistency reliability.
C. has low criterion-related validity.
D. has low incremental validity

A

Knowing that the kappa statistic (also known as the kappa coefficient) is a measure of inter-rater reliability would have enabled you to identify the correct response to this question.
a. CORRECT Reliability coefficients range from 0 to +1.0, so a coefficient of .90 indicates good reliability.
b. Incorrect The kappa statistic is a measure of inter-rater reliability, not internal consistency reliability.
(Test Construction)

478
Q
In a normal distribution, which of the following represents the lowest score?
A. percentile rank of 20 
B. z-score of -1.0
C. T score of 25
D. Wechsler IQ score of 70
A

To answer this question, you must be familiar with the relationship between percentile ranks, z-scores, T-scores, and IQ scores in a normal distribution.
a. Incorrect A percentile rank of 20 is slightly less than one standard deviation below the mean.
b. Incorrect A z-score of -1.0 is one standard deviation below the mean.
c. CORRECT A T score is a standardized score with a mean of 50 and a standard deviation of 10. Therefore, a T-score of 25 is two and one-half standard deviations below the mean and represents the lowest score of those given in the answers.
d. Incorrect Wechsler IQ scores have a mean of 100 and standard deviation of 15. Therefore, an IQ score of 70 is two standard deviations below the mean.
(Test Construction)

479
Q

When using principal component analysis:
A. the first principal component represents the largest share of the total variance.
B. the first principal component represents the smallest share of the total variance.
C. each component represents an equal share of the total variance.
D. the order of the components is not related to the share of total variance they represent.

A

This is an example of the “distant galaxy” questions that sometimes appear on the licensing exam and, fortunately, is more difficult than the majority of questions you’ll encounter.
a. CORRECT A characteristic of principal components analysis is that the components [factors] are extracted so that the first component reflects the greatest amount of variability, the second component the second greatest amount of variability, etc.
(Test Construction)

480
Q

In a distribution of percentile ranks, the number of examinees receiving percentile ranks between 20 and 30 is:
A. equal to the number of examinees receiving percentile ranks between 50 and 60.
B. greater than the number of examinees receiving percentile ranks between 50 and 60.
C. about equal to one-half the number of examinees receiving percentile ranks between 50 and 60.
D. about equal to one-fourth the number of examinees receiving percentile ranks between 50 and 60.

A

Knowing that a distribution of percentile ranks is flat (rectangular) would have helped you identify the correct answer to this question.
a. CORRECT The flatness of a percentile rank distribution indicates that scores are evenly distributed throughout the Full range of the distribution. In other words, at least theoretically, the same number of examinees Fall at each percentile rank. Consequently, the same number of examinees obtain percentile
ranks between the ranks of 20 and 30, 30 and 40, etc.(Test Construction)

481
Q
In test construction, shrinkage is associated with:
A. differential validity.
B. incremental validity.
C. cross-validation.
D. process validation.
A

In the context of test construction, shrinkage refers to a reduction in the magnitude of a measure’s validity coefficient.
a. Incorrect A test has differential validity when it has different validity coefficients for different groups.
b. Incorrect Incremental validity refers to the increase in decision-making accuracy that results from use of a predictor.
c. CORRECT Cross-validation refers to validating a predictor with a new sample Because the predictor is often “tailor-made” for the original validation sample (i.e., items are included in the predictor based on their correlation with the criterion for that sample), the chance factors that contributed to the original validity coefficient may not be present in the cross-validation sample. As a result, the validity coefficient for the cross-validation sample is usually smaller.
d. Incorrect Process validation refers to determining that a process will continue to meet pre-determined standards and is not relevant to the phenomenon known as shrinkage.
(Test Construction)

482
Q
Which of the following types of validity would you be most interested in when designing an aptitude test that will be used to predict the future job performance ratings ofjob applicants?
A. discriminant
B. content
C. construct
D. criterion-related
A

In this situation, you are developing a test that will be used to predict future job performance.
a. Incorrect Discriminant validity refers to the extent that scores on a test do not correlate with scores on tests measuring different traits. Discriminant validity is of concern for tests designed to measures hypothetical traits (constructs).
b. Incorrect Content validity is of concern for tests designed to measure a specific content or behavior domain.
c. Incorrect Construct validity is of concern for tests designed to measure hypothetical traits.
d. CORRECT When a test is being used to predict performance on a criterion, you would be most interested in the test’s criterion-related validity (e.g., in its correlation with the criterion measure).
(Test Construction)

483
Q

The primary advantage in using a percentile rank, z-score, or T-score is that these scores:
A. are easy to interpret because they reference an individual’s test performance to an absolute standard of performance.
B. are easy to interpret because they reference an individual’s test performance to the performance of other examinees.
C. are easy to interpret because they make it possible to predict which criterion group an examinee is likely to belong to.
D. “normalize” the raw score distribution so that parametric tests can be used to analyze test scores.

A

The scores listed in this question are all norm-referenced scores that indicate how well an examinee did in comparison to others in the norm group.
a. Incorrect This answer does not accurately describe norm-referenced scores.
b. CORRECT Because it is usually difficult to “ma ke sense” of raw scores, they are often transformed into scores that are easier to interpret. The advantage of norm-referenced scores (which are a type of transformed score) is that they make it possible to determine how well an examinee did in comparison to other examinees.
c. Incorrect This describes a criterion-referenced score, not a norm-referenced score.
d. Incorrect Although some norm-referenced scores are “normalized” (e.g., normalized z-scores), this is not true for all norm-referenced scores.
(Test Construction)

484
Q

The distribution of percentile ranks is always:
A. the same as the shape of the distribution of raw scores.
B. normal regardless of the shape of the distribution of raw scores.
C. rectangular (flat) regardless of the shape of the distribution of raw scores.
D. bimodal regardless of the shape of the distribution of raw scores.

A

For the exam, you want to be familiar with the shape of the normal distribution (bell-shaped) and the shape of a distribution of percentile ranks (rectangular).
c. CORRECT A distinguishing characteristic of percentile ranks is that their distribution is always rectangular (Flat) regardless of the shape of the distribution of raw scores.(Test Construction)

485
Q

A personnel director hires all job applicants who obtain a high score on a job selection test but, after using the test for six months, realizes that many of the new employees are obtaining low performance ratings. Assuming that the selection test has adequate criterion-related validity, the personnel direct can reduce the number of unsatisfactory workers that she hires using the test by:
A. lowering the selection test cutoff score and the job performance rating cutoff score.
B. raising the selection test cutoff score and the job performance rating cutoff score.
C. lowering selection test cutoff score.
D. raising the selection test cutoff score.

A

The applicants who are hired on the basis of their selection test scores but who perform poorly on the job are false positives.
d. CORRECT Raising the cutoff score on the selection test (predictor) should reduce the number of individuals who do poorly on the job — i.e., it will reduce the number of positives, including the number of false positives. Note that lowering the job performance rating (criterion) cutoff score would also reduce the number of false positives but that, in many work situations, an employer would not want to do so.(Test Construction)

486
Q
An assumption underlying \_\_\_\_\_\_ is that observed covariation among a set of variables is due to one or more underlying common dimensions.
A. factor analysis
B. path analysis
C. trend analysis
D. multiple regression
A

Covariation is another word for correlation, so this question is asking what technique is used to determine what accounts for the intercorrelations among a set of tests or other variables. You may have been able to answer this question using the process of elimination if you’re familiar with the techniques listed in the incorrect answers.
a. CORRECT The primary Function of Factor analysis is to identify the Factors (dimensions) that explain or underlie the intercorrelations among a set of tests or other variables.
b. Incorrect Path analysis is used to test causal models For the relationships among a set of variables.
c. Incorrect Trend analysis is a type of analysis of variance that is used when the independent variable is quantitative.
d. Incorrect Multiple regression is used when the goal is to predict a score on a criterion based on scores on two or more predictors.
(Test Construction)

487
Q

In factor analysis, a factor loading indicates the correlation between:
A. a test and an identified factor.
B. two different tests.
C. two factors measured by the same test.
D. two factors measured by different tests.

A

A factor loading provides information about a test’s factorial validity.
a. CORRECT In factor analysis, a factor loading is the correlation coefficient that indicates the correlation between a test and an identified factor.
(Test Construction)

488
Q

The standard error of measurement is used to estimate:
A. a test”s “true” reliability coefficient.
B. a test”s “true” criterion-related validity coefficient.
C. the range within which an examinee”s true test score is likely to fall given her obtained score.
D. the range within which an examinee”s true criterion score is likely to fall given her predicted criterion score.

A

Since no test is completely reliable, any test score may be in error (i.e., may not reflect the examinee’s “true” test score). Consequently, the best strategy is to interpret the examinee’s score in terms of a confidence interval.
c. CORRECT The standard error of measurement is an index of error and is used to construct an interval in which an examinee’s true test score is likely to Fall given his or her obtained test score.
d. Incorrect This answer describes the use of the standard error of estimate.
(Test Construction)

489
Q

In factor analysis, the original factor matrix is usually rotated in order to:
A. facilitate interpretation of the identified factors.
B. determine how many factors to extract.
C. cross-validate the factor analysis.
D. verify the causal relationships among the identified factors.

A

One characteristic of the original factor matrix is that it is usually difficult to interpret because it does not provide a clear pattern of factor loadings.
a. CORRECT The rotation of factors provides a clearer pattern of factor loadings - i.e., in the rotated matrix, some tests correlate most highly with one factor, while other tests correlate more highly with a different factor. This makes it easier to identify the factors (dimensions) that account for the intercorrelations
between the tests.
b. Incorrect The correlation matrix provides the information needed to determine how many factors to extract.
(Test Construction)

490
Q

The minimum and maximum values of the standard error of estimate are:
A. -1 and +1.
B. 0 and 1.
C. 0 and the standard deviation of the predictor.
D. 0 and the standard deviation of the criterion.

A

Knowing the formula for the standard error of estimate would have helped you identify the correct answer to this question.
d. CORRECT The standard error of estimate equals the standard deviation of the criterion scores times the square root of one minus the validity coefficient squared. This formula indicates that the standard error of estimate ranges from 0 (which occurs when the validity coefficient is 1.0) to the standard deviation of the criterion scores (which occurs when the validity coefficient is 0).
(Test Construction)

491
Q
\_\_\_\_\_ refers to the extent to which individual test items contribute to the overall purpose of the test.
A. Validity
B. Reliability
C. Discrimination
D. Relevance
A

Several standards are used to evaluate the usefulness of tests and test items, including the four standards listed in the answers to this question.
a. Incorrect Validity refers to the overall accuracy of the test rather than to the accuracy of individual test items. In other words, validity addresses the question, “Does the g measure what it was designed to measure?”
Consequently, this is not the best answer of those given.
b. Incorrect Reliability refers to the degree to which test scores are free from the effects of measurement error.
c. Incorrect Discrimination refers to the extent to which test items accurately distinguish between examinees having high and low levels of the attribute(s) measured by the test.
d. CORRECT This question describe relevance, which is determined by judging the extent to which each item assesses the target content or behavior domain and does so at the appropriate ability level.
(Test Construction)

492
Q
In a rnultitrait-rnultimethod matrix, the coefficient that indicates a test's reliability is the \_\_\_\_\_ coefficient.
A. heterotrait-heteromethod
B. heterotrait-monomethod
C. monotrait-heterornethod
D. monotrait-monomethod
A

The multitrait-multirnethod matrix contains four types of correlation coefficients.
a. Incorrect The heterotrait-heteromethod coefficient is a measure of divergent validity.
b. Incorrect This heterotrait-monomethod coefficient is also a measure of divergent validity.
c. Incorrect The monotrait-heteromethod coefficient is a measure of convergent validity.
d. CORRECT The monotrait-monomethod coefficient indicates the correlation of the test with itself and is a measure of the test’s reliability.
(Test Construction)

493
Q
Which of the following techniques would be most useful for combining test scores when superior performance on one test can compensate for poor performance on another test?
A. multiple cutoff
B. multiple regression
C. multiple hurdle
D. multidimensional scaling
A

This question is asking about the preferred technique for combining test scores when a high score on one measure can compensate for a low score on another measure.
a. Incorrect Multiple cutoff is a noncompensatory technique.
b. CORRECT Multiple regression is a compensatory technique for combining test scores. When using this technique, a low score on one test can be offset (compensated for) by a high score on another tests.
c. Incorrect Multiple hurdle is also a noncompensatory technique.
d. Incorrect Multidimensional scaling is similar to factor analysis and is used to identify the similarities and differences underlying a set of variables.
(Test Construction)

494
Q

To maximize the inter-rater reliability of a behavioral observation scale, you should make sure that coding categories:
A. are mutually exclusive.
B. are measured on an interval or ratio scale.
C. produce criterion-referenced scores.
D. produce scores that are normally distributed.

A

When a person’s behavior is to be observed and recorded, that behavior must be operationalized in order for the observations to be meaningful. For example, a psychologist interested in obtaining data about aggressiveness in children might record data using categories such as “hits others” or “destroys property.”
a. CORRECT Coding categories must be discrete and mutually exclusive. For example, if the behavioral categories For aggressiveness were “aggressive acts” and “emotional displays,” the same behavior might be recorded twice, and an unreliable picture of a child’s behavior would be obtained.
b. Incorrect It is ordinarily better For categories to be discrete rather than continuous (i.e., to be measured on a nominal rather than an interval or ratio scale).
c. Incorrect This is not a requirement For (or usual characteristic of} behavioral observation scales.
d. Incorrect Since coding categories are often discrete (nominal), they do not produce scores that are normally distributed.
(Test Construction)

495
Q
Which of the following item difficulty (p) levels maximizes the differentiation of examinees into high- and low-performing groups:
A. 0.0
B. .50
C. .90
D. 1.50
A

An item difficulty level (p) ranges in value from 0 to +1.0 with a value of 0 indicating a very difficult item and a value of +1.0 indicating a very easy item. A difficulty index of .50 indicates that 50% of examinees in the try-out sample answered the item correctly.
b. CORRECT When p equals .50, this means that the item provides maximum differentiation between the upper- and lower-scoring examinees — i.e., a large proportion of examinees in the upper group answered the item correctly, while a small proportion of examinees in the lower group answered it correctly.
(Test Construction)

496
Q
Which of the following scores does not “belong with" the other three?
A. stanine scores
B. z-scores
C. percentile ranks
D. percentage scores
A

This question requires you be able to distinguish between norm- and criterion-referenced scores.
d. CORRECT The scores listed in answers a, b, and c permit norm-referenced interpretation; that is, they permit an examinee’s score to be compared to the scores of others who are taking or have taken the same test. In contrast, percentage scores are a type of criterion-referenced score that reference an examinee’s score to the content of the exam and indicate how much of the content an examinee has mastered.
(Test Construction)

497
Q
An educational psychologist designs a screening test to identify underachieving first- and second-grade children who have a learning disability. The psychologist will probably be rnost concerned that her test has
adequate \_\_\_\_\_\_ validity.
A. content
B. construct
C. concurrent
D. predictive
A

When a test developer wants to use a screening test to estimate or evaluate current behavior, he/she will be most interested in the test’s concurrent validity.
a. Incorrect Content validity is of most concern when test items are expected to be a representative sample of a particular content domain.
b. Incorrect Construct validity is of concern for tests designed to measure a hypothetical trait or construct (e.g., motivation, self-esteem).
c. CORRECT As noted above, concurrent validity (a type of criterion-related validity) is of concern when the purpose of a test is to estimate an individual’s current status on some external criterion. Screening tests are usually used to estimate the results of a more thorough evaluation (the criterion) which, in this case, would be a more thorough diagnostic procedure.
d. Incorrect The screening test is being used to estimate current status, not to predict Future status. Therefore, concurrent validity would be of greater concern than would predictive validity.
(Test Construction)

498
Q
When a test's reliability has been estimated by splitting the test in half and correlating scores on the two halves, you would use which of the following to correct the resulting reliability coefficient?
A. correction for attenuation formula
B. standard error of measurement
C. Kuder-Richardson 20 
D. Spearman-Brown formula
A

Split-half reliability tends to underestimate a measure’s reliability, so a correction formula is often used to estimate what the measure’s reliability coefficient would be if it were based on the full length of the test.
a. Incorrect The correction for attenuation formula is used to estimate the effects of increasing a predictor’s and/or criterion’s reliability coefficient on the predictor’s validity coefficient.
b. Incorrect The standard error of measurement is used to calculate the range within which an examinee’s true score is likely to fall given his/her obtained score.
c. Incorrect This is a type of internal consistency reliability that is used when test items are scored dichotomously.
d. CORRECT The Spearman-Brown formula is commonly used to correct the split-half reliability coefficient but can also be used whenever the goal is to estimate the effects of lengthening or shortening a test on its reliability coefficient.
(Test Construction)

499
Q

The slope (steepness) of an item response curve indicates which of the following?
A. the item’s ability to discriminate between high and low achievers.
B. the item’s difficulty level
C. the susceptibility of the item to social desirability effects
D. the probability that the item can be answered correctly by guessing

A

The various item response theory models produce item response curves (ICCs) that provide information on either one, two, or three parameters - difficulty level, discrimination, and probability of guessing correctly.
a. CORRECT The item’s ability to discriminate between those who have high and low levels of the attribute measured by the item is indicated by the slope of the ICC — the steeper the slope, the greater its discrimination.
b. Incorrect An item’s difficulty level is indicated by the ability level (charted on the horizontal axis) at which 50% of examines in the sample answered correctly.
c. Incorrect Susceptibility to social desirability is not indicated by the ICC.
d. Incorrect The probability of guessing correctly is indicated by the point at which the ICC intercepts the vertical axis.
(Test Construction)

500
Q
Stella S. obtains a score of 50 on a test that has a standard deviation of 10 and a standard error of measurement of 5. The 95% confidence interval for Stella's score is approximately:
A. 45 to 55.
B. 40 to 60.
C. 35 to 65. 
D. 30 to 70.
A

The 95% confidence interval for an obtained test score is constructed by multiplying the standard error of measurement by 1.96 and adding and subtracting the result to and from the examinee’s obtained score.
b. CORRECT This interval is closest to the 95% confidence interval and was obtained by multiplying the standard error by 2.0 (instead of 1.96) and then adding and subtracting the result (10) to and from Stella’s score of 50.
(Test Construction)

501
Q

Criterion contamination has which of the following effects?
A. It artificially increases scores on the criterion.
B. It artificially reduces the criterion”s reliability coefficient.
C. It artificially increases the predictor”s criterion-related validity coefficient.
D. It artificially attenuates scores on the predictor and the criterion

A

Criterion contamination occurs when a rater’s knowledge of a predictor performance biases how he/she rates the person on the criterion.
a. Incorrect The effect of criterion contamination on an individual’s criterion score will depend on how well he/she did on the predictor (e.g., if the individual received a low score on the predictor, she will receive a low rating on the criterion).
b. Incorrect The effect on reliability will depend on the situation and the method used to establish the reliability of the criterion.
c. CORRECT Criterion contamination has the effect of artificially inflating the correlation between the predictor and the criterion.
d. Incorrect This response doesn’t really make sense since the meaning of “attenuation” in this context is rather vague.
(Test Construction)

502
Q

The best way to control consensual observer drift is to:
A. use the correction for attenuation formula.
B. use a true experimental research design.
C. videotape the observers.
D. alternate raters.

A

Consensual observer drift occurs when observers’ ratings become increasingly less accurate over time in a systematic way.
d. CORRECT Of the actions described in the answers to this question, this one is the best way to alleviate consensual observer drift, which occurs when raters who are working together influence each other’s ratings so that they assign ratings in increasingly similar (and idiosyncratic) ways.
(Test Construction)

503
Q

When assessing the test-retest reliability of your newly developed personality test, you obtain a reliability coefficient of .80. This rneans that:
A. 80% of variability in test scores is shared variability.
B. 64% of variability in test scores is shared variability.
C. 80% of variability in test scores is true score variability.
D. 64% of variability in test scores is true score variability.

A

A reliability coefficient is interpreted directly as a measure of true score variability.
c. CORRECT Reliability is a measure of the extent to which a test consistently measures the attribute(s) assessed by the test — i.e., the extent to which scores reflect true score variability. Note that the reliability coefficient is not squared in order to interpret it but is, instead, interpreted directly.
(Test Construction)

504
Q

You would use a “multitrait-multimethod matrix” in order to:
A. compare a test”s predictive and concurrent validity.
B. determine if a test has adequate convergent and discriminant validity
C. identify the common factors underlying a set of related constructs.
D. test hypotheses about the causal relationships among variables.

A

A multitrait-multimethod matrix contains the correlation coefficients between measures that do and do not purport to measure the same trait.
a. Incorrect Predictive and concurrent validity are types of criterion-related validity. The multitrait-multimethod matrix is used to assess a measure’s construct validity.
b. CORRECT When a measure correlates highly with other measures of the same trait, the measure has convergent validity; when it has low correlations with measures of different traits, it has discriminant (divergent) validity. Convergent and discriminant validity are used as evidence of construct validity and the multitrait-multimethod matrix contains correlation coefficients that provide information about a measure’s convergent and discriminant validity.
c. Incorrect This answer describes factor analysis, which is used to identify the common factors that underlie a set of tests, test items, or other variables.
d. Incorrect This answer doesn’t describe the purpose of the multitrait-multimethod matrix.
(Test Construction)

505
Q

A test developer would use the Kuder-Richardson Formula (KR-20) in order to:
A. evaluate a test’s internal consistency reliability.
B. evaluate a test’s test-retest reliability.
C. determine the impact of increasing a test’s reliability on its validity.
D. determine the impact of lengthening a test on its reliability.

A

KR-20 provides information on internal consistency reliability.
a. CORRECT KR-20 is used to determine a test’s internal consistency reliability when test items are scored dichotomously.
(Test Construction)

506
Q

Incrernental validity is a measure of:
A. decision-making accuracy.
B. “shrinkage”
C. the generalizability of research results
D. the costs involved in using a predictor.

A

The name “incremental validity” can help you remember what this term refers to.
a. CORRECT Incremental validity refers to the increase (“increment”) in decision-ma king accuracy that results from the use of a new predictor (e.g., the increase in accurate hiring decisions).
(Test Construction)

507
Q
The item difficulty index (p) represents which scale of measurement?
A. ordinal
B. interval
C. nominal
D. ratio
A

The item difficulty index (p) is calculated by dividing the number of individuals who answered the item correctly by the total number of individuals.
a. CORRECT To understand why the item difficulty index represents an ordinal scale, assume that items 1, 2, and 3 of a test are passed by 10, 20, and 30 percent of examinees, respectively, which will result in p values for these items of .10, .20, and .30. Although these values indicate that item 1 is more difficult than item 2 which, in turn, is more difficult than item 3, it is not possible to say that item 2 is twice as difficult as item 1 or that the difference in difficulty between items 1 and 2 is equal to the difference between items 2 and 3. Moreover, an item difficulty index of 0 would not mean that the item completely lacks difficulty (which doesn’t really make any sense). In other words, p values represent an ordinal scale because they do not have the property of equal
intervals or an absolute zero point.
(Test Construction)

508
Q

Which of the following best describes the relationship between validity and reliability?
A. A valid test is also a reliable test.
B. A valid test may or may not be a reliable test.
C. A reliable test is also a valid test.
D. An invalid test is not a reliable test.

A

Knowing that reliability is a necessary but not sufficient condition for validity would have helped you identify the correct answer to this question.

a. CORRECT Reliability sets an upper limit on validity, which means that a valid test it also be a reliable test. However, high reliability does not guarantee validity — i.e., a test can be free from the effects of measurement error but not measure the attribute it was designed to measure.
b. Incorrect A valid test must also be reliable.
c. Incorrect A reliable test may or may not be valid.
d. Incorrect An invalid test may or may not be reliable.(Test Construction)

509
Q
In a normal distribution, a T score of \_\_\_\_\_ is equivalent to a percentile rank of 16.
A. 10
B. 20
C. 30
D. 40
A

To identify the correct answer to this question, you need to be familiar with the areas under the normal curve and know that T scores have a mean of 50 and a standard deviation of 10.
d. CORRECT In a normal distribution, a percentile rank of 16 and a T score of 40 are both one standard deviation below the mean.
(Test Construction)

510
Q

When a test has been constructed on the basis of item response theory, an examinee’s total test score provides information about his/her:
A. status on a latent trait or ability.
B. predicted performance on an external criterion.
C. performance relative to other examinees included in the standardization sample.
D. current developmental level.

A

Item response theory is an alternative to classical test theory for the development of tests and interpretation of test scores.
a. CORRECT Scores on tests developed on the basis of item response theory are reported in terms of the examinee’s level on the trait or ability measured by the test rather than in terms of a total score. An advantage of this method of score reporting is that it makes it possible to compare scores from different sets of items and from different tests.
d. Incorrect Information on current developmental level might be provided by some, but not all, tests that are developed on the basis of item response theory.
(Test Construction)

511
Q
Which type of reliability would be rnost appropriate for estimating the reliability of a multiple-choice speecled test?
A. split-half
B. coefficient of concordance
C. alternate forms
D. coefficient alpha
A

Speeded tests are designed so that all items answered by an examinee are answered correctly, and the examinee’s total score depends primarily on his/her speed of responding. Because of the nature of
these tests, a measure of internal consistency will provide a spuriously high estimate of the test’s reliability.
a. Incorrect Split-half reliability is a type of internal consistency reliability and is not appropriate for speeded tests.
b. Incorrect The coefficient of concordance is a measure of inter-rater reliability for three or more raters when ratings are ranks. The test described in this question is a multiple-choice test, not a rating scale or other subjectively scored measure that would report scores in terms of ranks. Therefore, inter-rater reliability would not be a concern.
c. CORRECT Alternate-forms reliability is an appropriate method for establishing the reliability of speeded tests.
d. Incorrect Coefficient alpha is a type of internal consistency reliability and is less appropriate for speeded test than is alternate forms reliability.
(Test Construction)

512
Q

After reviewing the data collected on a new selection test during the course of a criterion-related validity study, an industrial psychologist decides to lower the selection test cutoff score. Apparently the psychologist is hoping to:
A. reduce the number of false negatives
B. increase the number of true positives
C. reduce the number of false positives ‘
D. increase the number of false negatives

A

By lowering the predictor cutoff, the psychologist will increase the number of people who are accepted on the basis of their selection test score.
a. Incorrect Lowering the predictor cutoff will decrease the number of false negatives. However, it is not likely that the personnel director will want to decrease the number of individuals who would have been successful on the criterion if they had been hired.
b. CORRECT Lowering the selection test (predictor) cutoff will increase the number of positives, including the number of true positives, who are individuals who will be selected on the basis of their test scores and will be successful on the criterion.
c. Incorrect Lowering the selection test cutoff will increase the number of false positives.
d. Incorrect Lowering the selection test cutoff will decrease the number of false negatives.
(Test Construction)

513
Q

To obtain a “coefficient of stability,” you would:
A. administer the same test twice to the same group of examinees on two separate occasions and correlate the two sets of scores.
B. administer a test to a group of examinees and determine the average inter-item correlation.
C. administer a test to two different random samples of examinees on two occasions and correlate the two sets of scores.
D. administer parallel forms of a test to the same group of examinees and correlate the two sets of scores.

A

The coefficient of stability is another name for the test-retest reliability coefficient.

a. CORRECT To obtain a coefficient of stability, the same measure is administered to the same group of examinees on two separate occasions and the scores obtained by the examinees are correlated. The result indicates the consistency (stability) of scores over time.
b. Incorrect This answer describes the procedure for obtaining a coefficient of internal consistency.
c. Incorrect When assessing test-retest reliability, the test is administered twice to the same group of examinees, not to different to different groups of examinees.
d. Incorrect This would provide a coefficient of equivalence (alternate forms reliability coefficient).(Test Construction)

514
Q

The assumption underlying convergent validity is that:
A. a measure of a characteristic should correlate highly with a different type of measure that is already known to assess the same characteristic.
B. a valid measure of a variable should have as much (or more) factorial validity as does an existing measure of the same variable.
C. a measure of a construct should correlate more highly with itself than with another measure of the same construct.
D. to be valid, a measure of a characteristic should correlate highly with the measure of the behavior it is designed to predict.

A

A test has convergent validity when it correlates highly with another measure of the same trait.
a. CORRECT One way to establish a test’s construct validity is to determine that it correlates highly with other measures that are already known to assess the same trait. When it does, the measure is said to have convergent validity.
(Test Construction)

515
Q

To evaluate the concurrent validity of a new selection test for clerical workers, you would:
A. conduct a factor analysis to confirm that the test measures the attributes it was designed to measure.
B. have supervisors and others familiar with the job rate test items for relevance to success as a clerical worker.
C. administer the test to a sample of current clerical workers and correlate their scores on the test with their recently assigned performance ratings.
D. administer the test to clerical workers when they are initially hired and six months after they are hired and then correlate the two sets of scores.

A

Concurrent validity is a type of criterion-related validity.
a. Incorrect This technique would be used to evaluate the test’s construct validity.
b. Incorrect This would help establish the test’s content validity.
c. CORRECT To evaluate a test’s criterion-related validity, scores on the predictor (in this case, the selection test) are correlated with scores on a criterion (measure of job performance). When scores on both measures are obtained at about the same time, they provide information on the test’s concurrent validity.
d. Incorrect This procedure would not provide information on the test’s concurrent validity.
(Test Construction)

516
Q

An achievement test is administered to 250 high school seniors. The test’s mean is 50, its standard deviation is 5, and the test scores are normally distributed. If you want to use test scores to select the top 16% of the
students, you will set the cutoff score at:
A. 55
B. 60
C. 65
D. 70

A

By the time you take the licensing exam, you’ll want to have memorized the areas under the normal curve so that you can answer questions like this one
a. CORRECT In a normal distribution, 50% of examinees obtain scores at the mean or below and an additional 34% obtain scores Falling between the mean and the score that is one standard deviation above the mean: 50% plus 34% equals 84%. Consequently, the score that is one standard deviation above the
mean divides the distribution into two sections: 84% of the people in the distribution obtain scores below that score and the remaining 16% obtain higher scores. The test described in this question has a mean of 50 and a standard deviation of 5, so the score that is one standard deviation above the mean is 55
(Test Construction)

517
Q

Your newly developed measure of integrity correlates highly with a well-
known and widely used measure of integrity. This correlation provides
evidence of your measure’s ______ validity.
A. incremental
B. internal
C. discriminat
D. convergent

A

In this situation, one measure of a specific construct correlates highly with another measure of the same construct.

a. Incorrect Incremental validity is a measure of decision-making accuracy and is associated with criterion-related validity.
b. Incorrect Internal validity is one of the standards used to evaluate research designs and is not relevant to the situation described in this question.
c. Incorrect Discriminant validity (also known as divergent validity) is the extent to which a test does not correlate with measures of different constructs.
d. CORRECT A high correlation between a new and an established measure of the same construct provides evidence of the new measure’s convergent validity (which, in turn, provides evidence of its construct validity).(Test Construction)

518
Q
The applicants for sales positions at the Acme Company complain that the selection test they are required to take is unfair because it doesn't “look like” it measures the knowledge and skills that are important for successful job performance. Their complaint suggests that the selection test is lacking which of the following?
A. incremental validity
B. differential validity
C. construct validity
D. face validity
A

In this situation, the selection test doesn’t appear to be measuring the skills and knowledge that are important for success as a salesperson.
d. CORRECT Face validity refers to the extent that a test appears to be valid to test-takers — i.e., to the extent that the test “looks like” it is measuring what it is supposed to be measuring.
(Test Construction)

519
Q

In factor analysis, comrnunality refers to:
A. the proportion of variance accounted in a single variable by a single factor.
B. the proportion of variance accounted in multiple variables by a single factor.
C. the proportion of variance accounted for in a single variable by all of the identified factors.
D. the total proportion of variance in all of the variables included in the analysis that is not due to error.

A

A communality is a measure of “common variance” and is a reflection of the amount of variance that a test has in common with the other tests included in the factor analysis.
c. CORRECT In factor analysis, a communality is calculated for each test (variable) included in the analysis. The communality indicates the total amount of variability accounted for in the test by all of the identified factors.
(Test Construction)

520
Q

A 200-item test that has been administered to 100 college students has a normal distribution, a mean of 145, and a standard deviation of 12. When the students’ raw scores have been converted to percentile ranks, Alex
obtains a percentile rank of 49, while his twin sister Alicia obtains a percentile rank of 90. The teacher realizes that she made a mistake in scoring Alex and Alicia’s tests: Both should have received a raw score that
was five points higher. In terms of their percentile ranks, when the teacher adds the five points to Alex and Alicia’s scores, she can expect that:
A. Alicia”s percentile rank will increase more than Alex”s.
B. Alex”s percentile rank will increase more than Alicia”s.
C. Alicia and Alex”s percentile ranks will increase by the same amount.
D. Alicia and Alex”s percentile ranks will not change.

A

A problem with percentile ranks is that, when the raw scores are normally distributed, raw score differences near the center of the distribution are exaggerated when they are converted to percentile ranks, while raw score differences at the extremes are reduced.
b. CORRECT Because of the above-described phenomenon, Alex’s percentile rank will increase more than Alicia’s. This makes sense if you think about the normal distribution: Since most of the scores are “piled up” near the center of the distribution, the increase in 5 points in Alex’s score will position him above a
larger number of examinee’s than the 5 point increase in Alicia’s score. This difference will be reflected in their percentile ranks.
(Test Construction)

521
Q

A final exam is developed to evaluate students’ comprehension of information presented in a high school history class. When the exam is administered to three classes of students at the end of the semester, all
students obtain failing scores. This suggests that the exam may have poor _____ validity.
A. concurrent
B. incremental
C. content
D. divergent

A

The first sentence of this question gives you the information you need to identify the correct answer to this question: It states that the purpose of the test is to assess the students‘ knowledge of the content
presented in the history course.
a. Incorrect Concurrent validity (a type of criterion-related validity) refers to the extent to which test scores correlate with scores on an external criterion. In this situation, performance on the test is the measure of performa nce, and test scores are not being correlated with scores on an external criterion. Consequently, concurrent validity is not relevant to this situation.
b. Incorrect Incremental validity is associated with criterion-related validity and refers to the increase in decision-ma king accuracy that results from use of a predictor. Incremental validity is not an issue in this situation.
c. CORRECT If all students do poorly on a test designed to assess their mastery of the course content, one possible reason is that the test questions do not represent that content; i.e., the test does not have adequate content validity. (There are, of course, other possible reasons, for the students’ low scores but, of
answers given, this is the best one.)
d. Incorrect A measure has divergent validity when scores on the measure do not correlate with scores on measures of unrelated traits. Divergent validity is not relevant to this situation.
(Test Construction)

522
Q

All other things being equal, which of the following tests is likely to have the largest reliability coefficient?
A. a multiple-choice test that consists of items that each have five answer opfions
B. a multiple-choice test that consists of items that each have four answer opfions
C. a multiple-choice test that consists of iterns that each have three answer options
D. a true-false test

A

For the licensing exam, you’ll want to be familiar with the factors that impact the magnitude of the reliability coefficient.
a. CORRECT All other things being equal, tests containing items that have a low probability of being answered correctly by guessing alone are more reliable than tests containing items that have a high probability of being answered correctly by guessing alone. Of the types of items listed, multiple-choice items with five answer options have the lowest probability of being answered correctly by guessing alone.
(Test Construction)

523
Q

When a test user uses a correction for guessing formula that involves subtracting points from each examinee’s scores, the resulting distribution of scores will have a _____ than the original (non-corrected) distribution.
A. higher mean and larger standard deviation
B. higher mean and smaller standard deviation
C. lower mean and larger standard deviation
D. lower mean and smaller standard deviation

A

This is a “distant galaxy” question that, fortunately, is more difficult than most of the questions on test construction that you’ll encounter on the licensing exam.
c. CORRECT The effect of this type of correction for guessing formula on a distribution’s mean is fairly easy to understand — i.e., use of the formula will result in reducing the scores of some examinees and, thereby, reduce the magnitude of the mean. To understand its effect on the standard deviation, assume that the lowest expected score on a test is 25% and that the highest score is 100%, which is obtained by at least one examinee. In this situation, as a result of the correction for guessing, some examinees will obtain scores g
than 25%, while the highest scorer will still receive a score of 100%. When this occurs, the range of scores will increase, and this will be reflected in the distribution’s standard deviation.
(Test Construction)

524
Q
The optimal item difficulty level (p) for a true/false test is:
A. +1.0
B. .75
C. .25
D. -1.0
A

The optimal item difficult level depends on several factors, including the probability that an examinee can select the correct answer by chance alone.
b. CORRECT When considering the probability that an examinee can select the correct answer by chance alone, the optimal difficulty level is halfway between 100% of examinees answering the item correctly and probability of answering the item correctly by chance alone. For a true/false item, the latter is 50%, so
the optimal item difficulty is 75% (.75), which is halfway between 100% and 50%.
(Test Construction)

525
Q

The correction for attenuation formula is used to measure the impact of increasing:
A. a test’s reliability on its validity.
B. a test’s validity on its reliability.
C. the number of test items on the test’s validity.
D. the number of test items on the test’s reliability.

A

For the exam, you’ll want to be familiar with the purpose of several formulas including the correction for attenuation formula.
a. CORRECT The correction for attenuation formula is used to determine the impact of increasing the reliability of the predictor (test) andfor the criterion on the predictor’s validity.
(Test Construction)

526
Q
A personnel director uses a mechanical aptitude test to hire machine shop workers. Several of the people hired using the test turn out to be less than adequate performers. These individuals are:
A. true positives.
B. true negatives.
C. false positives.
D. false negatives.
A

For the exam, you’ll want to be familiar with the definitions of the terms listed in the answers to this question.
a. Incorrect True positives are individuals who are predicted to perform satisfactorily by the predictor and, in fact, do well on the criterion.
b. Incorrect True negatives are individuals who are predicted to perform poorly by the predictor and, in fact, perform poorly on the criterion.
c. CORRECT False positives are individuals who are predicted to perform satisfactorily by the predictor but, in fact, perform poorly on the criterion. In other words, these individuals have been “falsely identified as positives.”
d. Incorrect False negatives are individuals who are predicted to perform poorly by the predictor but, in fact, do well on the criterion.
(Test Construction)

527
Q
You ask a group of experienced salespeople to review the test items included in a test you have developed to help select new sales applicants. You are apparently interested in determining the test's \_\_\_\_\_ validity.
A. incremental
B. content
C. concurrent
D. differential
A

Of the types of validity listed in the answers, only one is established primarily by having items reviewed by subject matter experts.
a. Incorrect Incremental validity refers to the degree to which use of a test increases decision-making accuracy.
b. CORRECT A test‘s content validity refers to the extent to which test items represent the domain of knowledge, skills, andfor abilities the test was designed to measure. Content validity is established primarily by having subject matter experts evaluate items in terms of their representativeness.
c. Incorrect Concurrent validity is a type of criterion-related validity and is evaluated by correlating scores on the test with scores on an external criterion.
d. Incorrect A test has differential validity when it has different validity coefficients for different groups.
(Test Construction)

528
Q
Assuming no constraints in terms of time, money, or other resources, the best (most thorough) way to demonstrate that a test has adequate reliability is by using which of the following techniques?
A. equivalent forms
B. test-retest
C. Cronbach”s alpha
D. Cohen"s kappa
A

The most thorough method for assessing reliability is the one that takes into account the greatest number of potential sources of measurement error.
a. CORRECT Because equivalent (alternate) Forms reliability takes into account error due to both time and content sampling, it is the most thorough method For establishing reliability and, consequently, is considered by some experts to be the best method.
(Test Construction)

529
Q

Cronbach’s alpha is an appropriate method for evaluating reliability when:
A. all test items are designed to measure the same underlying characteristic.
B. test items are subjectively scored.
C. the test will be administered to examinees at regular intervals over time.
D. there is a restriction in the range of scores.

A

To answer this question, you need to know that Cronbach’s alpha is another name for coefficient alpha and is used to assess internal consistency reliability.
a. CORRECT Cronbach’s alpha is an appropriate method for evaluating reliability when the test is expected to be internally consistent — i.e., when all test items measure the same or related characteristics.
(Test Construction)

530
Q

To maximize the ability of a test to discriminate among test takers, a test developer will want to include test items that vary in terms of difficulty. If the test developer wants to add more difficult items to her test, she will
include items that have an item difficulty index of:
A. .90
B. .50.
C. .10.
D. .00.

A

The item difficulty index ranges from 1.0 (which occurs when everyone in a sample answers the item correctly) to .0 (which occurs when no one answers the item correctly).
a. Incorrect When an item’s difficulty index is .90, this means that it is a very easy item — i.e., it was answered correctly by 90% of examinees in the sample.
b. Incorrect An item difficulty level of .50 indicates an item of moderate difficulty (50% of examinees answered the item correctly).
c. CORRECT An item difficulty level of .10 indicates a difficult item (only 10% of examinees in the sample answered it correctly) and is the best answer of those given.
d. Incorrect An item difficulty level of .0 means that no examinees answered this item correctly. Although this is a difficult item, it would not be useful for discriminating among examinees and, therefore, would not be included in a test.
(Test Construction)

531
Q

When using criterion-referenced interpretation of scores obtained on a knowledge test, you would most likely be interested in which of the following?
A. the total number of test items answered correctly by an examinee
B. an examinee”s performance relative to that of other examinees
C. an examinee”s standing on two or more measures designed to assess the same characteristic
D. ensuring that test items are based on a systematic job evaluation

A

As its name implies, criterion-referenced interpretation entails interpreting an examinee’s score in terms of a criterion, or standard of performance.
a. CORRECT One criterion that is used to interpret a person’s test score is the total number of correct items. This criterion is probably most associated with “mastery testing.” A person is believed to have mastered a content area when he/she obtains a predetermined minimum score on the test that is designed to assess knowledge of that area. There are other types of criteria that are external to the test itself that are used in criterion-referenced interpretation but none of the other responses addresses those types of interpretation; and, consequently, this answer is the best one.
c. Incorrect This really doesn’t make any sense.
d. Incorrect For some tests used in organizational settings, it might be important to base the content of the test on a job analysis (but not on a job evaluation, which is done to set wages and salaries).
(Test Construction)

532
Q

Factor analyses of tests designed to measure psychomotor skills have found that these skills:
A. are relatively independent.
B. reflect an underlying “g” (general) factor.
C. represent three interdependent factors (speed, strength, and coordination).
D. are strongly related to intelligence.

A

Measures of psychornotor skills are usually apparatus tests that assess speed, coordination, strength, and other movement responses.
a. CORRECT Factor analyses have shown that there is no “general psychomotor Factor” and that the various psychomotor skills are relatively independent.
(Psych Assess)

533
Q
When using Holland's Self-Directed Search, the degree of match between an examinee’s personality and the characteristics of a particular job or career is referred to as:
A. cornmonness
B. consistency
C. coherence
D.  congruence
A

The detailed score report for Holland’s Self-Directed Search (SDS) provides information on consistency, differentiation, cornmonness, and coherence.

a. Incorrect Commonness indicates the degree to which an examinee’s score code occurred in the standardization sample (i.e., the percent of people in the sample who obtained the same code).
b. Incorrect Consistency refers to the distance between the examinee’s first two code letters in the RIASEC hexagon. When the first two code letters are adjacent in the hexagon, the individual’s consistency is high.
c. Incorrect Coherence is the degree to which an individual’s vocational/career aspirations belong to the same Holland category.
d. CORRECT Congruence refers to the degree of match between a person’s personality (as measured by the SDS) and the characteristics of a particular job or career.(Psych Assess)

534
Q
To determine a student's instructional needs in terms of current course content, you would be best advised to use:
A. a norm-referenced test.
B. a standardized achievement test.
C. a functional analysis.
D. a curriculum-based measurement.
A

You may have been able to answer this question through the process of elimination — i.e., by knowing that the description in the question doesn’t fit three of the responses.
a. Incorrect A norm-referenced test is one that permits interpretation of an individual examinee’s performance in terms of the performance of other examinees in the norm sample.
b. Incorrect A standardized achievement test provides information about an examinee’s level of performance but usually doesn’t yield the kind of information that is needed to pinpoint a student’s instructional needs.
c. Incorrect A functional analysis is a type of behavioral assessment that is used to identify the characteristics of a target behavior as well as its antecedents and consequences in order to determine the best way to modify that behavior.
d. CORRECT As its name implies, curriculum-based measurement is closely linked to the curriculum of interest. It involves periodic assessment of school-aged children with brief standardized and validated measures of basic academic skills that reflect the school’s curriculum in order to identify the student’s
instructional needs.
(Psych Assess)

535
Q
The Leiter International Performance Scale would be most useful for measuring the intelligence of:
A. adults with visual impairments.
B. children under the age of two.
C. children with hearing impairments.
D. adults who are mentally retarded.
A

The Leiter International Performance Scale was developed as a culture-fair test for children ages 2 through 12.
a. Incorrect The Haptic Intelligence Scale can be used to assess the intelligence of individuals with visual impairments.
b. Incorrect Tests used to assess the intellectual development of infants under age 2 include the Bayley Scales of Infant Development and the Fagan Test of Intelligence.
c. CORRECT A distinctive feature of the Leiter International Performance Scale is its non-relia nce on verbal instructions or responses. Because of this feature, it is an appropriate test For individuals with hearing and language impairments.
d. Incorrect The intelligence of individuals with mental retardation is commonly assessed with the Stanford-Binet.
(Psych Assess)

536
Q

Which of the following are the routing subtests for the Stanford-Binet, 5th Edition?
A. Object Series/Matrices and Vocabulary
B. Quantitative Reasoning and Memory for Sentences
C. Vocabulary and Quantitative Reasoning
D. Picture Absurclities and Verbal Absurdities

A

The routing subtests are used to determine the appropriate starting point for the other Stanford-Binet, 5th Edition (SB5) subtests.
a. CORRECT Administration of each of the remaining subtests begins at a level at or slightly below the examinee’s ability level, as indicated by his/her performance on the Object Series,’Matrices and Vocabulary subtests.
(Psych Assess)

537
Q

An elevated score on the MMPI-2’s F scale is most suggestive of which of the following?
A. deliberate rnalingering
B. deliberate attempt to present oneself in a favorable light ‘
C. tendency toward social conformity
D. defensiveness or low stress tolerance

A

The MMPI-2 includes several validity scales that provide information on an examinee’s test-taking attitudes and style.
a. CORRECT An extremely high score on the F (frequency) scale indicates a deliberate attempt to “Fake bad,” carelessness in responding, or gross eccentricity.
b. Incorrect This is suggested by an elevated score on the L scale.
c. Incorrect This is suggested by a low score on the F scale.
d. Incorrect This is also suggested by an elevated L scale score.
(Psych Assess)

538
Q

Dynamic testing is most useful for:
A. identifying an examinee’s ability to recall specific information from a well-defined content domain.
B. identifying the antecedents and consequences that are maintaining a behavior.
C. determining if test items accurately predict performance on an external criterion.
D. determining if an examinee would benefit from assistance or instruction.

A

Dynamic testing involves deviating from standardized procedures for a test after the test has been administered to the examinee using the standardized procedures.
d. CORRECT Dynamic testing is based on the assumption that cognitive abilities and processes are modifiable and is used to assess an examinee’s learning potential — i.e., to determine if an examinee’s test performance improves when he/she receives assistance or instruction.
(Psych Assess)

539
Q
To assess the general intelligence of a six-year-olcl child with severe hearing impairment, you would use which of the following tests?
A. Halstead-Reitan 
B. Kaufman Assessment Battery
C. Haptic Intelligence Test
D. Hiskey-Nebraska
A

Of the tests listed in the answers, only one is useful for evaluating intelligence of a child with impaired hearing.
a. Incorrect The Halstead-Reitan is used to identify individuals with neurological impairment.
b. Incorrect The Kaufman Assessment Battery (KAB) is a measure of cognitive ability for individuals aged 3 to 18. Administration of the KAB involves verbal instructions, so it would not be useful for children with hearing impairments.
c. Incorrect The Haptic Intelligence Test is a measure of intelligence for adolescents and adults with impaired vision.
d. CORRECT A number of tests have been developed as alternatives to the Stanford-Binet and Wechsler tests for assessing the general intellectual ability of individuals with disabilities. The Hiskey-Nebraska test was developed for use with children aged 3 to 16 who have hearing impairments.
(Psych Assess)

540
Q

A 10-year-old who has received a diagnosis of ADHD is likely to obtain the lowest scores on which of the following WISC-IV subtests?
A. Cancellation, Arithmetic, and Coding
B. Arithmetic, Word Reasoning, and Similarities
C. Comprehension, Symbol Search, and Coding
D. Cancellation, Letter-Number Sequencing, and Picture Completion

A

The WISC-IV Technical’ Manual provides scores patterns that are characteristic of children with ADHD and other disorders.
a. CORRECT Children with ADHD are likely to obtain the lowest scores on the Cancellation, Arithmetic, and Coding subtests and the highest scores on the Picture Concepts, Picture Completion, Word Reasoning, and Similarities subtests.
(Psych Assess)

541
Q
Performance on which of the following is least likely to be adversely affected by increasing age in adulthood?
A. a measure of implicit memory
B. a dichotic listening task
C. a measure of processing speed
D. a test of visual acuity
A

Increasing age is associated with decrements in a number of cognitive and physical abilities.
a. CORRECT Although explicit memory shows age-related declines, implicit memory is relatively unaffected by increasing age.
b. Incorrect Dichotic listening assesses divided attention which does show age-related declines.
c. Incorrect Slowing in the speed of processing and responding is one of the most consistent findings of research investigating the impact of increasing age on cognitive abilities.
d. Incorrect Visual acuity begins to decline somewhat in the 40s and then shows a rapid decline between 60 and 80.
(Psych Assess)

542
Q

A test developer would most likely administer test items to samples of people who vary in terms of age, gender, race/ethnicity, and socioeconomic status for the purpose of:
A. standardization.
B. assessing incremental validity.
C. establishing criterion-related validity.
D. domain-referenced testing.

A

This is a fairly straightforward question, but you may have had to be careful not to be misled by an answer you’re not familiar with.
a. CORRECT The purpose of standardization is to develop norms, which are scores obtained by clearly described samples. Norms facilitate test score interpretation by making it possible to compare an examinee’s test performance to that of the performance of people in an appropriate norm group (e.g.,
examinees of the same age and race/ethnicity).
b. Incorrect A measure has incremental validity when its use results in an increase in decision-making accuracy.
c. Incorrect Establishing criterion-related validity involves correlating test (predictor) and criterion scores.
d. Incorrect Domain-referenced testing is also known as criterion- or content-referenced testing and refers to a method of score interpretation that indicates what an examinee knows or can do in terms of a particular content or behavior domain.
(Psych Assess)

543
Q

According to Sternberg (1996), “successful intelligence” consists of:
A. eight intelligences (e.g., linguistic, spatial, interpersonal).
B. “g” (general intelligence) plus specific cognitive abilities related to the task.
C. three abilities — analytical, creative, and practical.
D. a combination of fluid and crystallized intelligence.

A

Sternberg’s triarchic theory of intelligence (more recently referred to as successful intelligence) distinguishes between three components.
c. CORRECT According to Sternberg, successful intelligence consists of the three abilities described in this answer (analytical, creative, and practical). He has criticized traditional intelligence tests on the ground that they focus on analytical ability only.
(Psych Assess)

544
Q
An examinee whose highest score on Holland's Self-Directed Search is on the realistic scale would probably be least interested in pursuing a career as a:
A. farmer.
B. bookkeeper.
C. technical writer. 
D. social worker.
A

The Self-Directed Search provides scores on six occupational themes that are conceptualized as being similar to one another to varying degrees.
a. Incorrect Agricultural work is consistent with the realistic theme.
b. Incorrect Bookkeeper is associated with the conventional theme, which is similar to the realistic theme.
c. Incorrect Technical writer is associated with the investigative theme, which is also similar to the realistic theme.
d. CORRECT Social work is appropriate For people who score highest on the social theme which is the theme most dissimilar to the realistic theme.
(Psych Assess)

545
Q
A score of 50 on the Beck Depression Inventory-II suggests:
A. no or minimal depression.
B. mild depression.
C. moderate depression.
D. severe depression.
A

The Beck Depression Inventory-II consists of 21 items that the examinee’s rates on a four-point scale ranging from 0 (minimal) to 3 (severe).
d. CORRECT An examinee’s score on the BDI-II can range from 0 to 63, and scores of 29 and above are considered indicative of severe depression.
(Psych Assess)

546
Q

A psychologist administers the MMPI-2 to a 36-year old man who has exhibited a number of symptoms suggestive of neurosis. The man receives a very low K Scale score, and, on the basis of this score, the psychologist can tentatively conclude that the man:
A. tried to present himself in a favorable light.
B. is very self-critical.
C. has a personality disorder.
D. may have an organic brain disorder.

A

The K (Correction) Scale is one of the MMPI’s validity scales.
a. Incorrect An elevated K score might indicate an attempt to “fake good.”
b. CORRECT A very low score on the K scale indicates excessive frankness or self-criticism or an attempt to “fake bad.”
c. Incorrect The MMPI-2’s validity scales were not designed to provide information about personality disorders.
d. Incorrect The MM PI-2’s validity scales were not designed to detect organicity
(Psych Assess)

547
Q

Longitudinal studies have generally found that scores on infant intelligence tests administered prior to the age of 18 months have what relation to later IQ test scores?
A. They have high validity for predicting childhood and adolescent IQ scores.
B. They have moderate validity for predicting childhood, adolescent, and adult IQ scores.
C. They have little validity for predicting adult IQ scores.
D. They tend to overestimate adolescent and adult IQ scores.

A

Note that this question is asking about intelligence test scores obtained prior to age 18 months.
c. CORRECT Studies have found that infant and preschool intelligence tests administered after age 24 months have moderate validity for predicting future
IQ scores but that these tests have little or no validity for predicting future IQ when administered prior to 24 months of age. Therefore, of the answers given,
this one best describes the results of the research.
(Psych Assess)

548
Q
When administering the \_\_\_\_\_, the examinee is presented with a list of color names that are printed in a color that differs from the name (e.g., the name red might be printed in blue or green ink) and the examinee is asked to go through the list and say the ink color rather than read the color name.
A. Beery Test
B. Benton Test
C. Stroop Test
D. Glasgow Scale
A

The test described in this question is a measure of cognitive flexibility and selective attention.
a. Incorrect The Beery Test of Visual-Motor Integration is a measure of visual-motor skills.
b. Incorrect The Benton Visual Retention Test is used to assess visual memory.
c. CORRECT The Stroop Color-Word Association Test assesses the degree to which the examinee can suppress a prepotent response in Favor of an unusual one (i.e., can name the print color rather than the color name). Poor performance on the Stroop Test has been linked to Frontal lobe damage, ADHD, autism, and depression.
d. Incorrect The Glasgow Coma Scale is used to assess consciousness following a traumatic brain injury.
(Psych Assess)

549
Q
A school psychologist will administer which of the following tests to evaluate the intelligence of a 13-year old who received a diagnosis of mental retardation when she was 6 years old?
A. WPPSI-III
B. WAIS-III
C. SB5
D. CMMS
A

For the licensing exam, you’ll want to be familiar with the purpose and age ranges of the four tests listed in the answers to this question.
a. Incorrect The WPPSI-III is appropriate for individuals aged 2 years, 6 months to 7 years, 3 months.
b. Incorrect The WAIS-III is appropriate for individuals aged 16 through 89.
c. CORRECT The SB5 (Stanford-Binet Intelligence Scales, Fifth Edition) was designed for use with individuals aged 2 to 85+ and is considered a reliable measure for lower Functioning individuals.
d. Incorrect The CMMS (Columbia Mental Maturity Scale) is a measure of general reasoning For individuals aged 3 through 10.
(Psych Assess)

550
Q

The Occupational Scales of the Kuder Occupational Interest Survey:
A. are appropriate only for individuals considering careers that do not require a college education.
B. provide information on the similarity of the examinee’s interests with those of satisfied workers in different occupations.
C. provide an overview of the examinee’s occupational interests in terms of ten broad areas that are similar to Holland’s themes.
D. indicate the degree to which the examinee’s personal characteristics match the work environments of particular jobs.

A

The Kuder Occupational Interest Survey (KOIS) provides scores on four scales - Occupational Scales, College Major Scales, Vocational Interest Estimates, and Dependability Indices.
a. Incorrect The Occupational Scales include a broad range of occupations.
b. CORRECT This answer accurately describes the Occupational Scales.
c. Incorrect This answer describes the Vocational Interest Estimates.
d. Incorrect This answer does not describe the Occupational Scales.
(Psych Assess)

551
Q

Which of the following Wechsler subtests is incorrectly described in terms of the abilities it measures?
A. Comprehension: common sense and social judgment
B. Similarities: logical, abstract thinking
C. Picture Arrangement: nonverbal reasoning and social awareness
D. Object Assembly: long-term memory and visual planning

A

For the exam, you’ll want to be familiar with the abilities measured by the Wechsler subtests. Note that this question is asking which subtest is described incorrectly.
a. Incorrect The Comprehension subtest is a measure of common sense and social judgment.
b. Incorrect The Similarities subtest assesses abstract thinking.
c. Incorrect Nonverbal reasoning and social awareness are measured by the Picture Arrangement subtest.
d. CORRECT The Object Assembly subtest is a measure of visual-motor speed and coordination and the ability to see part/whole relationships.
(Psych Assess)

552
Q

Which of the following is the best conclusion about the research investigating the effects of examiner race on the IQ test scores obtained by African-American children?
A. The research has consistently found that African-American children obtain higher scores when the examiner is of the same race.
B. The research has consistently found that African-American children obtain lower scores when the examiner is of the same race.
C. The research has found that African-American children tend to exhibit greater variability in subtest scores when the examiner is of the same race.
D. The research has not found a consistent effect of examiner race on the scores of African-American children.

A

Like much of the research on racial/ethnic matching, the research on the effects of examiner race on examinee IQ test scores has provided inconsistent results.
d. CORRECT The research results on this issue are far from consistent. Consequently, the best conclusion that can be reached is that there is no consistent effect of a match or mismatch of the examiner and examinee in terms of race/ethnicity on the examinee’s IQ test scores.
(Psych Assess)

553
Q

When administering the Rorschach Inkblot Test, the purpose of the inquiry phase is to:
A. initially find out what the examinee sees in the inkblot.
B. obtain information needed to score the test.
C. determine what questions to ask during the free association phase.
D. give the examinee an opportunity to ask questions about the test’s purpose and procedures.

A

The inquiry phase is the second phase in the administration of the Rorschach test.
a. Incorrect This is a better description of the initial Free association phase.
b. CORRECT During the inquiry phase, the examiner actively questions the examinee about the Features of the inkblots that determined the examinee’s responses in order to Facilitate scoring.
c. Incorrect This answer does not describe the purpose of the inquiry phase.
d. Incorrect This answer does not describe the purpose of the inquiry phase.
(Psych Assess)

554
Q

An examinee’s score on the Benton Visual Retention Test (BVRT) reflects his/her ability to:
A. reproduce geometric figures from memory.
B. inhibit prepotent responses.
C. distinguish figure from ground.
D. verbally describe visual stimuli.

A

The BVRT is used to assess visual memory, spatial perception, and visual motor skills for the purpose of identifying brain injury.
a. CORRECT Administration of the BVRT requires the examinee to reproduce (draw) simple geometric figures from memory. The greater the number of errors, the more likely that the examinee has a brain injury.
(Psych Assess)

555
Q
Which of the following cognitive abilities is least likely to show a noticeable decline after age 60?
A. abstract reasoning
B. attention span
C. mathematical ability
D. concept formation
A

Crystallized abilities are less affected by increasing age during adulthood than are fluid abilities.
c. CORRECT Mathematical ability, vocabulary, and general information are crystallized abilities and are less affected by normal aging than are the fluid abilities listed in answers a, b, and d.
(Psych Assess)

556
Q

Subtest scatter of three to five points on the WISC indicates:
A. pathology of an organic etiology.
B. borderline intellectual functioning, mental retardation, or a learning disability.
C. variability of strengths and weaknesses in cognitive abilities.
D. the effects of measurement error only.

A

The interpretation of Wechsler test performance in terms of subtest scatter is controversial because of the effects of measurement error (i.e., a certain amount of scatter can be expected simply as the result of measurement error).
c. CORRECT A common technique for interpreting subtest scores is to calculate the examinee’s mean verbal and performance subtest scores and determine how many subtests in the verbal and performance scales deviate from the mean. When doing so, discrepancies of three points or more are considered significant and are interpreted in terms of strengths and weaknesses rather than in terms of specific pathology.
d. Incorrect Although measurement error is a source of subtest scatter, discrepancies of three points or more are considered significant (i.e., to be indicative of more than the effects of measurement error only).
(Psych Assess)

557
Q
Administration of the \_\_\_\_\_\_ involves asking the examinee to make up stories about pictures that include one or more human figures in order to obtain information about unconscious aspects of the examinee’s personality.
A. Thematic Apperception Test
B. Myers-Briggs Type Indicator
C. Rorschach Inkblot Test
D. Bender-Gestalt Test
A

This question describes a projective personality test, which eliminates two of tests listed in the answers.
a. CORRECT The Thematic Apperception Test (TAT) is a projective personality test that consists of picture cards that depict one or more human figures. The examinee is asked to make up a story about each picture and his/her response are scored and interpreted using one of several scoring and interpretation systems.
b. Incorrect The Myers-Briggs Types Indicator is an objective personality test that assesses four personality dimensions derived from Jung’s personality type - i.e., extraversionfintroversion, sensate/intuitive, thinking/feeling, and judging/perceiving.
c. Incorrect The Rorschach is a projective personality test that presents the examinee with ten bilaterally symmetrical inkblots. The examinee is asked to describe what he/she sees in each inkblot.
d. Incorrect The Bender-Gestalt test is a measure of visual-motor integration that requires the examinee to copy geometric figures.
(Psych Assess)

558
Q

A person who received a Level 4 rating on the Rancho Los Amigos Scale of Cognitive Functioning:
A. is nonresponsive to visual or auditory stimuli and seems to be in a state of deep sleep.
B. is confused and incoherent, may exhibit bizarre behavior, and is unable to care for him/herself.
C. is alert and oriented and can remember and integrate remote and recent events but may have some impairment in judgment, planning, and abstract reasoning.
D. is functioning at an intellectual level that is superior for his/her age, education, and demographic background.

A

The Rancho Los Amigos Scale of Cognitive Functioning was developed as a method for monitoring recovery from head trauma and involves rating the individual in terms of eight levels of functioning.
a. Incorrect This describes behaviors characteristic of Level 1.
b. CORRECT The behaviors described in this answer are characteristic of Level 4.
c. Incorrect This answer describes Level 8 Functioning.
d. Incorrect This answer does not accurately describe any of the levels of Functioning assessed by the Rancho Los Amigos Scale.
(Psych Assess)

559
Q
The concordance rate for IQ for biological (non-twin) siblings who are reared together is:
A. .85
B. .67
C. .45
D. .22
A

For the licensing exam, you’ll want to be familiar with the IQ concordance rates listed in the Psychological Assessment chapter of the written study materials.
a. Incorrect Bouchard and Gue (1981) report a concordance rate of .85 for IQ For identical twins reared together.
b. Incorrect The concordance rate For identical twins reared apart is .67.
c. CORRECT Bouchard and Gue report a concordance rate of .45 for biological (non-twin) siblings reared together.
cl. Incorrect The concordance rate For biological parent and child living apart is .22.
(Psych Assess)

560
Q

When empirical criterion keying is used to develop a test, items chosen for inclusion are those that:
A. are consistent with the constructs identified by a specific theory or model.
B. distinguish between preselected groups or subgroups.
C. have been identified as representative of the content domain by subject matter experts.
D. accurately rank people from “low” to “high” on a predefined criterion.

A

The original MMPI clinical scales and the Occupational scales of the Strong Interest Inventory were developed on the basis of empirical criterion keying.
a. Incorrect This describes the theoretical method of test construction.
b. CORRECT When using empirical criterion keying to construct scales, items that distinguish between different criterion groups or between criterion groups and a control (general reference) group are included in each scale.
c. Incorrect This answer does not accurately describe empirical criterion keying
d. Incorrect This answer does not accurately describe empirical criterion keying
(Psych Assess)

561
Q
The developer of a new occupational interest test conducts a study to determine which test items accurately distinguish between people who are employed in different occupations. This test developer is using which of the following methods of scale construction?
A. empirical criterion keying
B. homogeneous keying
C. factor analysis 
D. multiple regression analysis
A

The test developer described in this question is trying to determine which items distinguish between specific criterion groups (i.e., people in different occupations).
a. CORRECT Empirical criterion keying involves choosing items for different scales on the basis of how well the items distinguish between different criterion groups. This method was used in the development of the Occupational scales of the Strong Interest Inventory.
b. Incorrect When using homogeneous keying, items are selected on the basis of the degree to which they measure the same domain.
c. Incorrect When factor analysis is used for scale construction, items that correlate highly with the same factor are included in the same scale.
d. Incorrect Multiple regression analysis is the appropriate multivariate technique when the goal is to use two or more predictors to predict or estimate an individual’s score on a criterion.
(Psych Assess)

562
Q
A psychologist is designing a study that will investigate the impact of age on intelligence. The results of the psychologist's study will most likely indicate that intelligence test scores begin to decline in early adulthood if she uses which of the following research designs?
A. cross-sequential
B. cross-sectional
C. longitudinal 
D. analogue
A

The research has confirmed that the method used to investigate the impact of age on IQ can bias the results of the study.

a. Incorrect See explanation for response b.
b. CORRECT Early cross-sectional studies on the relationship between age and IQ suggested that IQ scores begin to drop in late adolescence or early adulthood. Subsequent longitudinal and cross-sequential studies found, however, that this result was due more to methodology than to actual IQ score declines — i.e., the apparent age-related declines identified in cross-sectional studies were due to cohort (intergenerational) effects.(Psych Assess)

563
Q

Which of the following is the best conclusion that can be drawn from the research comparing actuarial and clinical approaches to test score interpretation?
A. The actuarial method is more accurate than the clinical method.
B. The clinical method is more accurate than the actuarial method.
C. The actuarial and clinical methods are equivalent in terms of accuracy.
D. A strategy that combines actuarial and clinical methods is substantially more accurate than is either method alone.

A

Actuarial (statistical) interpretations are based on empirically validated relationships between test results and target criteria, while clinical interpretations are based on the decision-maker’s intuition, experience etc.
a. CORRECT Although the research has not provided entirely consistent results, in general, the studies have shown that the actuarial method alone is more accurate than clinical judgment alone and about equally accurate as a combination of actuarial and clinical methods (e.g., Grove and Meehl, 1996).
(Psych Assess)

564
Q
Which of the following WAIS-III factor indices is most sensitive to (most adversely affected by) traumatic brain injury?
A. processing speed
B. working memory
C. perceptual organization
D. verbal comprehension
A

The research has confirmed that the WAIS-III is useful as an assessment tool for traumatic brain injury (TBI).
a. CORRECT The Technical Manual for the WAIS-III reports that, of the four factor indices, processing speed is most adversely affected by TBI, and this finding has been confirmed by several researchers.
b. Incorrect Although memory is affected by TBI, working memory tends to be less adversely affected than is processing speed.
(Psych Assess)

565
Q
An exarninee whose highest clinical scale scores on the MMPI-2 are on scales \_\_\_\_\_ is most likely to have a history of alcohol and drug abuse, antisocial behavior, and/or sex offenses.
A. 2 and 6
B. 4 and 9
C. 9 and 0
D. 2 and 1
A

Knowing the names of the MMPI-2’s clinical scales may have helped you identify the correct answer to this question:
a. Incorrect Highest scores on Scales 2 and 6 (Depression and Paranoia) suggest hopelessness, hostility, and paranoia.
b. CORRECT Scale 4 is the Psychopathic Deviate scale, and Scale 9 is the Hypomania scale. The 4-9, 9-4 code (highest scores on Scales 4 and 9) is characteristic of people with a history of alcohol and drug abuse, antisocial behaviors, andfor sex offenses.
c. Incorrect Highest scores on scales 9 and 0 (Hypomania and Social Introversion) suggest egocentrism and grandiosity.
d. Incorrect Highest scores on scales 2 and 1 (Depression and Hypochondriasis) suggest depression, worry, and pessimism.
(Psych Assess)

566
Q

To determine if a predictor is biased against members of a particular group, you would compare:
A. the means of the different groups on the predictor.
B. the means of the different groups on the criterion.
C. the regression lines for the different groups.
D. the Y-intercepts for the different groups.

A

A predictive bias is suggested when a predictor has different validity coefficients for different groups.
a. Incorrect Differences in mean scores on either the predictor or criterion may or may not be indicative of a predictive bias.
c. CORRECT When a test has different validity coefficients for different groups, it will also have different regression lines since the slope of the regression line is directly related to the magnitude of the validity coefficient.
d. Incorrect Different Y-intercepts do not necessarily indicate bias but may, instead, reflect real differences in criterion performance.
(Psych Assess)

567
Q
A criticism of the Scholastic Aptitude Test (SAT) is that it is a relatively poor predictor of the college grades of students who:
A. score in the lower third. 
B. score in the upper third.
C. score in the lower and upper thirds.
D. score in the mid-range.
A

You may have been able to make an educated guess when choosing the answer to this question if you’re unfamiliar with the SAT but are familiar with scatterplots: Often, in a scatterplot there is a slight
“bulge” in the middle of the distribution, indicating that the relationship between variables is not quite as strong in the mid-region of the scores as it is at the extremes.
d. CORRECT A shortcoming of the SAT is that it is less accurate in predicting the college GPA of examinees who obtain SAT scores in the mid-range (between 900 and 1200) than it is for predicting the GPA of examinees who obtain higher or lower scores.
(Psych Assess)

568
Q

The Peabody Picture Vocabulary Test, 3rd Edition (PPVT-III) was designed as a measure of intelligence for:
A. adolescents and adults with visual impairments.
B. children and adolescents with hearing impairments.
C. adolescents and adults whose first language is not English.
D. children, adolescents, and adults with orthopedic disabilities.

A

For the licensing exam, you’ll want to be familiar with the tests designed for individuals with physical disabilities that are described in the Psychological Assessment chapter of the study materials.
a. Incorrect The Ha ptic Intelligence Scale was designed for adolescents and adults with visual impairments.
b. Incorrect The Hiskey-Nebraska Test of Learning Aptitude is a measure of intelligence For children and adolescents with hearing impairments.
c. Incorrect The PPVT-III requires the examinee to understand standard English.
d. CORRECT The PPVT-III requires the examinee to point to (or otherwise indicate) the picture that best corresponds to the word verbally provided by the examiner. It was originally developed For examinees ages 2-112 to 85 who have orthopedic disabilities.
(Psych Assess)

569
Q
A score of 24 or higher on the Mini Mental State Exam (MMSE) suggests which of the following?
A. normal functioning
B. depression
C. delirium
D. dementia or psychosis
A

The MMSE consists of 11 items designed to assess six aspects of cognitive functioning in older adults: orientation, registration, attention and calculation, recall, language, and visual construction.
a. CORRECT Scores on the MMSE range from 0 to 30, with higher scores indicating better cognitive Functioning. A score above 23 suggests normal cognitive functioning.
(Psych Assess)

570
Q
Development of which of the following personality tests was based on Murray's 15 needs (e.g., achievement, autonomy, affiliation)?
A. 16 PF 
B. CPI
C. EPPS
D. NEO-PI-R
A

For the licensing exam, you’ll want to be familiar with the purpose and assumptions underlying the four tests listed in the answers to this question.
a. Incorrect The developers of the 16 PF (Sixteen Personality Factor Questionnaire) used factor analysis to categorize adjectives that describe personality traits.
b. Incorrect The CPI (California Psychological Inventory) was developed as a measure of normal personality traits for adolescents and adults. Items for each scale were chosen on the basis of their ability to distinguish between groups of people who differed with regard the trait measured by the scale.
c. CORRECT The developer of the EPPS (Edwards Personal Preference Schedule) used the 15 needs identified by Murray to generate items for the test.
d. Incorrect The NEO-PI-R (NEO Personality Inventory — Revised) assesses the big five personality traits.
(Psych Assess)

571
Q

The Wide-Range Achievement Test is:
A. a brief achievement test that measures reading comprehension, spelling, and arithmetic computation.
B. a brief achievement test that measures the basic content domains (e.g English, science, math) taught in elementary and junior high school.
C. a measure of academic achievement that is used to predict high school grades.
D. a measure of academic achievement that is used to predict undergraduate grades.

A

The Wide-Range Achievement Test (WRAT) is a brief achievement test that can be administered to children, adolescents, and adults. The most recent version is the WRAT-4, which was published in 2006.
a. CORRECT The WRAT assesses achievement in reading, spelling, and arithmetic only.
(Psych Assess)

572
Q

Raven’s Progressive Matrices is a:
A. measure of simultaneous and sequential processing
B. measure of inductive and deductive reasoning.
C. nonverbal measure of general intelligence.
D. culture-fair measure of academic achievement.

A

Raven’s Progressive Matrices requires the examinee to solve problems involving abstract figures and designs by indicating which of several alternatives complete a given matrix.
c. CORRECT Raven’s Progressive Matrices is a nonverbal measure of general intelligence (g). Although it is considered a culture-Fair test, it does not evaluate academic achievement.
(Psych Assess)

573
Q

You would conduct a functional analysis in order to:
A. identify an examinee’s strengths and weaknesses with regard to a specific content domain.
B. identify the antecedents and consequences that control or maintain a specific behavior.
C. determine the expectations of others (e.g., parents or teachers) regarding a child’s performance.
D. determine the knowledge and skills required to perform a job or task successfully.

A

As its name implies, a functional analysis (also known as functional assessment) is conducted to identify the functions of a behavior.
b. CORRECT A functional analysis is conducted to identify the environmental factors (antecedents and consequences) that control the target behavior.
(Psych Assess)

574
Q

Recent research suggests that, with regard to metamemory, older adults are:
A. more likely than younger adults to underestimate their actual memory deficits.
B. more likely than younger adults to overestimate their actual memory deficits.
C. just as able as younger adults to accurately estimate their actual memory deficits.
D. better than younger adults at estimating their actual memory deficits.

A

The research on this topic is limited, but it appears that while older adults do, in fact, experience memory problems, they may overestimate the extent of those problems.
b. CORRECT Metamemory refers to the ability to monitor one’s own memory processes. There is evidence that, compared to younger adults, older adults tend to view their memory loss as worse than it actually is.
(Psych Assess)

575
Q
On the WAIS-III, a Verbal IQ that is substantially higher than the Performance IQ is rnost suggestive of which of the following?
A. a learning disability
B. low socioeconomic status
C. delinquency
D. depression
A

On the WAIS-III, a Verbal-Performance discrepancy of at least 12 points is statistically significant, but Kaufman and Lichtenberger (1999) suggest that a discrepancy be considered clinically significant only when it is at least 17 points.
d. CORRECT This is the only answer given that describes a condition that is associated with a higher Verbal IQ score. The conditions listed in answers a, b, and c are associated with a higher Performance IQ score.
(Psych Assess)

576
Q

During the oral administration of a standardized scholastic achievement test to an eight-year old child, the child tells the administrator, a school psychologist, that she doesn’t understand what the psychologist has asked. When the child responds in this way to a question, the psychologist should:
A. rephrase the question in terms the child can understand.
B. repeat the question but speak more slowly.
C. say “do the best you can.”
D. go on to the next question.

A

When administering a standardized test, valid results are obtained only when the standardized administration and scoring procedures are used.
a. Incorrect Changing the wording of test questions might invalidate the test and, therefore, make the results uninterpretable. Consequently, it would be inappropriate to rephrase the test questions.
b. CORRECT Of the answers given, this is the best one. In most testing situations, it would be acceptable to repeat the question.
c. Incorrect Although this might be appropriate in some situations (e.g., when a written test is being administered and an examinee tells the examiner that he/she doesn’t understand a question), answer b is a better choice for the situation described in this question.
d. Incorrect This might be appropriate in some situations (e.g., when the examiner has already repeated a question several times), but answer b is a better choice.
(Psych Assess)

577
Q

Some experts argue that achievement tests can be used as substitutes for aptitude tests because:
A. “achievement” and “aptitude” are essentially synonymous.
B. both types of tests are measures of past learning in controlled situations.
C. future accomplishments are largely the result of innate capacities.
D. past accomplishments are a good indicator of what can be expected in the future.

A

A distinction is often made between achievement and aptitude tests. Strictly speaking, achievement tests assess the extent of past learning in classrooms or other controlled settings, while aptitude tests focus on future behavior, in particular the capacity to learn.
d. CORRECT Advocates of the position described in this question contend that past learning is due to a substantial degree to aptitude and, therefore, view achievement test performance as an indicator of aptitude (i.e., view past performance as a good predictor of future learning). Although not all experts agree with this position, most do agree that there is not a clear distinction between the two and that achievement is affected by aptitude and vice versa.
(Psych Assess)

578
Q
The Bender-Gestalt II was designed to be a measure of:
A. fine motor skills.
B. visual-motor integration.
C. cognitive flexibility.
D. suicide risk.
A

The Bender-Gestalt II is a brief assessment of visual-motor integration.
b. CORRECT The Bender-Gestalt II is a drawing test that requires the examinee to copy several geometric Figures. It is used to assess visual-motor integration and to assist in the assessment of perceptual disorders, organic brain dysfunction, and psychopathology.
(Psych Assess)

579
Q
The Wonclerlic Personnel Test is a:
A. measure of psychomotor skills.
B. measure of clerical skills.
C.brief measure of cognitive ability
D. brief interest inventory.
A

The Wonderlic Personnel Test is a 12-minute, 50-item exam.
c. CORRECT The Wonderlic Personnel Test assesses cognitive ability (general intelligence) and is used in organizations as a selection test.
(Psych Assess)

580
Q

Which of the following describes a requirement of Public Law 94-142?
A. Children and adolescents with disabilities must be provided with education in the least restrictive environment.
B. Parents have the right to request a school to amend their minor child’s school records if they believe the record is inaccurate or misleading.
C. Employers must make reasonable accommodations when testing examinees with disabilities.
D. Tests used to make selection and other employment decisions must have evidence of adequate criterion-related validity.

A

Public Law 94-142 (the Education of all Handicapped Children Act) was passed in 1975, has been amended several times, and is now known as the Individuals with Disabilities Education Act (IDEA).
a. CORRECT The purpose of PL 94-142 was to improve the educational opportunities For children and adolescents with disabilities, and this answer accurately describes one of its provisions.
b. Incorrect This answer describes one of the provisions of the Family Educational Rights and Privacy Act (FERPA).
c. Incorrect This answer describes one of the provisions of the Americans with Disabilities Act.
d. Incorrect This answer does not describe a provision of PL 94-142.
(Psych Assess)

581
Q

Which of the following is not true about the impact of increasing age in adulthood on cognitive ability?
A. Performance IQ is more negatively impacted than verbal IQ.
B. Free recall memory is more negatively impacted than recognition memory.
C. Fluid intelligence is more negatively impacted than crystallized intelligence.
D. Primary memory is more negatively impacted than secondary memory

A

Studies suggest that there is considerable heterogeneity in the impact of aging on cognitive abilities. However, some general conclusions can be drawn.
a. Incorrect This is true.
b. Incorrect This has also been Found to be true.
c. Incorrect This is a consistent finding of the research.
cl. CORRECT This is the opposite of what is true. Recent long-term (secondary) memory is more negatively impacted by increasing age than is primary memory (memory span).
(Psych Assess)

582
Q

A psychologist would use the Mini Mental State Exam (MMSE) to:
A. determine if a child has a learning disability.
B. determine if an adult has a psychotic disorder.
C. evaluate the quality of life of an adult with a physical disability.
D. evaluate the cognitive functioning of an older adult.

A

The MMSE was developed as a measure of cognitive functioning for older adults and is also used as a screening tool for dementia.
d. CORRECT The MMSE assesses six aspects of cognitive functioning: orientation, registration, attention and calculation, recall, language, and visual construction.(Psych Assess)

583
Q

In the context of cognitive ability tests, “testing-the-limits” refers to:
A. readministering the test to an examinee using standardized procedures after a period of formal instruction or training.
B. readministering the test (or portions of it) to an examinee while modifying the standardized procedures.
C. determining the accuracy of test results for predicting academic or job performance.
D. determining the accuracy of test results for a highly heterogeneous group of examinees.

A

Testing-the-limits is done in order to obtain additional information about an examinee and involves deviating from standardized testing procedures.
b. CORRECT Testing-the-limits is done to obtain additional qualitative information about an examinee (e.g., how the examinee derived his/her responses or how the examinee responds when given cues or additional time to answer items). Testing-the-limits is done after the test has been administered using standardized procedures.
(Psych Assess)

584
Q

The Slosson Intelligence Test — Revised 3rd Edition for Children and Adults was developed to be a:
A. screening test for crystallized intelligence.
B. screening test for analytical, creative, and practical intellectual abilities.
C. measure of receptive language.
D. culture-fair measure of aptitude.

A

The Slosson tests include the Slosson Intelligence Test Primary and the Slosson Intelligence Test — Revised 3rd Edition for Children and Adults.
a. CORRECT The SIT-R3-1 was designed as a quick screening test of crystallized (verbal) intelligence For individuals aged 4 through 65.
(Psych Assess)

585
Q

Development of the ______ was based on the Cattell-Horn-Carroll theory of cognitive abilities.
A. Kaufman Assessment Battery for Children, Second Edition
B. Peabody Picture Vocabulary Test, Third Edition
C. Naglierli and Das’s Cognitive Assessment System
D. Sternberg’s Multidimensional Abilities Test

A

Of the cognitive ability tests listed, only one was developed on the basis of the Cattell-Horn-Carroll theory of cognitive abilities.
a. CORRECT The Kaufman Assessment Battery For Children, Second Edition (KABC-II) was based on two theories — Luria’s neuropsychological model and the Cattell-Horn-Carroll theory of cognitive abilities.
(Psych Assess)

586
Q

When an interest test yields ipsative scores, this means that scores indicate:
A. the absolute strengths of an examinee”s interests.
B. the relative strengths of an exarninee”s interests.
C. the occupational group that an exarninee most closely resembles.
D. the consistency of an exarninee”s interests.

A

Ipsative scores permit intraindividual comparisons but not interindividual comparisons.
b. CORRECT When an interest test yields ipsative scores, an individual’s scores indicate his/her relative standing in terms of the interests measured by the test (i.e., they indicate which interest is the strongest for the examinee, which is the second strongest, etc.).
(Psych Assess)

587
Q
The Wisconsin Card Sorting Test is most useful for assessing \_\_\_\_\_ lobe functioning.
A. parietal
B. temporal
C. occipital
D. frontal
A

The Wisconsin Card Sorting Test (WCST) cletects perseverative thinking and deficits in abstract reasoning.
d. CORRECT The WCST is considered to be a measure of executive cognitive abilities and is used by neuropsychologists to assess the Functioning of the Frontal lobes.
(Psych Assess)

588
Q

As defined by Horn and Cattell (1966) intellectual ability consists of:
A. three general components — analytical, creative, and practical.
B. two general components — fluid and crystallized.
C. a “g” (general) factor plus specific factors unique to the task.
D. eight distinct abilities (e.g., linguistic, spatial, interpersonal).

A

For the licensing exam, you’ll want to be familiar with the major theories of intelligence described in the answers to this question.
a. Incorrect Sternberg describes intelligence as consisting of analytical, creative, and practical components.
b. CORRECT Horn and Cattell distinguish between two general intelligences: Fluid intelligence enables the individual to solve novel problems and perceive relations and similarities and does not depend on specific instruction. In contrast crystallized intelligence consists of acquired knowledge and skills and is atfected by educational and cultural experiences.
c. Incorrect This answer describes Spearman’s theory of intelligence.
d. Incorrect This answer describes Gardner’s theory of intelligence.
(Psych Assess)

589
Q
In general, interest inventories are least useful for predicting:
A. job choice.
B. job satisfaction.
C. job persistence.
D. job performance.
A

Studies suggest that job performance is more closely related to ability than to interests.
d. CORRECT Interest test scores are not good predictors of job perforrnance since, as noted above, performance is more dependent on ability than interests Interest test scores are, however, good predictors of job choice, satisfaction, and persistence.
(Psych Assess)

590
Q

In the case of Larry P. v. Riles, the court ruled that IQ tests:
A. are culturally fair.
B. are racially and culturally biased.
C. are valid predictors ofjob performance for members of minority groups when used in conjunction with other measures.
D. are valid measures of intelligence for members of minority groups when administered by an examiner of the same race/ethnicity.

A

In the Larry P. case, a suit was brought against the California State Department of Education and the San Francisco School Board on the ground that the use of standard IQ tests as a placement technique resulted in an over-representation of African American children in special education classes.
b. CORRECT Judge Peckham’s ruling in the case of Larry P. reflected his conclusion that IQ tests are racially and culturally biased and, therefore, should not be used as placement tools for minority children.
(Psych Assess)

591
Q

Scores on the MMPI-2’s clinical scales are most commonly interpreted:
A. by considering each clinical scale score separately.
B. in terms of a profile analysis.
C. by referring to expectancy tables.
D. in terms of percentile bancls.

A

The MMPI-2 provides scores on 10 clinical scales.
a. Incorrect Many test items are included in more than one clinical scale. Thus, the validity of considering each clinical scale without reference to the others is questionable.
b. CORRECT Since interpretation of clinical scales in isolation is dubious, most experts agree that the MMPI clinical scales are most effectively interpreted in terms of score patterns; i.e., in terms of profile (pattern) analysis. A number of available coding systems facilitate this type of interpretation.
(Psych Assess)

592
Q
Examinees with Alzheimer's dementia would be likely to obtain the highest score on which of the following WAIS-III factor indexes?
A. Verbal Comprehension
B. Working Memory
C. Perceptual Organization
D. Processing Speed
A

Individuals with Alzheimer’s dementia tend to obtain higher Verbal than Performance IQ scores.
a. CORRECT The WAIS-III Technical Manual reports the highest score For individuals with Alzheimer’s dementia For the Verbal Comprehension index, Followed by the Working Memory index, the Perceptual Organization index, and the Processing Speed index.
(Psych Assess)

593
Q
Which of the following methods was used to identify the “big five” personality traits?
A. multidimensional scaling
B. homogeneous keying
C. actuarial approach 
D. lexical hypothesis
A

Identification of the big five personality traits was based on the lexical hypothesis, which predicts that all socially-relevant traits have been incorporated into language (are included in the dictionary).
d. CORRECT The big Five personality traits — extroversion/introversion, conscientiousness, openness to experience, emotional stability, and agreeableness — were originally identified through Factor analyses of personality characteristics that are included in the dictionary.
(Psych Assess)

594
Q
Which of the following MMPI-2 scales is considered to be a “suppressor” variable?
A. Cannot Say (?) Scale
B. L (Lie) Scale
C. F (Frequency) Scale
D. K (Correction) Scale
A

A suppressor variable affects scores or status on another variable but is irrelevant to that variable.
d. CORRECT The K Scale is considered a suppressor variable because it correlates with defensiveness and other characteristics (education level and SES) that are irrelevant to what is being measured by several the MMPI-2’s clinical scales but that may affect scores on those scales.. Consequently, an exarninee’s score on the K scale is used to correct his/her scores on certain clinical scales and thereby increase their validity.
(Psych Assess)

595
Q

Which of the following best describes the Flynn effect (Flynn, 1998)?
A. IQ scores gradually decline over the lifespan.
B. Performance IQ declines more than Verbal IQ over the lifespan.
C. IQ test scores have gradually increased in developed nations in recent decades.
D. IQ test scores have gradually decreased in developed nations in recent decades.

A

Research by Flynn (1998) and others has shown that IQ test scores have generally increased from one generation to the next, especially in developed countries.
c. CORRECT Flynn proposes that increases in IQ test scores from one generation to the next represent increases in abstract problem solving as the result of environmental Factors (e.g., improved education).
(Psych Assess)

596
Q

In terms of age and reading level, the MMPI-A is appropriate for:
A. children and adolescents aged 9-12 who can read at a 3rd grade level or above.
B. children and adolescents aged 10-16 who can read at a 6th grade level or above.
C. adolescents aged 14-18 who can read at a 6th grade level or above.
D. adults who can read at the 8th grade level or above.

A

The MMPI-A is a downward extension of the MMPI for adolescents.
c. CORRECT The MMPI-A was designed to assist in problem identification, diagnosis, and treatment planning for adolescents aged 14 through 18. One requirement for the test is that the examinee can read at the 6th grade level or above.
(Psych Assess)

597
Q
On the Rorschach Inkblot Test, confabulation is rnost suggestive of which of the following?
A. giftedness
B. antisocial personality disorder
C. an attempt to “fake bad” 
D. organic brain dysfunction
A

Confabulation on the Rorschach refers to the tendency to overgeneralize from a part of an inkblot to the entire inkblot.
d. CORRECT Confabulation is associated with cognitive impairment, especially impairment related to organic brain dysfunction.
(Psych Assess)

598
Q
A "speed" test can be viewed as the opposite of a  \_\_\_\_\_ test
A. mastery
B. performance
C. power
D. context
A

A speed (or speeded) test contains a large number of questions that are so easy that most examinees would be able to answer them correctly if given sufficient time. However, a strict time limit is imposed so that an examinee’s test score reflects his/her speed of responding.
c. CORRECT Power tests can be viewed as the opposite of speed tests. Power tests contain items that are relatively difficult and that could not all be answered correctly by most or all examinees regardless of how much time they are given to complete the test.
(Psych Assess)

599
Q

With regard to performance on tasks assessing verbal and nonverbal intelligence, the research has found that, as people get older, they ordinarily exhibit:
A. no or minimal deficits on verbal tasks but declines on nonverbal tasks beginning in the 4th or 5th decade.
B. no or minimal deficits on nonverbal tasks but declines on verbal tasks beginning in the 4th or 5th decade.
C. declines on both verbal and nonverbal tasks beginning in the 4th or 5th decade.
D. improvements in performance on both verbal and nonverbal tasks until the 6th or 7th decade when declines begin.

A

Overall, cognitive ability tasks assessing accumulated experience and knowledge (verbal tasks) are less affected by increasing age than are tasks requiring rapid processing or the use of novel information.
a. CORRECT verbal tasks show little or no age-related cognitive decline throughout the lifespan, while many nonverbal (performance) tasks begin to show some decline beginning in the 4th or 5th decade.
(Psych Assess)

600
Q
Of the “big five” personality traits, \_\_\_\_\_has most consistently been linked to job performance across a variety of jobs.
A. emotional stability
B. agreeableness
C. conscientiousness
D. openness to experience
A

The big five personality traits are extraversion/introversion, emotional stability, agreeableness, conscientiousness, and openness to experience.
c. CORRECT This was the conclusion reached, for example, by M. R. Barrick and M. K. Mount, based on their meta-analysis of the research (The big Five personality dimensions and job performance: A meta-a nalysis, Personnel Psychology, 1991, 44, 1-25).
(Psych Assess)

601
Q
The \_\_\_\_\_ provides information on the four cognitive abilities identified by Luria (1980) — i.e., planning, attention, simultaneous Processing, and sequential processing.
A. Slosson Intelligence Test Primary
B. Cognitive Assessment System
C. Kuhlman-Anderson Test
D. Woodcock Johnson III
A

For the licensing exam, you’ll want to be familiar with the purpose of the tests listed in the answers to this question.
a. Incorrect The Slosson Intelligence Test Primary is a screening test of intelligence for children aged 2 through 7 years, 11 months. It provides Verbal and Performance Scores and a Total Standard Score.
b. CORRECT The Cognitive Assessment System assesses basic cognitive processes that are central to learning. It is based on the PASS model of intelligence that distinguishes between the four cognitive functions identified by Luria (1980) — planning, attention, simultaneous processing, and sequential processing.
c. Incorrect The Kuhlman-Anderson Test is a group intelligence test for children in grades K through 12.
d. Incorrect The Woodcock Johnson III Tests of Cognitive Ability assess general and specific cognitive abilities and are based on the Cattell-Horn-Carroll model of intelligence.
(Psych Assess)

602
Q

Raymond R. and his wife have not been getting along for some time, and Raymond has recently started thinking about seeking a divorce. The problem with getting a divorce is that it will require Raymond to either divide the business he and his wife have or to sell his share of the business to her, neither of which appeals to him. The problem with staying with his wife is that Raymond will have to continue putting up with her nagging and his feeling that he has no “life of his own.” Assuming that Raymond is experiencing an “avoidance-avoidance conflict,” it is most likely that he will
do which of the following in the near future?
A. choose one of his two options and feel confident that it was the correct one
B. vacillate between the two options, first choosing one and then the other
C. antagonize his wife in order to force her to make the decision to get a divorce
D. decide that “no one is really satisfied anyway” and, consequently, stay with his wife

A

Dollard and Miller (1950) distinguished between three types of conflict — avoidance-avoidance, approach-approach, and approach-avoidance.
a. Incorrect This is the more likely outcome in an approach-approach conflict.
b. CORRECT An avoidance-avoidance conflict produces vacillation between the two courses of action. Even if you’re not familiar with Dollard and Miller’s work, you’ve probably had a personal experience that could help you understand why this is the correct answer.
c. Incorrect This is a possibility, of course, but is not consistent with what Dollard and Miller predicted about the outcome of an avoidance-avoidance conflict.
d. Incorrect This is also not consistent with the predictions of Dollard and Miller’s theory.
(Social Psychology)

603
Q
The three components of attitudes are:
A. affect, cognition, and behavior.
B. aptitude, affect, and cognition.
C. evaluation, experience, and application
D. knowledge, evaluation, and action.
A

Attitudes can be defined as relatively stable evaluative responses to an entity or situation.
a. CORRECT Most researchers interested in attitudes describe them as consisting of three components: affective (evaluative), cognitive, and behavioral (conative).(Social Psychology)

604
Q
According to Baumeister, Catanese, and Wallace (2002), a man may feel that his personal freedom is being restricted when a woman refuses his sexual advances and, as a result, become aggressive toward the woman. In other words, Baumeister and his colleagues suggest that, in some circumstances, sexual aggression toward women by men may be attributable to which of the following?
A. sublimation
B. reactance 
C. coercive power
D. behavioral catharsis
A

This question is referring to a study by R. F. Baumeister, K. R. Catanese, and H. M. Wallace on the causes of sexual coercion (Conquest by force: A narcissistic reactance theory of rape and sexual coercion, Review of Genera.-‘ Psychology, 2002, 6, 92-135). However, you may have been able to identify the correct answer even if you’re
not familiar with this study as long as you’re familiar with the concept being asked about.
b. CORRECT The phrase “feel that his personal freedom is being restricted” should have helped you identify the correct response to this question. Reactance (also known as psychological reactance) occurs when a person acts in a way that is opposite of what is being requested because the person feels that hisfher freedom-of-choice is being restricted. Based on their research, Baumeister and his colleagues conclude that the attempts of sexually coercive men to force women to engage in sexual behavior is the result of a combination of reactance and narcissism.
(Social Psychology)

605
Q
During a family therapy session, the therapist tells the 8-year old son that he should keep annoying his sister, even though it makes her very mad. The boy says, "I don't have to ifI don't want to." The boy’s response to the therapist's request is best described as a manifestation of which of the following?
A. paradox
B. double-bind
C. source derogation
D.  psychological reactance
A

Note that this question is asking about the boy’s behavior, not the therapist’s.
a. Incorrect The therapist is using paradox, but this question is asking about the boy’s behavior.
b. Incorrect Double-bind communication presents a “mixed message” (e.g., contradictory verbal and nonverbal messages) and is not relevant to the situation described in this question.
c. Incorrect Source derogation refers to defending oneself against persuasion by criticizing the source of the message.
d. CORRECT Psychological reactance occurs when we do the opposite of what is requested or desired by another person because we feel our freedom-of-choice is being restricted. In family therapy, paradox is sometimes used to elicit resistance (psychological reactance) in order to promote change.
(Social Psychology)

606
Q
The tendency for interrupted and unfinished tasks to be remembered better than completed tasks are when the tasks are performed under non-stressful conditions is referred to as the:
A. Zeigarnik effect. 
B. Hawthorne effect.
C. Rosenthal effect.
D. von Restorff effect.
A

This question describes the Zeigarnik effect. Note that the opposite may occur when tasks are performed under stressful (versus non-stressful) conditions.
a. CORRECT The Zeigarnik effect is associated with Lewin’s field theory, which views behavior as a function of the interaction between the person and the environment.
b. Incorrect The Hawthorne effect refers to the tendency of research participants to behave differently simply because they are participating in a research study.
c. Incorrect The Rosenthal effect is also known as the “self-fulfilling prophecy” effect and is not relevant to the phenomenon described in this question.
d. Incorrect The von Restorff effect refers to the tendency to pay attention to or recall stimuli that are distinct from other stimuli (e.g., a noun in a list of verbs).
(Social Psychology)

607
Q
A co-worker says, “It never fails. Every time I plan to go hiking with my children, it rains.” Assuming that your co-worker's claim is not really true, her statement is best described as a manifestation of which of the following?
A. false consensus bias
B. hedonic relevance
C. base rate fallacy
D.  illusory correlation
A

In this situation, your co-worker is overestirnating the relationship between two events (hiking and raining).
a. Incorrect The false consensus bias is the tendency to overestimate the extent to which others agree with us.
b. Incorrect Hedonic relevance is a potential source of bias when making attributions about the behavior of another person and refers to the extent to which the other person’s behavior has positive or negative consequences for us.
c. Incorrect The base rate fallacy refers to the tendency to consider statistical probability rates as false and, as a result, to rely on irrelevant information rather than statistical information when making a judgment.
d. CORRECT The tendency to overestimate a relationship between two variables is referred to as illusory correlation. Illusory correlation is most likely to occur when the two events or other variables are distinctive (e.g., occur relatively infrequently).
(Social Psychology)

608
Q

Aronson and Linder’s (1965) gain-loss theory predicts that we will like a person most when his/her evaluations of us are:
A. initially positive and remain positive.
B. initially negative but become positive.
C. initially positive but become negative.
D. initially and subsequently neutral.

A

Gain-loss theory (Aronson and Linder, 1965) is a theory of personal attraction.
b. CORRECT Gain-loss theory predicts that an evaluation sequence involving a change from a negative to a positive evaluation will result in greater liking For the evaluator than will a consistent positive evaluation sequence and, conversely, that an evaluation sequence involving a change from a positive to a negative evaluation will result in greater dislike for the evaluator than will a consistent negative evaluation sequence.(Social Psychology)

609
Q

Which of the following strategies would probably be least effective for reducing aggressiveness in children?
A. exposure to a nonaggressive model
B. opportunities for catharsis
C. social skills training
D. a threat of retaliation from a high-status person

A

Note that this question is asking for the most ineffective method for reducing aggression.
a. Incorrect The research has shown that exposure to models can increase both positive and negative behaviors.
b. CORRECT Catharsis has not been found to be an effective way to reduce aggressive behavior. In fact, there is evidence that it may actually increase aggression.
c. Incorrect In many cases, aggressiveness is due to a lack of alternative behaviors. Consequently, social skills training is often effective because its goal is to replace undesirable behaviors (e.g., aggression) with more desirable ones.
d. Incorrect The research has shown that a threat of retaliation reduces aggression, at least in certain circumstances (e.g., when the threat comes from a person with high status or power and is not accompanied by provocation).
(Social Psychology)

610
Q
In general, a communicator of a persuasive message will produce the greatest amount of attitude change in a listener when the communicator is \_\_\_\_\_\_ in credibility and the discrepancy between the listener's initial position and the position advocated by the communicator is \_\_\_\_\_.
A.  high; moderate 
B. high; small
C. low; large
D. low; moderate
A

The ability of a communicator to change a listener’s attitude depends on several factors including the communicator’s credibility and the amount of discrepancy between the listener’s initial attitude and the attitude advocated by the communicator.
a. CORRECT The optimal combination of communicator credibility and communication discrepancy For attitude change is a highly credible communicator and a moderate level of discrepancy. With regard to the discrepancy, a small discrepancy will not require much change in attitude on the part of the listener, while a large discrepancy is likely to produce no change.
(Social Psychology)

611
Q

Self-verification theory predicts that a person with low self-esteem will prefer to receive evaluations from others that:
A. confirm his/her own negative self-evaluations.
B. refute his/her own negative self-evaluations.
C. avoid addressing his/her self-evaluations.
D. are clearly non-evaluative.

A

Swann, Pelham, and Krull’s (1989) self-verification theory predicts that people prefer confirmation of their self-concept.
a. CORRECT Self-verification theory predicts that a person with low self-esteem will prefer evaluations from others that confirm his/her own negative self-evaluations.(Social Psychology)

612
Q
According to the learned helplessness model, children who consistently have extreme negative emotional reactions to failure in school are most likely attributing their academic problems to:
A. lack of ability.
B. lack of effort.
C. bad luck.
D. task difficulty.
A

The negative reactions of the children described in this question are characteristic of learned helplessness.
a. CORRECT The learned helplessness model predicts that, when people view their Failures as the result of global, stable, and internal causes (e.g., lack of ability), they are likely to respond to Failure with depression or other strong negative emotion.
(Social Psychology)

613
Q
As a supervisor, Jason J. always tries to modify his communication and managerial style so that they “match” the style and characteristics of the particular employee he is interacting with. Based on this information, you can conclude that Jason:
A. is high in self-monitoring.
B. is high in initiating structure.
C. has low self-efficacy beliefs.
D. has an internal locus of control.
A

The individual described in this question is adjusting his behavior to fit the behavior of each employee that he supervises.
a. CORRECT High self-monitors are concerned about the impression they make and, as a result, pay close attention to the behaviors of other people so that they can adjust their own behavior to fit the expectations, style, and other characteristics of those people.
b. Incorrect Initiating structure refers to the degree to which a leader is goal (task) oriented and is not relevant to the situation described in this question.
(Social Psychology)

614
Q
The tendency to take credit for our successes but to blame other people or external circumstances for our failures is referred as the:
A. fundamental attribution bias.
B. confirmation bias.
C. self-serving bias.
D. optimism bias.
A

The answers to this question are all biases that affect our attributions or other aspect of our cognitive processing.
a. Incorrect The fundamental attribution bias is the tendency to attribute another person’s behavior to dispositional factors.
b. Incorrect The confirmation bias involves focusing on and seeking out information that confirms our preconceptions.
c. CORRECT The self-serving bias applies to the attributions we make for our own behaviors. It refers to the tendency to take credit for our successes (i.e., make dispositional attributions) but to blame the circumstances or other people for our failures (i.e., make situational attributions).
d. Incorrect As its name suggests, the optimism bias refers to a tendency to be overly optimistic about the outcomes of our behaviors.
(Social Psychology)

615
Q

Solomon Asch (1958) found that a participant’s conformity to group norms was substantially reduced when:
A. the stimulus (task) was unambiguous.
B. only one of the group members (confederates) deviated from the group norm.
C. the participant was not directly pressured (requested) to conform to the group norm.
D. the discrepancy between the group’s judgment and the participant’s judgment increased overtime.

A

In Asch’s (1958) studies, group members (the participants and confederates) were shown a vertical line and asked to indicate which of three other lines it matched in terms of length. Results indicated that when the confederates offered their estimates first, the participant conformed to the confederates’ estimates about one-third of the time.
a. Incorrect Asch’s studies used an unambiguous stimulus (a vertical line), and the lack of ambiguity did not eliminate conformity to the group norm.
b. CORRECT The participant was more likely to conform when the confederates were unanimous in theirjudgment of the line length than when even one confederate deviated from the group norm.
c. Incorrect Participants in Asch’s study were not directly pressured (required) to conform to the group norm.
d. Incorrect An increase in the discrepancy over time actually increased the likelihood that the participant would conform to the group norm.
(Social Psychology)

616
Q

With regard to attitude change, inoculation is useful for:
A. increasing a person’s resistance to persuasion.
B. reducing a communicator”s anxiety prior to delivering a message to a hostile audience.
C. distracting a person from a communicator”s message.
D. ensuring that attitude change represents internalization rather than mere compliance.

A

The concept of inoculation (McGuire, 1969) was derived from the medical model and is based on the assumption that a person will be better able to resist a persuasive communication when he/she has been “inoculated” against it. Inoculation involves providing weak arguments against a position and counterarguments refuting those arguments.
a. CORRECT Inoculation has been shown to be an effective method For increasing an individual’s resistance to persuasion. McGuire Found, For example, that it is more effective than providing the individual only with arguments that support his/her initial position.
b. Incorrect This is not the goal of inoculation.
c. Incorrect Distraction is associated with increased persuasiveness when a listener is initially opposed to a message and decreased persuasiveness when a listener is initially in Favor of a message. Inoculation does not involve distraction
d. Incorrect This is not a goal of inoculation.
(Social Psychology)

617
Q
Sherif (1935) used which of the following to investigate conformity to group norms?
A. visual cliff
B. jigsaw classroom 
C. door-in-the-face technique
D. autokinetic effect
A

Sherif’s (1935) study is a classic in social psychology, and you’ll want to be familiar with it for the licensing exam.
d. CORRECT The autokinetic eFl’ect is a perceptual phenomenon in which a stationary point of light appears to move in a darkened room. Sherif found that, in certain circumstances, research participants conformed to the judgments of confederates regarding how far the light had moved (even though the light was actually stationary).(Social Psychology)

618
Q
Cognitive dissonance theory is most useful for understanding
A. interpersonal attraction.
B. minority influence.
C. self attributions.
D. attitude change
A

According to cognitive dissonance theory, inconsistencies in cognitions produce tension (dissonance), which motivates the individual to modify one of his/her cognitions.
d. CORRECT Cognitive dissonance theory is useful for understanding the circumstances in which a person changes his/her attitudes or beliefs — i.e., it predicts that attitude change is, in some cases, an attempt to alleviate dissonance.
(Social Psychology)

619
Q

Which of the following best illustrates deindividuation?
A. A usually quiet, reserved person acts uncharacteristically violent in a crowd because he is able to do so anonymously.
B. A victim of a crime is more likely to receive assistance when the crime occurs in a rural area rather than in a city.
C. In a group decision-making situation, there is a tendency for critical-thinking to be suspended when group cohesiveness is very strong.
D. A group member reduces his/her effort when he/she thinks other members are not exerting maximum effort.

A

Deindividuation occurs in conditions that foster a loss of personal identity.
a. CORRECT This is the kind of behavior that was originally linked with deindividuation — i.e., the research Found that people who are normally non-aggressive may act in aggressive ways in crowds as the result of their ability to act anonymously.
b. Incorrect This describes bystander apathy.
c. Incorrect This sounds like groupthink.
d. Incorrect This is referred to as the sucker effect.
(Social Psychology)

620
Q
Participants in a research study are injected with epinephrine which produces rnild arousal. One-half of the participants are told to expect arousal while the other half are told that the injection will have no physiological side effects. Each participant is then placed in a waiting room with a confederate who has been instructed to act in an angry manner. Subsequently, participants who were told to expect arousal from the epinephrine report no change in their emotional state, while those who were told to expect no side effects report feeling angry. Results of this study provide evidence for which of the
following?
A. attribution theory
B. dissonance theory
C. self-serving bias
D. self-perception theory
A

In the study described in this question, participants in one group have an explanation for their arousal (the epinephrine), while participants in the other group do not.
d. CORRECT Results of the study are consistent with self-perception theory, which predicts that, in ambiguous situations, an indiviclual’s self-attributions are based on observations of cues in the external environment (e.g., the behavior of others).
(Social Psychology)

621
Q

The elaboration likelihood model predicts that the recipient of a persuasive message is more likely to use the peripheral route of information processing when:
A. the person delivering the message is a well-liked and trusted celebrity.
B. the recipient is in a neutral or slightly negative mood.
C. the message is considered important or personally relevant by the recipient.
D. the recipient has a high need for cognition.

A

The elaboration likelihood model (Petty and Cacioppo, 1980) distinguishes between central and peripheral processing routes. The central route involves active, effortful cognitive processing (cognitive elaboration), while the peripheral route involves reliance on simple decision-making rules.
a. CORRECT A listener is more likely to use the peripheral route (i.e., rely on peripheral cues} when the person delivering the message is attractive or well-liked,
b. Incorrect A recipient who is in a neutral or slightly negative mood is more likely to use the central route For processing a communication.
c. Incorrect A person is more likely to use the central route when the message is important or personally relevant.
d. Incorrect This is the opposite of what is true. People with a high need For cognition (i.e., those who usually think carefully about issues) are more likely to rely on the central route.
(Social Psychology)

622
Q
Sherif and Hovland's (1961) social judgment theory is useful for understanding which of the following?
A. interpersonal attraction
B. attitude change
C. attribution of cause
D. impression formation
A

Social judgment theory distinguishes between three “categories of judgment” that determine our susceptibility to persuasion — the latitude of acceptance, the latitude of non-commitment, and the latitude of rejection.
b. CORRECT Social judgment theory predicts that we are most likely to be persuaded (i.e., to change our attitude) when the position advocated by a message is within our latitude of acceptance.
(Social Psychology)

623
Q

In Zimbardo’s (1972) prison study, students were randomly assigned to enact the role of either a prisoner or prison guard and were placed in an environment that simulated an actual prison. Results of the study indicate: that:
A. introverted prisoners and extroverted prison guards more easily adapted to their roles than did extroverted prisoners and introverted prison guards.
B. prisoners and prison guards more easily adapted to their roles when they were paid for participating in the study than when they were not paid.
C. prisoners and prison guards both quickly adapted to their assigned roles to such an extreme that the study had to be terminated early.
D. prisoners and prison guards actively resisted their assigned roles and often deliberately engaged in “role reversal.”

A

Zimbardo’s (1972) prison study confirmed that people often alter their behaviors so that they are consistent with assigned roles.
c. CORRECT Within the first six days of Zimbardo’s prison simulation study, students assigned the role of guard became very aggressive and authoritarian (e.g., they issued arbitrary commands to prisoners to do push-ups and refused requests to go to the bathroom), while those assigned the role of prisoner quickly lapsed into depression and helplessness and began exhibiting signs of both physical and emotional distress. Because of the rapid decay of the behavior of both prison guards and prisoners, the study was halted prematurely.
(Social Psychology)

624
Q

Superorclinate goals have been found useful for:
A. clarifying roles.
B. maximizing satisfaction and motivation.
C. reducing intergroup conflict.
D. increasing feelings of self-efficacy.

A

Superordinate goals can be achieved only through intergroup cooperation.
c. CORRECT In their Robber’s Cave study, Sherif and his colleagues (1961) Found that the introduction of superordinate goals was the most effective way for reducing intergroup hostility.
(Social Psychology)

625
Q
\_\_\_\_\_ refers to publicly acting in ways that are consistent with what is requested by another person in order to obtain a reward or avoid punishment while privately disagreeing with the request.
A. Identification
B. Internalization
C. Commitment
D. Compliance
A

Research has found that social influence can have one of three effects on an individual’s opinions and behavior — compliance, identification, and internalization.
a. Incorrect Identification occurs when a person changes his/her opinion or behavior in response to a request because he/she wants to be liked by or identified with the person ma king the request.
b. Incorrect Internalization occurs when the person changes his/her opinion or behavior because the person actually (privately) accepts the beliefs or opinions of another person.
c. Incorrect Commitment is not one of the terms that is usually used to describe responses to persuasion but, in terms of its general meaning, is most similar to internalization.
d. CORRECT Compliance occurs when a person publicly changes his/her opinion to obtain a reward or avoid punishment but does not actually (privately) accept that opinion.
(Social Psychology)

626
Q
People rely on which of the following to determine how to act in specific situations (e.g., at a faculty meeting, at a wedding, or in a restaurant)?
A. heuristics
B. prototypes
C. scripts
D. life space
A

Researchers interested in social knowledge distinguish between several cognitive phenomena including schemas and prototypes.
a. Incorrect Heuristics are “mental shortcuts” that help us process complex information.
b. Incorrect Prototypes are similar to stereotypes and consist of a set of characteristics that are commonly associated with a particular category of people, objects, or events.
c. CORRECT Scripts (also known as event schemas) are a type of schema that consist of a sequence of events. Scripts provide the information needed to determine the appropriate behaviors For familiar events or situations.
d. Incorrect Life space is a key concept in Lewin’s (1951) Field theory and consists of the individual and his/her environment.
(Social Psychology)

627
Q
Schachter’s (1959) conclusion that "misery loves miserable company" is most consistent with the predictions of which of the following?
A. social impact theory
B. the overjustification hypothesis
C. the notion of psychological reactance
D. social comparison theory
A

Schachter found that, when fearful participants who were anxious about the study they were about to participate in were given a choice of waiting for the study alone or with another anxious participant, they preferred to wait with other another anxious participant. However, if anxious participants were given a choice of waiting alone or with a non-anxious individual, they chose to wait alone.
a. Incorrect Social impact theory predicts that the Force to comply is determined by the strength, immediacy, and number of sources of social influence that are present.
b. Incorrect The overjustification hypothesis predicts that providing a person with external rewards For engaging in a behavior diminishes the person’s internal motivation For doing so.
c. Incorrect Psychological reactance is the tendency to do the opposite of what is requested because one Feels the request is violating one’s Freedom-of-choice.
d. CORRECT Social comparison theory predicts that we reduce uncertainty about our Feelings, opinions, etc. by observing those of others in similar circumstances.
(Social Psychology)

628
Q

In his studies on obedience, Stanley Milgram (1963) found that research participants assigned the role of teacher would deliver electric shock to another person when the experimenter:
A. let them know they were not really harming the learner.
B. took responsibility for their (the teachers’) actions.
C. modeled the behavior by delivering high levels of shock to the learner.
D. rewarded them for doing so.

A

Participants in Milgram’s (1963) obedience to authority studies were assigned the role of teacher and asked to deliver electric shock to a learner (actually a confederate) when the learner made a mistake while recalling a list of words.
a. Incorrect The participants did not know they were not actually hurting the confederate.
b. CORRECT OF the answers given, this one best describes a conclusion that can be drawn from Milgram’s research — i.e., participants believed the experimenter was responsible For the events that occurred during the study.
c. Incorrect The experimenter did not shock the learner during the course of the study.
d. Incorrect Participants were not rewarded by the experimenter For shocking the learner.
(Social Psychology)

629
Q

A person who is low in self-monitoring (Snyder, 1987) will rely on which of the following when deciding how to act in a particular social situation?
A. his/her life scripts.
B. his/her own feelings, attitudes, and beliefs.
C. the emotional reactions and behaviors of the people he/she is interacting with.
D. internal working models that were acquired during infancy and early childhood.

A

As defined by Snyder (1987), self-monitoring refers to the degree to which people are concerned about the impressions they convey to others in social situations and are able to control those impressions.
b. CORRECT A person who is low in self-monitoring relies on his/her own feelings, attitudes, and beliefs when determining how to act in social situations and, as a result, acts similarly in different social situations.
c. Incorrect People who are high in self-monitoring are concerned about the impressions that others have of them and, as a result, adapt their behaviors to the emotional reactions and behaviors of the people in each social situation.
(Social Psychology)

630
Q
A father is quite upset about his 11-year old son's recent unwillingness to do as he is tolcl, and he tells his son, "I’m the boss around here and you must do as I say." The father is relying on which type of social power to control his son’s behavior?
A. expert
B. referent
C. legitirnate
D. coercive
A

French and Raven (1959) distinguished between six types of social power — coercive, reward, expert, referent, informational, and legitimate.
a. Incorrect When using expert power, an individual emphasizes his/her superior knowledge or expertise. The father would be using expert power if he had said, “You’ll do what I say because I know what’s best for you.”
b. Incorrect Referent power refers to the ability of an individual or group to control the behavior of others because of their desire to identify with that individual or group. The father would be relying on referent power if he said “You should do what I say because you’re part of this family.”
c. CORRECT When using legitimate power, the individual emphasizes his/her position of power or authority and the other person’s obligation to comply. In this situation, the father is relying on his position as “boss” to control his son’s behavior.
d. Incorrect Coercive power is being used when an individual influences the behavior of others through the control of punishments. The father would be using coercive power if he said, “If you don’t do what I say, you’ll have to stay in your room all weekend.”
(Social Psychology)

631
Q

The elaboration likelihood model (ELM) of persuasion predicts that:
A. central route processing produces attitude change that is more persistent over time than does peripheral route processing.
B. in terms of short-term effects, central route processing produces a greater amount of attitude change than does peripheral route processing.
C. central route processing relies more on environmental cues than does peripheral route processing.
D. central route processing produces positive attitude change while peripheral route processing is more likely to produce negative attitude change.

A

Petty and Cacioppo’s (1980) elaboration likelihood model distinguishes between central and peripheral processing routes. The two routes differ in terms of the amount of effortful cognitive processing (elaboration) they require and their impact on attitude change.

a. CORRECT The model predicts (and the research has confirmed) that attitude change resulting from central (versus peripheral] route processing results in more persistent attitude change as well as attitudes that are more predictive of behavior.
b. Incorrect In terms of immediate (short-term) effects, the two routes can produce the same amount of attitude change.
c. Incorrect This is the opposite of what is true. It is peripheral processing that relies on environmental (peripheral) cues — for example, the length of the message or the attractiveness of the person delivering the message.
d. Incorrect Both routes can produce either positive or negative (favorable or unfavorable) attitude change.(Social Psychology)

632
Q
Research has found that which of the following is rnost important  reducing the negative effects of crowding on behavior?
A. diffusion of responsibility
B. level of self-monitoring
C. a sense of control
D. behavioral incentives
A

The impact of crowding has been linked to a number of individual and environmental characteristics. Of the phenomena listed in the answers, however, only one has been identified as a key determinant of the impact of crowding on behavior.
c. CORRECT The research has found that a sense of control over the situation mediates the impact of crowding on behavior. For example, Sherrod (1974) found that, when people working on a task in a crowded situation could press an “escape button” in order to leave the situation, their performance was enhanced even though they never actually pressed the button.
(Social Psychology)

633
Q

The jigsaw technique is a(n):
A. individual decision-making strategy.
B. cooperative learning strategy.
C. method for reducing susceptibility to persuasion.
D. method for decreasing conformity to group norms

A

The jigsaw method was first used by Aronson and colleagues (1978).
b. CORRECT The jigsaw classroom was developed by Aronson and his colleagues as a method for enhancing learning and motivation and reducing racial/ethnic conflict in classrooms.
(Social Psychology)

634
Q

According to Sherif and Hovland’s (1961) social judgment theory, a person’s latitudes of rejection, non-commitment, and acceptance are affected by which of the following?
A. the person’s level of involvement with the topic addressed by the persuasive message
B. the person’s perceptions about the consequences of altering his/her opinion or behavior
C. the person’s self-efficacy beliefs
D. the person’s level of cognitive dissonance

A

Social judgment theory (Sherif and Hovland, 1961) distinguishes between three “categories of judgment” that determine our susceptibility to persuasion — the latitude of acceptance, the latitude of
non-commitment, and the latitude of rejection.
a. CORRECT Social judgment theory predicts that the size of the latitudes of acceptance, non-commitment, and rejection are affected by the individual’s level of ego-involvernent (i.e., the extent to which the person is personally involved with the topic addressed by the persuasive message). When the individual’s ego-involvement is high, his/her latitude of rejection is wide.(Social Psychology)

635
Q
The results of a research study on persuasion reveal that, one month following exposure to a persuasive message, participants in the study can remember the message but have forgotten the source of the message. This result is a manifestation of which of the following?
A. primacy effect
B. deindividuation
C. sleeper effect
D. correspondence bias
A

The phenomenon described in this question is probably one that you have experienced yourself. It has been used to explain why the attitude change produced by a high-credible communicator decreases over time, while the attitude change produced by a low-credible communicator
increases over time.
a. Incorrect The primacy effect refers to the tendency to remember stimuli that occurred first better than stimuli that occurred last.
b. Incorrect Deindividuation refers to the tendency for people to be more willing to act in antisocial ways when they can do so anonymously.
c. CORRECT This question describes the sleeper effect — i.e., the tendency to remember a message over time but to forget its source.
d. Incorrect The correspondence bias is another name for the fundamental attribution bias and is not relevant to the phenomenon described in this question.
(Social Psychology)

636
Q
You are convinced that you are psychic because you're able to predict when close friends are going to call you on the phone. As a result, you always notice the times when your predictions are correct but ignore those times when they’re not. This is an example of which of the following?
A. correspondence bias
B. confirmation bias
C. self-fulfilling prophecy effect 
D. post hoc fallacy
A

In the situation described in this question, you are noticing only those phone calls that that confirm your belief about your psychic ability.
a. Incorrect The correspondence bias is another name for the fundamental attribution bias, which is the tendency to attribute the behaviors of others to dispositional factors.
b. CORRECT This situation illustrates the confirmation bias, which predicts that we’re most likely to pay attention to events that confirm our preconceptions.
c. Incorrect The self-fulfilling prophecy effect occurs when a person’s expectations alter the behaviors of the targets of those expectations.
d. Incorrect The post hoc fallacy occurs when we conclude that Event A ca used Event B simply because Event A preceded Event B.
(Social Psychology)

637
Q

Moscovici (1985) found that individuals with a minority opinion are most likely to change the minds of those holding the majority opinion when those with the minority opinion:
A. express their opinion as consistently as possible.
B. use ingratiation techniques to gain the acceptance of members of the majority.
C. initially agree with the majority position and gradually introduce their own opinion.
D. point out the ways in which they agree with the majority.

A

Moscovici (1985) was interested in the factors that increase the likelihood that a minority can sway the opinion of the majority.

a. CORRECT Moscovici found consistency to be the key factor in determining whether a minority would successfully change the opinion of the majority.
b. Incorrect This was not found by Moscovici to be an effective way for a minority to alter the opinion of the majority.
c. Incorrect Maintaining a consistent position is more effective than is initially agreeing with the majority.
d. Incorrect This has not been found to be an effective way for a minority to alter the opinion of the majority.(Social Psychology)

638
Q

A therapy client is exhibiting a depressive attributional style when she consistently attributes the negative events she experiences to:
A. internal, stable, and global factors.
B. external, stable, and global factors.
C. internal, unstable, and specific factors.
D. external, unstable, and specific factors.

A

Researchers interested in causal attribution distinguish between three attributional dimensions: internal/external, stable/unstable, and global/specific.
a. CORRECT Abramson, Seligman, and Teasdale (1978) Found that people who are depressed often exhibit signs of learned helplessness. In other words, they tend to attribute negative events to internal, stable, and global Factors.
(Social Psychology)

639
Q

With regard to the reduction of racial/ethnic discrimination, Gordon Allport (1954) concluded that:
A. folkways must always precede stateways.
B. stateways must always precede stateways.
C. it is not always necessary for folkways to precede stateways.
D. it is necessary for folkways and stateways to change simultaneously.

A

As defined by Allport (1954), folkways are personal attitudes and beliefs, while stateways are laws.
c. CORRECT Allport concluded that stateways can effectively precede Folkways. In other words, he believed that laws prohibiting discrimination can be effective even when they do not reflect public consensus.
(Social Psychology)

640
Q
Studies investigating the impact of viewing violent pornography have supported the predictions of which of the following?
A. frustration-aggression hypothesis
B. catharsis hypothesis
C. social learning theory
D. vicious circle hypothesis
A

The research has found that exposure to pornography with violent themes increases the viewers’ aggressive behaviors as well as acceptance of violence against women.
c. CORRECT Social learning theory predicts that we acquire behaviors by observing others engage in those behaviors. Therefore, its predictions are consistent with the results of studies investigating the effects of viewing violent pornography.
(Social Psychology)

641
Q
Research on consumer behavior has found that providing a bonus to customers for purchasing a product increases the sales of that product. However, if the bonus is subsequently discontinued, product loyalty declines substantially. This phenomenon is predicted by which of the following?
A. availability heuristic
B. dilution effect
C. psychological reactance 
D. overjustification hypothesis
A

In the situation described in this question, removal of an external reinforcement produces a decrease in the target behavior.
a. Incorrect The availability heuristic refers to the tendency to judge the frequency of an event based on how easy it is to think of examples of that event
b. Incorrect The dilution effect refers to the weakening of an inference about someone due to the presence of irrelevant information about him/her.
c. Incorrect Reactance occurs when a person does not comply with a request because he/she feels that the request violates his/her sense of personal freedom.
d. CORRECT The overjustification hypothesis predicts that a person’s internal motivation to perform a behavior will be weakened when he/she is provided with external reinforcement for doing so.
(Social Psychology)

642
Q

The theory of planned behavior (Ajzen, 1991) predicts that:
A. attitudes are the result of behaviors.
B. attitudes are the result of subjective norms, behavioral intentions, and behaviors.
C. behaviors are the result of attitudes.
D. behaviors are the result of attitudes, subjective norrns, and behavioral intentions.

A

Ajzen’s (1991) theory of planned behavior (also known as the theory of reasoned action) predicts that behaviors are the result of several factors.
c. Incorrect This answer is only partially correct and, therefore, not the best answer of those given.
d. CORRECT The theory of planned behavior attributes behaviors to three factors: attitudes toward the behavior; beliefs about people’s opinions of the behavior and motivation to comply with people’s expectations (subjective norms); and the intention to perform the behavior (behavioral intention).
(Social Psychology)

643
Q

According to social comparison theory:
A. we feel better when people like and appreciate us.
B. we often judge our own actions by looking at those of other people.
C. we tend to imitate other people who we perceive to be most like us
D. we are most attracted to people who compare favorably with us.

A

Even if you’re unfamiliar with social comparison theory, you may have been able to identify the correct answer to this question since its name describes what it predicts.
b. CORRECT Social comparison theory “sounds like” what it is: It predicts that people make judgments about themselves by comparing their behaviors and beliefs to those of other people.
(Social Psychology)

644
Q
In a research study, prison inmates and counselors working at the prison were asked to explain why the inmates had committed their crimes. In response, the inmates cited situational factors, while the counselors attributed the offenses to the dispositional characteristics of the inmates. Which of the following predicts the outcome of this study?
A. fundamental attribution bias
B. self-serving bias
C. actor-observer effect
D. false consensus effect
A

The answers to this question are all attribution biases or other cognitive biases that you’ll want to be familiar with for the licensing exam.

a. Incorrect The fundamental attribution bias is the tendency to overestimate the role of dispositional factors when ma king attributions about the behaviors of others. Since the fundamental attribution bias only explains the counselors attributions about the crimes committed by the inmates, this answer is only partially correct and not the best answer of those given.
b. Incorrect The self-serving bias refers to the tendency to make dispositional attributions for our successes but situational attributions for our failures.
c. CORRECT The actor-observer effect addresses the discrepancy in an actor’s and observer’s attributions for the actor’s behavior. It predicts that actors tend to attribute their own behaviors to situational factors, while observers tend to attribute the actors’ behaviors to dispositional factors.
d. Incorrect The false consensus effect is the tendency to overestimate the extent to which others agree with us.(Social Psychology)

645
Q
The tendency to seek out and interpret information in ways that verify our existing beliefs is referred to as the:
A. hindsight bias.
B. primacy effect.
C. focusing effect.
D. confirmation bias.
A

Even if you’re unfamiliar with the cognitive bias being asked about by this question, you may have been able to identify the correct answer (confirmation bias) because it “sounds like” what is it.

a. Incorrect The hindsight bias refers to the tendency to believe that, after an outcome occurs, you could have predicted the outcome beforehand.
b. Incorrect The primacy effect refers to the tendency to remember stimuli that occurred first better than stimuli that occurred last.
c. Incorrect The focusing effect is the tendency to place too much importance (focus) on one aspect of an event or situation when predicting a future outcome
d. CORRECT The confirmation bias is a cognitive bias which, as its name implies, involves focusing on and seeking out information that confirms our preconceptions.(Social Psychology)

646
Q
The predictions of social exchange theory are most applicable to our relationships with:
A. close friends.
B. parents.
C. romantic partners.
D. business associates.
A

Even if you’re unfamiliar with the predictions of social exchange theory you may have been able to identify the correct answer to this question by noticing that one of the answers is unlike the others.
d. CORRECT The research suggests that social exchange theory does not apply to people we are very close to (e.g., close friends, pa rents, and romantic partners) but, instead, to individuals we have more Formal relationships with or have recently met (e.g., business associates, acquaintances, and strangers).
(Social Psychology)

647
Q
An employee expects greater consistency in the behavior of his boss than is warranted. The employee's expectation is best accounted for by which of the following?
A. overjustification hypothesis
B. self-serving bias
C. fundamental attribution bias
D. self-verification theory
A

This question is vague, and you may have had to rely on the process of elimination to identify the correct answer.
a. Incorrect The overjustification hypothesis predicts that internal motivation to perform a specific action is weakened when one is given external reinforcement for performing the action.
b. Incorrect The self-serving bias applies to the attributions one makes about one’s own behavior and is not relevant to the situation described in this question
c. CORRECT The fundamental attribution bias is the tendency to attribute another person’s behavior to dispositional factors (e.g., traits) and, therefore, to expect a great deal of consistency in behavior.
d. Incorrect Self-verification theory predicts that people prefer receiving information about themselves from others that is consistent with their self-evaluations.
(Social Psychology)

648
Q
Asch (1946) found that some characteristics (e.g., warm and cold) influence the impressions people form of others more than other characteristics do, and he referred to these influential characteristics as:
A. stable attributes.
B. central traits.
C. schemata. 
D. stereotypes.
A
Solomon Asch (1946) was among the first to conduct research on impression formation. The results of his studies confirmed that some characteristics have a greater impact on impression formation than others do.
b. CORRECT When Asch presented participants in his studies with a list of adjectives that supposedly described another person, he found that certain traits were the most important determinants of the pa rticipa nts’ impressions of that person. Asch concluded that these central (influential) traits are characteristics that are highly associated with other characteristics and, as a result, have a strong impact on impression formation.
(Social Psychology)
649
Q

In a research study on bystander intervention, a participant hears someone (a confederate) fall and cry out in pain in an adjacent room. In this situation, the participant is most likely to respond to this apparent need for help when the participant is:
A. alone.
B. with a friend.
C. with a stranger.
D. with three or more other participants.

A

Studies on bystander intervention have found that a person in need is most likely to receive help when there is only one bystander.
a. CORRECT Research has shown that a bystander is more likely to intervene when alone than when in the presence of others, regardless of whether the others are strangers or friends.
d. Incorrect The greater the number of bystanders, the greater the bystander apathy (i.e., the less likely a person will receive assistance).
(Social Psychology)

650
Q
Research on \_\_\_\_\_ has found that people tend to pay more attention to information that confirms their beliefs about themselves than to information that contradicts those beliefs.
A. propositions
B. heuristics 
C. schemas
D. metamemory
A

Researchers interested in how people interpret their experiences distinguish between several phenomena including schemas, prototypes, and scripts.
a. Incorrect Researchers interested in declarative knowledge define propositions as simple ideas that are either true or false.
b. Incorrect Heuristics are “mental shortcuts” that help us process complex information.
c. CORRECT Schemas (schemata) are organized mental networks of information that are based on prior experience or knowledge and that influence our interpretation of or reaction to current experience. Research on schemas has found that people tend to pay attention to and recall schema-consistent information better than information that is inconsistent with their current schemas. For example, a person who considers herself to be a very independent person will be able to recall many examples of her independence but few, if any, examples of times when she acted in dependent ways.
d. Incorrect Metamemory refers to awareness of one’s own memory processes.
(Social Psychology)

651
Q
Zimbardo's (1972) prison study illustrated the impact of which of the following on behavior?
A. psychopathology
B. group conformity
C. deindividuation
D. social roles
A

In Zirnbardo’s (1972) prison study, students were randomly assigned to enact the role of either a prisoner or prison guard and were placed in an environment that simulated an actual prison.
d. CORRECT Zimba rdo’s (1972) prison study confirmed that people often alter their behaviors so that they are consistent with assigned roles: Even though students had been randomly assigned to the role of prisoner or guard, they quickly adopted behaviors consisted with their roles.(Social Psychology)

652
Q
A person with symbolic racist views is least likely to oppose which of the following?
A. affirmative action
B. school integration.
C. welfare
D. bilingual election ballots
A
Symbolic racism (Sears, 1988) represents a combination of anti-black affect and conservative values, especially individualism. Note that this question is asking which type of opposition is g characteristic of symbolic racism.
b. CORRECT In contrast to “old-fashioned" racism, symbolic racism is characterized by more subtle forms of prejudice and discrimination. A person with symbolic racist views is not likely to oppose school integration because doing so represents an overt form of racism. However, he/she is likely to oppose other efforts designed to promote equality or to benefit members of minority groups — e.g., affirmative action, welfare, and bilingual ballots — on the ground that they violate traditional American values.
(Social Psychology)
653
Q

According to Berscheid’s (1991) emotion-in-relationships model, strong emotions in romantic relationships occur when:
A. interactions between partners deviate from expected patterns.
B. interactions between partners become coercive.
C. a partner’s internal working model of intimate relationships is not effective.
D. a partner’s illusions (unrealistic expectations) about romantic relationships are contradicted.

A

Berscheid’s emotion-in-relationships model (ERM) is one of several models that have been proposed as explanations for affect in close relationships.
a. CORRECT Berscheid’s model proposes that strong positive or negative emotions occur when there is a disruption in interpersonal scripts — i.e., when a partner violates expectations regarding important couple or personal goals.
d. Incorrect Although ERM addresses the violation of expectations, it does not attribute affect only to a violation of unrealistic expectations, so this is not the best answer of those given.
(Social Psychology)

654
Q
According to Herek (1991), harassment of and violence against lesbians and gay men is the result of:
A. an antisocial predisposition.
B. extreme authoritarianism.
C. homophobia.
D. heterosexism.
A

Herek (1991) describes the harassment of and violence against lesbians and gay men as an extension of the heterosexism that pervades society.
d. CORRECT Herek (1991) defines heterosexism as “an ideological system that denies, denigrates, and stigmatizes among nonheterosexual forms of behavior, identity, relationships, or community” (p. 150). His research has linked a combination of individual (psychological) and cultural heterosexism to violence against lesbians and gay men.
(Social Psychology)

655
Q

When a listener is exposed to both sides of an argument, a primacy effect is most likely to occur when:
A. the first side of the argument is presented immediately before the second side and the listener’s attitude is measured immediately after presentation of the second argument.
B. the first side of the argument is presented immediately before the second side and the listener’s attitude is measured at a later time.
C. there is a delay between presentation of the first and second sides of the argument and the listener’s attitude is measured immediately after resentation of the second argument.
D. there is a delay between presentation of the first and second sides of the argument and the listener’s attitude is measured at a later time.

A

The research has found that the order of presentation of two sides of an argument and the time at which a listener’s attitude is measured determine whether a primacy or recency effect will occur.
b. CORRECT When both sides of an argument are presented, a primacy effect occurs (the listener is swayed more by the first argument) when the second argument immediately follows the first and the attitude measure is administered at a later time. In contrast, a recency effect occurs (the listener is swayed more by the second argument) when there is a delay between presentation of the two sides of the argument and the attitude measure is administered immediately after the second argument.
(Social Psychology)

656
Q

Which of the following is true about a person who is faced with an approach-avoidance conflict?
A. The closer the person gets to his/her goal, the stronger the desire to approach it.
B. The closer the person gets to his/her goal, the stronger the desire to avoid it.
C. Regardless of his/her proximity to the goal, the person’s desire to avoid the goal is stronger than his/her desire to approach it.
D. Regardless of his/her proximity to the goal, the person’s desire to approach it is equal to the desire to avoid it.

A

Lewin (1948) distinguished between three motivational conflicts: approach-approach, avoidance-avoidance, and approach-avoidance.
b. CORRECT The approach-avoida nce conflict occurs when a goal has both positive and negative aspects. This kind of conflict is difficult to resolve because, as a person moves closer to the goal, the desire to avoid it becomes stronger and, as the person moves away from the goal, the desire to approach it increases.
(Social Psychology)

657
Q

According to Bem’s (1972) self-perception theory:
A. people experience discomfort (dissonance) when their behaviors are not consistent with their attitudes, beliefs, or values.
B. people enhance their own sense of self-worth by associating or identifying with others who are successful.
C. people tend to attribute their successes to dispositional factors and failures to situational factors.
D. people infer their own attitudes and emotions by observing their own behaviors and the circumstances in which those behaviors occur.

A

Self-perception theory predicts that people judge or determine their own internal states in the same way that they determine the states of other people — i.e., by considering external cues.
a. Incorrect This is predicted by cognitive dissonance theory but not by self-perception theory.
b. Incorrect This is not a prediction of self-perception theory.
c. Incorrect This answer describes the self-serving bias.
d. CORRECT According to self-perception theory, we’re most likely to rely on external information for determining or judging our internal states when internal cues are ambiguous, weak, or difficult to interpret.
(Social Psychology)

658
Q
Dr. Smith, a school psychologist, is working with a student who has recently failed an important exam. When she asks the student why he failed, he says it's because the test items were very tricky and poorly worded. The student's statement is a manifestation of which of the following?
A. learned helplessness
B. low self-monitoring
C. external locus of control
D. hindsight bias
A

According to Rotter (1966), people differ with regard to the amount of control they feel they have over their own lives. Specifically, people with an internal locus of control feel that their behavior is controlled by personal choice and free will, while people with an external locus of control feel that their behavior is controlled by fate and other external factors.
a. Incorrect Lea med helplessness refers to the belief that negative consequences are unavoidable and is associated with internal, stable, and global attributions. In the situation described in this question, the student is ma king an external attribution (the exam items were tricky and poorly worded).
b. Incorrect People who are low in self-monitoring use their own values and beliefs to guide their behavior. In contrast to those who are high in self-monitoring, these individuals do not look for external cues to determine the most appropriate behaviors for the situation.
c. CORRECT A student with an external locus of control would be likely to attribute his test failure to factors external to himself (e.g., to difficult exam questions).
d. Incorrect The hindsight bias refers to the tendency to believe that, after an outcome occurs, you could have predicted the outcome beforehand.
(Social Psychology)

659
Q

When Rosenhan’s (1973) pseudopatients were admitted to a mental health facility after reporting that they were “hearing voices,” they were:
A. recognized as being “normal” by most staff members but not by the other patients.
B. recognized as being “normal” by the other patients more often than by the staff members.
C. not recognized as being “normal” by the staff members or patients until they stopped faking their symptoms.
D. not recognized as being “normal” by the staff members or patients even after they stopped faking their symptoms.

A

Rosenhan and seven of his assistants had themselves admitted to different mental health facilities by complaining of “hearing voices.” Once they were admitted, they stopped faking their symptoms.
b. CORRECT Nearly 1/3 of the patients identified the pseudopatients as sane but the hospital staff members diagnosed all but one of the pseudopatients as schizophrenic.
(Social Psychology)

660
Q

People who frequently view violent shows on television are most likely to:
A. exhibit high levels of empathy for the victims of violence.
B. judge aggressive retaliation as unacceptable and unjustifiable.
C, report that they have been the victims of unprovoked violence.
D. overestimate the likelihood that they will be a victim of violence.

A

Most of the research on media violence has produced results that are consistent with social learning theory — i.e., that we acquire attitudes, beliefs, and behaviors through observation.
a. Incorrect This is the opposite of what is true. Long-term exposure to media violence has been linked to a decrease in both empathy and concern for the victims of violence.
b. Incorrect This is also the opposite of what is true. People who are often exposed to media violence are more likely to judge aggressive retaliation more positively than do people who are infrequently (or not) exposed to media violence.
c. Incorrect Although long-term exposure to violence has been linked to a tendency to overestimate one’s risk for being a victim of violence, it has not been linked to an increase in reported rates of being a victim.
d. CORRECT The research has confirmed that long-term exposure to media violence is associated with a tendency to overestimate the amount of violence in society as well as the likelihood of being a victim of violence.
(Social Psychology)

661
Q

Berkowitz (1971) revised the original frustration-aggression hypothesis by proposing that frustration leads to aggression only when:
A. the aggressor anticipates positive consequences for acting aggressively.
B. there are aggressive cues in the environment.
C. the recipient of the aggression has low status.
D. the recipient of the aggression provoked the aggressor.

A

The original frustration-aggression hypothesis proposed by Dollard et al (1939) was subsequently modified by several researchers including Berkowitz.
b. CORRECT Berkowitz’s (1971) research found that Frustration leads to a “readiness for aggression” but that the actual expression of aggression requires anger arousal plus the presence of appropriate environmental (aggressive) cues
(Social Psychology)

662
Q

Lewin’s (1951) field theory predicts that human behavior is a function of:
A. the person’s attitude, values, and beliefs.
B. the person’s actual and perceived abilities.
C. the characteristics of the person and the person’s environment.
D. the antecedents and consequences of the behavior.

A

Lewin’s (1951) field theory is expressed by the following formula: B =f(P,E).
c. CORRECT Lewin described behavior as a function of interactions between the person’s characteristics (e.g., heredity, personality, health) and the characteristics of the person’s social environment.
(Social Psychology)

663
Q

In a research study, boys observe a model act aggressively toward an inflatable “Bobo” doll. For some boys, the model is rewarded for acting aggressively; for others, the model is punished; and, for others, the model is neither rewarded nor punished. Subsequently, all boys are offered incentives for acting aggressively toward the doll. Based on your knowledge of Bandura’s (1983) research on social learning theory, you predict that:
A. only boys who observed the model being rewarded will act aggressively toward the doll.
B. only boys who observed the model being rewarded or receiving no consequence will act aggressively toward the doll.
C, only boys who observed the model being punished will act aggressively toward the doll.
D. all boys (those who saw the model rewarded, punished, or receiving no consequence) will act aggressively toward the doll.

A

The study described in this question was originally conducted by Bandura (1983).
d. CORRECT Ba ndura found that, when no incentive was offered to the boys for imitating the model’s behavior, only those in the reward or no consequence condition did so. However, when boys were offered an incentive for imitating the model, all boys acted aggressively toward the Bobo doll. These results confirmed that learning and performance are distinct — i.e., all boys learned the aggressive behavior by observing the model regardless of whether the model received a reward, punishment, or no consequence for acting aggressively.
(Social Psychology)

664
Q

In a research study, a social psychologist offers participants either $1.00 or $20.00 to tell potential participants that a dull experiment was very interesting. With regard to cognitive dissonance theory and self-perception theory, which of the following is true?
A. Cognitive dissonance theory predicts that participants in the $1.00 condition will subsequently report greater liking for the dull experiment, while self-perception theory predicts that participants in
the $20.00 condition will subsequently report greater liking for the dull experiment.
B. Self-perception theory predicts that participants in the $1.00 condition will subsequently report greater liking for the dull experiment, while cognitive dissonance theory predicts that participants in the $20.00 condition will subsequently report greater liking for the dull experiment.
C. Cognitive dissonance theory and self-perception theory both predict that participants in the $1.00 condition will subsequently report greater liking for the dull experiment than will those in the $20.00 condition.
D. Cognitive dissonance theory and self-perception theory both predict that participants in the $20.00 condition will subsequently report greater liking for the dull experiment than will those in the $1.00 condition.

A

Cognitive dissonance theory and self-perception theory make the same prediction about the outcome of the study described in this question but do so for different reasons.
c. CORRECT Cognitive dissonance theory predicts that participants in the $1.00 condition will report liking the dull experiment more than will those in the $20.00 condition in order to reduce the dissonance they feel about describing the dull study as interesting to potential participants. In contrast, self-perception theory predicts that participants in the $1.00 condition will report greater liking for the dull experiment because they will evaluate their actual opinion of the study by
looking at their overt behavior (telling others that the experiment was interesting).
(Social Psychology)

665
Q
A manager or consultant interested in maximizing her influence on employees should be aware that the productivity of employees is most likely to be positively affected if she relies on which bases of social power?
A. expert and informational
B. informational and legitimate
C. legitimate and referent
D. expert and referent
A

French and Raven (1959) distinguished between six bases of social power: reward, coercive, legitimate, referent, informational, and expert
a. Incorrect Expert power is important for a manager or consultant (see response d), but the problem with informational power is that, once the employee has the information, the manager or consultant would no longer have power.
b. Incorrect A person has legitimate power when he/she is accepted as the “boss” and informational power when he/she has needed information. Over the long-run, neither of these sources of power would be the most effective.
c. Incorrect Referent power is important (see response d), but legitimate power is not associated with increased productivity.
d. CORRECT Several authorities have suggested that incremental power (which is a combination of expert and referent power) is optimal for managers and consultants because it has the greatest positive effects on employee performance. A person has expert power when he/she has needed expertise and referent power when he/she is liked and respected.
(Social Psychology)

666
Q
Which of the following emphasizes the role of the costs and rewards of a relationship on a person's decision to stay in the relationship?
A. gain-loss theory
B. expectancy theory
C. social exchange theory
D. social cornparison theory
A

For the licensing exam, you’ll want to be familiar with all of the phenomena listed in the answers to this question.
a. Incorrect Gain-loss theory (also known as the gain-loss effect) refers to the tendency to like or be most attracted to people who initially evaluate us negatively but subsequently evaluate us in positive ways.
b. Incorrect Expectancy theory predicts that motivation is the result of three beliefs — expectancy, instrumentality, and valence.
c. CORRECT As its name implies, social exchange theory is based on the premise that human behavior is affected by the exchange of rewards between actors. Specifically, it predicts that the decision to stay in a relationship depends on the balance of costs and rewards: When a person believes that costs exceed rewards, the person will leave the relationship, but when the person believes that rewards exceed the costs, he/she will stay in the relationship.
d. Incorrect Social comparison theory predicts that people make judgments about themselves by comparing their behaviors, attitudes, and beliefs to those of other people.
(Social Psychology)

667
Q

Research on persuasion has found that people who argue against their own self-interest are:
A. usually viewed by listeners as untrustworthy.
B. usually viewed by listeners as credible.
C. viewed as reliable by uninformed listeners only.
D. viewed as knowledgeable by listeners who have low-esteem.

A

The studies have found that a comrnunicator’s credibility is affected by a number of factors including his/her apparent motives for advocating a particular position.
b. CORRECT Walster, Aronson, and Abrahams (1966) found that communicators are usually considered credible by listeners when they are arguing against their own self-interest, apparently because they are perceived as more trustworthy when doing so.
(Social Psychology)

668
Q
A movie viewer is most likely to report feeling uncomfortable in a crowded movie theater when he/she is viewing a \_\_\_\_\_ film.
A. frightening horror
B. sexuaflyarouwng
C. humorous
D. boring
A

The consequences of crowding depend on several factors including the nature of the environment and task.
d. CORRECT Research by Worchel and Brown (1984) found that crowding is experienced as less uncomfortable or stressful in attention-grabbing, arousing situations than in uninteresting situations. They conclude that this difference is due to the fact that crowding produces arousal and, when the arousal can be attributed to the circumstances, it is not experienced as unpleasant. However, when the arousal cannot be attributed to the circumstances (e.g., when the movie is boring), it is experienced as unpleasant.
(Social Psychology)

669
Q

Research on the intergroup contact hypothesis has demonstrated that antagonism between members of two groups is most likely to be reduced when:
A. members of the two groups are initially provided with numerous opportunities for casual (superficial) contact.
B. members of the two groups are provided with incentives for getting along while interacting.
C. the leaders from each group meet to discuss the issues underlying the antagonism before group members interact.
D. interactions between group members provide opportunities to disconfirm negative stereotypes.

A

The research has shown that, for interpersonal contact to reduce intergroup hostility, certain conditions must be met.
a. Incorrect Casual contact may actually provide opportunities for continued expressions of hostility.
b. Incorrect The research on the intergroup contact hypothesis has not found this to be an effective method for reducing intergroup hostility.
c. Incorrect In this situation, group leaders would likely be subsequently viewed by group members with suspicion, resulting in even more problems.
d. CORRECT Of the answers given, this one is most consistent with the research. Intergroup contact is most likely to reduce hostility when it provides opportunities for learning that negative stereotypes about members of the opposing group are untrue.
(Social Psychology)

670
Q
The belief that watching someone else act aggressively will serve to reduce one's own aggressiveness is most consistent with the notion of:
A. inoculation.
B. vicarious reinforcement.
C. catharsis.
D. vicarious desensitization.
A

A number of approaches have been investigated as means for reducing aggression.
c. CORRECT Catharsis involves the release of emotion, and some experts believe that one way to reduce aggression, anger, and hostility is through catharsis. Note, however, that the research has not been very supportive of this proposal.
(Social Psychology)

671
Q

The results of research on the jigsaw classroom are consistent with which of the following?
A. the results of Sherif’s Robber’s Cave study
B. the results of Zirnbardo’s deindividuation study
C. the predictions of Berkowitz’s frustration-aggression hypothesis
D. the predictions of Ajzen and Fishbein’s theory of reasoned action

A

In a jigsaw classroom, students work together in teams to complete an assignment (i.e., to achieve a superordinate goal). The research has found that the jigsaw classroom helps reduce hostilities related to racial, ethnic, or cultural differences.

a. CORRECT Sherif found that cooperation in achieving a superordinate goal reduced hostility between groups of boys.
b. Incorrect Zimbardo’s deindividuation study found that people are more willing to act in antisocial ways when they can do so anonymously.
c. Incorrect Berkowitz found that frustration may lead to aggression, especially in the presence of aggressive cues.
d. Incorrect According to the theory of reasoned action, people consider two factors when deciding whether to behave in certain ways: their attitude toward the behavior and the subjective norms that apply to the situation.(Social Psychology)